Está en la página 1de 296

GROUP-1

CLASS XII: MATHEMATICS

BLUE PRINT

S.No. Topics 1 Mark 2 Marks 4 Marks 6 TOTAL


Marks
1 a) RELATIONS AND FUNCTIONS 1 1

10
b) INVERSE TRIGONOMETRIC 1 2
FUNCTIONS
2 a) MATRICES 1 1
13
b) DETERMINANTS 1 1
3 a) CONTINUITY & DIFFERENTIABILITY 1 1
b) APPLICATION OF DERIVATIVES 1 3
c) INTEGRATION 1 1 1 44
d) APPLICATION OF INTEGRALS 1
e) DIFFERENTIAL EQUATIONS 1 1
4 a) VECTORS 1 1 1
17
b) 3-DIMENTIONAL GEOMETRY 1 1
5 LINEAR PROGRMMING 1 6
6 PROBABILITY 1 2 10
TOTAL 4X1 = 8X2 = 11X4 = 44 6X6=36 100(29)
4 16
KENDRIYA VIDYALAYA SANGATHAN
MODEL QUESTION PAPER 2017-18
MATHEMATICS
CLASS: XII Max.Marks:100 Time Allowed: 3 hours
General Instructions:
1. All questions are compulsory
2. This question paper consists of 29 questions divided into three sections A, B and C and D.
Section A comprises of 4 questions of one mark each, section B comprises of 8 questions of
two marks each, section C comprises of 11 questions of four marks each and section D
comprises 6 questions of six marks each.
3. All questions in Section A are to be answered in one word, one sentence or as per the exact
requirement of the question
4. There is no overall choice. However, internal choice has been provided in 03 questions of
four marks each and 02 questions of six marks each. You have to attempt only one of the
alternatives in all such questions.
5. Use of calculators is not permitted. You may ask for logarithmic tables, if required
SECTION – A
1. Find gf(-3) if f(x) =|𝒙| and g(x) = |𝟓𝒙 − 𝟐|
3𝜋
2. Find the principal value of 𝑠𝑖𝑛−1 [𝑠𝑖𝑛 5 ]
3 1
3. If A = ( ), then find |𝑎𝑑𝑗𝐴|
2 −3
4. If 𝑎⃗ = 𝑖̂ + 2𝑗̂ − 3𝑘̂ and 𝑏⃗⃗ = 2𝑖̂ + 4𝑗̂ + 9𝑘̂, find a unit vector parallel to𝑎⃗ + 𝑏⃗⃗
SECTION – B
sin 𝛼 cos 𝛼
5. If 𝐴 = ( ), verify that A’ A = I.
−𝑐𝑜𝑠𝛼 sin 𝛼
1 1 1 1
6. Evaluate: 𝑡𝑎𝑛−1 + 𝑡𝑎𝑛−1 + 𝑡𝑎𝑛−1 + 𝑡𝑎𝑛−1
5 7 3 8
3𝑎𝑥 + 𝑏, 𝑖𝑓𝑥 > 1
7. If the function f(x) given by: 𝑓(𝑥) = { 11 𝑥 = 1 , is continuous at x = 1, find the value
5𝑎𝑥 − 2𝑏𝑖𝑓𝑥 < 1
of a and b.
8. Find the intervals in which the function f given by f(x) =𝑥 2 − 𝑥 + 6 is: Strictly decreasing.
2+sin 2𝑥
9. Evaluate:∫ 1+cos 2𝑥 𝑒 𝑥 𝑑𝑥
10. Form the differential equation of the family of hyperbolas having foci on x-axis and center at
origin.
11. Find the area of the parallelogram whose adjacent sides are determined by the vectors
 ˆ ˆ 
a  i  j  3kˆ and b  2iˆ  7 ˆj  kˆ .
12. A die is thrown twice and the sum of numbers appearing is observed to be 7. What is the
conditional probability that the number 2 has appeared at least once?
SECTION – C
13. By using properties of determinants, show that
a b c
a2
b c 2  (a  b)(b  c)(c  a)(a  b  c)
2

bc ca ab


14. If y = 𝑙𝑜𝑔{𝑥 + √𝑥 2 + 𝑎2 } prove that (x2 + a2) y2 + x y1 = 0
 (𝑥 2 +1)(𝑥 2 +4)
15. Evaluate: ∫ (𝑥 2 +3)(𝑥 2 −5) 𝑑𝑥
OR
(3sin   2) cos  d
Evaluate:  (5  4 sin   cos 2
)
16. A window is in the form of a rectangle surrounded by a semi-circular opening. The total
perimeter of window is 10 meters. Find the dimensions of the window so as to admit maximum
light through the whole opening. Your friend is constructing a house for him. What will you
suggest him in order to have enough light and air?
17. Find the intervals in which the function f given by f(x) = sinx+cos x, 0 ≤ x ≤ 2 is strictly increasing
or strictly decreasing.
OR
Prove that the curves x = y2 and xy = k cut at right angles if 8k2 = 1.
18. Show that semi-vertical angle of a right circular cone, of given surface area and maximum
1
volume, is sin−1 3 .
19. Solve the following differential equation: 𝑥 𝑑𝑦 − 𝑦 𝑑𝑥 = √(𝑥 2 + 𝑦 2 ) 𝑑𝑥
 
20. Let a = î + 4ĵ + 2 k̂ , b = 3 î - 2ĵ + 7 k and c = 2î - ĵ + 4 k . Find a vector d which is perpendicular
to

both a and b , and c . d = 15.


21. Find the shortest distance between the lines :
x 1 y 1 z 1 x 3 5 y z 7
  and  
7 6 1 1 2 1
22. In a bolt factory, machines A, B and C manufacture 25%, 35% and 40% of the total. Of their
output 5%, 4%, 2% are defective. A bolt is drawn at random from the product. a) What is the
probability that the bolt drawn is defective? b) If the bolt drawn is found to be defective, find
the probability that it is a product of machine B?
23. An insurance company insured 2000 scooter drivers, 4000 car drivers and 6000 truck drivers. The
probabilities of their accidents are 0.01, 0.03 and 0.15 respectively. One of the insured persons
met with an accident. What is the probability that he is a scooter driver? What moral value will
you assign to all?
SECTION – D
24. Using Matrices, solve the following system of equations:
2x – 3y + 5z = 11; 3x + 2y – 4z = -5 ; x + y – 2z = -3
𝜋 𝑥
25. Evaluate: ∫0 𝑎2 𝑐𝑜𝑠2 𝑥+ 𝑏2 𝑠𝑖𝑛2 𝑥dx
26. Find the area of the region: {(x , y) : x2 + y2 ≤ 1 ≤ x + y}
27. Find the distance of the point A (-2, 3, -4) from the line (x+2)/3 = (2y+3)/3 =(3z+4)/5 measured
parallel to the plane 4x+12y-3z+1=0 ?
28. A manufacturer produces two products A and B. Both the products are processed on two
different machines. The available capacity of the I machine is 12 hours and that of II machine is 9
hours. Each unit of product A requires 3 hrs. On both machine and each unit of product B
requires 2 hrs on I machine and 1 hr. on II machine. Each unit of product A is sold at a profit of
Rs. 5 and B at a profit of Rs. 6. Find the production level for maximum profit graphically.
n 1
f : N  N  2 , if n is odd
29. Let be defined by f ( x)  
n
 , if n is even
 2
Examine whether f is one to one, onto or bijective, Justify your answer.

Solutions/Marking Scheme
SECTION A
1. gf(x) = |5|𝑥| − 2| gf(-3) = 13
3𝜋 2𝜋 2𝜋 2𝜋 𝜋 𝜋
2. 𝑠𝑖𝑛 = 𝑠𝑖𝑛 [𝜋 − ] = 𝑠𝑖𝑛 , ∈ [− , ]
5 5 5 5 2 2
2𝜋
PRINCIPAL VALUE =
5
−3 −1
3. 𝑎𝑑𝑗 𝐴 = ( ) |𝑎𝑑𝑗 𝐴| = -11
−1 3
𝑎 + 𝑏⃗⃗ =3 𝑖̂ + 6𝑗̂ + 6𝑘̂
4. ⃗⃗⃗⃗
⃗⃗⃗⃗
𝑎+𝑏⃗⃗ ̂
3 𝑖̂+6𝑗̂ +6𝑘
Parallel unit vector = ⃗⃗|
=
|⃗⃗⃗⃗
𝑎+𝑏 9
SECTION B
sin 𝛼 − cos 𝛼 sin 𝛼 cos 𝛼 1 0
5. 𝐴′ 𝐴 = ( )( )= ( )=𝐼
𝜋
cos 𝛼 sin 𝛼 −𝑐𝑜𝑠𝛼 sin 𝛼 0 1
6. 6
7. a=3,b=2
8. Decreasing xε(-2,3)
9. 𝑒 𝑥 𝑡𝑎𝑛𝑥
𝑥2 𝑦2
10. The equation of family of hyperbolas: 𝑎2 − =1
𝑏2
Differentiate twice to eliminate a and b
The required D.E is xy y`` + x (y`)2 – y y` = 0
11. Area of Parallelogram =a x b
𝒊 𝒋 𝒌
=|𝟏 −𝟏 𝟑| = 15√2 sq. units
𝟐 −𝟕 𝟏
12. A={(1,6), (6,1), (2,5), (5,2), (3,4), (4,3)}
B={(1,2),(2,1),(2,2),(2,3),(3,2),(4,2),(2,4),(5,2),(2,5),(6,2),(2,6)}
A B = (5,2) (2,5)
2
P(A/B)= 11
SECTION C
13. Apply C1→C1-C2, C2→C2-C3
1 1 𝑐
(a-b)(b-c). |𝑎 + 𝑏 𝑏 + 𝑐 𝑐2 |
−1 −1 𝑎+𝑏
Apply R1→R1+R3
0 0 𝑐+𝑎+𝑏
(a-b)(b-c). |𝑎 + 𝑏 𝑏 + 𝑐 𝑐2 |
−1 −1 𝑎+𝑏
Expanding along R1 and getting
(a-b)(b-c)(c-a)(a+b+c)
14. Find the first derivative and cross multiplying : y1√𝑥 2 + 𝑎2 = 1
Find the second derivate and getting the answer
15. PUT y = x2
(𝑥 2 +1)(𝑥 2 +4) 𝑦 2 + 5𝑦+4 7𝑦+19
= =1+
(𝑥 2 +3)(𝑥 2 −5) 𝑦 2 − 2𝑦−15 (𝑦+3)(𝑦−5)
7𝑦+19 1 27
1+ (𝑦+3)(𝑦−5)
= 1 + 4(𝑥 2 +3) + 4(𝑥 2 −5)
(𝑥 2 +1)(𝑥 2 +4) 1 1 27 𝑥− √5
∫ (𝑥2 +3)(𝑥2 −5) = 𝑥 + 4√3
𝑡𝑎𝑛−1 3 + 8√5 𝑙𝑜𝑔 | 𝑥+√5 |+ c

OR
Put sin 𝜃= t
3𝑡−2 𝐴 𝐵
Writing (𝑡−2)2=𝑡−2+(𝑡−2)2
3 4
Writing ∫ 𝑡−2+∫ (𝑡−2)2
4
3log | sin 𝑥 − 2|- |𝑠𝑖𝑛𝑥−2| + c
16. Let x and y are length and breadth of the rectangle
2x+2r+πr = 10
2x+(2 + π)r = 10 [1]
1
Area = 2πx + πr 2
2
1
= 10 – (2 + π)r 2 + 2 πr 2 [1]
dA
= 10 − (4 + π)r
dr
10
For maximum area r = [1]
4+π
d2 A
<0 [1]
dr2

Dimension of the rectangle are


10 20
m,
4+π 4+π
m

17. F(x)=sin 𝑥 +cos 𝑥


𝑓 −1 (x)=cos 𝑥-sin 𝑥
𝑓 −1 (x)=0
Getting interval
𝜋 𝜋 5𝜋 5𝜋
(0, 4 ), (4 , 4 ),( 4 ,𝜋)
𝜋 5𝜋
Getting increasing in(0, 4 ), ( 4 ,𝜋)
𝜋 5𝜋
Getting decreasing in(4 , 4 )
OR
2 1
Finding point of intersection(𝑘 , 𝑘 ) 3 3

𝑑𝑦
Finding 𝑑𝑥
1 𝑑𝑦 𝑑𝑦 −𝑦
= &
2𝑦 𝑑𝑥 𝑑𝑥
=
𝑥
1
Multiply - 2𝑥 =-1
Writing 8𝑘 2=1
18. Surface area of the cone S = πrl + πr 2
=πr(l + r)
s
l= −r
πr
1
V = πr 2 h
3
1
V = 9 s(sr 2 − 2πr 4 )
2

1
Z = s(sr 2 − 2πr 4 ) [1]
9
dz 2
dr
= 9
s(sr − 4πr 3 )
d2 z 2
dr2
= 9 s(s − 12πr 2 ) [1]
dz s
dr
= 0, implies r = √4π
d2 z
<0 [1]
dr2
1
θ= sin−1 3 [1]
dy y+√x2 +y2 y y 2
19. dx = =x+√1 + (x)
x
Homo generous differential equation
Put y=vx
dy dv
= v + x dx
dx
dv
v+xdx = v + √1 + v 2
dv dx
√1+v2
= x
log(v+√1 + v 2 ) = logcx
y y2
+ √1 + x2 = cx
x
20. Writing d̂ = xî+yĵ+zk̂
Writing x+4y+2z=0
3x-2y+7z=0
2x-y+4z=15
Writing x=32
y =⅄
z = -14⅄
Writing ⅄ =
= + -
21. - = 4i+6j+8k
Writing * =-4i-6j-8k
Find | * | = 116
Find SD=
22.
) + P(B) P (D / B) + P(C) P(D / C)
= 0.0345
P(B / D) = using Baye’s theorem
23. P(E1) =
P(E2)=
P(E3)=
A = Ensured person meets accident
P(A/E1)= .01 P(A/E2)= .03
P(A/E3)= .15
P(E1/A)=

=
Moral – every driver should have insurance to protect family’s future and expenditures on
treatment after accident.
SECTION D
24. Expressing : AX = B

Getting

x = 1; y = 2; z = 3
25. Using the property of definite integrals:I = dx
Multiply and divide by Sec2x, put t = tan x
Change the limits

I=
26. Points of intersection (1,0)
Area =
Area =
3 4
𝑥+2 𝑦+ 𝑧+
2 3
27. Equation of the line = = 5
3 2
3
3 4 5
Coordinates of any point P on this line be (-2+3t, -2 +2t, -3 +3t )
9 5 8
Direction ratios of AP are 3t, 2t− 2 ,3t+ 3 .
Direction ratios of normal to the plane 4x+12y-3z+1=0 are (4, 12, -3)
Formulae: a1a2+b1b2+c1c2=0, t=2
5
Therefore point P(4,2,2)
17
AP = units
2

28. Let number of product A be x and of product be y. The data can be represented as under:
A(x) B(y) Available capacity
Machine I 3 hrs 2 hrs 12 hrs
Machine II 3 hrs 1 hrs 9 hrs

L.P.P. is maximize P = 5x + 6y
Subject to constraints
3x + 2y ≤ 12
3x + y ≤ 9
x ≥ 0, y ≥ 0
The vertices O(0, 0), A(3, 0), B(2, 3) and
From the feasible region.
The maximum value of P is 36 at x = 0 & y = 6 i.e. the no. of units of A = 0, B = 6
29. Consider 1,2  N
11
As 1 is odd so f (1)  1
2
2
And 2 is even so, f (2)   1
2
Thus the different elements 1 and 2 of the domain of the function f have same image 1, therefore, the
function f is not one-one.
2m  1  1
Clearly for every m  N, there exists 2m-1  N such that f (2m  1)   m Thus every m  co
2
domain N is the image of some elements (here 2m-1) of domain N under f ,Hence f is ontoSince f is
not one-one, therefore f is not bijective.
GROUP-2

SAMPLE QUESTION PAPER-1

MATHEMATICS

CLASS XII 2017-18

TIME : 3 HOURS Maximum Marks: 100

General Instructions:

(i) All questions are compulsory.


(ii) This question Paper contains 29 questions.
(iii) Question 1 - 4 in Section A are very short-answer type questions carrying 1 mark each.
(iv) Question 5 - 12 in section B are short answer type question carrying 2 marks each.
(v) Question 13 - 23 in section C are long answer type I question carrying 4 marks each .
(vi) Question 24 - 29 in section D are long answer type II question carrying 6 marks each .

SECTION - A

1. State the reason why the Relation R = { ( a , b ) : a ≤ 𝑏 3 } on the set R of real numbers is not

Reflexive.
0
2. Evaluate |cos 700 sin 200 |
𝑠𝑖𝑛70 𝑐𝑜𝑠200

𝑎 = 2𝑖̂ - 𝑗̂ + 2𝑘̂ and 𝑏⃗⃗ = -𝑖̂ + 𝑗̂ + 3𝑘̂


3. Find the unit vector in the direction of the sum of the vectors ⃗⃗⃗⃗

4. The binary operation * on R is denoted by a * b = 2a + b . Find ( 2 * 3 )

SECTION – B
1 1 2
5. Prove that tan−1 7 + tan−1 13 = tan−1 9

5 2 3 6
6. Find Matrix X and Y , if X + Y = | | and X - Y = | |
0 9 0 −1
2𝑥
7. Differentiate with respect to x , sin−1 (1+ 𝑥2 )

8. Find the rate of change of area of a circle with respect to its radius ‘ r ‘ when r = 6 cm.

9. Evaluate ∫ sin−1(cos 𝑥) 𝑑𝑥

10. Find the differential of the family of all straight lines.

11. Find the position vector of the mid – point of the vector joining the points P ( 2 , 3 , 4 ) and

Q ( 4 , 1 , -2 ).
3 7 9
12. If A and B are the two events such that P ( A ) = , P(B)= and P ( A ∪ 𝐵 ) = , then find
5 10 10

P(A∩𝐵).

SECTION – C

13. Using properties of determinants , show that

1  a 2  b2 2ab 2b
 1  a 2  b 2 
3
2ab 1  a 2  b2 2a
2b 2a 1  a 2  b2

sin 𝑥
+ cos 𝑥, 𝑥 > 0
𝑥
14. Show that the function f (𝑥) given by 𝑓(𝑥) = 2 ,𝑥 = 0
4(1−√1−𝑥)
{ ,𝑥 < 0
𝑥

is continuous at 𝑥 = 0.

OR
1−𝑐𝑜𝑠4𝑥
, 𝑥>0
𝑥2
If the function f defined by f(x) = 𝑎 , 𝑥=0
√𝑥
, 𝑥>0
{ √16+√𝑥 −4 }
is continuous at x = 0, find the value of a.
−1 𝑥 𝑑2𝑦 𝑑𝑦
15. If y = 𝑒 𝑚 cos , prove that (1-𝑥 2 ) -x = m2y.
𝑑𝑥 2 𝑑𝑥

16. Find the intervals in which the function

1 − 12𝑥 − 9𝑥 2 − 2𝑥 3 is increasing or decreasing.

OR
Find the equation of the normal line to the curve 𝑦(𝑥 − 2)(𝑥 − 3) − 𝑥 + 7 = 0 at the point where

it meets the 𝑥 −axis.

17. Show that height of the cylinder of maximum volume that can be inscribed in a

2R
sphere of radius R is . Also find the maximum volume.
3

x 4 dx
18. Evaluate   x  1  x2  1

19. Find the general solution of the differential equation: (1 + 𝑦 2 )𝑑𝑥 = (tan−1 𝑦 − 𝑥)𝑑𝑦.

OR
𝑥 𝑥
Solve the differential equation : 2𝑦. 𝑒 𝑦 𝑑𝑥 + (𝑦 − 2𝑥𝑒 𝑦 ) 𝑑𝑦 = 0 .
20. Express the vector 𝑎⃗ = 5𝑖̂ − 2𝑗̂ + 5𝑘̂ as the sum of two vectors such that one is parallel to the

vector 𝑏⃗⃗ = 3𝑖̂ + 𝑘̂ and other is perpendicular to 𝑏⃗⃗ .

21. Find the shortest distance between the lines whose vector equations are

𝑟⃗ = (1 − 2𝑡)𝑖̂+(1 − 𝑡)𝑗̂+(𝑡)𝑘̂ and


𝑟⃗ = (2 + 3𝑠)𝑖̂ +(1 − 5𝑠)𝑗̂+(2𝑠 − 1)𝑘̂

22. A speaks truth in 70% of the cases and B speaks truth in 80 % of the cases .In what

percentage of the cases :-

(i) They contradict each other in stating the same fact?

(ii) They agree each other in stating the same fact?

Truth is very closely related with our national movement. How?

23. By examining the chest X-ray , the probability that TB is detected when a person actually

suffering is 0.99 . The probability of incorrect diagnosis is 0.001. In a certain city one in

thousand persons suffer from TB . A person selected at random and is diagnosed to have TB.

What is the chance that he actually has TB.

SECTION – D

24. Let f : N → R be a function defined as f (x ) = 4x2 + 12x + 15 . Show that f : N → 𝑆 , where S is


the range of f , is invertible. Also find the inverse of f .

OR

A binary operation * is defined on the set X = R – { - 1 } by x * y = x + y + xy , ∀ x , y ∈ X.

Check whether * is commutative and Associative. Find its identity element and also find the
inverse of each element of X.

25. Two school A and B decided to award prizes to their students for three values

honesty(x),punctuality (y) and obedience(z).School A decided to award a total of Rs 11000

for the three values of 5,4 and 3 students respectively while school B decided to award

Rs 10700 for the three values of 4, 3 and 5 students respectively. If all the three prizes

together amount to Rs. 2700, then:

(i) Represent the above situation by a matrix equation and form linear equations using matrix
multiplication.
(ii) Is it possible to solve the system of equation so obtained using matrix? If yes, find the
award money for each value

OR
Using the properties of determinants , prove that:

𝑥 𝑥2 1 + 𝑝𝑥 3
|𝑦 𝑦2 1 + 𝑝𝑦 3 | = (1 + 𝑝𝑥𝑦𝑧)(𝑥 − 𝑦)(𝑦 − 𝑧)(𝑧 − 𝑥)
𝑧 𝑧2 1 + 𝑝𝑧 3

26. Using integration, find the area of the triangle whose vertices are 𝐴(1,0), 𝐵(2,2) 𝑎𝑛𝑑 𝐶(3,1)

 /2 x
27. . Evaluate 0 sin x  cos x
dx

OR
2
Evaluate the following integral as a limit of a sum   3x  1 dx.
2

28. Find the equation of the plane passing through the point (-1, 3, 2) and perpendicular to each

of the planes 𝑥 + 2𝑦 + 3𝑧 = 5 and 3𝑥 + 3𝑦 + 𝑧 = 0. Also find the angle between this plane

and x-axis.

29. A dealer wishes to purchase number of fans and sewing machines . He has only Rs. 5760 to
invest and has a space for at most 20 items . A fan cost him Rs. 360 and sewing machine Rs. 240 .

His expectation is that .He can sell a fan at profit of Rs. 22and sewing machine at a profit of

Rs.18 . Assuming that he can sell all the items that he can buy , how should invest his money in

order to maximize the profit ? Formulate this as a linear programming problem and solve it

graphically .

MARKING SCHEME

1. For Reflexivity ( a , a ) ∈ R
1 1
Clearly ½ is a real no. , but is not less than or equal to ( )3
2 2
1 1
(2 , 2 ) not belongs to R
Hence , R is not Reflexive. 1 mark
2. Cos 700.cos200 - sin 700 sin 200
= cos (70 + 20 ) = 0 1 mark
3. For getting unit vector 1mark
4. a * b = 2a + b
2*3=7 1 mark
1 1
5. tan−1 7 + tan−1 13
Applying formula 1 mark
2
= tan−1 91 mark
4 4
6. Getting value of X = [ ] 1 mark
0 4
1 −2
Y =[ ] 1 mark
0 5
7. Taking x = tan𝜃 ,
Getting y = 2𝜃 1 mark
𝑑𝑦
For getting =2 / ( 1+x2) 1 mark
𝑑𝑥
8. For getting dA/dr = 2𝜋r 1 mark
(dA/dr ) at r = 6 = 12 𝜋 cm2/cm 1 mark
9. ∫ sin−1 cos x dx
𝜋
Putting cos x = sin ( − x ) 1mark
2

𝜋
To get the result x – x2 / 2 + c 1 mark
2

10. For equation y = mx + c 1 mark


𝑑𝑦
𝑑𝑥
=m & for second derivative = 0 1 mark
11. For writing 𝑎⃗ =2i +3j+4k and 𝑏 ⃗⃗ = 4𝑖̂ + 𝑗 − 2𝑘̂ 1 mark
For finding P.V.ofmid point= 3i+2j+k 1 mark
12. For writing P(AUB)= P(A)+p(B)-P(A∩B) 1 mark
2
For getting correct answer P(A∩B) = 1 mark
5

1  a 2  b2 2ab 2b
2ab 1 a  b
2 2
2a
13.
2b 2a 1  a 2  b2
R1  R1  bR2 , R2  R2  aR3

………………………………(1+1/2)

1  a 2  b2 0 b 1  a 2  b 2 
0 1  a 2  b2 a 1  a 2  b 2 
2b 2a 1  a 2  b2

1 0 b
1  a 2
b 
2 2
0 1 a ………………………………(1+1/2)
2b 2a 1  a 2  b 2

1  a  b  1  a
2 2 2 2
 b2  2a 2  2b2 
………………………………..(1)
 1  a  b 
2 2 3

14. For finding: LHL=2 (1 mark)

For finding: RHL=2 (1 mark)

For finding:f (0) =2 (1 mark)

LHL=RHL= f(0) , therefor function is continuous at x=0 (1 mark)

OR
To find LHL= lim− 𝑓(𝑥) = 8
𝑥→0
To find RHL= lim+ 𝑓(𝑥) = 8
𝑥→0
Value = a =8
𝑑𝑦 𝑚𝑦
15. =-
𝑑𝑥 √1−𝑥 2

𝑑𝑦 2𝑥
√1−𝑥 2 ∗(−𝑚) −(−𝑚𝑦)
𝑑2𝑦 𝑑𝑥 2√1−𝑥2
= 2
𝑑𝑥 2 √1−𝑥 2

𝑑2𝑦 𝑑𝑦
(1-𝑥 2 ) -x = m2y.
𝑑𝑥 2 𝑑𝑥

16.

 𝑓 ′(𝑥) = −6(𝑥 + 1)(𝑥 + 2)  1


 Finding critical points 𝑥 = −1, −2  1
 Finding intervals for increasing and decreasing functions  1
 Writing answer: Increasing in (−2, −1), Decreasing in (−∞, −2) ∪ (−1, ∞)  1

OR

 Finding the point (7,0)


 1
𝑑𝑦
 Finding 𝑑𝑥
 1
𝑑𝑦 1
 Finding 𝑑𝑥 at (7,0) = 20 
1
2

 Slope of normal −20 


1
2

 Finding equation of normal:20𝑥 + 𝑦 = 140


 1

17.

Let r and h be the radius and height of the cone respectively inscribed in a sphere of radius R.

1
mark

Let V be the volume of the cone.


Then,

Height of the cone is given by,

h = R + AB 1mark
1
mark

2
marks

∴ By second derivative test, the volume of the cone is the maximum when

1
mark
1
mark

𝑋4 𝑥 4 − 1+1
18. ∫ (𝑥−1)(𝑥2 +1 ) 𝑑𝑥 = ∫ (𝑥−1)(𝑥2 +1 ) 𝑑𝑥

𝑥4− 1 1
= ∫ (𝑥−1)(𝑥2 +1 ) 𝑑𝑥 + ∫ (𝑥−1)(𝑥2 +1 ) 𝑑𝑥

1
= ∫(𝑥 + 1 )𝑑𝑥 + ∫ (𝑥−1)(𝑥2 +1 ) 𝑑𝑥

𝑥2 1 𝑑𝑥 1 𝑥+1
= + 𝑥 + ∫ − ∫ 𝑑𝑥
2 2 𝑥−1 2 (𝑥 2 +1 )

𝑥2 1 1 1
= + 𝑥 + 𝑙𝑜𝑔|𝑥 − 1| − log|(𝑥 2 + 1 )| - tan−1 𝑥 + 𝑐
2 2 4 2
19

𝑑𝑥
Writing the equation in the form: 𝑑𝑦 + 𝑃(𝑦). 𝑥 = 𝑄(𝑦) 𝑖. 𝑒.
𝑑𝑥 𝑥
+ 1+𝑦2 =  1
𝑑𝑦
tan−1 𝑦
1+𝑦 2
……… (1)


1
Writing 𝑃(𝑦) = 1+𝑦2 , 𝑄(𝑦) =
tan−1 𝑦  1/2
1+𝑦 2

−1 𝑦
 Integrating factor = 𝑒 tan  1

 Multiplying integrating factor in (1) and changing the equation in form:  1/2

−1 𝑦 tan−1 𝑦 −1 𝑦
𝑒 tan . 𝑥 = ∫ 𝑒 tan
𝑑𝑦 .
1 + 𝑦2
−1
 Finding the general solution: 𝑥 = tan−1 𝑦 − 1 + 𝑐 𝑒 −tan 𝑦  1

OR
𝑥
𝑑𝑥 2𝑥𝑒 𝑦 −𝑦
 Writing D.E. in the form: 𝑑𝑦
= 𝑥 ………. (1)
2𝑦.𝑒 𝑦
 1
 Putting 𝑥 = 𝑣𝑦 𝑎𝑛𝑑
𝑑𝑥
=𝑣+
𝑑𝑣
𝑦 𝑑𝑦 ………….. (2)  1
𝑑𝑦
and converting D.E. in the form variable separable as
 1
𝑑𝑦
2𝑒 𝑣 𝑑𝑣 = −
𝑦

 Integrating both sides and getting general solution


𝑥
2. 𝑒 𝑦 = − log|𝑦| + 𝐶  1

⃗⃗⃗⃗1 +𝑏
20. . Let 𝑎⃗=𝑏 ⃗⃗⃗⃗⃗2

⃗⃗⃗⃗
𝑏1 =𝛽(3𝑖̂+𝑘̂)

⃗⃗⃗⃗⃗
𝑏2 =𝑥̂i+𝑦̂j+𝑧̂ k Marks 1

𝐹raming equation 3x+z=0 Marks 1

Ans𝛽=2 x=-1,y=-2,z=3 Marks 2

21. ⃗⃗⃗⃗⃗ 𝑎1 = 𝑖̂ − 𝑘̂
𝑎2 − ⃗⃗⃗⃗⃗ (1 mark)

⃗⃗⃗⃗⃗⃗⃗⃗⃗⃗⃗⃗⃗⃗⃗⃗
𝑏1 𝑋𝑏⃗⃗⃗⃗⃗2 = 3𝑖̂ − 𝑗̂ − 7𝑘̂ (1 mark)

⃗⃗⃗⃗1 𝑋𝑏
|𝑏 ⃗⃗⃗⃗⃗2 | = √59 (1 mark)
10
Shortest distance between the line:√59 (1 mark)

22. (i) They contradict each other in stating the same fact = 38% ( 1.5 mark)
(ii) They agree each other in stating the same fact= 0.62% (1.5 marks)

For value (1 mark)

23. Consider A1 : suffers from TB, P  A1   0.001 ------------------------------------(1)

A2 : person donot suffer fromTB, P  A2   0.999        (1)

C: Doctor diagnoses correctly

Then P(C / A1 )  0.99 and P(C / A2 )  0.001        (1)

P( A1 ).P(C / A1 ) 110
By Baye ’s theorem P( A1 / C )          (1)
P( A1 ).P(C / A1 )  P( A2 ).P(C / A2 ) 221

24. For proving one-one (2 Marks)

For proving onto (2 Marks)

For finding inverse (2 Marks)

OR

5 4 3 𝑥 11000
𝑦
25. [4 3 5] [ ] = [10700]
1 1 1 𝑧 2700
5x+4y+3z=11000

4x+3y+5z=10700

x +y + z=2700 (1marks)

for getting |A|= -3 (1 marks)

−2 −1 11
−1
getting A-1= 3 [ 1 2 −13] (2 marks)
1 −1 −1
getting x=1000,y=900 and z=800 (2 marks)

OR

For Commutative (2 marks)

For Associative (2 marks)

For finding Identity (1 mark)

For finding Inverse (1 mark)

25.
𝑥 𝑥2 1 𝑥 𝑥2 𝑝𝑥 3
|𝑦 𝑦2 1|+|𝑦 𝑦2 𝑝𝑦 3 | Mark 2
𝑧 𝑧2 1 𝑧 𝑧2 𝑝𝑧 3

Similarly 2 marks for using each properties

Getting proof (Marks 2)

26. Drawing correct rough sketch and obtaining point of intersection of given

lines ( 2 marks)
2 3 3 𝑥−1
𝑎𝑟𝑒𝑎 𝑜𝑓 ∆𝐴𝐵𝐶 = ∫1 2(𝑥 − 1)𝑑𝑥 + ∫2 (4 − 𝑥)𝑑𝑥 − ∫1 𝑑𝑥 (2 marks)
2

3
𝑎𝑟𝑒𝑎 𝑜𝑓 ∆𝐴𝐵𝐶 = 2 (2 marks)


2
x
27. I   dx
0
sin x  cos x


 2
1
2  sin x  cos x
2I  dx ……………………………….(1)
0


 2
1
2
2I  dx …………………………(1)
2 1 1
0 sin x  cos x
2 2

 2
  
2 
2I  cos ec  x   dx ………………………….(2)
2 0  4


I
2 2
log  2 1  …………………………………………(2)

OR

By definition of limit sum we have

  3x  1dx  lim h  f 1  f 1  h   f 1  2h       f 1   n  1 h  


2
h 0
1

where nh  1              (1)

3 1  1  1     ntimes   1  1      ntimes  
 
= lim h 
h 0 

 3 h 2  22 h 2        n  1 h 2
2

      (1)


  3.2  h  2h        n  1 h  

 n  n  1 2n  1 n(n  1) 
= lim h  2n  3h   6h    6            (2)
2
h0
 6 2 
28. Equation of plane passing through (-1, 3,2) is A(x+1)+B(y-3) +C(z-2)=0 (1 mark)

Finding (A, B,C) = (-7, 8 -3) (2 marks)

Getting equation of plane: 7x-8y+3z+25=0 (2 marks)


7
Finding angle with x axis 𝑐𝑜𝑠 −1 √122 (1 mark)

29. Let x be the no. of fans and y be the no. of sewing machines

If p be the total profit ,

P= 22x+18y

360x+240y≤5760. For correct graph, ---------- (1)

i.e., 3x+2y≤48.

x+y≤20.

x≥0,y≥0 -------------------------------- (2)

E(0,24)

(0,20)D

C(8,12)

O (16,0)A (20,0)

po int s valueof p
(0, 0) 0
(16, 0) 352
(8,12) 392
(0, 20) 360

the dealer gets a maximum profit of Rs. 392 when he purchase and sells 8 fans and 12 sewing machines.

Investment in fans =360.×8=2880.


Investment in sewing machines =240×12= 2880.-------------------(3)

SAMPLE QUESTION PAPER-2

MATHEMATICS

CLASS XII 2017-18

Max.Marks:100

Time Allowed: 3 hours

General Instructions:
1. All questions are compulsory.

2. This question paper consists of 29 questions divided into Four sections A, B, C andD. Section A comprises of
4 questions of one mark each, section B comprises of 8 questions of 02 marks each and section C comprises of
11 questions of 04 marks each and section D consists of 06 questions of 06 marks each.

3. All questions in Section A are to be answered in one word, one sentence or as per the exact requirement of
the question.

4. There is no overall choice. However, internal choice has been provided in 03 Questions of four marks each
and 02 questions of six marks each. You have to attempt only one of the alternatives in all such questions.

5. Use of calculators is not permitted. You may ask for logarithmic tables, if required.

________________________________________________________________________

SECTION - A (1 mark questions)

1. Let * be a binary operation on Q given by a*b=a+ab where a,b  Q .Is * commutative?


1  4 
2. Evaluate Sin  cos 1   
2  5 
3. Given that A, B are two symmetric matrices such that AB =BA .Is AB symmetric?
 x  y  z   9
   
4. Find the values of x, y and z if  x  z    5 
 y  z   7
   
SECTION - B (2 mark questions)
Sin30 0 Cos30 0
5. Evaluate
 sin 60 0 Cos60 0
6. Find a point on the curve y=x2-4x+5 where the tangent to the curve is parallel to the x axis
x  cos 6 x
7. Evaluate  3x 2
 Sin6 x
dx
      
8. Find the projection of the vector a  i  3 k on the vector b  3 i  j  4 k
     
9. If a is a unit vector and ( x  a )( x  a )  15 find x

2x  1 y2 z3
10. The Cartesian equation of a line AB is   .Find the direction cosines of a line
3 2 3
parallel to AB

1
11. Let f: R  R be defined by f(x) = 3x+2.Show that f is invertible. Also find f :R  R
 1 
12. Write in the simplest form : tan
1
  where x  1
 
 x2  1 
SECTION - C (4 marks questions)

x 6 1
13. Show that x = 2 is a root of the equation 2  3 x x  3  0 and solve it completely.
 3 2x x2
 1  Cos2 x x0

 x2
14. Discuss the continuity of the function at x = 0: f ( x)  


5 x0

 
15. Verify Rolle’s theorem for the function f(x)= Sin2x in 0, 
 2
dy 1  y2
16. If 1  x 2  1  y 2  a( x  y ) prove that 
dx 1  x2

OR

If y=x2 +4 and x changes from 2 to 2.1 find the approximate change in y

2
17. Evaluate  e x dx as a limit of sum
0
OR

5
Evaluate   x  1x 2
4 dx
dy
18. Solve the differential equation Cos x
2
 y  tan x
dx
19. Find the differential equation for the family of circles which passes through the origin and have their
centre on the x axis
   
20. Express the vector a  5 i  2 j  5 k as the sum of two vectors such that one is parallel to the vector
   
b  3 i  k and the other is perpendicular to b
Or

       
If a  b  c  0 and a  3 ; b  5 c  7 then show that the angle between a and b is 600


21. Find the vector equation of the plane passing through the intersection of the planes r .
      
( 2 i  7 j  4 k )  3 and r . ( 3 i  5 j  4 k )  11  0 and passing through the point (-2,1,3)
22. A coin is tossed three times and all the possible outcomes are assumed to be equally likely. Let E and F
be two events given by E: “both tail and head have occurred” F: “at most one tail has occurred”. Show
that E and F are independent.

23. The sum of three numbers is 6.Twice the third number when added to the first number gives 7.On
adding the sum of the second and third numbers to thrice of the first number we get 12.Find the
numbers using inverse of a matrix.
Or

 1 2 3 

Using elementary transformations find the inverse of the matrix 2 5 7 

  2  4  5

SECTION - D (6 mark questions)

24. Let R be a relation on N  N defined by (a, b)R (c, d)  ad=bc, for all (a,b) and (c,d)  N . Show that R is
an equivalence relation .
OR

Let * be the binary operation defined on QxQ by (a,b)*(c,d) = (ac,b+ad) where Q is the set of
rational numbers. Determine whether * is commutative and associative . Find the indentiy
element for * and the inverttible elements of QxQ

25. An open box with a square base is to be made out of a given quantity of metal sheet of area c2.Show
c3
that the maximum volume of the box is
6 3
1
26. Evaluate  Sinx(2  Cosx )dx
Or


2
Evaluate  log Sinxdx
0

27. Find the area of region included between the parabolas y2=4ax and x2=4ay where a>0
28. Find the foot of the perpendicular drawn from the point A(1,0,3) to the line joining the points B(4,7,1)
C(3,5,3)
29. Mona wants to invest at most Rs 12000 in Savings Certificates (SC) and National Saving Bond (NSB). She
has to invest at least Rs 2000 in SC and at least Rs 4000 in NSB. If the rate of interest on Sc is 8% and the
rate of interest on NSB is 10% per annum, how much money should she invest to earn maximum yearly
income?

ANSWER KEY WITH MARKING SCHEME

Q.No Value points Marks Total


Marks

1 Not Commutative (as a*b=a+ab  b+ba=b*a) 1M 1M

2 1 1 4 4  1M 1M
(put cos =   cos  = Given expression =Sin =
10 5 5 2
4
1
1  Cos 5= 1
=
2 2 10

3 t t
AB is Symmetric. (AB) =B A =BA=AB
t 1M 1M

4 x=2 ; y= 4 ; z= 3 1M 1M

( by equating corresponding entries we get x+y+z = 9 ; x+z =5 ; y+z


= 7)

5 0 00
Sin30 Cos 60 +Cos30 Sin60
0 1M 2M

=Sin 900=1 IM
6 dy 1M 2M
(2,1) is the point .  2 x  4 =0  x  2 putting x =2 in
dx
y=x2-4x+5 we get y = 1

7 1 1 1M 2M
6 t
Put 3 x  Sin6 x = u  Given integral =
2
dt =

1
log 3 x 2  Sin6 x  c
6

8     1M 2M
projection of the vector a  i  3 k on the vector b 
 
   a .b
3 i  j 4 k =

b

3  0  12 15
= =
9  1  16 26
1M

9    1M 2M
=15  x
2 2 2
x -a


=15+1  x = 4 1M

10 1 1M 2M
x
2  y  2  z  3 dr’s of AB are 3 ,4,6
3 4 6

3 4 6
Hence dc’s of AB are , ,
55 55 55
1M

11 f ( x1 )  f ( x2 )  3 x1  2  3 x 2  2  x 1 =x 2 Thus f is one 1M
to one

y2 y2 1M
y=3x+2  x= Given any y  R there exists =x  R
3 3
s.t. f(x)=y showing that the function is onto.
2
1 x2
f (x)= is the inverse of f
3

1
12 x=Cosec    =Cosec x
 1   1  1M
tan
1
  = tan 1  
   
 x2  1   Co sec 2
  1 

  1M 2
 = tan 1  
 = tan Tan = 
1 1
= tan
1
 1
   Cot 
 Cot  
2

1
=Cosec x

13 x  2 3x  6  x  2 1M
5  5x  5  0 by performing R1  R1  R2 and
3 2x x2
R2  R2  R3

1 3 1 1M
5(x-2) 1  x 1  0
 3 2x x2

0 3 1 1M
5(x-2) 0 x  1  0 C1  C1  C 3 4
x  1 2x x2

0 3 1 ½M
5(x-2)(x-1) 0  x 1  0
1 2x x2

= -5(x-2)(x-1)(x+3) = 0  x  2,1,3 ½M

14 2 Sin 2 x
Here f(0) = 5 and Lt f(x) = Lt =2
x 0 x 0 x2

Sinx Sinx
2 Lt Lt =2  1  1 =2
x 0 x x 0 x

As Lt f(x)  = f(0) ,f is not continuous


x 0

15   ½M
Consider the function f(x) = Sin2x in 0,  . Here f is
 2
 
continuous in 0,  as f(x)= Sin x is continuous
 2

'   1M
f (x) = 2 Cos2x exists in ( 0, ) thus f is differentiable on ( 0, )
2 2

  1M
f(0)=Sin (0) = 0 and f   =Sin  = 0
 2
Conditions for Rolle’sthm are satisfied. Hence there should be at ½M
 
least one c   0,  such that f (c ) = 0
'

 2
4
   1M
Let 2Cos2c = 0  c =   0, 
4  2

16 Let x=Sin A and y=Sin B ½M

Cos A + Cos B =a(Sin A –Sin B) ½M

A B A B A B A B 1M
2Cos Cos =a  2Cos Sin
2 2 2 2 4

A B 1M
Cot =a  Sin-1x-Sin-1y=2Cot-1a
2

1 1 ½M
Differentiating , - y ' =0
1  x2 1  y2

½M
dy 1  y2

dx 1  x2

OR

x= 2 x  0.1 ½M

y  f ' ( x )x =2x x =2  2 0.1 =0.4 1½M

When x=2 y = 22+4=8 ½M 4

y+  y =8+0.4=8.4 1M

y changes from 8 to 8.4 ½M

17. ba 2 ½M
a=0 b=2 h=  nh=2 f(x)=ex
n n
2 1M
 e dx =
x

Lt h  f (0)  f (0  h)  f (0  2h)  ......  f (0  (n  1)h)


h 0

h 0

= Lt h 1  e  e  e  .....  e
h 2h 3h  n 1 h
 ½M

Lt h.
 
n
1. e h  1  1M
4
h 0 eh  1

= Lt
h

e 2  1 =e 2 -1  1M
h 0 e 1
h
Or

5 A Bx  C 1M
 2
 =

 x  1 x  4 x  1 x  4
2

To get A =1 B = -1 and C = 1 1M

5 1  x 1 ½M
  x  1 x 2  4 dx =  dx  
  x1 x2  4
dx

x 1 1M
=log x  1 -  x2  4
dx   x 2  4dx 4

1 1 x ½M
=log x  1 - log x 2  4  tan 1  c
2 2 2

18 dy 1M
Dividing Cos x
2
 y  tan x by Cos2x to get
dx
dy
 ySec 2 x  tan xSec 2 x
dx

I.F. =etanx ½M
4

Solution is y(IF)= tan xSec 2 x etanxdx+ k 1M

 Put tan x =u  y etanx=  ue u du  k ½M

 y etanx=e u ( u  1)  k ½M

 y =tanx-1+ke  tan x ½M

19 1M

Let a be the radius Then centre is


(a,0)

= n to the Circle is (x-a) +y =a  x  y  2ax  0 ….(*) 1M


2 2 2 2 2

Differentiating ,2x+2yy’-2a=0  a=x+yy =a 1M


'
4

Putting this value of a in (*) we get 2xyy -y  x =0 1M


' 2 2

20   ½M
Any vector parallel to b is of the form k b for some scalar k
     1M
Let a = k b + c where c is perpendicular to b

    ½M
Then c =( a - k b )  b

1M
      
 ( a - k b ). b  ( a . b )-k( b . b )=0  k=2
4
    ½M
k b =2 b = 6 i  2 k

      ½M
c =( a - 2 b )=(  i  2 j  k )

OR

      ½M
a b c  0  a b   c
   ½M
 ( a  b ) 2  ( c ) 2

      ½M
 ( a  b ).( a  b )  (  c ).(  c )
     1M
2 2 2
a +b +2 a . b = c

  15 ½M
a .b =
2
  1M
a .b 15 1
Cos  = =    60
0
=
 
2 3 5 2
a b 4

21     1½M
r . { ( 2  3 ) i  ( 7  5 ) j  (4  4 ) k }  3  11  0 …(*)

4
   1½M
(*) Passes through the point with position vector ( 2 i  j  3 k )
1
 
6
 1M
Putting the value of  in (*) we get r .
  
(15 i  47 j  28 k )  7  0

22. E: “both tail and head have occurred” ½M+½M


={HTT,THT,TTH,HHT,HTH,THH}

F: “at most one tail has occurred”= {HHH,HHT,THH,HTH}


E  F ={HHT,HTH,THH} ½M

6 4 3 1½M
P(E) = P(F)= P( E  F )=
8 8 8 4

3 6 4 1M
As P( E  F )=P(E) P(F)    is true ,we conclude that E
8 8 8
and F are independent.

23 Let the first, second and third numbers be x, y and z respectively. 1M


Then x + y + z=6 : x + 2z =7 ; 3x + y + z=12

 1 1 1  x 6 ½M
     
 AX=B where A=  1 0 2  X=  y  B=  7 
 3 1 1 z  12 
     

A  4  0  A is invertible ½M

 2 0 2  1M
  4
adjA=  5  2  1 
 1 2  1 

 2 0 2   6   3 1M
1 1     
X=A B=  5  2  1  7  =  1 
A  1
 2  1   12   2 

The required numbers are 3 ,1 and 2 ½M

OR

 1 2 3  ½M

Let A= 2 5 7 

  2  4  5

 1 2 3  1 0 0 1M

A=IA  2 5 7  = 0 1 0 A

  2  4  5 0 0 1 6

1 0 0
 
By performing elementary transformations ,to get 0 1 0 =
 
0 0 1
 3  2  1
  4 1  1 A 3½M
 
 2 0 1 
 3  2  1 1M
1 
 A =   4 1  1
 2 0 1 

24

(a,b) R (a,b)  ab=ba Reflexive 1.5M

(a,b) R (c,d)  ad=bc  cb=da  (c,d) R (a,b) symmetric 1.5M 6

(a,b) R (c,d) and (c,d) R (e,f)  ad=bc and cf=de  ad.cf=bc.de 2½ M


 af=be  (a,b) R (e,f) Transitive

Thus R is an equivalence relation ½M

25 Let x be the side of the square base and y be the height. 1½M

c2  x2
V=x2y ,Surface area S= x2+4xy=c2  y  4 x

 d 2V  3

1 dV 1 2 1½M
V= (c x  x )  c  3x2 
2 3
x
4 dx 4 dx 2 2

c d V  3c 1M
2
dV
0 x  2 
 0
dx 3 dx  x  c 2 3
3

c c 1M
V is Maximum when x  . Then y 
3 2 3 6

c3 1M
V Max =
6 3

26 Sinx ½M
 Sin x( 2  Cosx )
2
dx by multiplying the Nr and Dr by Sinx

Sinx 1 1M
=  (1  Cos 2 x)(2  Cosx)dx =  (1  t 2 )( 2  t )dx by letting
Cosx=t

1 ½M
=  (1  t )(1  t )( 2  t )dx
1 A B C 1M
=  
(1  t )(1  t )( 2  t ) 1  t 1  t 2  t

1 1 1 1½M
To get A= B= C=
6 2 3
1 1M 6
 (1  t )(1  t )( 2  t )dx =
1 1 1
log 1  t  log 1  t  log 2  t  k
6 2 3

1
 sin x(2  Cosx )dx =
1 1 1
 log 1  Cosx  log 1  Cosx  log 2  Cosx  c
6 2 3
½M

OR

   1M
2 2   2
I=  log Sinxdx =  log Sin  x dx =  log Cosxdx
0 0 2  0

  1M
2 2 2 SinxCosx
2I=  log SinxCosxdx =  log dx
0 0 2

 2M
2 2 SinxCosx
=  log dx =
0 2 6

2  
 log Sin2 xdx  log 2  I 1  log 2 ………(*)
0 2 2

 1M
1  1 2
I1 =  log S int dt =  2  log Sinxdx =I
2 0 2 0

  1M
(*)  2I=I- log 2  I= log 2
2 2

27 2M
Solving the two equations given we get x=0 and x=4a 1M

4a 4a x 2 2M
Required area =  4ax dx   dx
0 0 4a

1M 6
32a 2 16a 2 16a 2
=   sq.units
3 3 3

28 Let P be the foot of the perpendicular from A on BC. If P divides BC ½M


 3k  4 5k  7 3k  1 
in the ratio k:1 then P is  , , 
 k 1 k 1 k 1 
1M

Dr’s of BC are 1,2,-2 ½M

3k  4 5k  7 3k  1 1M
Dr’s of AP are  1,  0, 3
k 1 k 1 k 1

7 2M
Since AP  BC ,dot product =0  k 
4

 5 7 17  1M
So the foot of the perpendicular is  , ,  6
3 3 3 

29 Suppose that she invests Rs x in SC and Rs y in NSB ½M

8 x 10 y 1½M
Then LPP is to maximize Z=  subjecting to constraints x
100 100
 2000 y  4000 x+y  12000

2½M

(To draw the


graph, to identify the feasible region and to get coordinates)

8 x 10 y 1M
To put the coordinates of the vertices in Z=  to get
100 100
6
values Rs 560, Rs 1040 Rs 1160

Rs 2000 should be invested in in savings certificates and Rs 10000 ½M


in National Savings Bonds to get a maximum yearly income of Rs
1160
Model Question Paper: -3
CLASS XII
Class :XII Max Marks: 100

Subject: Maths Max Time:03 hours

General Instructions
 All questions are compulsory
 The question paper consists of 29 questions divided into three sections A, B ,C&D
 Section A contains 4questions of one mark each, Section B is of 8 questions of two
marks each and Section C comprises of 11 questions of four marks each,section D
comprises 6 questions of six marks each.
 There is no overall choice. However, internal choice has been provided in 3 questions of
four marksand two questions of six marks .You have to attempt only one of the
alternatives in all such questions.
 Use of calculators is not permitted

SECTION – A

1. Find f -1 when f={(1,a) (2,c) (3,b) }


3𝜋
2. Find the principal value of 𝑠𝑖𝑛−1 [𝑠𝑖𝑛 ]
5
3. If A and B are symmetric matrices of the same order, then show that AB – BA is a skew
symmetric matrix.
4. Find a unit vector parallel to the sum of vectors a  2iˆ  4 ˆj  5kˆ and b  iˆ  2 ˆj  3kˆ

SECTION – B
5. Define a Symmetric and Skew Symmetric Matrices with an example
1 1
6. Evaluate: 𝑡𝑎𝑛−1 + 𝑡𝑎𝑛−1
5 7

𝑑2 𝑦
7. If x=a cost and y=b sint then find 𝑑𝑥 2.
8. The volume of a cube is increasing at a rate of 9cubic cms per sec. How fast is the surface area
increasing when thelengh of an edge is 10 cm?
𝝅
𝒔𝒊𝒏𝟒 𝒙
9 Eevaluate . ∫𝟎𝟐 𝒔𝒊𝒏𝟒 𝒙 + 𝒄𝒐𝒔𝟒 𝒙dx

2 2
 dy   dy 
10. .Find the degree and order y  x   a 1   
 dx   dx 
11. A family has two children. What is the probability that both the children are boys given that at least
one of them is a boy?
12. Using vectors ,find the area of the triangle points A(1 , 1, 1), B(1,2,3) and C(2,3,1) as its vertices?

SECTION C
13. By using properties of determinants, show that
a b c
a2
b c 2  (a  b)(b  c)(c  a)(a  b  c)
2

bc ca ab

𝑑𝑦 √1+𝑠𝑖𝑛𝑥+ √1−𝑠𝑖𝑛𝑥
14. Find for y = 𝑡𝑎𝑛 −1 [ ] xε[0,π/4]
𝑑𝑥 √1+𝑠𝑖𝑛𝑥− √1−𝑠𝑖𝑛𝑥
15. If y = 𝑙𝑜𝑔{𝑥 + √𝑥 2 + 𝑎2 } prove that (x2 + a2) y2 + x y1 = 0
𝜋 𝑥
16. Evaluate: ∫ dx
0 𝑎 𝑐𝑜𝑠 𝑥+ 𝑏 2 𝑠𝑖𝑛2 𝑥
2 2
17.Using vector, show that the points A(2, 3, 4), B(-1, -2, 1) and C(5, 8, 7) are collinear.
18. In a family the husband tells a lie in 30% cases and the wife in 35% cases. Find the probability that both
state the same fact.

What value inherent above question?


19. An open top box is to be constructed by removing equal squares from each corners of a 3
cm by 8 cm rectangular shape of aluminium and folding up the sides.Find the volume of
the largest such box.
(OR)
Manufacturer can sell x items at a price of Rs (5-x/100)each.The cost price of x items is Rs
(x/5)+100.Find the number of items he should sell to earn maximum profit?
20. Water is running in to a conical vessel, 15 cm deep and 5 cm in radius, at the rate of 0.1
cm3/sec. When the water is 6 cm deep, find at what rate is the level of water increasing.
OR
Find the intervals in which the function  2 x 3  9 x 2  12 x  1 is strictly increasing or decreasing.
21 Solve the differential equation: (tan-1y – x)dy = (1 + y2) dx
22 Find the equation of the plane passing through the line of intersection of the planes 2x + 6y
+12 = 0 and 3x – y + 4z = 0 which are at a unit distance from the origin.
23 .A bag, A contains 8 white and 7 black balls while the other bag B contains 5 white and 4
black balls. One ball is randomly picked up from bag A and mixed up with the balls in the bag B.
Then a ball is randomly drawn from it. Find the probability the ball drawn is white.
(OR)
Find the mean and variance of the number of heads in a two tosses of a coin
SECTION - D

24. Consider f: R+ [-5, ∞) given by f (x) = 9 x2 + 6x-5 show that f is invertible . Also find f-1.
Or
𝑎 + 𝑏, 𝑖𝑓 𝑎 + 𝑏 < 6
Define a binary operation * on the set {0, 1,2 3, 4, 5} as 𝑎 ∗ 𝑏 = {
𝑎 + 𝑏 − 6, 𝑖𝑓 𝑎 + 𝑏 ≥ 6

Show that zero is the identity for this operation and each non zero element 𝑎 of the set is invertible with 6 − 𝑎
being the inverse of 𝑎.

25 Using Matrices, solve the following system of equations: 2x – 3y


+ 5z = 11 ; 3x +2 y – 4z = -5 ; x + y – 2z = -3

26. Find the area lying above x-axis and included between the circle x2+y2=8x and Parabola y2=4ax.

27. Find the distance of the pointA (-2, 3, -4) from the line (x+2)/3 = (2y+3)/3 =(3z+4)/5 measured
parallel to the plane 4 x +12 y -3 z +1=0 ?
28. A diet is to contain at least 80 units of vitamin A and 100 units of minerals. Two foods F1 and F2
are available. Food F1 cost Rs.4/- per unit and F2 costs Rs.6/- per unit. One unit of food F1 contains 3
units of vitamin A and 4 units of minerals. One unit of food F2 contains 6 units of vitamin A and 3
units of minerals. Formulate this as a linear programming problem. Find the minimum cost of the diet
that consists of mixture of these two foods and also meets the minimal nutritional requirements.
 /4
29 Evaluate:  log 1  tan xdx
0 or

1
 sin x(2  cos x) dx
MARKING SCHEME

SECTION A

Q.1 For correct answer f-1 = {(a,1),(c,2),(b,3)}


2𝜋
Q.2 For correct answer 5

Q.3 Proving correctly AB-BA is a skew symmetric matrix


3 6 −2
Q.4 7 𝑖 + 7 𝑗 + 𝑘
7

SECTION B

. Q.5 For correct definition and example of symmetric and skew symmetric matrix

Q.6 tan-16/17

𝑑𝑦 −𝑏
Q.7 (i) for correct = 𝑐𝑜𝑡𝑡
𝑑𝑥 𝑎

(ii) for correct d2y/dx2 =(-b cosec3t)/a 2

𝑑𝑣
Q8.(i) For writing 𝑑𝑡 =9 cm3/sec

𝑑𝑥 3
(ii). V = x3 and S = 6x2 , for calculating = 𝑥2
𝑑𝑡

𝑑𝑆 36
(iii) For =
𝑑𝑡 𝑥

𝑑𝑆
(iv) For = 3.6 cm2/sec
𝑑𝑡

Q 9.π/4

Q10 order 1 degree 4

Q.11 S= { (b,b),(g,b),(b,g)(g,g)}

E = both the children are boys, F= atleast one of the child is a boy
3 1
P(f) =4 , P(E∩ 𝐹) = 4

1
P(E/F)= 3

Q.12 AB = j +2k , AC= i+2j

ABx AC = -4i+2j-k
1 1
Area of triangle = 2 |ABx AC|= 2 √21

Q.13 LHS= for applying C1→ C1-C3 , C2→ C2-C3

𝑎−𝑐 𝑏−𝑐 𝑐
|𝑎2 − 𝑐 2 𝑏2 − 𝑐 2 𝑐2 |
𝑐−𝑎 𝑐−𝑏 𝑎+𝑏

C1→ C1+C2 ,

0 1 𝑐
(c-a)(b-c)|𝑏 − 𝑎 𝑏 + 𝑐 𝑐2 |
0 −1 𝑎 + 𝑏
Expanding along c1 = (a-b)(b-c)(c-a)(a+b+c)

𝑥 𝑥 𝑥 𝑥
Q14. (i). For putting 1 = sin22 + cos22 , sinx = 2sin2 cos2

𝜋 𝑥
(ii). For writing tan-1{tan(2 - 2)}

𝑑𝑦
(iii) for𝑑𝑥 = - ½

𝜋 𝑑𝑥
Q16. 2I=𝜋 ∫0 𝑎2 𝑐𝑜𝑠2 𝑥+𝑏 2 𝑠𝑖𝑛2 𝑥
𝜋/2 𝑠𝑒𝑐 2 𝑥
I=𝜋 ∫0 𝑎2 +𝑏2𝑡𝑎𝑛2 𝑥 𝑑𝑥
𝜋2
I= 2𝑎𝑏

Q.18 0.56

For correct value answer

Q19 Let x m be the length of a side of the removed square. Then l = 8-2x, b= 3-2x

and h = x.V(x) = 4x3 -22x2+24x

V’(x) = 4(x-3)(3x-2)

V’’(x) = 24x-44
Critical point x = 2/3

V’’(2/3) < 0
200
Max. volume = m3
27

Writing correct values

OR

LetS(x) be the selling price of x items and let C(x) be the cost price of x items.

𝑥2 𝑥
S(x) = 5x- , C(x) = 5 + 500
100

24 𝑥2
P(x) = 𝑥 - -500
5 100

24 𝑥 1
P’(x) = - 50 P’’(x) = - 50 Critical
5
point x = 240

1
P’’(240)= - 50< 0

Writing correct values

20. V = Volume of water in the cone


𝑟 5
tan = ℎ = 15
h = 3r
𝜋
V = 27 ℎ3
𝑑ℎ 2.7 1
= 2
=
𝑑𝑡 3𝜋ℎ 40𝜋
OR
f ' x   6 x 2  18x  12   6x  2x  1
For intervals f ' x  0
 6x  2x  1  0
x  1 and x  2
Interval Value of f ' x  Sign of f ' x  Nature of
function
 ,2 f '  3  12 Negative Decreasing
 2,1 f '  1.5  1.5 Positive Increasing
 1,  f ' 0  6 Negative Decreasing
𝑑𝑥 𝑥 𝑡𝑎𝑛−1 𝑦
21. The given DE is + =
𝑑𝑦 1+𝑦 2 1+𝑦 2
𝑡𝑎𝑛−1 𝑦
I.F = 𝑒
−1
Solution: x = (tan-1y – 1 ) + 𝑐𝑒 𝑡𝑎𝑛 𝑦

22. Required plane :(2x + 6y + 12) +  (3x – y + 4z) = 0


Use perpendicular distance from (0,0,0) = 1
 = 2
The required planes are 2x + y + 2z + 3 = 0 and x – 2y +2 z – 3 = 0

23.
A = event of transferring a white ball
B = event of transferring a black ball
8 7
P(A) = 15 and P(B) = 15
E = event of selecting white ball from II bag
P(E) = P(AE or BE) = P(AE) + P(BE)
8 6 7 5 83
= 15 × 10 + 15
× 10 = 150
OR
X 0 1 2
P(X) 𝟏 𝟏 𝟏
𝟒 𝟐 𝟒
3
Mean = 2
3
Variance = 4

√𝑦+6−1
Q24. ( i ). For correct x= g(y)= 3

(ii) gof = IR+

(iii)fog = I[-5, ∞ )

(iv) f-1 =g

Q.25 i) 2x-3y+5z= 11, 3x +2y -4z = -5 and x +y-2z = -3

ii)Write in matrix form AX = B

0 1 −2
iii) for correct A -1 = [−2 9 −23]
−1 5 −13
iv) For calculation and solution x=1,y=2 and z=3

Q.26 (i) For drawing the correct curve line and circle

(ii) for point of intersection of two curves ( 0,0) and (4,4), (4 √2,0)

4 4 √2
(iii)For writing ∫0 (𝑥 𝑑𝑥 + ∫4 (√32 − 𝑥 2 𝑑𝑥

(iv) For evaluating the answer 4𝜋 square unit.


4 1 4
Q16 ∫0 |𝑥 − 1|𝑑𝑥 = ∫0 −(𝑥 − 1)𝑑𝑥 + ∫1 (𝑥 − 1)𝑑𝑥

−𝑥 2 𝑥2
=[ + 𝑥] 10 + [ 2 − 𝑥] 41
2

=5

Q17 I=∫ √4 − (𝑥 + 1)2 dx

Put x+1=y;dx=dy

I=∫ √4 − 𝑦 2 𝑑𝑦
1 𝑥+1
I= (𝑥 + 1)√3 − 2𝑥 − 𝑥 2 + 2𝑠𝑖𝑛−1 ( )+𝑐 2
2 2

OR

S = { 1,2,3,4,5,6 } X is the random variable which can take values 1,2,3,4,5or6.

X 1 2 3 4 5 6

1/6 1/6 1/6 1/6 1/6 1/6


P(X)

21 91
For getting E(x)= 6 , E(x2)= 6 ,

35
For getting var(x)=12

SECTION C

3 4
𝑥+2 𝑦+ 𝑧+
2 3
Q.23 Equation of the line = = 5
3 2
3

3 4 5
Coordinates of any point P on this line be (-2+3t, -2 +2t, -3 +3t )

9 5 8
Direction ratios of AP are 3t,2t-2 ,3 t +,3 .

Direction ratios of normal to the plane 4x+12y-3z+1=0 are (4,12,-3)

Formulae a1a2+b1b2+c1c2=0
t=2

5
therefore point P(4,2,2)

17
AP = units
2

27Equation of plane containing the given point is

A(x-1) +B(y-1)+C(z-2)=

2A+3B-2C=0

A + 2B – 3C = 0

5x-4y-z= 7

Q28. Let x units of food F1 and y units of F2 be mixed to get the desire diet.

Min Z = 4x +6y

Subject to the constraints

3x +6y ≥ 80

4x +3y ≥ 100

x, y ≥ 0

For finding correct feasible region

Corner points : Value of Z = 4x +6y

80 2
A( 3 , 0) 1063

4 104 (Min)
B(24, 3)

100 200
C(0, )
3

The feasible region has no point common with 4x +6y < 104.

Min cost of diet = Rs.104.

For correct value answer


a a
29. Applying  f x dx   f a  x dx correctly
0 0

Getting up to log 2x   I


4
0


Getting I= log 2
8

OR

1 1 1
log( 1  cos x)  log( 1  cos x)  log( 2  cos x)  c
6 2 3
SAMPLE PAPER -4
CLASS XII : MATHEMATICS
BLUE PRINT
S Topics 1 2 4 6 T
. m marks m O
N m ar a T
o a ks r A
. r k L
k s
1c) RELATIONS AND 1 1
FUNCTIONS 1

d) INVERSE TRIGONOMETRIC 1 2 0

FUNCTIONS
2c) MATRICES 1 1 1
d) DETERMINANTS 1 1 3

3f) CONTINUITY & 1 1


DIFFERENTIABILITY
g) APPLICATION OF 1 3
DERIVATIVES
4
h) INTEGRATION 1 1 1
4
i) APPLICATION OF 1
INTEGRALS
j) DIFFERENTIAL 1 1
EQUATIONS
4c) VECTORS 1 1 1 1
d) 3-DIMENTIONAL 1 1 7
GEOMETRY
5LINEAR PROGRMMING 1 6

6PROBABILITY 1 2 1
0

TOTAL 4 8x 11x4 = 6 1
x 2 44 * 0
1 = 6 0
16 =
= 3 (
6 2
4 9
)

SAMPLE PAPER : -4
CLASS XII : MATHEMATICS

Time: 03hours Max Marks: 100

SECTION – A

1. Find gf (-3) if f(x) =|𝒙| and g(x) = |𝟓𝒙 − 𝟐|


3𝜋
2. Find the principal value of 𝑠𝑖𝑛−1 [𝑠𝑖𝑛 ]
5
3 1
3. If A = ( ), then find |𝑎𝑑𝑗 𝐴|
2 −3
4. If 𝑎⃗ = 𝑖̂ + 2𝑗̂ − 3𝑘 ̂ and 𝑏⃗⃗ = 2𝑖̂ + 4𝑗̂ + 9𝑘̂, find a unit vector parallel to⃗⃗⃗⃗
𝑎 + 𝑏⃗⃗
SECTION – B
sin 𝛼 cos 𝛼
5.If𝐴 = ( ), verify that A’ A = I.
−𝑐𝑜𝑠𝛼 sin 𝛼

1 1 1 1
6. Evaluate: 𝑡𝑎𝑛−1 + 𝑡𝑎𝑛−1 + 𝑡𝑎𝑛−1 + 𝑡𝑎𝑛−1
5 7 3 8
3𝑎𝑥 + 𝑏, 𝑖𝑓 𝑥 > 1
7.If the function f(x) given by: 𝑓(𝑥) = { 11 𝑥=1
5𝑎𝑥 − 2𝑏 𝑖𝑓 𝑥 < 1
is continuous at x = 1, find the value of a and b.
8 .Find the intervals in which the function f given by f(x) =𝑥 2 − 𝑥 + 6 is:

strictly decreasing.

2+sin 2𝑥
9.Evaluate:∫ 𝑒 𝑥 𝑑𝑥
1+cos 2𝑥

10 Form the differential equation of the family of hyperbolas having foci on x-axis and center at origin.
11.Find the area of the parallelogram whose adjacent sides are determined by the vectors
 
a  iˆ  ˆj  3kˆ and b  2iˆ  7 ˆj  kˆ .

12. A die is thrown twice and the sum of numbers appearing is observed to be 7. What is the conditional
probability that the number 2 has appeared at least once.
SECTION C

13,By using properties of determinants, show that

a b c
a 2
b c 2  (a  b)(b  c)(c  a)(a  b  c)
2

bc ca ab


14.If y = 𝑙𝑜𝑔{𝑥 + √𝑥 2 + 𝑎2 } prove that (x2 + a2) y2 + x y1 = 0

(𝑥 2 +1)(𝑥 2 +4)
15.Evaluate: ∫
(𝑥 2 +3)(𝑥 2 −5)
𝑑𝑥
OR
(3sin   2) cos  d
Evaluate:  (5  4sin   cos2  )
16A window is in the form of a rectangle surrounded by a semi-circular opening. The total perimeter
of window is 10 meters. Find the dimensions of the window so as to admit maximum light through the
whole opening.
Your friend is constructing a house for him. What will you suggest him in order to have enough
light and air?
17. Find the intervals in which the function f given by f(x) = sinx +cos x, 0 ≤ x ≤ 2 is strictly
increasing
or strictly decreasing.

OR

Prove that the curves x = y2 and xy = k cut at right angles if 8k2 = 1.

18.Show that semi-vertical angle of a right circular cone, of given surface area and maximum volume , is
1
sin−1 3 .

19. Solve the following differential equation: 𝑥 𝑑𝑦 − 𝑦 𝑑𝑥 = √(𝑥 2 + 𝑦 2 ) 𝑑𝑥

 
20. Let a = î + 4ĵ + 2 k̂ , b = 3 î - 2ĵ + 7 k and c = 2î - ĵ + 4 k . Find a vector d which is perpendicular
to

both a and b , and c . d = 15.

21.Find the shortest distance between the lines :

x 1 y 1 z 1 x 3 5 y z 7
  and  
7 6 1 1 2 1
22.In a bolt factory, machines A, B and C manufacture 25%, 35% and 40% of the total. Of their output
5%, 4%, 2% are defective. A bolt is drawn at random from the product. a) What is the probability that
the bolt drawn is defective? b) If the bolt drawn is found to be defective, find the probability that it is a
product of machine B?

23.An insurance company insured 2000 scooter drivers, 4000 car drivers and 6000 truck drivers. The
probability accidents is 0.01, 0.03 and 0.15 respectively. One of the insured persons met with an
accident. What is the probability that he is a scooter driver? What moral value will you assign to
all?

.
SECTION D
24.Using Matrices, solve the following system of equations:
2x – 3y + 5z = 11; 3x + 2y – 4z = -5 ; x + y – 2z = -3
𝜋 𝑥
25Evaluate: ∫ dx
0 𝑎2 𝑐𝑜𝑠2 𝑥+ 𝑏 2 𝑠𝑖𝑛2 𝑥
26Find the area of the region: {(x , y) : x2 + y2 ≤ 1 ≤ x + y}
(OR)
Compute the area bounded by the lines x +2y = 2 ; y – x = 1and 2x + y = 7
𝑥
27.Find the distance of the point (1, 2, 3) from the plane x – y + z = 5 measured parallel to the line =
2
𝑦 𝑧
=
3 −6
OR

 
Find the equation of the plane which contains line of intersection of planes r . i  2 j  3k  4  0 ,

 
r . 2i  j  k  5  0 and which passes through the

Point (1,0,-2).

28. A manufacturer produces two products A and B. Both the products are processed on two different
machines. The available capacity of the I machine is 12 hours and that of II machine is 9 hours. Each unit of
product A requires 3 hrs. On both machine and each unit of product B requires 2 hrs on I machine and 1 hr.
on II machine. Each unit of product A is sold at a profit of Rs. 5 and B at a profit of Rs. 6. Find the production
level for maximum profit graphically.

n 1
 , if n is odd
29. Let f : N  N be defined by f ( x)   2
n
 , if n is even
 2

Examine whether f is one to one, onto or bijective, Justify your answer


Answers:

1.gf(x) = |5|𝑥| − 2| gf(-3) = 13


3𝜋 2𝜋 2𝜋 2𝜋 𝜋 𝜋
2. 𝑠𝑖𝑛 = 𝑠𝑖𝑛 [𝜋 − ]= 𝑠𝑖𝑛 , ∈ [− , ]
5 5 5 5 2 2
2𝜋
PRINCIPAL VALUE =
5

−3 −1
3. 𝑎𝑑𝑗 𝐴 = ( ) |𝑎𝑑𝑗 𝐴| = -11
−1 3
⃗⃗ =3 𝑖̂ + 6𝑗̂ + 6𝑘̂
4.𝑎⃗ + 𝑏
⃗⃗
𝑎⃗⃗ + 𝑏 ̂
3 𝑖̂+6𝑗̂ +6𝑘
Parallel unit vector = ⃗⃗|
=
|⃗⃗⃗⃗
𝑎+𝑏 9
sin 𝛼 − cos 𝛼 sin 𝛼 cos 𝛼 1 0
5.𝐴′ 𝐴 = ( )( )= ( )=𝐼
cos 𝛼 sin 𝛼 −𝑐𝑜𝑠𝛼 sin 𝛼 0 1
𝜋
6. 6

7. a=3,b=2

8.Decreasing xε(-2,3)

9. 𝑒 𝑥 𝑡𝑎𝑛𝑥

𝑥2 𝑦2
10. The equation of family of hyperbolas: − =1
𝑎2 𝑏2
Differentiate twice to eliminate a and b
The required D.E is xy y`` + x (y`)2 – y y` = 0

11. . Area of Parallelogram =a x b

𝒊 𝒋 𝒌
=|𝟏 −𝟏 𝟑|= 15√2 sq. units
𝟐 −𝟕 𝟏

12. A={(1,6), (6,1), (2,5), (5,2), (3,4), (4,3)}

B={(1,2),(2,1),(2,2),(2,3),(3,2),(4,2),(2,4),(5,2),(2,5),(6,2),(2,6)}

A B = (5,2) (2,5)
2
P(A/B)= 11

13. Apply C1→C1-C2, C2→C2-C3

1 1 𝑐
(a-b)(b-c). 𝑎+𝑏 𝑏+𝑐 𝑐2
−1 −1 𝑎+𝑏

Apply R1→R1+R3

0 0 𝑐+𝑎+𝑏
(a-b)(b-c). 𝑎+𝑏 𝑏+𝑐 𝑐2
−1 −1 𝑎+𝑏
Expanding along R1 and getting

(a-b)(b-c)(c-a)(a+b+c)

14. Find the first derivative and cross multiplying :y1√𝑥 2 + 𝑎2 = 1

Find the second derivate and getting the answer

15. PUT y = x2
(𝑥 2 +1)(𝑥 2 +4) 𝑦 2 + 5𝑦+4 7𝑦+19
= =1+
(𝑥 2 +3)(𝑥 2 −5) 𝑦 2 − 2𝑦−15 (𝑦+3)(𝑦−5)
7𝑦+19 1 27
1+ (𝑦+3)(𝑦−5)
= 1 + 4(𝑥 2 +3) + 4(𝑥 2 −5)
(𝑥 2 +1)(𝑥 2 +4) 1 1 27 𝑥− √5
∫ (𝑥2 +3)(𝑥2 −5) = 𝑥 + 4√3
𝑡𝑎𝑛−1 3 + 8√5 𝑙𝑜𝑔 | 𝑥+√5 | OR

Put sin 𝜃= t
3𝑡−2 𝐴 𝐵
Writing (𝑡−2)2=𝑡−2+(𝑡−2)2

3 4
Writing ∫ 𝑡−2+∫ (𝑡−2)2

4
3log | sin 𝑥 − 2|- |𝑠𝑖𝑛𝑥−2|

16. Let x and y are length and breadth of the rectangle


20−(2+ 𝜋)𝑥 2 𝜋𝑥 2
Area of the figure = 4
+ 8
20
When A’(x) 0 gives x =
4+ 𝜋
Second derivative is negative at this value of x
Area is maximum when
20
Length = 4+ 𝜋
10
Breadth =
4+ 𝜋
10
Radius of semi-circle =4+ 𝜋

17. F(x) = sin 𝑥 +cos 𝑥

𝑓 −1 (x)=cos 𝑥 -sin 𝑥

𝑓 −1 (x)=0

Getting interval
𝜋 𝜋 5𝜋 5𝜋
(0, 4 ), ( 4 , 4 ),( 4 ,𝜋)

Getting increasing in
𝜋 5𝜋
(0, 4 ), ( 4 ,𝜋)
Getting decreasing in
𝜋 5𝜋
(4 , 4 )

OR

Finding point of intersection

(𝑘32 , 𝑘31)
𝑑𝑦
Finding 𝑑𝑥

1 𝑑𝑦 𝑑𝑦 −𝑦
= &
2𝑦 𝑑𝑥 𝑑𝑥
= 𝑥

1
Multiply - 2𝑥 =-1

Writing 8𝑘 2=1

18. Show that semi-vertical angle of a right circular cone, of given surface

1
area and maximum volume , is sin−1 3 .

Surface area of the cone S = πrl + πr 2

=πr(l + r)

s
l = πr − r ……………………. [1]

1
V =3 πr 2 h

1
V 2 = s(sr 2 − 2πr 4 )
9

1
Z = 9 s(sr 2 − 2πr 4 ) [1]

dz 2
= s(sr − 4πr 3 ) [1]
dr 9

d2 z 2
dr2
= 9 s(s − 12πr 2 ) [1]

dz s
dr
= 0, implies r = √4π
d2 z
<0 [1]
dr2

1
θ = sin−1 3 [1]

OR

2x+2r+πr = 10

2x+(2 + π)r = 10 [1]

1
Area = 2πx + πr 2
2

1
= 10 – (2 + π)r 2 + 2 πr 2 `[1]

dA
= 10 − (4 + π)r [1]
dr

10
For maximum area r = 4+π [1]

d2 A
<0 …………… [1]
dr2

Dimension of the rectangle are

10 20
m , m For correct graph ……………………………. [1]
4+π 4+π

4x 2 + 4y 2 = 9

x2= y solving we get

1 3
y= 2
, x = √2, − √2 radius of the circle =2 [1]

1 3
3 2
Area = 2 ∫02 2√y dy + 2 ∫12 2√( ) − y 2 dy [1]
2
2

For correct integration ………………………………………….. [2]

1 9π 9 1
Area= [3 + 8 − 4 sin−1 3]sq.unit [1]
√2

OR

For correct graph ………………………………… [1]

y = 4x + 5

y = −x + 5
1 5
y = x + [2]
4 4

0 3 3 x 5
Area= ∫−1(4x + 5)dx + ∫0 (−x + 5)dx − ∫−1 (3 + 4) dx [1]

415
= 2
sq.units.

𝒅𝒚 𝒚+√𝒙𝟐 +𝒚𝟐 𝒚 𝒚 𝟐
19. . = = + √𝟏 + ( )
𝒅𝒙 𝒙 𝒙 𝒙

Homo generous differential equation


Put y=vx
𝑑𝑦 𝑑𝑣
=𝑣+𝑥
𝑑𝑥 𝑑𝑥
𝑑𝑣
v+x = 𝑣 + √1 + 𝑣 2
𝑑𝑥

𝑑𝑣 𝑑𝑥
=
√1 + 𝑣 2 𝑥

Log(v+√1 + 𝑣 2 ) = logcx 2 Marks

𝑦 𝑦2
+ √1 + = cx
𝑥 𝑥2

20. Writing 𝑑̂= 𝑥𝑖̂+y𝑗̂+𝑧𝑘̂

Writing x+4y+2z=0

3x-2y+7z=0

2x-y+4z=15

Writing x = 32

y=⅄

z = -14⅄
15
writing⅄ = 7

180 15 220
𝑑̂= 7 𝑖̂ + 7 𝑗̂- 7 𝑘̂

⃗⃗⃗⃗⃗2 -⃗⃗⃗⃗⃗
21.𝑎 𝑎1 = 4i+6j+8k
Writing 𝑏⃗⃗⃗⃗1 * 𝑏⃗⃗⃗⃗⃗2 =-4i-6j-8k

Find | 𝑏⃗⃗⃗⃗1 * 𝑏⃗⃗⃗⃗⃗2 | = 116

Find SD=√116

22.
25 35 40
𝑃( 𝐴) = ; 𝑃(𝐵) = ; 𝑃(𝐵) =
100 100 100
a) 𝑃(𝐷) = 𝑃(𝐴)𝑃(𝐷/𝐴) + P(B) P (D / B) + P(C) P(D / C)
= 0.0345
28
b) P(B / D) = 69 using Baye’s theorem
2
23.P(E1) = 12

4
P(E2)= 12

6
P(E3)=12

A = Ensured person meets accident

P(A/E1)= .01 P(A/E2)= .03

P(A/E3)= .15

2
∗ .01
12
P(E1/A)= 2 4 6
∗ .01+ ∗ .03+ ∗ .15
12 12 12

2
= 51

Moral – every driver should have insurance to protect families future and expenditures on treatment after
accident.

24. Expressing : AX = B
0 1 −2
Getting 𝐴−1 = (−2 9 −23)
−1 5 −13
x=1; y=2;z=3

𝜋 1
25. Using the property of definite integrals:I =∫ dx
0 𝑎2 𝑐𝑜𝑠2 𝑥+ 𝑏 2 𝑠𝑖𝑛2 𝑥
Multiply and divide by Sec2x, put t = tan x
Change the limits
𝜋2
I=
2𝑎𝑏
26.points of intersection (1,0)

1
Area = ∫0 (√1 − 𝑥 2 − (1 − 𝑥))𝑑𝑥

𝜋 1
Area = 4 − 2

27. The equaiton of the line through A and parallel to given line: 28.Let
number of product A be x and of product be y. The data can be
represented as under :

28.Let number of product A be x and of product be y. The data can be


represented as under :

A(x) B(y) Available capacity

Machine I 3 hrs 2 hrs 12 hrs

Machine II 3 hrs 1 hrs 9 hrs

L.P.P. is maximize P = 5x + 6y C(0,6)

Subject to constraints B(2,3)

3x + 2y ≤ 12

3x + y ≤ 9

x ≥ 0, y ≥ 0 A(3,0)

The vertices O(0, 0), A(3, 0), B(2, 3) and

from the feasible region.


Vertex P = 5x + 6y

O(0, 0) 0+0=0
The maximum value of P is 36 at x = 0 & y = 6 i.e.
A(3, 0) 15 + 0 = 15
the no. of units of A = 0, B = 6
B(2, 3) 10 + 18 = 28

C(0, 6) 0 + 36 = 36 Maximum
29 Consider 1,2  N
11
As 1 is odd so f (1)  1
2

2
And 2 is even so, f (2)  1
2

Thus the different elements 1 and 2 of the domain of the function f have same image 1, therefore, the
function f is not one-one

Clearly for every m  N, there exists 2m-1  N such that

2m  1  1
f (2m  1)  m
2

Thus every m  co domain N is the image of some elements (here 2m-1) of domain N under f ,Hence f is onto

Since f is not one-one, therefore f is not bijective.

SAMPLE PAPER -5
CLASS XII : MATHEMATICS
BLUE PRINT
S Topics 1 2 4 6 T
. m marks m O
N m ar a T
o a ks r A
. r k L
k s
1e) RELATIONS AND 1 1
FUNCTIONS 1

f) INVERSE TRIGONOMETRIC 1 2 0

FUNCTIONS
2e) MATRICES 1 1 1
f) DETERMINANTS 1 1 3

3k) CONTINUITY & 1 1


DIFFERENTIABILITY
l) APPLICATION OF 1 3
DERIVATIVES
4
m) INTEGRATION 1 1 1
4
n) APPLICATION OF 1
INTEGRALS
o) DIFFERENTIAL 1 1
EQUATIONS
4e) VECTORS 1 1 1 1
f) 3-DIMENTIONAL 1 1 7
GEOMETRY
5LINEAR PROGRMMING 1 6

6PROBABILITY 1 2 1
0

TOTAL 4 8x 11x4 = 6 1
x 2 44 * 0
1 = 6 0
16 =
= 3 (
6 2
4 9
)

SAMPLE PAPER : -5
CLASS XII : MATHEMATICS

Time: 03hours Max Marks: 100

SECTION – A

1. Find gf (-3) if f(x) =|𝒙| and g(x) = |𝟓𝒙 − 𝟐|


3𝜋
2. Find the principal value of 𝑠𝑖𝑛−1 [𝑠𝑖𝑛 ]
5
3 1
3. If A = ( ), then find |𝑎𝑑𝑗 𝐴|
2 −3
4. If 𝑎⃗ = 𝑖̂ + 2𝑗̂ − 3𝑘 ̂ and 𝑏⃗⃗ = 2𝑖̂ + 4𝑗̂ + 9𝑘̂, find a unit vector parallel to⃗⃗⃗⃗
𝑎 + 𝑏⃗⃗
SECTION – B
sin 𝛼 cos 𝛼
5.If𝐴 = ( ), verify that A’ A = I.
−𝑐𝑜𝑠𝛼 sin 𝛼

1 1 1 1
6. Evaluate: 𝑡𝑎𝑛−1 + 𝑡𝑎𝑛−1 + 𝑡𝑎𝑛−1 + 𝑡𝑎𝑛−1
5 7 3 8
3𝑎𝑥 + 𝑏, 𝑖𝑓 𝑥 > 1
7.If the function f(x) given by: 𝑓(𝑥) = { 11 𝑥=1
5𝑎𝑥 − 2𝑏 𝑖𝑓 𝑥 < 1
is continuous at x = 1, find the value of a and b.
9 .Find the intervals in which the function f given by f(x) =𝑥 2 − 𝑥 + 6 is:

strictly decreasing.

2+sin 2𝑥
9.Evaluate:∫ 𝑒 𝑥 𝑑𝑥
1+cos 2𝑥

11 Form the differential equation of the family of hyperbolas having foci on x-axis and center at origin.
11.Find the area of the parallelogram whose adjacent sides are determined by the vectors
 
a  iˆ  ˆj  3kˆ and b  2iˆ  7 ˆj  kˆ .

12. A die is thrown twice and the sum of numbers appearing is observed to be 7. What is the conditional
probability that the number 2 has appeared at least once.

SECTION C

13,By using properties of determinants, show that

a b c
a2 b2 c 2  (a  b)(b  c)(c  a)(a  b  c)
bc ca ab


14.If y = 𝑙𝑜𝑔{𝑥 + √𝑥 2 + 𝑎2 } prove that (x2 + a2) y2 + x y1 = 0

(𝑥 2 +1)(𝑥 2 +4)
15.Evaluate: ∫
(𝑥 2 +3)(𝑥 2 −5)
𝑑𝑥
OR
(3sin   2) cos  d
Evaluate:  (5  4sin   cos 2
)

16A window is in the form of a rectangle surrounded by a semi-circular opening. The total perimeter
of window is 10 meters. Find the dimensions of the window so as to admit maximum light through the
whole opening.
Your friend is constructing a house for him. What will you suggest him in order to have enough
light and air?
17. Find the intervals in which the function f given by f(x) = sinx +cos x, 0 ≤ x ≤ 2 is strictly
increasing
or strictly decreasing.

OR

Prove that the curves x = y2 and xy = k cut at right angles if 8k2 = 1.

18.Show that semi-vertical angle of a right circular cone, of given surface area and maximum volume , is
1
sin−1 3 .

19. Solve the following differential equation: 𝑥 𝑑𝑦 − 𝑦 𝑑𝑥 = √(𝑥 2 + 𝑦 2 ) 𝑑𝑥

 
20. Let a = î + 4ĵ + 2 k̂ , b = 3 î - 2ĵ + 7 k and c = 2î - ĵ + 4 k . Find a vector d which is perpendicular
to

both a and b , and c . d = 15.

21.Find the shortest distance between the lines :

x 1 y 1 z 1 x 3 5 y z 7
  and  
7 6 1 1 2 1

22.In a bolt factory, machines A, B and C manufacture 25%, 35% and 40% of the total. Of their output
5%, 4%, 2% are defective. A bolt is drawn at random from the product. a) What is the probability that
the bolt drawn is defective? b) If the bolt drawn is found to be defective, find the probability that it is a
product of machine B?

23.An insurance company insured 2000 scooter drivers, 4000 car drivers and 6000 truck drivers. The
probability accidents is 0.01, 0.03 and 0.15 respectively. One of the insured persons met with an
accident. What is the probability that he is a scooter driver? What moral value will you assign to
all?

.
SECTION D
24.Using Matrices, solve the following system of equations:
2x – 3y + 5z = 11; 3x + 2y – 4z = -5 ; x + y – 2z = -3
𝜋 𝑥
25Evaluate: ∫ dx
0 𝑎2 𝑐𝑜𝑠2 𝑥+ 𝑏 2 𝑠𝑖𝑛2 𝑥
26Find the area of the region: {(x , y) : x2 + y2 ≤ 1 ≤ x + y}
(OR)
Compute the area bounded by the lines x +2y = 2 ; y – x = 1and 2x + y = 7
𝑥
27.Find the distance of the point (1, 2, 3) from the plane x – y + z = 5 measured parallel to the line =
2
𝑦 𝑧
=
3 −6
OR

 
Find the equation of the plane which contains line of intersection of planes r . i  2 j  3k  4  0 ,

 
r . 2i  j  k  5  0 and which passes through the

Point (1,0,-2).

28. A manufacturer produces two products A and B. Both the products are processed on two different
machines. The available capacity of the I machine is 12 hours and that of II machine is 9 hours. Each unit of
product A requires 3 hrs. On both machine and each unit of product B requires 2 hrs on I machine and 1 hr.
on II machine. Each unit of product A is sold at a profit of Rs. 5 and B at a profit of Rs. 6. Find the production
level for maximum profit graphically.

n 1
 2 , if n is odd
29. Let f : N  N be defined by f ( x)  
n
 , if n is even
 2

Examine whether f is one to one, onto or bijective, Justify your answer


Answers:
1.gf(x) = |5|𝑥| − 2| gf(-3) = 13
3𝜋 2𝜋 2𝜋 2𝜋 𝜋 𝜋
2. 𝑠𝑖𝑛 = 𝑠𝑖𝑛 [𝜋 − ]= 𝑠𝑖𝑛 , ∈ [− , ]
5 5 5 5 2 2
2𝜋
PRINCIPAL VALUE =
5

−3 −1
3. 𝑎𝑑𝑗 𝐴 = ( ) |𝑎𝑑𝑗 𝐴| = -11
−1 3
⃗⃗ =3 𝑖̂ + 6𝑗̂ + 6𝑘̂
4.𝑎⃗ + 𝑏
⃗⃗
𝑎⃗⃗ + 𝑏 ̂
3 𝑖̂+6𝑗̂ +6𝑘
Parallel unit vector = ⃗⃗|
=
|⃗⃗⃗⃗
𝑎+𝑏 9
sin 𝛼 − cos 𝛼 sin 𝛼 cos 𝛼 1 0
5.𝐴′ 𝐴 = ( )( )= ( )=𝐼
cos 𝛼 sin 𝛼 −𝑐𝑜𝑠𝛼 sin 𝛼 0 1
𝜋
6. 6
7. a=3,b=2
1
8.Decreasing xε(-∞, 2)

9. 𝑒 𝑥 𝑡𝑎𝑛𝑥

𝑥2 𝑦2
10. The equation of family of hyperbolas: − =1
𝑎2 𝑏2
Differentiate twice to eliminate a and b
The required D.E is xy y`` + x (y`)2 – y y` = 0

11. . Area of Parallelogram =a x b

𝒊 𝒋 𝒌
=|𝟏 −𝟏 𝟑|= 15√2 sq. units
𝟐 −𝟕 𝟏

12. A={(1,6), (6,1), (2,5), (5,2), (3,4), (4,3)}

B={(1,2),(2,1),(2,2),(2,3),(3,2),(4,2),(2,4),(5,2),(2,5),(6,2),(2,6)}

A B = (5,2) (2,5)
2
P(A/B)= 11

13. Apply C1→C1-C2, C2→C2-C3

1 1 𝑐
(a-b)(b-c). 𝑎+𝑏 𝑏+𝑐 𝑐2
−1 −1 𝑎+𝑏

Apply R1→R1+R3

0 0 𝑐+𝑎+𝑏
(a-b)(b-c). 𝑎+𝑏 𝑏+𝑐 𝑐2
−1 −1 𝑎+𝑏
Expanding along R1 and getting

(a-b)(b-c)(c-a)(a+b+c)

14. Find the first derivative and cross multiplying :y1√𝑥 2 + 𝑎2 = 1

Find the second derivate and getting the answer

15. PUT y = x2
(𝑥 2 +1)(𝑥 2 +4) 𝑦 2 + 5𝑦+4 7𝑦+19
= =1+
(𝑥 2 +3)(𝑥 2 −5) 𝑦 2 − 2𝑦−15 (𝑦+3)(𝑦−5)
7𝑦+19 1 27
1+ (𝑦+3)(𝑦−5)
= 1 + 4(𝑥 2 +3) + 4(𝑥 2 −5)
(𝑥 2 +1)(𝑥 2 +4) 1 1 27 𝑥− √5
∫ (𝑥2 +3)(𝑥2 −5) = 𝑥 + 4√3
𝑡𝑎𝑛−1 3 + 8√5 𝑙𝑜𝑔 | 𝑥+√5 | OR

Put sin 𝜃= t
3𝑡−2 𝐴 𝐵
Writing (𝑡−2)2=𝑡−2+(𝑡−2)2

3 4
Writing ∫ 𝑡−2+∫ (𝑡−2)2

4
3log | sin 𝑥 − 2|- |𝑠𝑖𝑛𝑥−2|

16. Let x and y are length and breadth of the rectangle


20−(2+ 𝜋)𝑥 2 𝜋𝑥 2
Area of the figure = 4
+ 8
20
When A’(x) 0 gives x =
4+ 𝜋
Second derivative is negative at this value of x
Area is maximum when
20
Length = 4+ 𝜋
10
Breadth =
4+ 𝜋
10
Radius of semi-circle =4+ 𝜋

17. F(x) = sin 𝑥 +cos 𝑥

𝑓 −1 (x)=cos 𝑥 -sin 𝑥

𝑓 −1 (x)=0

Getting interval
𝜋 𝜋 5𝜋 5𝜋
(0, 4 ), ( 4 , 4 ),( 4 ,𝜋)

Getting increasing in
𝜋 5𝜋
(0, 4 ), ( 4 ,𝜋)
Getting decreasing in
𝜋 5𝜋
(4 , 4 )

OR

Finding point of intersection

(𝑘32 , 𝑘31)
𝑑𝑦
Finding 𝑑𝑥

1 𝑑𝑦 𝑑𝑦 −𝑦
= &
2𝑦 𝑑𝑥 𝑑𝑥
= 𝑥

1
Multiply - 2𝑥 =-1

Writing 8𝑘 2=1

18. Show that semi-vertical angle of a right circular cone, of given surface

1
area and maximum volume , is sin−1 3 .

Surface area of the cone S = πrl + πr 2

=πr(l + r)

s
l = πr − r ……………………. [1]

1
V =3 πr 2 h

1
V 2 = s(sr 2 − 2πr 4 )
9

1
Z = 9 s(sr 2 − 2πr 4 ) [1]

dz 2
= s(sr − 4πr 3 ) [1]
dr 9

d2 z 2
dr2
= 9 s(s − 12πr 2 ) [1]

dz s
dr
= 0, implies r = √4π
d2 z
<0 [1]
dr2

1
θ = sin−1 3 [1]

OR

2x+2r+πr = 10

2x+(2 + π)r = 10 [1]

1
Area = 2πx + πr 2
2

1
= 10 – (2 + π)r 2 + 2 πr 2 `[1]

dA
= 10 − (4 + π)r [1]
dr

10
For maximum area r = 4+π [1]

d2 A
<0 …………… [1]
dr2

Dimension of the rectangle are

10 20
m , m For correct graph ……………………………. [1]
4+π 4+π

4x 2 + 4y 2 = 9

x2= y solving we get

1 3
y= 2
, x = √2, − √2 radius of the circle =2 [1]

1 3
3 2
Area = 2 ∫02 2√y dy + 2 ∫12 2√( ) − y 2 dy [1]
2
2

For correct integration ………………………………………….. [2]

1 9π 9 1
Area= [3 + 8 − 4 sin−1 3]sq.unit [1]
√2

OR

For correct graph ………………………………… [1]

y = 4x + 5

y = −x + 5
1 5
y = x + [2]
4 4

0 3 3 x 5
Area= ∫−1(4x + 5)dx + ∫0 (−x + 5)dx − ∫−1 (3 + 4) dx [1]

415
= 2
sq.units.

𝒅𝒚 𝒚+√𝒙𝟐 +𝒚𝟐 𝒚 𝒚 𝟐
19. . = = + √𝟏 + ( )
𝒅𝒙 𝒙 𝒙 𝒙

Homo generous differential equation


Put y=vx
𝑑𝑦 𝑑𝑣
=𝑣+𝑥
𝑑𝑥 𝑑𝑥
𝑑𝑣
v+x = 𝑣 + √1 + 𝑣 2
𝑑𝑥

𝑑𝑣 𝑑𝑥
=
√1 + 𝑣 2 𝑥

Log(v+√1 + 𝑣 2 ) = logcx 2 Marks

𝑦 𝑦2
+ √1 + = cx
𝑥 𝑥2

20. Writing 𝑑̂= 𝑥𝑖̂+y𝑗̂+𝑧𝑘̂

Writing x+4y+2z=0

3x-2y+7z=0

2x-y+4z=15

Writing x = 32

y=⅄

z = -14⅄
15
writing⅄ = 7

180 15 220
𝑑̂= 7 𝑖̂ + 7 𝑗̂- 7 𝑘̂

⃗⃗⃗⃗⃗2 -⃗⃗⃗⃗⃗
21.𝑎 𝑎1 = 4i+6j+8k
Writing 𝑏⃗⃗⃗⃗1 * 𝑏⃗⃗⃗⃗⃗2 =-4i-6j-8k

Find | 𝑏⃗⃗⃗⃗1 * 𝑏⃗⃗⃗⃗⃗2 | = 116

Find SD=√116

22.
25 35 40
𝑃( 𝐴) = ; 𝑃(𝐵) = ; 𝑃(𝐵) =
100 100 100
c) 𝑃(𝐷) = 𝑃(𝐴)𝑃(𝐷/𝐴) + P(B) P (D / B) + P(C) P(D / C)
= 0.0345
28
d) P(B / D) = 69 using Baye’s theorem
2
23.P(E1) = 12

4
P(E2)= 12

6
P(E3)=12

A = Ensured person meets accident

P(A/E1)= .01 P(A/E2)= .03

P(A/E3)= .15

2
∗ .01
12
P(E1/A)= 2 4 6
∗ .01+ ∗ .03+ ∗ .15
12 12 12

2
= 51

Moral – every driver should have insurance to protect families future and expenditures on treatment after
accident.

24. Expressing : AX = B
0 1 −2
Getting 𝐴−1 = (−2 9 −23)
−1 5 −13
x=1; y=2;z=3

𝜋 1
25. Using the property of definite integrals:I =∫ dx
0 𝑎2 𝑐𝑜𝑠2 𝑥+ 𝑏 2 𝑠𝑖𝑛2 𝑥
Multiply and divide by Sec2x, put t = tan x
Change the limits
𝜋2
I=
2𝑎𝑏
26.points of intersection (1,0)

1
Area = ∫0 (√1 − 𝑥 2 − (1 − 𝑥))𝑑𝑥

𝜋 1
Area = 4 − 2

27. The equaiton of the line through A and parallel to given line: 28.Let
number of product A be x and of product be y. The data can be
represented as under :

28.Let number of product A be x and of product be y. The data can be


represented as under :

A(x) B(y) Available capacity

Machine I 3 hrs 2 hrs 12 hrs

Machine II 3 hrs 1 hrs 9 hrs

L.P.P. is maximize P = 5x + 6y C(0,6)

Subject to constraints B(2,3)

3x + 2y ≤ 12

3x + y ≤ 9

x ≥ 0, y ≥ 0 A(3,0)

The vertices O(0, 0), A(3, 0), B(2, 3) and

from the feasible region.

The maximum value of P is 36 at x = 0 & y = 6 i.e. the no. of units of A = 0, B = 6

29 Consider 1,2  N

11
As 1 is odd so f (1)  1
2
2
And 2 is even so, f (2)  1
2

Thus the different elements 1 and 2 of the domain of the function f have same image 1, therefore, the
function f is not one-one

Clearly for every m  N, there exists 2m-1  N such that

2m  1  1
f (2m  1)  m
2

Thus every m  co domain N is the image of some elements (here 2m-1) of domain N under f ,Hence f is onto

Since f is not one-one, therefore f is not bijective.

Vertex P = 5x + 6y

O(0, 0) 0+0=0
Note: New pattern Sample Question is available at
CBSE A(3, 0) 15 + 0 = 15 website: cbse.nic.in->public portal->
academic- >circular 32/2016.
B(2, 3) 10 + 18 = 28

C(0, 6) 0 + 36 = 36 Maximum

GROUP- 3

Submitted by ; A P SHRIVASTAVA

KV VF JABALPUR

SAMPLE PAPER

SUBJECT: MATHEMATICS(041)

BLUE PRINT : CLASS XII

Chapter VSA Short answer Long answer Long answer Total


(1 mark) (2 marks) - I (4 marks) - II (6 marks)

Relations and Functions 2(2) -- -- 6(1) 8(3)

Inverse Trigonometric
-- 2(1) -- -- 2(1)
Functions

Matrices -- 2(1) -- -- 2(1)

Determinants 1(1) -- 4(1) 6(1) 11(3)

Continuity &
-- 2(1) 8(2) -- 10(3)
Differentiability

Applications of
-- 2(1) 8(2) -- 10(3)
Derivatives

Integrals -- 2(1) 4(1) 6(1) 12(3)

Applications of the
-- -- -- 6(1) 6(1)
Integrals

Differential Equations -- 2(1) 4(1) -- 6(2)

Vector Algebra 1(1) 2(1) 4(1) -- 7(3)

Three-Dimensional
-- -- 4(1) 6(1) 10(2)
Geometry

Linear Programming -- -- -- 6(1) 6(1)

Probability -- 2(1) 8(2) -- 10(3)


Total 4(4) 16(8) 44(11) 36(6) 100(29)

Submitted by ; A P SHRIVASTAVA

KV VF JABALPUR

SAMPLE PAPER

SUBJECT: MATHEMATICS MAX. MARKS : 100

CLASS : XII DURATION : 3 HRS

General Instruction:

(i) All questions are compulsory.

(ii) This question paper contains 29 questions.


(iii) Question 1- 4 in Section A are very short-answer type questions carrying 1 mark each.

(iv) Question 5-12 in Section B are short-answer type questions carrying 2 marks each.
(v) Question 13-23 in Section C are long-answer-I type questions carrying 4 marks each.
(vi) Question 24-29 in Section D are long-answer-II type questions carrying 6 marks each.

SECTION A
Q.N. QUESTION MARKS
1 Find the number of all one - one functions from set 1
{1, 2, 3} to itself.
2. Check whether the relation given by R = {(1,2),(3,4)} 1
defined in the set {1,2,3,4} is transitive or not.
3. 𝒂 𝒃 𝒄 1
𝑭𝒊𝒏𝒅 𝒕𝒉𝒆 𝒗𝒂𝒍𝒖𝒆 𝒐𝒇 |𝒂 + 𝟐𝒙 𝒃 + 𝟐𝒚 𝒄 + 𝟐𝒛|
𝒙 𝒚 𝒛
4. ̂ and ⃗𝒃⃗ = 𝒊̂ + 𝒋̂ + 𝒌
⃗⃗ = 𝒊̂ + 𝒋̂ − 𝟐𝒌
If 𝒂 ̂ , then find the 1
angle between 𝒂 ⃗⃗⃗⃗ 𝒂𝒏𝒅 ⃗𝒃⃗.
SECTION B
5 Find the equation of normal to the curve 𝒂𝒚𝟐 = 𝒙𝟑 at 2
the point (𝒂𝒎𝟐 , 𝒂𝒎𝟑 ).
6 Find the area of the parallelogram having diagonals 2
̂ and 𝒊̂ − 𝟑𝒋̂ + 𝟒𝒌
3𝒊̂ + 𝒋̂ − 𝟐𝒌 ̂.
7 Probability of solving specific problem by A and B 2
are ½ and 1/3 respectively. If both try to solve the
problem independently , find the probability that
1. The problem is solved, and
2.Exactly one solves the problem.
8 Find the integrating factor of the differential 2
equation
𝒚 𝒅𝒙 − (𝒙 + 𝟐𝒚𝟐 )𝒅𝒚 = 𝟎.
9 𝟒 𝟓 𝟏𝟔 𝝅 2
Prove that, 𝒔𝒊𝒏−𝟏 ( ) + 𝒔𝒊𝒏−𝟏 ( ) + 𝒔𝒊𝒏−𝟏 ( ) =
𝟓 𝟏𝟑 𝟔𝟓 𝟐
10 𝟓 2
𝟑 −𝟔
Compute[𝟏 𝟐] [ ] + [𝟎 𝟑 – 𝟏 𝟐] [ ].
𝟒 𝟖
𝟐
11 Differentiate 𝒔𝒊𝒏 𝒙 𝒘𝒊𝒕𝒉 𝒓𝒆𝒔𝒑𝒆𝒄𝒕 𝒆𝒄𝒐𝒔𝒙 .
𝟐 2
12 𝟑𝒙−𝟏 2
Evaluate : ∫ (𝒙+𝟐)𝟐 𝒅𝒙
SECTION C
13 𝒙 𝒔𝒊𝒏−𝟏 𝒙 4
Solve : ∫ 𝒅𝒙.
√𝟏− 𝒙𝟐
14 Discuss the continuity of the function 4
𝒇 = {𝒙𝟑 + 𝟑 𝒊𝒇 𝒙 ≠ 𝟎 𝒂𝒏𝒅 𝟏 𝒊𝒇 𝒙 = 𝟎
At x =0
Or
Prove that the function f given by
𝒇(𝒙) = |𝒙 − 𝟏|, 𝒙 𝝐 𝑹
is not differentiable at x =1.
15 𝟏 𝒂 𝒂𝟑 4
𝑷𝒓𝒐𝒗𝒆 𝒕𝒉𝒂𝒕 |𝟏 𝒃 𝒃𝟑 |
𝟏 𝒄 𝒄𝟑
= (𝒂 − 𝒃)(𝒃 − 𝒄)(𝒄 − 𝒂)(𝒂 + 𝒃 + 𝒄)
16 An unbiased coin is tossed 4 times. Find the mean 4
and variance of heads obtained
17 Solve the differential equation : 4
(𝒕𝒂𝒏−𝟏 𝒚 − 𝒙)𝒅𝒚 = (𝟏 + 𝒚𝟐 )𝒅𝒙.
𝒐𝒓
Find the particular solution of the differential
𝒅𝒚 𝒙𝒚
equation = . Given that y=1 and x = 0.
𝒅𝒙 𝒙𝟐 +𝒚𝟐
18 A man is known to speak truth 3 out of 4 times. He 4
throws a die and reports that it is a six. Find the
probability that it is actually a six.
19 If 𝑰𝒇 𝒚 = 𝒔𝒊𝒏(𝒍𝒐𝒈𝒙), 𝒕𝒉𝒆𝒏 𝒔𝒉𝒐𝒘 𝒕𝒉𝒂𝒕 4
𝟐
𝒅𝟐 𝒚 𝒅𝒚
𝒙 + 𝒙 + 𝒚 = 𝟎.
𝒅𝒙𝟐 𝒅𝒙
20 Find the local maxima or local minima, if any of the 4
function
𝝅
𝒇(𝒙) = 𝒔𝒊𝒏𝒙 + 𝒄𝒐𝒔 𝒙, 𝟎 < 𝑥 < .
𝟐
21 Find the equation of the plane through the intersection 4
of planes 3x-y+2z-4=0 and x+y+z-2 =0 and the point
(2,2,1).
22
Show that (𝒂 𝒃)𝟐 = |𝒂
⃗⃗ × ⃗⃗⃗⃗⃗ ⃗⃗. 𝒂
⃗⃗ 𝒂⃗⃗. ⃗𝒃⃗| 4
⃗⃗. ⃗𝒃⃗ ⃗𝒃⃗. ⃗𝒃⃗
𝒂
23 Show that the volume of the largest cone that can be 4
inscribed in a sphere of radius R is 8/27 of the volume
of the sphere.
SECTION D
24 Prove that the relation R on the set 𝑵 × 6
𝑵 𝒅𝒆𝒇𝒊𝒏𝒆𝒅 𝒃𝒚
(𝒂, 𝒃)𝑹(𝒄, 𝒅) ⇔ 𝒂 + 𝒅 = 𝒃 + 𝒄
𝒇𝒐𝒓 𝒂𝒍𝒍 (𝒂, 𝒃), (𝒄, 𝒅) 𝝐 𝑵 × 𝑵
Is an equivalence relation.
OR
Consider 𝒇 ∶ 𝑹+ → [ −𝟓, ∞) given by 𝒇(𝒙) =
𝟗𝒙𝟐 + 𝟔𝒙 − 𝟓, show that 𝒇 is invertible with
√𝒚+𝟔− 𝟏
𝒇−𝟏 (𝒚) =
𝟑
25 Using the properties of determinants, prove that : 6
(𝒃 + 𝒄)𝟐 𝒂𝟐 𝒃𝒄
| (𝒄 + 𝒂)𝟐 𝒃𝟐 𝒄𝒂 | =
(𝒂 + 𝒃)𝟐 𝒄𝟐 𝒂𝒃
(𝒂𝟐 + 𝒃𝟐 + 𝒄𝟐 )(𝒂 + 𝒃 + 𝒄)(𝒃 − 𝒄)(𝒄 − 𝒂)(𝒂 − 𝒃)
OR
Solve using matrix method x+2y+z =7 , x+3z= 11,
2x-3y =1.
26 1. Find the area bounded by the lines 𝒚 = 𝟐𝒙 + 6
𝟏, 𝒚 = 𝟑𝒙 + 𝟏 𝒂𝒏𝒅
𝒙 = 𝟒 𝒖𝒔𝒊𝒏𝒈 𝒊𝒏𝒕𝒆𝒈𝒓𝒂𝒕𝒊𝒐𝒏.
Or
𝟒
Evaluate as a limit of sum ∫𝟎 (𝒙 + 𝒆𝟐𝒙 )𝒅𝒙
𝝅
27 6
Evaluate : ∫𝟎 𝒍𝒐𝒈 𝒔𝒊𝒏 𝒙 𝒅𝒙
𝟐

28 𝒙+𝟏 6
Find the shortest distance between lines =
𝟕
𝒚+𝟏 𝒛+𝟏 𝒙−𝟑 𝒚−𝟓 𝒛−𝟕
= 𝒂𝒏𝒅 = =
−𝟔 𝟏 𝟏 −𝟐 𝟏
29 𝑴𝒂𝒙𝒊𝒎𝒊𝒛𝒆 𝒛 = 𝟐𝒙 + 𝟓𝒚 6
𝑺𝒖𝒃𝒋𝒆𝒄𝒕 𝒕𝒐 ∶ 𝟐𝒙 + 𝟒𝒚 ≤ 𝟖,
𝟑𝒙 + 𝟒𝒚 ≤ 𝟔,
𝒙 + 𝒚 ≤ 𝟒,
𝒙 ≥ 𝟎, 𝒚 ≥ 𝟎.
Submitted by ; A P SHRIVASTAVA
KV VF JABALPUR

MARKING SCHEME
Q.No. Steps Marks
1 Correct answer 6. 1

2 Relation is transitive 1

3 Correct answer 0 1

4 𝟗𝟎𝟎 1

5 Slope of normal = −
𝟐 ½
𝟑𝒎
Equation of normal ½
Final answer 𝟐𝒙 + 𝟑𝒎𝒚 − 𝒂𝒎𝟐 (𝟑𝒎𝟐 + 𝟐) = 𝟎. 1
6 Application of formula and calculation 1 each
7 Each part 1 each
8 Conversion into standard form 1
Correct answer 1/y 1
9 𝟒 𝟓 𝟔𝟑 1
𝒔𝒊𝒏−𝟏 ( ) + 𝒔𝒊𝒏−𝟏 ( ) = 𝒔𝒊𝒏−𝟏 ( )
𝟓 𝟏𝟑 𝟔𝟓
−𝟏 𝟔𝟑 −𝟏 𝟏𝟔
𝒔𝒊𝒏 ( ) + 𝒔𝒊𝒏 ( ) =
𝝅 1
𝟔𝟓 𝟔𝟓 𝟐
10 Multiplication of matrices 1
Correct answer [-11] 1
11 Derivative of each function w.r.t. x 𝟏
Final answer 1
12 Partial fraction 1
Integration of both parts 1
𝟕
( answer 𝟑𝒍𝒐𝒈|𝒙 + 𝟐| + + 𝒄)
𝒙+𝟐
13 Substitution 1 marks
Calculation and simplification 1marks
Integration by parts 1 marks
Answer 1 marks
14 Each part (LHL & RHL : LHD & RHD) 2 marks
15. Getting 0 in second and third row 1
Simplification and factorization 2
Final answer 1
16 Each part 2 marks
17 𝒅𝒙
Conversion in the form of + 𝑷𝒙 = 𝑸 1
𝒅𝒚
Integrating Factor 1
Solution 2
Or
Homogeneous method 1
Substitution 1
Answer 2
18 Correct formula and correct usage 1
Evaluation of all components 2
Calculation and answer = 3/8 1
19 Taking log and differentiating once 2
Differentiating twice 2
20 𝒅𝒚 1
= 𝒄𝒐𝒔 𝒙 − 𝒔𝒊𝒏𝒙
𝒅𝒙
𝝅 1
𝒙=
𝟒
Second derivative 1
Maximum value 1
21 Formation of plane 2
Evaluation of the constant 1
Answer 7x-5y+4z-8=0 1
22 Formula of cross product 1
𝒗𝒂𝒍𝒖𝒆 𝒐𝒇 (𝒂 ⃗⃗⃗⃗⃗𝟐
⃗⃗ × 𝒃) 2
Answer 1
23 𝟏 1
𝑽 = 𝝅(𝑹𝟐 − 𝒙𝟐 )(𝑹 + 𝒙)
𝟑
Value of R 1
Result 2
24 Reflexive, symmetrix ,transitive 2 each
OR
One – one 2 marks
Onto 2 marks
Inverse 2marks
25 𝑪𝟏 → 𝑪𝟏 + 𝑪𝟐 − 𝟐𝑪𝟑 2
Taking (𝒂𝟐 + 𝒃𝟐 + 𝒄𝟐 ) 𝒄𝒐𝒎𝒎𝒐𝒏 𝒇𝒓𝒐𝒎 2
𝑪𝟏 𝒂𝒏𝒅 𝒂𝒑𝒑𝒍𝒚𝒊𝒏𝒈 𝑹𝟏 → 𝑹𝟏 − 𝑹𝟑 𝒂𝒏𝒅
𝑹𝟐 → 𝑹𝟐 − 𝑹𝟑
Simplification and final answer 2
OR
|𝑨| = 𝟏𝟖 1 mark
AdjA 1 mark
Inverse 2 mark
Answer x = 2, y = 1, z = 3 2 mark
26 Diagram 1 mark
Identification of required region 1 mark
Application of formula 2 mark
Calculation and answer = 8 sq units 2 marks
Or
Adoption of formula 2 marks
Calculation 3 marks
Answer 1 mark
27 Application of f(a-x) = f(x) 2
Adding both integrals 1
Simplification 2
Correct integration 1
28 Conversion, calculation and correct answer 2√𝟐𝟗 2 each
29 Graph 2 marks
vertices 1 marks
Evaluation 3 marks
( x = 0,y = 2, max z = 10 )

SAMPLE QUESTION PAPER

MATHEMATICS

CLASS XII 2016-17

TIME : 3 HOURS Maximum Marks: 100

General Instructions:

(vii) All questions are compulsory.


(viii) This question Paper contains 29 questions.
(ix) Question 1 - 4 in Section A are very short-answer type questions carrying 1 mark each.
(x) Question 5 - 12 in section B are short answer type question carrying 2 marks each.
(xi) Question 13 - 23 in section C are long answer type I question carrying 4 marks each .
(xii) Question 24 - 29 in section D are long answer type II question carrying 6 marks each .

SECTION - A

2. State the reason why the Relation R = { ( a , b ) : a ≤ 𝑏 3 } on the set R of real numbers is not

Reflexive.
0
2. Evaluate |cos 700 sin 200 |
𝑠𝑖𝑛70 𝑐𝑜𝑠200

𝑎 = 2𝑖̂ - 𝑗̂ + 2𝑘̂ and 𝑏⃗⃗ = -𝑖̂ + 𝑗̂ + 3𝑘̂


3. Find the unit vector in the direction of the sum of the vectors ⃗⃗⃗⃗

4. The binary operation * on R is denoted by a * b = 2a + b . Find ( 2 * 3 )

SECTION – B
1 1 2
5. Prove thattan−1 7 + tan−1 13 = tan−1 9

5 2 3 6
6. Find Matrix X and Y , if X + Y = | | and X - Y = | |
0 9 0 −1
2𝑥
7. Differentiate with respect to x , sin−1 (1+ 𝑥2 )

8. Find the rate of change of area of a circle with respect to its radius ‘ r ‘ when r = 6 cm.

9. Evaluate ∫ sin−1(cos 𝑥) 𝑑𝑥

10. Find the differential of the family of all straight lines.

11. Find the position vector of the mid – point of the vector joining the points P ( 2 , 3 , 4 ) and
Q ( 4 , 1 , -2 ).
3 7 9
12. If A and B are the two events such that P ( A ) = , P(B)= and P ( A ∪ 𝐵 ) = , then find
5 10 10

P ( A∩ 𝐵 ) .

SECTION – C

13. Using properties of determinants , show that

1  a 2  b2 2ab 2b
 1  a 2  b 2 
3
2ab 1  a 2  b2 2a
2b 2a 1  a 2  b2

sin 𝑥
+ cos 𝑥, 𝑥 > 0
𝑥
14. Show that the function f (𝑥) given by 𝑓(𝑥) = 2 ,𝑥 = 0
4(1−√1−𝑥)
{ ,𝑥 < 0
𝑥

is continuous at 𝑥 = 0.

OR
1−𝑐𝑜𝑠4𝑥
, 𝑥>0
𝑥2
If the function f defined by f(x) = 𝑎 , 𝑥=0
√𝑥
, 𝑥>0
{ √16+√𝑥 −4 }
is continuous at x = 0, find the value of a.
−1 𝑥 𝑑2𝑦 𝑑𝑦
15. If y = 𝑒 𝑚 cos , prove that (1-𝑥 2 ) -x = m2y.
𝑑𝑥 2 𝑑𝑥

16. Find the intervals in which the function

1 − 12𝑥 − 9𝑥 2 − 2𝑥 3 is increasing or decreasing.

OR
Find the equation of the normal line to the curve 𝑦(𝑥 − 2)(𝑥 − 3) − 𝑥 + 7 = 0 at the point where

it meets the 𝑥 −axis.

17. Show that height of the cylinder of maximum volume that can be inscribed in a

2R
sphere of radius R is . Also find the maximum volume.
3

x 4 dx
18. Evaluate   x  1  x2  1

19. Find the general solution of the differential equation: (1 + 𝑦 2 )𝑑𝑥 = (tan−1 𝑦 − 𝑥)𝑑𝑦.

OR
𝑥 𝑥
Solve the differential equation : 2𝑦. 𝑒 𝑦 𝑑𝑥 + (𝑦 − 2𝑥𝑒 𝑦 ) 𝑑𝑦 = 0 .

20. Express the vector 𝑎⃗ = 5𝑖̂ − 2𝑗̂ + 5𝑘̂ as the sum of two vectors such that one is parallel to the

vector 𝑏⃗⃗ = 3𝑖̂ + 𝑘̂ and other is perpendicular to 𝑏⃗⃗ .

21. Find the shortest distance between the lines whose vector equations are

𝑟⃗ = (1 − 2𝑡)𝑖̂+(1 − 𝑡)𝑗̂+(𝑡)𝑘̂ and


𝑟⃗ = (2 + 3𝑠)𝑖̂ +(1 − 5𝑠)𝑗̂+(2𝑠 − 1)𝑘̂

22. A speaks truth in 70% of the cases and B speaks truth in 80 % of the cases .In what

percentage of the cases :-

(i) They contradict each other in stating the same fact?

(ii) They agree each other in stating the same fact?

Truth is very closely related with our national movement. How?

23. By examining the chest X-ray , the probability that TB is detected when a person actually

suffering is 0.99 . The probability of incorrect diagnosis is 0.001. In a certain city one in

thousand persons suffer from TB . A person selected at random and is diagnosed to have TB.

What is the chance that he actually has TB.

SECTION – D

24. Let f : N → R be a function defined as f (x ) = 4x2 + 12x + 15 . Show that f : N → 𝑆 , where S is


the range of f , is invertible. Also find the inverse of f .

OR

A binary operation * is defined on the set X = R – { - 1 } by x * y = x + y + xy , ∀ x , y ∈ X.

Check whether * is commutative and Associative. Find its identity element and also find the
inverse of each element of X.

25. Two school A and B decided to award prizes to their students for three values

honesty(x),punctuality (y) and obedience(z).School A decided to award a total of Rs 11000

for the three values of 5,4 and 3 students respectively while school B decided to award

Rs 10700 for the three values of 4, 3 and 5 students respectively. If all the three prizes

together amount to Rs. 2700, then:

(iii) Represent the above situation by a matrix equation and form linear equations using matrix
multiplication.
(iv) Is it possible to solve the system of equation so obtained using matrix? If yes, find the
award money for each value
OR

Using the properties of determinants , prove that:

𝑥 𝑥2 1 + 𝑝𝑥 3
|𝑦 𝑦2 1 + 𝑝𝑦 3 | = (1 + 𝑝𝑥𝑦𝑧)(𝑥 − 𝑦)(𝑦 − 𝑧)(𝑧 − 𝑥)
𝑧 𝑧2 1 + 𝑝𝑧 3

26. Using integration, find the area of the triangle whose vertices are 𝐴(1,0), 𝐵(2,2) 𝑎𝑛𝑑 𝐶(3,1)

 /2 x
27. . Evaluate  0 sin x  cos x
dx

OR
2

  3x  1 dx.
2
Evaluate the following integral as a limit of a sum
1

28. Find the equation of the plane passing through the point (-1, 3, 2) and perpendicular to each

of the planes 𝑥 + 2𝑦 + 3𝑧 = 5 and 3𝑥 + 3𝑦 + 𝑧 = 0. Also find the angle between this plane

and x-axis.

29. A dealer wishes to purchase number of fans and sewing machines . He has only Rs. 5760 to
invest and has a space for at most 20 items . A fan cost him Rs. 360 and sewing machine Rs. 240 .

His expectation is that .He can sell a fan at profit of Rs. 22and sewing machine at a profit of

Rs.18 . Assuming that he can sell all the items that he can buy , how should invest his money in

order to maximize the profit ? Formulate this as a linear programming problem and solve it

graphically .

MARKING SCHEME

13. For Reflexivity ( a , a ) ∈ R


1 1
Clearly ½ is a real no. , but is not less than or equal to ( )3
2 2
1 1
(2 , 2 ) not belongs to R
Hence , R is not Reflexive. 1 mark
14. Cos 700.cos200 - sin 700 sin 200
= cos (70 + 20 ) = 0 1 mark
15. For getting unit vector 1mark
16. a * b = 2a + b
2*3=7 1 mark
1 1
17. tan−1 + tan−1
7 13
Applying formula 1 mark
2
= tan−1 91 mark
4 4
18. Getting value of X = [ ] 1 mark
0 4
1 −2
Y =[ ] 1 mark
0 5
19. Taking x = tan𝜃 ,
Getting y = 2𝜃 1 mark
𝑑𝑦
For getting 𝑑𝑥
=2 / ( 1+x2) 1 mark
20. For getting dA/dr = 2𝜋r 1 mark
(dA/dr ) at r = 6 = 12 𝜋 cm2/cm 1 mark
21. ∫ sin−1 cos x dx
𝜋
Putting cos x = sin ( − x ) 1mark
2

𝜋
To get the result x – x2 / 2 + c 1 mark
2

22. For equation y = mx + c 1 mark


𝑑𝑦
=m & for second derivative = 0 1 mark
𝑑𝑥
23. For writing 𝑎⃗ =2i +3j+4k and 𝑏 ⃗⃗ = 4𝑖̂ + 𝑗 − 2𝑘̂ 1 mark
For finding P.V.ofmid point= 3i+2j+k 1 mark
24. For writing P(AUB)= P(A)+p(B)-P(A∩B) 1 mark
2
For getting correct answer P(A∩B) = 1 mark
5

1  a 2  b2 2ab 2b
2ab 1 a  b
2 2
2a
13.
2b 2a 1  a 2  b2
R1  R1  bR2 , R2  R2  aR3

………………………………(1+1/2)

1  a 2  b2 0 b 1  a 2  b 2 
0 1  a 2  b2 a 1  a 2  b 2 
2b 2a 1  a 2  b2

1 0 b
1  a 2
b 
2 2
0 1 a ………………………………(1+1/2)
2b 2a 1  a 2  b 2

1  a  b  1  a
2 2 2 2
 b2  2a 2  2b2 
………………………………..(1)
 1  a  b 
2 2 3

14. For finding: LHL=2 (1 mark)

For finding: RHL=2 (1 mark)

For finding:f (0) =2 (1 mark)

LHL=RHL= f(0) , therefor function is continuous at x=0 (1 mark)

OR
To find LHL= lim− 𝑓(𝑥) = 8
𝑥→0
To find RHL= lim+ 𝑓(𝑥) = 8
𝑥→0
Value = a =8
𝑑𝑦 𝑚𝑦
15. =-
𝑑𝑥 √1−𝑥 2

𝑑𝑦 2𝑥
√1−𝑥 2 ∗(−𝑚) −(−𝑚𝑦)
𝑑2𝑦 𝑑𝑥 2√1−𝑥2
= 2
𝑑𝑥 2 √1−𝑥 2

𝑑2𝑦 𝑑𝑦
(1-𝑥 2 ) -x = m2y.
𝑑𝑥 2 𝑑𝑥

16.

 𝑓 ′(𝑥) = −6(𝑥 + 1)(𝑥 + 2)  1


 Finding critical points 𝑥 = −1, −2  1
 Finding intervals for increasing and decreasing functions  1
 Writing answer: Increasing in (−2, −1), Decreasing in (−∞, −2) ∪ (−1, ∞)  1

OR

 Finding the point (7,0)


 1
𝑑𝑦
 Finding 𝑑𝑥
 1
𝑑𝑦 1
 Finding 𝑑𝑥 at (7,0) = 20 
1
2

 Slope of normal −20 


1
2

 Finding equation of normal:20𝑥 + 𝑦 = 140


 1

17.

Let r and h be the radius and height of the cone respectively inscribed in a sphere of radius R.

1
mark

Let V be the volume of the cone.


Then,

Height of the cone is given by,

h = R + AB 1mark
1
mark

2
marks

∴ By second derivative test, the volume of the cone is the maximum when

1
mark
1
mark

𝑋4 𝑥 4 − 1+1
18. ∫ (𝑥−1)(𝑥2 +1 ) 𝑑𝑥 = ∫ (𝑥−1)(𝑥2 +1 ) 𝑑𝑥

𝑥4− 1 1
= ∫ (𝑥−1)(𝑥2 +1 ) 𝑑𝑥 + ∫ (𝑥−1)(𝑥2 +1 ) 𝑑𝑥

1
= ∫(𝑥 + 1 )𝑑𝑥 + ∫ (𝑥−1)(𝑥2 +1 ) 𝑑𝑥

𝑥2 1 𝑑𝑥 1 𝑥+1
= + 𝑥 + ∫ − ∫ 𝑑𝑥
2 2 𝑥−1 2 (𝑥 2 +1 )

𝑥2 1 1 1
= + 𝑥 + 𝑙𝑜𝑔|𝑥 − 1| − log|(𝑥 2 + 1 )| - tan−1 𝑥 + 𝑐
2 2 4 2
19

𝑑𝑥
Writing the equation in the form: 𝑑𝑦 + 𝑃(𝑦). 𝑥 = 𝑄(𝑦) 𝑖. 𝑒.
𝑑𝑥 𝑥
+ 1+𝑦2 =  1
𝑑𝑦
tan−1 𝑦
1+𝑦 2
……… (1)

 Writing 𝑃(𝑦) = 1+𝑦2 , 𝑄(𝑦) =


1 tan−1 𝑦  1/2
1+𝑦 2

−1 𝑦
 Integrating factor = 𝑒 tan  1

 Multiplying integrating factor in (1) and changing the equation in form:  1/2

−1 𝑦 tan−1 𝑦 −1 𝑦
𝑒 tan . 𝑥 = ∫ 𝑒 tan
𝑑𝑦 .
1 + 𝑦2
−1
 Finding the general solution: 𝑥 = tan−1 𝑦 − 1 + 𝑐 𝑒 −tan 𝑦  1

OR
𝑥
𝑑𝑥 2𝑥𝑒 𝑦 −𝑦
 Writing D.E. in the form: 𝑑𝑦
= 𝑥 ………. (1)
2𝑦.𝑒 𝑦
 1
 Putting 𝑥 = 𝑣𝑦 𝑎𝑛𝑑
𝑑𝑥
=𝑣+
𝑑𝑣
𝑦 𝑑𝑦 ………….. (2)  1
𝑑𝑦
and converting D.E. in the form variable separable as
 1
𝑑𝑦
2𝑒 𝑣 𝑑𝑣 = −
𝑦

 Integrating both sides and getting general solution


𝑥
2. 𝑒 𝑦 = − log|𝑦| + 𝐶  1

⃗⃗⃗⃗1 +𝑏
20. . Let 𝑎⃗=𝑏 ⃗⃗⃗⃗⃗2

⃗⃗⃗⃗
𝑏1 =𝛽(3𝑖̂+𝑘̂)

⃗⃗⃗⃗⃗
𝑏2 =𝑥̂i+𝑦̂j+𝑧̂ k Marks 1

𝐹raming equation 3x+z=0 Marks 1

Ans𝛽=2 x=-1,y=-2,z=3 Marks 2

21. ⃗⃗⃗⃗⃗ 𝑎1 = 𝑖̂ − 𝑘̂
𝑎2 − ⃗⃗⃗⃗⃗ (1 mark)

⃗⃗⃗⃗⃗⃗⃗⃗⃗⃗⃗⃗⃗⃗⃗⃗
𝑏1 𝑋𝑏⃗⃗⃗⃗⃗2 = 3𝑖̂ − 𝑗̂ − 7𝑘̂ (1 mark)

⃗⃗⃗⃗1 𝑋𝑏
|𝑏 ⃗⃗⃗⃗⃗2 | = √59 (1 mark)
10
Shortest distance between the line:√59 (1 mark)

22. (i) They contradict each other in stating the same fact = 38% ( 1.5 mark)
(ii) They agree each other in stating the same fact= 0.62% (1.5 marks)

For value (1 mark)

23. Consider A1 : suffers from TB, P  A1   0.001 ------------------------------------(1)

A2 : person donot suffer fromTB, P  A2   0.999        (1)

C: Doctor diagnoses correctly

Then P(C / A1 )  0.99 and P(C / A2 )  0.001        (1)

P( A1 ).P(C / A1 ) 110
By Baye ’s theorem P( A1 / C )          (1)
P( A1 ).P(C / A1 )  P( A2 ).P(C / A2 ) 221

24. For proving one-one (2 Marks)

For proving onto (2 Marks)

For finding inverse (2 Marks)

OR

5 4 3 𝑥 11000
25. [4 3 5] [𝑦] = [10700]
1 1 1 𝑧 2700
5x+4y+3z=11000

4x+3y+5z=10700

x +y + z=2700 (1marks)

for getting |A|= -3 (1 marks)

−2 −1 11
−1
getting A-1= 3 [ 1 2 −13] (2 marks)
1 −1 −1
getting x=1000,y=900 and z=800 (2 marks)

OR

For Commutative (2 marks)

For Associative (2 marks)

For finding Identity (1 mark)

For finding Inverse (1 mark)

25.
𝑥 𝑥2 1 𝑥 𝑥2 𝑝𝑥 3
|𝑦 𝑦2 1|+|𝑦 𝑦2 𝑝𝑦 3 | Mark 2
𝑧 𝑧2 1 𝑧 𝑧2 𝑝𝑧 3

Similarly 2 marks for using each properties

Getting proof (Marks 2)

26. Drawing correct rough sketch and obtaining point of intersection of given

lines ( 2 marks)
2 3 3 𝑥−1
𝑎𝑟𝑒𝑎 𝑜𝑓 ∆𝐴𝐵𝐶 = ∫1 2(𝑥 − 1)𝑑𝑥 + ∫2 (4 − 𝑥)𝑑𝑥 − ∫1 𝑑𝑥 (2 marks)
2

3
𝑎𝑟𝑒𝑎 𝑜𝑓 ∆𝐴𝐵𝐶 = 2 (2 marks)


2
x
27. I   dx
0
sin x  cos x


 2
1
2  sin x  cos x
2I  dx ……………………………….(1)
0


 2
1
2
2I  dx …………………………(1)
2 1 1
0 sin x  cos x
2 2

 2
  
2 
2I  cos ec  x   dx ………………………….(2)
2 0  4


I
2 2
log  2 1  …………………………………………(2)

OR

By definition of limit sum we have

  3x  1dx  lim h  f 1  f 1  h   f 1  2h       f 1   n  1 h  


2
h 0
1

where nh  1              (1)

3 1  1  1     ntimes   1  1      ntimes  
 
= lim h 
h 0 

 3 h 2  22 h 2        n  1 h 2
2

      (1)


  3.2  h  2h        n  1 h  

 n  n  1 2n  1 n(n  1) 
= lim h  2n  3h   6h    6            (2)
2
h0
 6 2 
28. Equation of plane passing through (-1, 3,2) is A(x+1)+B(y-3) +C(z-2)=0 (1 mark)

Finding (A, B,C) = (-7, 8 -3) (2 marks)

Getting equation of plane: 7x-8y+3z+25=0 (2 marks)


7
Finding angle with x axis 𝑐𝑜𝑠 −1 √122 (1 mark)

29. Let x be the no. of fans and y be the no. of sewing machines

If p be the total profit ,

P= 22x+18y

360x+240y≤5760. For correct graph, ---------- (1)

i.e., 3x+2y≤48.

x+y≤20.

x≥0,y≥0 -------------------------------- (2)

E(0,24)

(0,20)D

C(8,12)

B O (16,0)A (20,0)

po int s valueof p
(0, 0) 0
(16, 0) 352
(8,12) 392
(0, 20) 360

the dealer gets a maximum profit of Rs. 392 when he purchase and sells 8 fans and 12 sewing machines.

Investment in fans =360.×8=2880.

Investment in sewing machines =240×12= 2880.-------------------(3)


SAMPLE PAPER
CLASS: XII

MATHEMATICS Max.Marks:100

Time Allowed: 3 hours

General Instructions:
1. All questions are compulsory.

2. This question paper consists of 29 questions divided into Four sections A, B, C andD. Section A comprises of
4 questions of one mark each, section B comprises of 8 questions of 02 marks each and section C comprises of
11 questions of 04 marks each and section D consists of 06 questions of 06 marks each.

3. All questions in Section A are to be answered in one word, one sentence or as per

the exact requirement of the question.

4. There is no overall choice. However, internal choice has been provided in 03

Questions of four marks each and 02 questions of six marks each. You have to

attempt only one of the alternatives in all such questions.

5. Use of calculators is not permitted. You may ask for logarithmic tables, if

required.

________________________________________________________________________

SECTION - A

30. Let * be a binary operation on Q given by a*b=a+ab where a,b  Q .Is * commutative?
1  4 
31. Evaluate Sin  cos 1   
2  5 
32. Given that A,B are two symmetric matrices such that AB =BA .Is AB symmetric?
 x  y  z   9
   
33. Find the values of x,y and z if  x  z    5 
 y  z   7
   
SECTION - B
Sin30 0 Cos30 0
34. Evaluate
 sin 60 0 Cos60 0
35. Find a point on the curve y=x2-4x+5 where the tangent to the curve is parallel to the x axis
x  cos 6 x
36. Evaluate  3x 2
 Sin6 x
dx
      
37. Find the projection of the vector a  i  3 k on the vector b  3 i  j  4 k
     
38. If a is a unit vector and ( x  a )( x  a )  15 find x

2x  1 y2 z3
39. The Cartesian equation of a line AB is   .Find the direction cosines of a line
3 2 3
parallel to AB

40. Let f: R  R be defined by f(x) = 3x+2.Show that f is invertible. Also find f  1 :R  R


 1 
41. Write in the simplest form : tan  1   where x  1

 x 2
 1 
SECTION - C

x 6 1
42. Show that x = 2 is a root of the equation 2  3 x x  3  0 and solve it completely.
 3 2x x2
 1  Cos2 x x0

 x2
43. Discuss the continuity of the function at x = 0: f ( x)  


5 x0

 
44. Verify Rolle’s theorem for the function f(x)= Sin2x in 0, 
 2
dy 1  y2
45. If 1  x 2  1  y 2  a( x  y ) prove that 
dx 1  x2

OR

If y=x2 +4 and x changes from 2 to 2.1 find the approximate change in y

2
46. Evaluate  e x dx as a limit of sum
0
OR

5
Evaluate   x  1x 2
4 dx
dy
47. Solve the differential equation Cos 2 x  y  tan x
dx
48. Find the differential equation for the family of circles which passes through the origin and have their
centre on the x axis
   
49. Express the vector a  5 i  2 j  5 k as the sum of two vectors such that one is parallel to the vector
   
b  3 i  k and the other is perpendicular to b
Or

       
If a  b  c  0 and a  3 ; b  5 c  7 then show that the angle between a and b is 600


50. Find the vector equation of the plane passing through the intersection of the planes r .
      
( 2 i  7 j  4 k )  3 and r . ( 3 i  5 j  4 k )  11  0 and passing through the point (-2,1,3)
51. A coin is tossed three times and all the possible outcomes are assumed to be equally likely. Let E and F
be two events given by E: “both tail and head have occurred” F: “at most one tail has occurred”. Show
that E and F are independent.

52. The sum of three numbers is 6.Twice the third number when added to the first number gives 7.On
adding the sum of the second and third numbers to thrice of the first number we get 12.Find the
numbers using inverse of a matrix.
Or

 1 2 3 

Using elementary transformations find the inverse of the matrix 2 5 7 

  2  4  5

SECTION - D

53. Let R be a relation on N  N defined by (a, b)R (c, d)  ad=bc, for all (a,b) and (c,d)  N . Show that R
is an equivalence relation .
OR

Let * be the binary operation defined on QxQ by (a,b)*(c,d) = (ac,b+ad) where Q is the set of

rational numbers. Determine whether * is commutative and associative . Find the indentiy

element for * and the inverttible elements of QxQ

54. An open box with a square base is to be made out of a given quantity of metal sheet of area c 2.Show
c3
that the maximum volume of the box is
6 3
1
55. Evaluate  Sinx(2  Cosx )dx
OR

2
Evaluate  log Sinxdx
0
56. Find the area of region included between the parabolas y2=4ax and x2=4ay where a>0
57. Find the foot of the perpendicular drawn from the point A(1,0,3) to the line joining the points B(4,7,1)
C(3,5,3)
58. Mona wants to invest at most Rs 12000 in Savings Certificates (SC) and National Saving Bond (NSB). She
has to invest at least Rs 2000 in SC and at least Rs 4000 in NSB. If the rate of interest on Sc is 8% and the
rate of interest on NSB is 10% per annum, how much money should she invest to earn maximum yearly
income?

MARKING SCHEME

Q.No Value points Marks Total


Marks

1 Not Commutative (as a*b=a+ab  b+ba=b*a) 1M 1M

2 1 4 4  1M 1M
(put cos  1 =   cos  = Given expression =Sin =
10 5 5 2
4
1
1  Cos 5= 1
=
2 2 10

3 AB is Symmetric. (AB) t =B t A t =BA=AB 1M 1M

4 x=2 ; y= 4 ; z= 3 1M 1M

( by equating corresponding entries we get x+y+z = 9 ; x+z =5 ; y+z


= 7)

5 Sin30 0 Cos 60 0 +Cos30 0 Sin60 0 1M 2M

=Sin 900=1 IM

6 dy 1M 2M
(2,1) is the point .  2 x  4 =0  x  2 putting x =2 in y=x2-
dx
4x+5 we get y = 1

7 1 1 1M 2M
6 t
Put 3 x 2  Sin6 x = u  Given integral = dt =
1
log 3 x 2  Sin6 x  c
6

8     1M 2M
projection of the vector a  i  3 k on the vector b 
 
   a .b
3 i  j 4 k =

b

3  0  12 15
= =
9  1  16 26
1M

9    1M 2M
x 2
-a 2
=15  x 2


=15+1  x = 4 1M

10 1 1M 2M
x
2  y  2  z  3 dr’s of AB are 3 ,4,6
3 4 6

3 4 6
Hence dc’s of AB are , ,
55 55 55
1M

11 f ( x1 )  f ( x2 )  3 x1  2  3 x2  2  x 1 =x 2 Thus f is one 1M
to one

y2 y2 1M
y=3x+2  x= Given any y  R there exists =x  R
3 3
s.t. f(x)=y showing that the function is onto.
2
1 x2
f (x)= is the inverse of f
3

12 x=Cosec    =Cosec 1 x

  1  1  1M
tan  1   = tan 1 
 


 x 1  Co sec   1 
2 2

2
  1M
= tan 1  1  = tan 1  1  = tan 1 Tan = 
   Cot 
 Cot  
2
=Cosec 1 x

13 x  2 3x  6  x  2 1M
5  5x  5  0 by performing R1  R1  R2 and
3 2x x2
R2  R2  R3

1 3 1 1M
5(x-2) 1  x 1  0
 3 2x x2

0 3 1 1M
5(x-2) 0 x  1  0 C1  C1  C 3 4
x  1 2x x2

0 3 1 ½M
5(x-2)(x-1) 0  x 1  0
1 2x x2

= -5(x-2)(x-1)(x+3) = 0  x  2,1,3 ½M

14 2 Sin 2 x
Here f(0) = 5 and Lt f(x) = Lt =2
x 0 x 0 x2

Sinx Sinx
2 Lt Lt =2  1 1 =2
x 0 x x 0 x

As Lt f(x)  = f(0) ,f is not continuous


x 0

15   ½M
Consider the function f(x) = Sin2x in 0,  . Here f is
 2
 
continuous in 0,  as f(x)= Sin x is continuous
 2

  1M
f ' (x) = 2 Cos2x exists in ( 0, ) thus f is differentiable on ( 0, )
2 2

  1M
f(0)=Sin (0) = 0 and f   =Sin  = 0
 2

Conditions for Rolle’sthm are satisfied. Hence there should be at ½M


 
least one c   0,  such that f (c ) = 0
'
4
 2

   1M
Let 2Cos2c = 0  c =   0, 
4  2
16 Let x=Sin A and y=Sin B ½M

Cos A + Cos B =a(Sin A –Sin B) ½M

A B A B A B A B 1M
2Cos Cos =a  2Cos Sin
2 2 2 2 4
A B 1M
Cot =a  Sin-1x-Sin-1y=2Cot-1a
2

1 1 ½M
Differentiating , - y ' =0
1  x2 1  y2

½M
dy 1  y2

dx 1  x2

OR

x= 2 x  0.1 ½M

y  f ' ( x )x =2x x =2  2 0.1 =0.4 1½M

When x=2 y = 22+4=8 ½M 4

y+  y =8+0.4=8.4 1M

y changes from 8 to 8.4 ½M

17. ba 2 ½M
a=0 b=2 h=  nh=2 f(x)=ex
n n
2 1M
 e dx =
x

Lt h  f (0)  f (0  h)  f (0  2h)  ......  f (0  (n  1)h)


h 0

h 0

= Lt h 1  e  e  e  .....  e
h 2h 3h  n 1 h
 ½M

Lt h.

1. e h  1 
n
 1M
4
h 0 eh  1

= Lt
h

e 2  1 =e 2 -1  1M
h 0 e 1
h

OR

5 A Bx  C 1M
 2
 =

 x  1 x  4 x  1 x  4
2

To get A =1 B = -1 and C = 1 1M
5 1  x 1 ½M
  x  1 x 2  4 dx = 
  x1
dx  
x2  4
dx

x 1 1M
=log x  1 - x 2
4
dx   2
x 4
dx
4
1 1 x ½M
=log x  1 - log x 2  4  tan 1  c
2 2 2

18 dy 1M
Dividing Cos 2 x  y  tan x by Cos2x to get
dx
dy
 ySec 2 x  tan xSec 2 x
dx

I.F. =etanx ½M

4
Solution is y(IF)=  tan xSec 2 x etanxdx+ k 1M

 Put tan x =u  y etanx=  ue u du  k ½M

 y etanx=e u ( u  1)  k ½M

 y =tanx-1+ke  tan x ½M

19 1M

Let a be the radius Then centre is


(a,0)

= n to the Circle is (x-a) 2 +y 2 =a 2  x  y  2ax  0 ….(*) 1M


2 2

Differentiating ,2x+2yy’-2a=0  a=x+yy ' =a 1M 4

Putting this value of a in (*) we get 2xyy ' -y 2  x 2 =0 1M

20   ½M
Any vector parallel to b is of the form k b for some scalar k

     1M
Let a = k b + c where c is perpendicular to b

    ½M
Then c =( a - k b )  b
1M
      
 ( a - k b ). b  ( a . b )-k( b . b )=0  k=2 4
    ½M
k b =2 b = 6 i  2 k

      ½M
c =( a - 2 b )=(  i  2 j  k )

OR

      ½M
a b c  0  a b   c
   ½M
 ( a  b ) 2  ( c ) 2

      ½M
 ( a  b ).( a  b )  (  c ).(  c )

     1M
2 2 2
a +b +2 a . b = c

  ½M
15
a .b =
2
  1M
a .b 15 1
Cos  = = =    60 0
 
2 3 5 2 4
a b

21     1½M
r . { ( 2  3 ) i  ( 7  5 ) j  (4  4 ) k }  3  11  0 …(*)

4
   1½M
(*) Passes through the point with position vector ( 2 i  j  3 k )
1
 
6
 1M
Putting the value of  in (*) we get r .
  
(15 i  47 j  28 k )  7  0

22. E: “both tail and head have occurred” ½M+½M


={HTT,THT,TTH,HHT,HTH,THH}

F: “at most one tail has occurred”= {HHH,HHT,THH,HTH}

E  F ={HHT,HTH,THH} ½M

6 4 3 1½M
P(E) = P(F)= P( E  F )= 4
8 8 8
3 6 4 1M
As P( E  F )=P(E) P(F)    is true ,we conclude that E
8 8 8
and F are independent.

23 Let the first, second and third numbers be x, y and z respectively. 1M


Then x + y + z=6 : x + 2z =7 ; 3x + y + z=12

 1 1 1  x 6 ½M
     
 AX=B where A=  1 0 2  X=  y  B=  7 
 3 1 1 z  12 
     

A  4  0  A is invertible ½M

 2 0 2  1M
  4
adjA=  5  2  1 
 1 2  1 

 2 0 2   6   3 1M
1     
X=A  1 B=  5  2  1  7  =  1 
A  1
 2  1   12   2 

The required numbers are 3 ,1 and 2 ½M

OR

 1 2 3  ½M

Let A= 2 5 7 

  2  4  5

 1 2 3  1 0 0 1M

A=IA  2 5 7  = 0 1 0 A

  2  4  5 0 0 1 6

1 0 0
 
By performing elementary transformations ,to get 0 1 0 =
 
0 0 1
 3  2  1
  4 1  1 A 3½M
 
 2 0 1 

 3  2  1 1M
 A =   4 1  1
1

 2 0 1 
24

(a,b) R (a,b)  ab=ba Reflexive 1.5M

(a,b) R (c,d)  ad=bc  cb=da  (c,d) R (a,b) symmetric 1.5M 6

(a,b) R (c,d) and (c,d) R (e,f)  ad=bc and cf=de  ad.cf=bc.de 2½ M


 af=be  (a,b) R (e,f) Transitive

Thus R is an equivalence relation ½M

25 Let x be the side of the square base and y be the height. 1½M

c2  x2
V=x2y ,Surface area S= x2+4xy=c2  y  4 x

 d 2V  3

1 dV 1 2 1½M
V= (c 2 x  x 3 )  c  3x2  x
4 dx 4 dx 2 2

dV c d 2V   3c 1M
0 x  2 
 0
dx 3 dx  x  c 2 3
3

c c 1M
V is Maximum when x  . Then y 
3 2 3 6

c3 1M
V Max =
6 3

26 Sinx ½M
 Sin x( 2  Cosx )
2
dx by multiplying the Nr and Dr by Sinx

Sinx 1 1M
=  (1  Cos 2 x)(2  Cosx)dx =  (1  t 2 )( 2  t )dx by letting
Cosx=t

1 ½M
=  (1  t )(1  t )( 2  t )
dx

1 A B C 1M
=  
(1  t )(1  t )( 2  t ) 1  t 1  t 2  t

1 1 1 1½M
To get A= B= C=
6 2 3 6
1 1M
 (1  t )(1  t )( 2  t )
dx =
1 1 1
log 1  t  log 1  t  log 2  t  k
6 2 3

1
 sin x(2  Cosx )dx =

1 1 1
 log 1  Cosx  log 1  Cosx  log 2  Cosx  c
6 2 3
½M

OR

   1M
2 2   2
I=  log Sinxdx =  log Sin  x dx =  log Cosxdx
0 0 2  0

  1M
2 2 2 SinxCosx
2I=  log SinxCosxdx =  log dx
0 0 2

 2M
2 2 SinxCosx
=  log dx =
0 2 6

2  
 log Sin2 xdx  log 2  I 1  log 2 ………(*)
0 2 2

 1M
1 1 2
I1 =  log S int dt =  2  log Sinxdx =I
2 0 2 0

  1M
(*)  2I=I- log 2  I= log 2
2 2

27 2M

Solving the two equations given we get x=0 and x=4a 1M


4a 4a x 2 2M
Required area =  4ax dx   dx
0 0 4a

1M 6
32a 2 16a 2 16a 2
=   sq.units
3 3 3

28 Let P be the foot of the perpendicular from A on BC. If P divides BC ½M


 3k  4 5k  7 3k  1 
in the ratio k:1 then P is  , , 
 k 1 k 1 k 1 
1M

Dr’s of BC are 1,2,-2 ½M

3k  4 5k  7 3k  1 1M
Dr’s of AP are  1,  0, 3
k 1 k 1 k 1

7 2M
Since AP  BC ,dot product =0  k 
4

 5 7 17  1M
So the foot of the perpendicular is  , ,  6
3 3 3 

29 Suppose that she invests Rs x in SC and Rs y in NSB ½M

8 x 10 y 1½M
Then LPP is to maximize Z=  subjecting to constraints x
100 100
 2000 y  4000 x+y  12000

2½M

(To draw the


graph, to identify the feasible region and to get coordinates)

8 x 10 y 1M
To put the coordinates of the vertices in Z=  to get
100 100
values Rs 560, Rs 1040 Rs 1160 6

Rs 2000 should be invested in in savings certificates and Rs 10000 ½M


in National Savings Bonds to get a maximum yearly income of Rs
1160
KENDRIYA VIDYALAYA SANGATHAN
MODEL QUESTION PAPER-I (Based on New Pattern)
CLASS XII: MATHEMATICS
BLUE PRINT

S.No. Topics 1 Mark 2 Marks 4 Marks 6 Marks TOTAL

g) RELATIONS AND FUNCTIONS - - - 1


1 10
h) INVERSE TRIGONOMETRIC
- - 1 -
FUNCTIONS
g) MATRICES 1 1 1 -
2 13
h) DETERMINANTS - 1 1 -
p) CONTINUITY &
- 1 2 -
DIFFERENTIABILITY
q) APPLICATION OF
- 2 - 1
DERIVATIVES
3 44
r) INTEGRATION 1 2 2 -
s) APPLICATION OF INTEGRALS - - - 1
t) DIFFERENTIAL EQUATIONS 1 - 1 -
g) VECTORS 1 1 1 -
4 17
h) 3-DIMENTIONAL GEOMETRY - - 1 1
5 LINEAR PROGRMMING - - - 1 6
6 PROBABILITY - - 1 1 10
TOTAL 4X1 = 4 8X2 = 16 11X4 = 44 6X6=36 100(29)

Prepared by:
Mr.Vinod Kumar, PGT (Maths)
K.V.No.5 (I-Shift), Mansarovar, Jaipur
KENDRIYA VIDYALAYA SANGATHAN
MODEL QUESTION PAPER-I (Based on New Pattern)
CLASS: XII, MATHEMATICS

Time: 3 hrs Max.Marks:100

General Instructions:
6. All questions are compulsory
7. This question paper consists of 29 questions divided into three sections A, B and C and D. Section
A comprises of 4 questions of one mark each, section B comprises of 8 questions of two marks
each, section C comprises of 11 questions of four marks each and section D comprises 6
questions of six marks each.
8. All questions in Section A are to be answered in one word, one sentence or as per the exact
requirement of the question
9. There is no overall choice. However, internal choice has been provided in 03 questions of
four marks each and 03 questions of six marks each. You have to attempt only one of the
alternatives in all such questions.
10. Use of calculators is not permitted. You may ask for logarithmic tables, if required.

SECTION – A
1 2 3 1 7 11
1. If [ ][ ] =[ ], then find the value of k.
3 4 2 5 𝑘 23

1
2. Evaluate ∫−1 𝑠𝑖𝑛5 𝑥 𝑐𝑜𝑠 4 𝑥 𝑑𝑥

3. Find the order and degree of the differential equation:


2 2
 dy   dy 
y  x    a 1  
 dx   dx 
4. Find the area of a parallelogram whose adjacent sides are î − ĵ + k̂ and3î + 4ĵ −
5k̂.
SECTION – B
1
5. If A is a 3 x 3 matrix, whose elements are given by 𝑎𝑖𝑗 = |-3i + j| then write the
3
value ofa23 + a31.

6. If area of triangle is 35 square units with vertices (2, -6), (5, 4) and (k, 4). Then find
the value of k.
dy
7. Find when x = acos3t, y = bsin3t
dx
𝑥−1
8. Find the slope of tangent to the curve y = , x ≠ 2 at x = 10.
𝑥−2
9. Find the rate of change of the area of a circle with respect to its radius ‘r’. When r =
4 cm.
10. Find the integral of  x 2e x cos(e x )dx
3 3

𝜋
3+5 cos 𝑥
11. Solve the integral: ∫02 𝑙𝑜𝑔 [ ] 𝑑𝑥
3+5 sin 𝑥

     
12. If a and b are two vectors such that a.b  a  b , then find the angle between a and

b.

SECTION – C
5 12
13. Simplify 𝑠𝑖𝑛−1 ( 𝑐𝑜𝑠𝑥 + 𝑠𝑖𝑛𝑥 ).
13 13

2 3 4 4 0 5
14. If A=[ 5 7 9], B=[1 2 0], verify that(𝐴𝐵)𝑇 = 𝐵𝑇 𝐴𝑇 .
−2 1 1 0 3 1
a2 bc ac  c 2

15. Using properties of determinants, Prove that: a 2  ab b 2 ac  4 a 2b 2 c 2

ab b 2  bc c2
16. If .

17. If .

1 x
18. Evaluate:  tan 1 dx
1 x

𝜋 𝑥 𝑡𝑎𝑛 𝑥
19. 𝐸𝑣𝑎𝑙𝑢𝑎𝑡𝑒: ∫0 𝑑𝑥
𝑠𝑒𝑐 𝑥+𝑡𝑎𝑛 𝑥

20.
    dydx  0
1
Solve : 1  y 2  x  e tan y

OR

Solve the D.E.( x2  y 2 )dx  2 xydy  0; given that y  1 when x  1.

21. If𝛼⃗ = 3𝑖̂ + 4𝑗̂ + 5𝑘̂ and𝛽⃗ = 2𝑖̂ + 𝑗̂ − 4𝑘̂, then express 𝛽⃗in the form of𝛽⃗ = ⃗⃗⃗⃗⃗
𝛽1 +
⃗⃗⃗⃗⃗
𝛽2 , where ⃗⃗⃗⃗⃗
𝛽1 is parallel to 𝛼⃗ and ⃗⃗⃗⃗⃗
𝛽2 is perpendicular to𝛼⃗.
OR
2 2 2 2
For any vector a , prove that a  iˆ  a  ˆj  a  kˆ  2 a .
22. Find the distance of the point (1, -2, 3) from the plane x – y + z = 5 measured
x 1 y  3 z  2
parallel to the line   .
2 3 6

OR

x  2 2 y  3 3z  4
Find the distance of the point (– 2, 3, – 4) from the line  
3 4 5
measured parallel to the plane 4𝑥 + 12𝑦 − 3𝑧 + 1 = 0.

23. An insurance company insured 2000 scooter drivers, 4000 car drivers and 6000
truck drivers. The probability of a scooter, a car and a truck meeting an accident
are 0.01, 0.03 and 0.15 respectively. If one of the insured person meets with an
accident, find the probability that he is a scooter driver. What moral value can you
get from the question?
SECTION – D

24. Let R0 denote the set of all non-zero real numbers and let A = R0 x R0. If * is a
binary operation on A defined by: (a,b)*(c,d) = (ac,bd) for all (a,b), (c,d) A.
a) Show that * is both commutative and associative on A.
b) Find the identity element in A.
c) Find the invertible element in A.

25. Show that height of the cylinder of greatest volume which can be inscribed in a
right circular cone of height h and semi vertical angle α is one third that of the
4
cone and the greatest volume of cylinder is 𝜋ℎ3 𝑡𝑎𝑛2 𝛼.
27

OR

Show that the semi vertical angle of the cone of the maximum volume & of given
slant height is𝑡𝑎𝑛−1 √2 .

26. Using integration, find the area of the following region:


{(𝑥, 𝑦): |𝑥 + 2| ≤ 𝑦 ≤ √20 − 𝑥 2 }.

OR
𝑥2 𝑦2
Find the area of the smaller region bounded by the ellipse 2
+ = 1 and the
𝑎 𝑏2
𝑥 𝑦
straight line + = 1.
𝑎 𝑏
x 1 y  3 z  4
27. Find image of the line   in the plane 2x – y + z + 3 = 0.
3 1 5

OR

A line with direction ratios <2, 7, –5> is drawn to intersect the lines.

x 5 y7 z  2 x 3 y 3 z 6
   
3 1 1 , 3 2 4

Find the co-ordinates of the points of intersection and the length intercepted on it.

28. A toy company manufactures two types of dolls, A and B. Market tests and
available resources have indicated that the combined production level should not
exceed 1200 dolls per week and the demand for dolls of type B is at most half of
that for dolls of type A. Further, the production level of dolls of type A can exceed
three times the production of dolls of other type by at most 600 units. If the
company makes profit of Rs. 12 and Rs. 16 per dolls respectively on dolls A and B,
how many of each should be produced weekly in order to maximize the profit.

29. Find the probability distribution of “sum of the numbers” in an experiment of


throwing two dice simultaneous. Also find the mean & variance for this
distribution.
ANSWER KEY WITH MARKING SCHEME

Q.No. Value points Marks Total


Marks
1 k = 17 1M 1M
2 I=0 1M 1M
3 Order = 1 1
2
M
Degree = 4 1 1M
2
M
4 Area = √114 Sq Units 1M 1M
5 1
a23= 1 2
M
a31= 83 1
2
M
2M
a23 + a31 = 11
3
1M
6 Writing 1 2 −6 1
2
|5 4 1| = ±35 1M
𝑘 4 1 2M
𝑘 = 12, −2 1M
7 𝑑𝑥 1
M
= −3𝑎𝑐𝑜𝑠 2 𝑡 𝑠𝑖𝑛𝑡 2
𝑑𝑡
𝑑𝑥 1
= 3𝑏𝑠𝑖𝑛2 𝑡 𝑐𝑜𝑠𝑡 2
M 2M
𝑑𝑡
−𝑏
𝑑𝑦/𝑑𝑥 = 𝑡𝑎𝑛𝑡 1M
𝑎
8 −1 1M
𝑑𝑦/𝑑𝑥 = 2M
(𝑥 − 2)2
Slope = −1 1M
64
1
9 𝐴 = 𝜋 𝑟2 M
𝑑𝐴 2
1
= 2𝜋𝑟 M
𝑑𝑟 2 2M
[𝑑𝐴/𝑑𝑟]𝑟=4𝑐𝑚 = 8𝜋𝑐𝑚2 /𝑠𝑒𝑐. 1M
10 Substituting 𝑒 𝑥 3 = 𝑡 1M
Getting 3
𝑠𝑖𝑛(𝑒 𝑥 )
2M
𝐼 = 3 +𝑐 1M
𝜋
11 2 𝜋
3 + 5 cos ( − 𝑥) 1M
𝐼 = ∫ 𝑙𝑜𝑔 [ 2 ] 𝑑𝑥
𝜋 2M
3 + 5 sin ( − 𝑥) 1M
0 2
𝜋
2
3 + 5 sin 𝑥
= ∫ 𝑙𝑜𝑔 [ ] 𝑑𝑥
3 + 5 cos 𝑥
0
Getting 𝐼 = 0
12 |𝑎⃗. 𝑏⃗⃗| = |𝑎⃗||𝑏⃗⃗|𝑐𝑜𝑠𝜃 ½M
½M 2M
|𝑎⃗ × 𝑏⃗⃗| = |𝑎⃗||𝑏⃗⃗|𝑠𝑖𝑛𝜃
𝑡𝑎𝑛𝜃 = 1, 𝜃 = 𝜋 1M
4
13 Assuming 𝑠𝑖𝑛𝛼 = 5 , cosα = 12 1 M
13 13
Getting 𝑡𝑎𝑛𝛼 = 5 1 M
12
𝑦 = sin−1(sin(𝛼 + 𝑥)) 1 M 4M
5
𝑦 = tan−1 ( ) + 𝑥 1 M
12
11 18 14
𝐴𝐵 = [ 27 41 34 ] 2M
−7 5 −9
11 27 −7
14
(𝐴𝐵)′ = [18 41 5 ]
14 34 −9 1M 4M
4 1 0 2 5 −2 11 27 −7
𝐵′ 𝐴′ = [0 2 3] [3 7 1 ] = [18 41 5 ] 2M
5 0 1 4 9 1 14 34 −9
15 Taking a,b and c out from c1,c2 and c3 respectively
a2 bc ac  c 2 a c ac
a 2  ab b 2 ac  abc a  b b a 1M

ab b 2  bc c2 b bc c

Operate R1–R2–R3

 2b  2b 0
1M
abc a  b b a
b bc c

Taking (–2b) out from R1

1 1 0
 2ab 2c a  b b a 4M

b bc c

Operate c2–c1 we get

1 0 0
1M
 2ab 2c a  b  a a
b c c

Expand by R1 1M

 2ab2c  ac  ac  4a 2b2c 2

16
Let + ………………..

Where = Differentiating (i) w.r.t. ½M


we get

………………..
=

Taking log on both sides, we have

= x
1½M 4M
= ……..

Also,

Taking log on both sides, we have

Differentiating w.r.t. We get

=
1½ M
= …..

By substituting the values from (iii) and (iv) in (ii), we get

=
½M

17 We have

Differentiating w.r.t. we get

1M

1M
½M

4M

½M
Squaring on both sides ,We have

½M
Differentiating again w.r.t. we get

½M

18 1 x
I   tan 1 dx
1 x
1  cos 2t 1M
  tan 1 (2sin 2tdt )
1  cos 2t
2sin 2 t
 (2)  tan 1 sin 2tdt
2 cos 2 t
 (2)  tan 1 (tan t ).sin 2tdt
 (2)  t.sin 2t.dt 1M 4M

   cos 2t  1 
 (2) t      cos 2tdt  1M
  2  2 
sin 2t 1 1  x2
 t cos 2t   c  x cos 1 x  c
2 2 2
1M
19 
x tan x
I  dx      (1)
0
(sec x  tan x)

(  x) tan(  x)
 dx 1M
0
(sec(  x)  tan(  x))

()(  x) tan x
 dx
0
()(sec x  tan x)

(  x) tan x
I  dx      (2) 1M
0
(sec x  tan x)
Adding (1) & (2) we get ,
 
tan x tan x (sec x  tan x)
2I    dx    dx
0
(sec x  tan x) 0
(sec x  tan x) (sec x  tan x) 4M

   (sec x.tan x  tan 2 x)dx
0
1M

 
 
2
2 I   [sec x]0   (sec x  1)dx 
 2

 0

  (2)  [tan x]0  [ x]0 
2 I   (2   )
 1M
I   (  1)
2
    dydx  0
20 1
Given D.E.: 1  y 2  x  e tan y

 x  e  dydx   1  y 
tan 1 y 2

dy

 1 y2  
dx  1
x  e tan y 
dx  x  e

tan
 1
y

dy 1 y2  
1
dx x e tan y
 
dy 1 y2 1 y2   
1
dx x e tan y
 
   
1M
dy 1  y 2 1 y2
dx
( It is of the form  p( y) x  q( y)
dy
1  pdy  e tan 1 y
 pdy   1  y  dy  tan y  I .F .  e
1 1M 4M
2

Gen.Sol : x( I .F )   q ( y )( I .F )dy  c
1 1
e tan y tan 1 y e tan y
x.e tan 1 y
 e dy  c Put v  e tan 1 y
; dv  dy 1M

1 y2  1 y2
v2
  vdv  c  c
2

e 
2
tan 1 y
1 1M
x.e tan y
 c
2
OR Given D.E..( x 2  y 2 )dx  2 xydy  0
dy y 2  x 2 dy dv
 put y  vx;  vx
dx 2 xy dx dx
dv v 2 x 2  x 2 dv v 2  1
vx  vx 
dx 2 x.vx dx 2v
dv v  12
dv v  1  2v 2
2
x  v  x 
dx 2v dx 2v
dv 1 v 2
2vdv dx
x    1M
dx 2v 1 v 2
x
Integrating both sides,
2vdv dx 1M 4M
 1  v2   x
log(1  v 2 )   log x  log c
log(1  v 2 ) x  log c
(1  v 2 ) x  c
( x2  y 2 ) x
c
x2
( x2  y 2 )  c x when x  1, y  1, we get c  2
1M
(x  y )  2 x
2 2

but x  1, y  1 do not satisfy ( x 2  y 2 )  2 x


 Particular solution : x 2  y 2  2 x
1M

⃗⃗⃗⃗⃗1 ∥ 𝛼
2221 Equation
𝛽 of
⃗ given line:
x  1𝛽
⃗⃗⃗⃗⃗1 =y 𝜆𝛼
3⃗ z  2
 
2𝛽 ⃗⃗⃗⃗⃗1 = 3𝜆(3𝑖̂ + 4𝑗̂ 6 ……………………….(i)
+ 5𝑘̂) 1M
⃗⃗⃗⃗⃗
Equation
𝛽2 = of 𝛽⃗ −
line⃗⃗⃗⃗⃗1passing
𝛽 = (2 −through the−given
3𝜆)𝑖̂ + (1 − (4 + 5𝜆)𝑘̂ &parallel to the given line:
4𝜆)𝑗̂ point(1,-2,3)
x  1⃗⃗⃗⃗⃗
𝛽2 ⊥y 𝛼⃗2  z  3
𝛽2 . 𝛼⃗ 3= 0 6 ……………………….(ii)
2 ⃗⃗⃗⃗⃗
1 1M
Finding 𝜆of=any
Coordinates − 5point on this line: 1M 4M
(2k+1, 3k-2,1-6k+3)
⃗⃗⃗⃗⃗1 = − (3𝑖̂ + 4𝑗̂ + 5𝑘̂)
𝛽
If this point 5lies on the given plane ½1M
M 4M
13 9 𝑥– 𝑦 + 𝑧 = 5
⃗⃗⃗⃗⃗
𝛽2 = 𝑖̂ + 𝑗̂ − 3𝑘̂
=>(2k+1)-(3k-2)+(-6k+3)=5
5 5 1M
Let 𝑎⃗ = 𝑎1 𝑖̂ + 𝑎2 𝑗̂ + 𝑎3 𝑘̂
OR =>k=1/7
Therefore = √𝑎2 2 + 𝑎3of2 point Q lying on the plane
|𝑎⃗ × 𝑖̂|coordinates ½M
(9/7,|𝑎⃗-11/7, ½1M
M
× 𝑗̂| =15/7)
√𝑎1 2 + 𝑎3 2
½M
|𝑎⃗ × 𝑘̂Distance
Required | = √𝑎1 2=+1 𝑎unit 2
2
4M
2
|𝑎⃗ × 𝑖̂|2 + |𝑎⃗ × 𝑗̂|2 + |𝑎⃗ × 𝑘̂ | ½M
= 2(𝑎1 2 + 𝑎2 2 + 𝑎3 2 ) 11M
M
= 2𝑎⃗ 1M
OR Equation of line AB
x  2 y  3/ 2 z  4 / 3
   r ( say ) ½M
3 2 5/3
Any point on the line AB is
3 5 4
(3r  2, 2r  , r  )
2 3 3 ½M
Let it be Q and PQ is parallel to the plane 4𝑥 + 12𝑦 − 3𝑧 + 1 = 0
9 5 8
Direction ratios of PQ are 3r , 2r  , r  4M
2 3 3 1M
As PQ is parallel to the plane, PQ is perpendicular to the normal to the plane.
9 5 8
4(3r )  12(2r  )  3( r  )  0
2 3 3
 r  2
The coordinates of Q are (4, 5/2, 2) 1M
PQ = 17/2 Units

1M
23 Let E1, E2, E3 and A be the events as
E1=The insured person is a scooter driver.
E2=The insured person is a car driver.
E3=The insured person is a truck driver.
A= Insured person meets with an accident.
1M
P(E1)= 1/6, P(E2)= 1/3, P(E3)= 1/2
P(A/ E1)=0.01, P(A/ E2)=0.03, P(A/ E3)=0.15
By bayes theorem 1M 4M
A
𝐸1 P (𝐸 ) × P(𝐸1 )
1
P( ) =
A A A A ½M
P (𝐸 ) × P(𝐸1 ) + P (𝐸 ) × P(𝐸2 ) + P (𝐸 ) × P(𝐸3 )
1 2 3
𝐸1 1 ½M
P( ) =
A 52
1M
Moral Value
24 1. (a)
Let (𝑎, 𝑏), (𝑐, 𝑑)𝜖𝐴
(𝑎, 𝑏) ∗ (𝑐, 𝑑) = (𝑎𝑐, 𝑏𝑑)
= (𝑐𝑎, 𝑑𝑏)
= (𝑐, 𝑑) ∗ (𝑎, 𝑏)
Therefore * is commutative. 1M
(b)
Let (𝑎, 𝑏), (𝑐, 𝑑), (𝑒, 𝑓) 𝜖𝐴
(𝑎, 𝑏) ∗ {(𝑐, 𝑑) ∗ (𝑒, 𝑓)} = (𝑎, 𝑏) ∗ (𝑐𝑒, 𝑑𝑓)
= (𝑎𝑐𝑒, 𝑏𝑑𝑓)…………..(i)
and
{(𝑎, 𝑏) ∗ (𝑐, 𝑑)} ∗ (𝑒, 𝑓) = (𝑎𝑐, 𝑏𝑑) ∗ (𝑒, 𝑓)
= (𝑎𝑐𝑒, 𝑏𝑑𝑓)…………..(ii)
from (i) and (ii)
(𝑎, 𝑏) ∗ {(𝑐, 𝑑) ∗ (𝑒, 𝑓)} = {(𝑎, 𝑏) ∗ (𝑐, 𝑑)} ∗ (𝑒, 𝑓)
Therefore * is associative. 1M 6M
2. Let (𝑥, 𝑦) be the identity element of * in A
so (𝑎, 𝑏) ∗ (𝑥, 𝑦) = (𝑎, 𝑏)
=> (𝑎𝑥, 𝑏𝑦) = (𝑎, 𝑏)
=> 𝑎𝑥 = 𝑎, 𝑏𝑦 = 𝑏
=> 𝑥 = 1, 𝑦 = 1
=> (1, 1) is the identity element of * in A 2M
3. Let (𝑥′, 𝑦′) be the inverse of an element (a, b) of * in A
so (𝑎, 𝑏) ∗ (𝑥′, 𝑦′) = (1, 1)
=> (𝑎𝑥′, 𝑏𝑦′) = (1, 1)
=> 𝑎𝑥′ = 1, 𝑏𝑦′ = 1
=> 𝑥′ = 1/𝑎, 𝑦′ = 1/𝑏
=> (1/𝑎, 1/𝑏) is the inverse of * for an element (a, b) of A
Since 𝑎 ≠ 0, 𝑏 ≠ 0 , therefore all element of A are invertible
2M
25 Let the radius and height of cylinder is R & H
𝑅 = 𝑥𝑡𝑎𝑛𝛼, 𝐻 = ℎ − 𝑥
Volume of cylinder 𝑉 = 𝜋𝑅 2 𝐻
𝑉 = 𝜋(𝑥𝑡𝑎𝑛𝛼)2 (ℎ − 𝑥)
𝑉 = 𝜋ℎ𝑥 2 𝑡𝑎𝑛2 𝛼 − 𝜋𝑥 3 𝑡𝑎𝑛2 𝛼
𝑑𝑉 2M
= 2𝜋ℎ𝑥𝑡𝑎𝑛2 𝛼 − 3𝜋𝑥 2 𝑡𝑎𝑛2 𝛼
𝑑𝑥
𝑑𝑉 2ℎ 2ℎ ℎ
=0 ⇒𝑥= ,𝐻 = ℎ − = 2M 6M
𝑑𝑥 3 3 3
𝑑2 𝑉
= −𝑖𝑣𝑒 verification 1M
𝑑𝑥 2
Maximum Volume 𝑉 = 𝜋𝑅 2 𝐻
4
𝑉= 𝜋ℎ3 𝑡𝑎𝑛2 𝛼
27
1M
OR Let α be the semi vertical angle of a cone of given slant height ‘𝑙’
Also let height and radius of cone be ‘ℎ’ and ‘𝑟’.
Then ℎ = 𝑙𝑐𝑜𝑠𝛼, 𝑟 = 𝑙𝑠𝑖𝑛𝛼 1M
1
Then volume of cone 𝑉 = 3 𝜋𝑟 2 ℎ
1
=> 𝑉 = 𝜋(𝑙𝑠𝑖𝑛𝛼)2 (𝑙𝑐𝑜𝑠𝛼)
3
1 1M
=> 𝑉 = 𝜋𝑙 3 𝑠𝑖𝑛2 𝛼𝑐𝑜𝑠𝛼
3
𝑑𝑣 1 3 1M 6M
=> = 𝜋𝑙 (−3𝑠𝑖𝑛3 𝛼 + 2𝑠𝑖𝑛𝛼𝑐𝑜𝑠 2 𝛼)
𝑑𝛼 3
𝑑𝑣
At the maxima & minima 𝑑𝛼 = 0
1M
=> 𝑡𝑎𝑛𝛼 = √2
𝑑2 𝑣 1 3 1M
= 𝜋𝑙 𝑐𝑜𝑠 3 𝛼(2 − 7𝑡𝑎𝑛2 𝛼)
𝑑 ∝2 3
𝑑2𝑣 −4𝜋𝑙 3 1M
=> ( ) =
𝑑 ∝2 𝑡𝑎𝑛𝛼= 3√3√2
Thus V is maximum when , 𝑡𝑎𝑛𝛼 = √2 => 𝛼 = tan−1 √2
26 Consider corresponding equation of curves
𝑦 = |𝑥 + 2| …………………(i) ½M

𝑥 2 + 𝑦 2 = 20…………(ii) ½M
Correct graph of required area 2M
Solving equations (i) and (ii), get 1M
6M
𝑥 = −4, 2
2 −2 2
𝐴𝑟𝑒𝑎 = ∫ √20 − 𝑥 2 𝑑𝑥 − ∫ (−𝑥 − 2) 𝑑𝑥 − ∫ (𝑥 + 2)𝑑𝑥 1M
−4 −4 −2
Solving and getting required 𝐴𝑟𝑒𝑎 = (5𝜋 − 2)𝑠𝑞. 𝑢𝑛𝑖𝑡𝑠 1M

OR Consider corresponding equation of curves


𝑥2 𝑦2
Ellipse: + 𝑏2 = 1 ……………..(i)
𝑎2

𝑥 𝑦
Straight line: 𝑎 + = 1………..(ii)
𝑏
2M
Correct graph of required area 6M
𝑎 𝑎 1M
𝐴𝑟𝑒𝑎 = ∫ 𝑦𝐸𝑙𝑙𝑖𝑝𝑠𝑒 𝑑𝑥 − ∫ 𝑦𝑆𝑡𝑟𝑎𝑖𝑔ℎ𝑡𝑙𝑖𝑛𝑒 𝑑𝑥
0 0
𝑏𝑎 𝑎
𝑏 1M
𝐴𝑟𝑒𝑎 = ∫ √ 2 2
𝑎 − 𝑥 𝑑𝑥 − ∫ (𝑎 − 𝑥) 𝑑𝑥
0 𝑎 0 𝑎 2M
Solving and getting required 𝐴𝑟𝑒𝑎 = (5𝜋 − 2)𝑠𝑞. 𝑢𝑛𝑖𝑡𝑠
27 Showing that line is parallel to the given plane 1M
Since P(1, 3, 4) are the coordinates of any point on the line.
Let 𝑄 (𝑥1 , 𝑦1 , 𝑧1 ) be the foot of perpendicular from this point to the plane.
Direction ratios of PQ are 𝑥1 − 1, 𝑦1 − 3, 𝑧1 − 4
1M
Now since PQ is perpendicular to the given plane
𝑎 𝑏 𝑐 𝑥 −1 𝑦1 −3 𝑧 −4
Therefore by 𝑎1 = 𝑏1 = 𝑐1 => 12 = −1 = 11 = 𝑟(𝑙𝑒𝑡)
2 2 2
=> 𝑥1 = 2𝑟 + 1, 𝑦1 = −𝑟 + 3, 𝑧1 = 𝑟 + 4
Since Q lies in the plane 1M
=> 2(2𝑟 + 1) − (−𝑟 + 3) + (𝑟 + 4) + 3 = 0
=> 𝑟 = −1
1M 6M
So coordinates of Q are (-1, 4, 3)
Now if R(x’, y’, z’) is the image of P in the given plane, then
1 + 𝑥′
= −1 => 𝑥 ′ = −3
2
3 + 𝑦′
= 4 => 𝑦 ′ = 5
2
4 + 𝑧′
= 3 => 𝑧 ′ = 2
2
Coordinates of R(-3, 5, 2) 1M
x 3 y5 z 2
Therefore image of given line in the given plane will be  
3 1 5
1M

OR Given equations of lines are:


x 5 y 7 z 2
   r(let) …………………..(i)
3 1 1
x 3 y3 z 6
   s(let) ....………………(ii)
3 2 4
Coordinates of any point P on line (i) are:
½ M
𝑃(3𝑟 + 5, −𝑟 + 7, 𝑟 − 2)
Coordinates of any point Q on line (ii) are:
𝑄(−3𝑠 − 3, 2𝑠 + 3,4𝑠 + 6) ½ M 6M
Direction ratios of PQ are
−3𝑠 − 3𝑟 − 8, 2𝑠 + 𝑟 − 4,4𝑠 − 𝑟 + 8 1M
If direction ratios of line PQ are taken as 2, 7 ,-5
𝑎 𝑏 𝑐 −3𝑠−3𝑟−8 2𝑠+𝑟−4 4𝑠−𝑟+8
Then by 𝑎1 = 𝑏1 = 𝑐1 => = 7 = −5
2 2 2 2
1M
Solving get 𝑟 = −1, 𝑠 = −1
Therefore coordinates of P and Q are: 1M
𝑃(2, 8, −3) and 𝑄(0, 1, 2) 1M
Required distance 𝑃𝑄 = √78 1M
28 Let 𝑥 dolls of type A and 𝑦 dolls of type B should be produced to get the maximum
profit. ½ M
Therefore given problem can be formulated as:
𝑀𝑎𝑥. 𝑍 = 12𝑥 + 16𝑦 ½ M
𝑥 + 𝑦 ≤ 1200
𝑥
𝑦≤
2
𝑥 − 3𝑦 ≤ 600
𝑥, 𝑦 ≥ 0 2M 6M
Correct graph and shaded feasible region 2M
The values of Z at the corner points of feasible region are:
Corner Point Coordinates Value of Z
O (0, 0) Z=0
A (600, 0) Z = 7200
B (1050, 150) Z = 15000
C (800, 400) Z = 16000(Max.)
Thus, 800 dolls of type A and 400 dolls of type B should be produced to get the 2M
maximum profit of Rs. 16000.
29 Probability Distribution:
X 2 3 4 5 6 7 8 9 10 11 12
P(X 1/3 2/3 3/3 4/3 5/3 6/3 5/3 4/3 3/3 2/3 1/3 2M
) 6 6 6 6 6 6 6 6 6 6 6 ½M
Let x = Sum of digits on the two dies ½M
Mean of x = E(x)=∑𝑛𝑖=1 𝑥𝑖 𝑝(𝑥𝑖 )
= 2 × 1/36 + 3 × 2/36 + 4 × 3/36 + 5 × 4/36 + 6 × 5/36 + 7 × 6/36 + 8 × 5/36 + 9 × 4/36 + 10 × 3/36 + 11 1M 6M
𝑛
× 2/36 + 12 × 1/36 =7
½M
𝐸(𝑥 2 ) = ∑ 𝑥𝑖 2 𝑝(𝑥𝑖)
½M
𝑖=1
1M
Var(x)=𝐸(𝑥 2 ) − {𝐸(𝑥)}2
Finding Variance = 5.833, Finding S D = 2.415

BLUE PRINT

CLASS XII -MATHEMATICS

S.N Typology of Learning Very Short Long Long Mark %


questions Outcomes Short Answer Answer Answer s Weight
Testing Answer (2mark (4 II age
Competencies (1 ) mark) (6
mark) marks)

1 Remembering Reasoning 2 2 2 1 20 20%


2 Understanding Analytical 1 3 4 2 35 35%
skills
3 Application Critical 1 - 3 2 25 25%
thinking
4 Higher Order Derivative - 3 1 - 10 10%
Thinking Skills
5 Evaluation and - - 1 10 10%
Multi- 1(V
Disciplinary BQ)

1x4=4 8x2=16 11x4=4 6x6=36 100 100%


4
BLUE PRINT

CLASS XII SUBJECT MATHEMATICS

CHAPTERWISE

S.No. Topics VSA SA1(2) SA2(4) LA(6) TOTAL


(1)
1 i) RELATIONS AND 2 1
FUNCTIONS
j) INVERSE 2 12
TRIGONOMETRIC
FUNCTIONS
2 i) MATRICES 1 1
13
j) DETERMINANTS 1 1
3 u) CONTINUITY & 2
DIFFERENTIABILI
TY
v) APPLICATION OF 1 2
DERIVATIVES
42
w) INTEGRATION 1 1 1
x) APPLICATION OF 1
INTEGRALS
y) DIFFERENTIAL 1 1
EQUATIONS
4 i) VECTORS 1 1 1
j) 3-DIMENTIONAL 1 1 17
GEOMETRY
5 LINEAR 1
06
PROGRMMING
6 PROBABILITY 1 2 10
TOTAL 4x1=4 8x2=16 11*4=44 6*6=36 29(100)

Time: 3 hours Class XII (MATHEMATICS) MM: 100

General Instructions:

1. The question paper consists of four sections A, B and C and D. Section A,


consists 4 questions of 1 mark each, Section B, consists of 8 questions of 2
marks each And Section C, consists 11 questions of 4 marks each and section D
consist of 6 questions of 6 marks each.
2. Questions of Section A are to be answered in one word or one sentence or as
per exact requirement of the question.
3. All the questions are compulsory but in some of the questions internal choice
has been given so you have to attempt only question of choice.
______________________________________________________________________
_____
Section – A
1. If f: R R define by f(x)=(3 - x ) then find ( fof) (x).
3 1/3

2. A is a non-singular matrix of order 3 x 3 and |A|=9 find |adj A|.


3. Find the values of λ so that the vectors 𝒂 ̂ and ⃗𝒃⃗ = 𝒊̂ - 2𝒋̂ + 3𝒌
⃗⃗ =2 𝒊̂ + λ𝒋̂ + 𝒌 ̂ are
perpendicular to each other.
4. If a*b= a- b +4 find 2*5.
Section – B

1  1  y 2 
5.Write in the simplest form tan  sin 1   2x 1
2 
 cos 1  2 
2  1 x  2  1  y 
6.If A and B are symmetric matrices show that AB-BA is skew symmetric

Matrix.
𝒙
7.Simplify : 𝒕𝒂𝒏−𝟏 ( ).
√𝒂𝟐 −𝒙𝟐

8.Find the rate of change of area of a circle with respect to its radius ‘ r ‘ when r = 6
cm.

9.Evaluate ∫ 𝐬𝐢𝐧−𝟏 (𝐜𝐨𝐬 𝒙) 𝒅𝒙

10.Find the differential equation of the family of all straight lines.


11.Find the position vector of the mid – point of the vector joining the points P ( 2 , 3
, 4 ) and Q ( 4 , 1 , -2 ).
𝟑 𝟕
12.If A and B are the two events such that P ( A ) = , P(B)=
𝟓 𝟏𝟎

𝟗
and P ( A ∪ 𝑩 ) = , then find P ( A ∩ 𝑩 ) .
𝟏𝟎

Section – C

13.Using elementary row operations, find the Inverse of the following matrix:
2 5
1 3
 

14.Find the value of k so that the function f given by:


 k cos x 
   2 x , if x  2 
f ( x)   is continuous at x  .
 3,
 2
if x 
 2 or

( x  3)( x 2  4)
Differentiate with respect to x.
3x 2  4 x  5

𝒎 𝐜𝐨𝐬 −𝟏 𝒙 𝒅𝟐 𝒚 𝒅𝒚
15.If y = 𝒆 , prove that (1-𝒙𝟐 ) -x = m2y.
𝒅𝒙𝟐 𝒅𝒙

1
16 Find the intervals in which the function f given by f ( x)  x3  , x  0 is
x3
(i) increasing (ii) decreasing
OR
Find the equation of the normal line to the curve𝒚(𝒙 − 𝟐)(𝒙 − 𝟑) − 𝒙 + 𝟕 = 𝟎 at the
point where it meets the 𝒙 −axis.
17.Show that height of the cylinder of maximum volume that can be
2R
inscribed in a sphere of radius R is . Also find the maximum volume.
3

x 4 dx
18. Evaluate 
 x  1  x 2  1

19.Solve the differential equation

OR

d 2 y dy
Show that y = ae 2 x  be  x is a solution of the differential equation   2y  0
dx 2 dx

20.Show that the points with position vectors 2 i  ĵ  k̂, î  3 ĵ  5k̂ and 3î  4 ĵ  4k̂ are the
vertices of a right angled triangle.
21.Find the shortest distance between the lines whose vector equations are
̂ and
⃗⃗ = (𝟏 − 𝟐𝒕)𝒊̂+(𝟏 − 𝒕)𝒋̂+(𝒕)𝒌
𝒓
̂
⃗⃗ = (𝟐 + 𝟑𝒔)𝒊̂ +(𝟏 − 𝟓𝒔)𝒋̂+(𝟐𝒔 − 𝟏)𝒌
𝒓

22.An urn contains 4 red and 3 blue balls. Find the probability distribution of the
number of blue balls in a random draw of 3 balls, with replacement.

23.A company has two plants to manufacture machines. Plant A manufactures 70%and
plant B manufactures 30% machines. At plant A, 80% machines are rated of standard
quality and at plant B, 90% machines are rated of standard quality. A machine is chosen
at random and is found to be of standard quality. What is the probability that it was
manufactured by plant A ?

Section – D

24. Consider f : R  5,  given by f ( x)  9x2  6x  5 . Show that f is invertible and find
the inverse of f.
OR
A binary operation * is defined on the set X = R – { - 1 } by x * y = x + y + xy , ∀ x , y
∈ X.Check whether * is commutative and Associative. Find its identity element and
also find the inverse of each element of X.

25. Using the properties of determinants , prove that:

𝒙 𝒙𝟐 𝟏 + 𝒑𝒙𝟑
|𝒚 𝒚𝟐 𝟏 + 𝒑𝒚𝟑 | = (𝟏 + 𝒑𝒙𝒚𝒛)(𝒙 − 𝒚)(𝒚 − 𝒛)(𝒛 − 𝒙)
𝒛 𝒛𝟐 𝟏 + 𝒑𝒛𝟑

OR

1  1 2  - 2 0 1 
Use product 0 2  3  9 2 - 3 to solve the system of equations
3  2 4   6 1 - 2

x  y  2 z  1, 2 y  3z  1, 3x  2 y  4 z  2.

26. Find the area of the region lying between the parabolas y  4ax and x  4ay
2 2

,where a  0.
𝟐 𝟒
27. Evaluate the integral as a limit of sum :∫𝟏 (𝒙 -x) dx

OR
2x  3
Evaluate  x 2  2x  5
dx

28. Find the equation of the plane passing through the point (-1,-1,2) and
perpendicular to each of the planes: 2 x  3 y  3z  2 and 5 x  4 y  z  6.

29.Reshma wishes to mix two types of food P and Q in such a way that the
vitamin contents of the mixture contain at least 8 units of vitamin A and 11
units of vitamin B. Food P costs Rs 60/kg and Food Q costs Rs 80/kg. Food
P contains 3 units /kg of vitamin A and 5 units /kg of vitamin B while food
Q contains 4 units /kg of vitamin A and 2 units /kg of vitamin B. Determine
the minimum cost of the mixture? What diet would you prefer to maintain
good health? Explain the importance of balanced diet?

MARKING SCHEME

1 X
1M
2 81
1M
3 λ= 5/2
1M
4 1
1M

1  2x  1 1  1  y 
2

Take y  tan  sin 1  2 
 cos  2 
5 2  1 x  2  1  y 
 2x 
sin 1  2 
 2 tan 1 x
 1 x 
 1 y2 
cos 1  2 
 2 tan 1 y
 1  y 
1 

2
 1

then, y  tan  2 tan 1 x  2 tan 1 y 
2 

  x  y   x  y 
 tan  tan 1     
  1  xy    1  xy 

6 (AB-BA)’=B’A’-A’B’ 1M

=BA-AB=-(AB-BA) 1M
HENCE AB-BA is skew symmetric matrix

7. Put x=sinα 1M

Simplifying and getting result 1M

8 25. For getting dA/dr = 2𝝅r 1 mark


(dA/dr ) at r = 6 = 12 𝝅 cm2/cm
1 mark
9. ∫ 𝐬𝐢𝐧−𝟏 𝐜𝐨𝐬 𝑿 dx
𝝅
Putting cos x = sin ( − x )
𝟐
1mark
𝝅
To get the result x – x2 / 2 + c
𝟐
1 mark

10 For equation y = mx + c 1 mark


𝒅𝒚
=m & for second derivative = 0
𝒅𝒙
1 mark

11 For writing 𝒂 ⃗⃗ = 𝟒𝒊̂ + 𝒋 − 𝟐𝒌


⃗⃗ =2i +3j+4k and 𝒃 ̂
1 mark
For finding P.V.of mid point= 3i+2j+k
1 mark

12 For writing P(AUB)= P(A)+p(B)-P(A∩B)


1 mark
𝟐
For getting correct answer P(A∩B) =
𝟓
1 mark

13. We know, IA  A 1

1 0 2 5 1
Here   A 
0 1  1 3

Applying , R1  R1  R2
1  1 1 2
0 1  A  1 3
    1
Applying , R2  R2  R1
 1  1 1 2
 1 2  A  0 1
   

Applying , R1  R1  2 R2
 3  5 1 0
  1 2  A  0 1 
   
 3  5
 A 1   
 1 2  1

14  k cos x 
   2 x , if x  2 
Given, f ( x)   is continuous at x  .
 3,
 2
if x 
 2
1
 
Therefore, lim f ( x)  lim f ( x)  f  
 
  2
x x
2 2

1
 
 lim f   h   3
h 0  2 
 
k cos  h 
2 
 lim 3
h 0  
  2  h 
2 
k  sinh 
 lim  3
2 h0 h 
k 6

Let , y 
( x  3)( x 2  4) 11
2
3x 2  4 x  5

Taking logarithm on both side, we have


11
log y 
1
2
 
log( x  3)  log( x 2  4)  log(3x 2  4 x  5) 2

Differentiating b/s w.r.t. x


1 dy 1  1 2x 6x  4 
    
y dx 2  x  3 x 2  4 3x 2  4 x  5  𝒅𝒚 𝒎𝒚
. =-
dy 1 ( x  3)( x 2  4)  1 2x 6 x  4  𝒅𝒙 √𝟏−𝒙𝟐
     
dx 2 3x 2  4 x  5  x  3 x 2  4 3x 2  4 x  5 

√𝟏−𝒙𝟐 ∗(−𝒎)𝒅𝒚−(−𝒎𝒚) 𝟐𝒙
𝒅𝟐 𝒚 𝒅𝒙 𝟐√𝟏−𝒙𝟐
15. = 𝟐 2
𝒅𝒙𝟐 √𝟏−𝒙𝟐

𝒅𝟐 𝒚 𝒅𝒚
(1-𝒙𝟐 ) -x = m2y.
𝒅𝒙𝟐 𝒅𝒙 2
3( x 2  1)( x 4  1  x 2 ) 1M
f '( x) 
16. x4

3( x 4  1  x 2 )
But 0
x4

Therefore sign of f’(x) depends upon (x2-1) 1M

(i) for increasing ,f’(x)>0


 x  1 or x  1
Hence f i s in crea sin g in  , 1  1,  

(ii) for decreasing f’<0


 1  x  1 1M
 function decreases in  1,1
OR

 𝒇′(𝒙) = −𝟔(𝒙 + 𝟏)(𝒙 + 𝟐) 1M


 Finding critical points 𝒙 = −𝟏, −𝟐
 Finding intervals for increasing and decreasing functions
 Writing answer: Increasing in (−𝟐, −𝟏), Decreasing in (−∞, −𝟐) ∪
(−𝟏, ∞)

17 Let r and h be the radius and height of the cone respectively


inscribed in a sphere of radius R.

Let V be the volume of the cone.


1M
Then,

Height of the cone is given by,

h = R + AB
1mark

1M

∴ By second derivative test, the volume of the cone is the


maximum when

1m
1m

18 𝑿𝟒 𝒙𝟒 −𝟏+𝟏
∫ (𝒙−𝟏)(𝒙𝟐 +𝟏) 𝒅𝒙 = ∫ (𝒙−𝟏)(𝒙𝟐+𝟏) 𝒅𝒙

𝒙𝟒 − 𝟏 𝟏
= ∫ (𝒙−𝟏)(𝒙𝟐 ) 𝒅𝒙 + ∫ (𝒙−𝟏)(𝒙𝟐 ) 𝒅𝒙
+𝟏 +𝟏

𝟏
= ∫(𝒙 + 𝟏 )𝒅𝒙 + ∫ (𝒙−𝟏)(𝒙𝟐 ) 𝒅𝒙
+𝟏

𝒙𝟐 𝟏 𝒅𝒙 𝟏 𝒙+𝟏
= + 𝒙 + ∫ − ∫ 𝒅𝒙
𝟐 𝟐 𝒙−𝟏 𝟐 (𝒙𝟐 +𝟏 )

𝒙𝟐 𝟏 𝟏
= + 𝒙 + 𝒍𝒐𝒈|𝒙 − 𝟏| − 𝐥𝐨𝐠|(𝒙𝟐 + 𝟏 )| -
𝟐 𝟐 𝟒
𝟏
𝐭𝐚𝐧−𝟏 𝒙 + 𝒄
𝟐

19.The given equation can be written as


dx 1 tan 1 y
 x 
dy 1  y 2 1  y2
1 tan 1 y
Here P  and Q 
1  y2 1  y2
1
 1 y 2 dy 1
 I.F.  e  e tan y

 Its Solution is
 tan1 y 

1 1
x.e tan y   .e tan y dy  C.......(i)
 1  y2 
 
 tan1 y 

1

Consider I   . e tan y dy
 1 y 
2
 
Let tan-1y = t
1
 dy  dt
1  y2

I =  t ..e t dt

= t . et   t.t et dt ….. (By parts)


t t
= t.e  e  C

 From (i)
1 1 1
x . e tan y  tan1 y . e tan y  e tan y  C
1
 x  tan1 y  1  Ce tan y
Is the required solution.
OR

We are given by y = ae2x + be-x


dy
 2ae 2 x  be  x
dx
d2y
2
 4ae 2 x  be  x
dx
d 2 y dy
 2
dx dx
    
 2 y  4ae 2 x  be  x  2ae 2 x  be  x  2 ae 2 x  be  x 
 4ae 2 x  be  x  2ae 2 x  be  x  2ae 2 x  2be  x
0
Hence y = a e 2 x + b e  x is a solution the differential
d 2 y dy
equation   2y  0
dx 2 dx

20.Let A, B and C be the vertices of  ABC whose position vectors


with respect to any point O are :
OA  2î  ĵ  k̂, OB  î  3 ĵ  5k̂ and OC  3î  4 ĵ  4k̂

AB  OB  OA  î  2 ĵ  6k̂
BC  C  OB  2î  ĵ  k̂
CA  OA  OC  î  3 ĵ  5k̂

AB  BC  î  2 ĵ  6k̂  2î  ĵ  k̂


Here
 î  3 ĵ  5k̂  - CA

Now BC.CA  (2iˆ  ˆj  kˆ).(iˆ  3 ˆj  5kˆ)

= -2 -3 + 5 = 0
 BC and CA are at right angle to each other.

Hence  ABC is a right angled at C.

21.

⃗⃗⃗⃗⃗𝟐 − 𝒂
𝒂 ̂
⃗⃗⃗⃗⃗𝟏 = 𝒊̂ − 𝒌
(1 mark)

⃗⃗⃗⃗⃗𝟏 𝑿𝒃
𝒃 ⃗⃗⃗⃗⃗𝟐 = 𝟑𝒊̂ − 𝒋̂ − 𝟕𝒌
̂
(1 mark)

| ⃗⃗⃗⃗⃗𝟏 𝑿𝒃
𝒃 ⃗⃗⃗⃗⃗𝟐 | = √𝟓𝟗
(1 mark)
𝟏𝟎
Shortest distance between the line:
√𝟓𝟗
(1 mark)

22 Let X denote the no of blue balls. ½M

Then X can take values 0,1,2,3


3 4
P(blue ball)= ; P(a red ball)=
7 7
½M
4 4 4 64
Now P(X=0)=P(no blue ball)= 3C0    
7 7 7 343

3 4 4 144
P(X=1)=P(one blue and two red balls)= 3C1    
7 7 7 343
½M
3 3 4 108
P(X=2)= 3C2    
7 7 7 343

3 3 3 27
P(X=3)= 3C3    
7 7 7 343
½M
The required probability distribution is

X 0 1 2 3
½M
P(X=x) 64 144 108 27
343 343 343 343

½M
1M

23 Let E: machine is of standard quality.

7 3
P( A)  ; P( B) 
10 10
8 9 1M
P( E / A)  ; P( E / B) 
10 10
P( A).P( E / A)
P( A / E )  
P( A).P( E / A)  P( B).P( E / B)
1M
56
83

1M

1M

24 for x1 , x2  R
f ( x1 )  f ( x2 )  9( x12  x2 2 )  6( x1  x2 )  0
1M
 ( x1  x2 ) 9( x1  x2 )  6  0
As 9( x1  x2 )  6  0 therefore ( x1  x2 )  0
 x1  x2  f is 1  1

11/2
y   5,   there exists x  R , such that
M
Let for 1  6  y
y  9 x2  6 x  5  x 
3

6  y 1
 x  R
3 hence onto.

Therefore the function is one and onto showing that it is invertible


6  y 1 2½ M
f 1 ( y) 
3
1M

24 For Commutative (2 marks)

For Associative (2 marks)

For finding Identity (1 mark)

For finding Inverse (1 mark)

𝟐
𝒙 𝒙𝟐 𝟏 𝒙 𝒙 𝒑𝒙𝟑
| 𝒚 𝒚𝟐 𝟏|+|𝒚 𝒚𝟐 𝒑𝒚𝟑 | 2 Marks
𝒛 𝒛𝟐 𝟏 𝒛 𝒛𝟐 𝒑𝒛𝟑

Similarly 2 marks for using each properties

Getting proof
(2Marks )

OR 1  1 2  - 2 0 1  1 0 0
0 2  3  9 2 - 3  0 1 0
     
3  2 4   6 1 - 2 0 0 1

-1
1  1 2  - 2 0 1 
 0 2  3   9 2 - 3.......... .......... .......... .......... ....(1)
3  2 4   6 1 - 2

Given System of equation: x  y  2 z  1, 2 y  3z  1, 3x  2 y  4 z  2.

Corresponding matrix equation,


AX  B  X  A 1 B.......... .......... .......... .......... .......... (2)
2
1  1 2   x 1
Where A  0 2  3, X   y , B  1
3  2 4   z  2
1
 x   2 0 1  1 0 2
 y    9 2  3 1  
       5 
 z   6 1  2 2 3

Therefore, x = 0 ,y = 5 and z = 3. 11
2
1

26 Given equations of parabolas are: y  4ax ……….(1); x  4ay


2 2

…………..(2)
Solving (1) and (2),point of
intersections are ( 0, 0) and 4a,4a  2

Fig.
1m

4a 4a
x2
Required area=  4ax dx   dx
0 0 4a

= 16 a 2 Square Unit. 3m
3

27 a=1 ,b=4, h= 3 /n

 Lim h[ 0 + ( h2 +h) + 4h2 + 2h) + ….{ ( n-1)2 h2 + ( n - ) h}]


 h->0
 Lim h[h2( 1+4 + ….. n-1 2) +h ( 1+2+……n -1]
 h->=0
(𝒏−𝟏 )𝒏( 𝟐𝒏−𝟏) 𝒏(𝒏−𝟏)
 Lim h[ h2 + h. ]
𝟔 𝟐
 h-> 0
 27/2
OR
d 2
dx

x  2x  5  2x  2 
2x  3 2x  2  1
 dx   dx
x  2x  5
2
x 2  2x  5

2x  2 1
 dx   dx
x  2x  5
2
x  2x  5
2

Let x2 – 2x + 5 = t2 in I integral

 2x - 2dx  2tdt

2tdt 1
  dx
t x  12  4
1
 2 dt   dx
x  1 2
2 2

 2t  log ( x  1)  x 2  2x  5

= 2 x 2  2x  5  log ( x  1)  x 2  2x  5  C

28. Equation of plane passing through ( -1, -1, 2) is

A(x+1) +B (y +1) +C(z-2)=0………………………………………….(1) 1

Plane (1) is perpendicular to the planes: 2x+3y-3z=2 and 5x-4y+z =6

Therefore, 2A +3B – 3Z=0 ……………………………………………(2)

5A – 4B +C =0 …………………………………………….(3) 2
A B C
  
Solving (2) and (3), 3  12 2  15  8  15 (say)

 A  9, B  17 and C  23 2


Therefore required equation of plane is:
9 ( x  1)  17  ( y  1)  23 ( z  2)  0
 9 x  17 y  23 z  20  0

29 Let the mixture contain x kg of food P and y kg of food Q.


Therefore,

x ≥ 0 and y ≥ 0

The given information can be compiled in a table


as follows.

Vitamin A Vitamin B Cost (Rs/kg)


(units/kg) (units/kg)

Food P 3 5 60

Food Q 4 2 80

Requiremen 8 11
t (units/kg)

The mixture must contain at least 8 units of


vitamin A and 11 units of vitamin B. Therefore,
the constraints are

3x + 4y ≥ 8

5x + 2y ≥ 11

Total cost, Z, of purchasing food is, Z = 60x + 80y

The mathematical formulation of the given


problem is

Minimise Z = 60x + 80y … (1)

subject to the constraints,

3x + 4y ≥ 8 … (2)

5x + 2y ≥ 11 … (3)

x, y ≥ 0 … (4)

The feasible region determined by the system of


constraints is as follows.

1 mark

It can be seen that the feasible region is unbounded.

The corner points of the feasible region are


𝟏
. mark
𝟐

The values of Z at these corner points are as follows.

Corner Z = 60x +
point 80y

160

160

440

As the feasible region is unbounded, therefore, 160 may or may


not be the minimum value of Z.

For this, we graph the inequality, 60x + 80y< 160 or 3x + 4y< 8,


and check whether the resulting half plane has points in common
with the feasible region or not.

It can be seen that the feasible region has no common point with
3x + 4y< 8

Therefore, the minimum cost of the mixture will be Rs 160 at the

line segment joining the points .


𝟏
mark
𝟐

Importance : A balanced diet will a keep a man healthy and


will keep him off from diseases.

1 mark

SUBMITTED BY:
NAGESH KUMAR MISHRA
PGT(MATHS)
KV IIT KANPUR

BLUE PRINT
CBSE Sample Paper
Class – XII
Subject: Mathematics (041)

S.No. UNIT VSA SA LA-I LA-II TOTAL


1M 2M 4M 6M Marks
1(a) Relations and Functions 1 1
10
(b) Inverse Trigonometric Functions 1 1
2 Matrices & Determinants 2 1 1 1 14
3(a) Continuity and differentiability 1 2
(b) Application of derivatives 1 1 1
(c) Integration 1 1 1 44
(d) Application of integrations 1
(e) Differential Equations 1
4(a) Vectors 2 16
(b) 3 – D Geometry 1 1 1
5 Linear Programming 1 6
6 Probability 1 2 10
TOTAL 4 8 11 6 100
SAMPLE QUESTION PAPER
CBSE - BOARD EXAMINATION
CLASS:XII TIME: 3 hrs
SUBJECT: MATHEMATICS MAX.MARKS:100

General Instructions:
(i) All questions are compulsory.
(ii) This question paper contains 29 questions.
(iii) Question 1- 4 in Section A are very short-answer type questions carrying 1 mark each.
(iv) Question 5-12 in Section B are short-answer type questions carrying 2 marks each.
(v) Question 13-23 in Section C are long-answer-I type questions carrying 4 marks each.
(vi) Question 24-29 in Section D are long-answer-II type questions carrying 6 marks each.

SECTION – A

1. Let * be a binary operation defined by x*y = 5x + 7y. Find 2 * 4.

2. If Sin (sin—1(1/5) + cos—1x) = 1, then find the value of x.

3. If matrix A = (1 2 3), write AA’, where A’ is the transpose of the matrix A.

4. If f(x) = x2 – 5x + 7, write f(A), where A is a square matrix of order n.

SECTION – B

5. Evaluate: sin 100 – cos 100


sin 800 cos 800

6. Evaluate: ∫ 𝑡𝑎𝑛2 (2𝑥 + 1)𝑑𝑥

7. Find the derivative of log10 𝑥 w.r.t. x.

8. Find the angle ө between the vectors 𝑎⃗ and 𝑏⃗⃗ if | 𝑎⃗ . 𝑏⃗⃗ | = | 𝑎⃗ x 𝑏⃗⃗ |

9. Find the projection of the vector 𝑖̂ – 𝑗̂ on the vector 𝑖̂ + 𝑗̂ .

10. If a plane intersects the co-ordinate axes at (1,0,0), (0,1,0) and (0,0,1), find its equation.

11. A couple has 2 children. Find the probability that both are boys, if it is known that (i) one
of them is a boy (ii) the older child is a boy.

12. The sides of an equilateral triangle are increasing at the rate of 2 cm/sec. Find the rate at
which its area increases, when side is 10 cm long.
SECTION - C
4𝑥+3
13. If f(x) = , x ≠2/3, Show that f 0 f (x) = x, for all x ≠2/3. What is the inverse of f?
6𝑥−4
OR

Show that the function f:R→R defined by f(x) = 2x3 – 7, for x∊R is bijective.

14. Solve: 2 tan—1(cos x) = tan—1(2 cosec x).


OR
Prove that tan—1(63/16) = sin—1(5/13) + cos—1 (3/5).

15. Using properties of determinants, Prove that

a+b+2c a b
c b+c+2a b = 2(a+b+c)3
c a c+a+2b

16. Find the relation between a and b so that


f(x) = ax + 1, x ≤ 3
= bx + 3, x > 3 is continuous at x = 3.

𝑑𝑦
17. If x √1 + 𝑦 + y √1 + 𝑥 = 0, for |x| < 1, Prove that = –1 / (1 + x)2.
𝑑𝑥

OR
If x = a(cos t + t sin t) and y = a(sin t – t cos t), find d2y / dx2.

18. Find the intervals in which the function f(x) given by f(x) = sin x + cos x, 0 ≤ x ≤ 2π.
I) is strictly increasing ii) strictly decreasing

OR

Find the equation of tangent to the curve y = √3𝑥 − 2, which is parallel to the line
4x – 2y + 5 = 0.
1+sin 𝑥
19. Evaluate: ∫ 𝑒 𝑥 ( ) 𝑑𝑥
1+cos 𝑥

𝑥 𝑥
20. Solve: 2y 𝑒 𝑦 dx + ( y – 2x 𝑒 𝑦 ) dy = 0

21. Find the value of P so that the lines


1−𝑥 7𝑦−14 5𝑧−10 7−7𝑥 𝑦−5 6−𝑧
. = = and = = are perpendicular.
3 2𝑃 11 3𝑃 1 5

22. Given three identical boxes I, II and III, each containing two coins. In box I, both coins
are gold coins, in box II both are silver coins and in the box III there are one gold and one
silver coin. A person chooses a box at random and takes out a coin. If the coin is of gold,
what is the probability that the other coin in the box is also of gold?

23. Two numbers are selected at random (without replacement) from the first six positive
integers. Let X denotes the larger of the two numbers obtained. Find the expectation E(X)
of the random variable X.

SECTION – D

1 −1 2 −2 0 1
24. Use product (0 2 – 3) ( 9 2 −3) to solve the system of equations
3 −2 4 6 1 −2
x – y + 2z = 1, 2y – 3z = 1, 3x – 2y + 4z = 2.

25. Show that the height of the cylinder of greatest volume which can be inscribed in a right
circular cone of height h and semi vertical angle  is one-third that of the cone and the
4
greatest volume of cylinder is π h3 tan2.
27

(OR)

A tank with rectangular base and sides, open at the top is to be constructed so that its depth
is 2 m and volume is 8 m3. If building of tank costs Rs.70 per square meter for the base
and Rs.45 per square meter for sides. What is the cost of least expensive tank?
𝜋 𝑥 tan 𝑥
26. Evaluate : ∫0 𝑑𝑥.
sec 𝑥+tan 𝑥

27. Find the area of the region enclosed between the two circles x2 + y2 = 9 and (x –
3)2 + y2 = 9.
OR
Use the integration, find the area of triangle ABC, with the vertices A( --1, 0), B( 1, 3) and
C(3,2).
28. Find the equation of the plane passing through the intersection of the planes 2x + 3y – z +
1 = 0; x + y – 2z + 3 = 0 and perpendicular to the plane 3x – y – 2z – 4 = 0. Also find the
inclination of this plane with xy plane.

29. Two godowns A and B have grain capacity of 100 quintals and 50 quintals respectively.
They supply to 3 ration shops, D, E and F whose requirements are 60, 50, and 40 quintals
respectively. The cost of transportation per quintal from the godowns to the shops is given
in the following table.

Transportation cost per quintal ( in Rs)


From/To A B
D 6 4
E 3 2
F 2.50 3

How should the supplies be transported in order that the transportation cost is
minimum? What is the minimum cost? Write any one value reflected in the problem.

-----xxxx----
MARKING SCHEME
CBSE Sample Paper
Class – XII
Subject: Mathematics (041)

SECTION- A
1. 38 1M
2. 1/5 1M
3. AA1 = (14) 1M
4. f(A) = A2 – 5 A + 7 I, where I is an identity matrix of order n. 1M

SECTION - B

5. 1 2M
6. ½ tan(2x + 1) – x + c 2M
1
7. 𝑥 log10 𝑒 2M
8. π/4 2M
9. 0 2M
10. x + y + z = 1 2M
11. Sample space = { B1B2, B1G2, G1B2, G1G2} , B1 and G1 are the older boy and girl respectively.
Let E1 = both the children are boys;
E2 = one of the children is a boy ;
E3 = the older child is a boy
𝐸1∩𝐸2 1/4
Then, (i) P(E1/ E2) = P ( )= = 1/3 1M
𝐸2 3/4

𝐸1∩𝐸3 1/4
(ii) P(E1/ E3) = P ( 𝐸3
)= 2/4 =½ 1M
3
12. Here, Area(A) = √4 X2 where ‘x’ is the side of the equilateral triangle. 1M

𝑑𝐴
= 10√3 cm2/sec 1M
𝑑𝑡

SECTION – C

4𝑥+3
13. f0f(x) = f[f(x)] = f (6𝑥−4) ½M
f0f(x) = x 1½M
4𝑥+3
f—1(x) = 6𝑥−4 2M

OR
Proving one-one 2M
Proving onto 2M

2 cos 𝑥
14. tan−1 (1− 𝑐𝑜𝑠2 𝑥)= tan—1(2 cosec x) ½M
2 cos x/sin2x = 2 cosec x. ½M
2 cosecx (cot x – 1) = 0 1M
Cosec x = 0 or cot x = 1
Cosec x = 0 is not possible 1M
Cot x = 1
x = π/4 1M
OR
Sin 5/13 = tan—1 5/12
—1 cos—1 3/5 = tan—1 4/3 1M
5
+4/3
tan—1 5/12 + tan—1 4/3 = tan−1 ( 12
5 ) 2M
1− .4/3
12
63
= tan−1 16 1M
15. Operate: C1→ C1+ C2+ C3 1M
Taking 2(a+b+c) common from C1 ½M
R1 →R2 – R1, R3 →R3 – R1
Taking (a+b+c) common from R2 and R3 1M
For expanding the determinant and for result 1½M
16. LHL: Lt f(x) = 3a + 1 1M
x→3—
RHL: Lt f(x) = 3b + 3 1M
x→3+
since f is continuous at x = 3, LHL = RHL 1M
a = b + 2/3 1M
−𝑥
17. For bringing y = 1+𝑥 2M
dy/dx = –1 / (1 + x)2. 2M
OR

dx/dt = at cos t 1M

dy/dt = at sin t 1M
dy/dx = tan t 1M

d2y/dx2 = sec3 t/at 1M

18. f1(x) = cos x – sin x


f1(x) = 0 x = π/4, 5π/4 1M
f1(x) > 0 if x ∊(0, π/4) U (5π/4, 2π) 1M
f1(x) < 0 if x ∊ (π/4, 5π/4) 1M
f is increasing on (0, π/4) U (5π/4, 2π)
f is decreasing on (π/4, 5π/4) 1M

OR

Let (a, b) be the point of contact


b2 = 3a – 2 1M
𝑑𝑦
.(𝑑𝑥 )(a,b) = 3/2b 1M
3 −4
.2𝑏 = −2
a = 41/48, b = 3/4 1M
Equation of tangent: 48 x – 24 y – 23 = 0 1M

1 sin 𝑥
19. ∫ 𝑒 𝑥 (1+cos 𝑥 + 1+cos 𝑥) 𝑑𝑥 1M

1
= ∫ 𝑒 𝑥 (2 𝑠𝑒𝑐 2 𝑥/2 + tan 𝑥/2) 𝑑𝑥 1½M
𝑥
=𝑒 𝑥 tan 2 + c (∫ 𝑒 𝑥 (𝑓(𝑥) + 𝑓 1 (𝑥) ). 𝑑𝑥 = ex f(x) + c 1½M
𝑥
𝑑𝑥 2𝑥𝑒 𝑦 − 𝑦
20. 𝑑𝑦 = 𝑥
2𝑦𝑒 𝑦
𝑥 1
=𝑦– 𝑥 1M
2𝑒 𝑦
𝑥 𝑑𝑥 𝑑𝑣
Put 𝑦 = 𝑣 =𝑣+𝑦 ½M
𝑑𝑦 𝑑𝑦
𝑑𝑣 1
v + y 𝑑𝑦 = 𝑣 − ½M
2𝑒 𝑣
2 ∫evdv = -∫dy/y 1M
𝑥
2𝑒 𝑦 = -log 𝑦 + 𝑐 P 1M

21. Writing the given lines in standard form 2M


Perpendicular condition a1b1 + a2b2 + a3b3 = 0 ½M
Substitution of values ½M
Solving for P = 7 1M
22. Let E1, E2, E3 be the events that boxes I, II, III are chosen respectively.
A be the event that coin drawn is of gold. ½M
P(E1) = 1/3 P(A/E1) = 1
P(E2) = 1/3 P(A/E2) = 0
P(E3) = 1/3 P(A/E3) = ½ 1½M

By Bayes theorem

P(E1/A) = P(E1) P(A/E1)


P(E1) P(A/E1) + P(E2) P(A/E2) + P(E3) P(A/E3) ½M
= 1/3 x 1
1/3 x1 + 1/3 x 0 + 1/3 x ½ ½M
= 2/3 1M
23. First six positive integers are 1, 2, 3,4,5,6.
1 is the smallest positive integer
Therefore X = 2,3,4,5,6. 1M
P(X = 2) = 2/30 ½M
P(X = 3) = 4/30 ½M
P(X = 4) = 6/30 ½M
P(X = 5) = 8/30 ½M
P(X = 6) = 10/30 ½M
E(X) = ∑X P(X) 1M
= 2 x 2/30 + 3 x 4/30 + 4 x 6/30 + 5 x 8/30 +6 x 10/30 1M
= 14/3 ½M

24.
1 −1 2 −2 0 1
(0 2 – 3) ( 9 2 −3) = I 1½M
3 −2 4 6 1 −2

−2 0 1
A-1 = ( 9 2 −3) 1M
6 1 −2
Writing the given equations in matrix form AX = B
and X = A-1 B 1M
0
solving for X = (5) 1M
3

25. Let r be the radius and x be the height of the cylinder, then
𝑟
tan  = ℎ−𝑥 1M
Volume of cylinder V = πr2x
= π tan2 (h2x + x3 – 2h x2) 1M
𝑑𝑉
.𝑑𝑥 = π tan2  ( h2 + 3x2 – 4 hx) 1M
dV/dx = 0 x = h/3 1M
2 2
(d V/dx )x = h/3 < 0 1M
V is maximum when x = h/3 and
4
Maximum Volume = 27 π h3 tan2 1M
(OR)

Let length = x, breadth = y, h = 2 m

Given V = 8 m3 y = x/4 1M

Cost of tank = 70 xy + 45x 4(x + Y)

= 280 + 180(x + 4/x) = C (say) 1M


dC/dx = 180(1 – 4/x) 1M

dC/dx = 0 x=2 1M

(d2C / dx2)x=2 > 0 1M

Therefore cost is minimum when x = 2

Min. Cost = Rs.1000. 1M


𝜋 𝑥 tan 𝑥
26. I = ∫0 𝑑𝑥.
sec 𝑥+tan 𝑥

𝜋 (𝜋−𝑥) tan(𝜋−𝑥)
= ∫0 𝑑𝑥. 1M
sec(𝜋−𝑥)+tan(𝜋−𝑥)
𝜋 tan 𝑥
2I = 𝜋 ∫0 sec 𝑥+tan 𝑥 𝑑𝑥. 1½M
= π{sec x – tanx + x}0π 2M
= π(π – 2) 1M
𝜋
I = 2 (π – 2) ½M
27. Y2 = 9 – x2 -------- (1)
Y2 = 9 – (x – 3)2 ----------- (2)
Solving (1) and (2), x = 3/2
Correct figure 1½M
3/2 3
Area = 2{∫0 √9 − (𝑥 − 3)2 𝑑𝑥 + ∫3/2 √9 − 𝑥 2 𝑑𝑥 } 1M
𝑥−3 9 𝑥−3 3/2 𝑥 9 𝑥 3
2[ √9 − (𝑥 − 3)2 + sin−1 ( )] + 2[2 √9 − 𝑥 2 + 2 sin−1 (3)] 1½M
2 2 3 0 3/2
.
−3 9 9 −1 9
2[ 4 √9 − 4 + sin−1 ( 2 ) − 2 sin−1(−1)] +
2
.
.
9 −1 (1) 3 9 9 1
2[2 sin − 4 √9 − 4 − sin−1 (2)] 1M
2
.
= 6π – 9√3/2 square units. 1M
(OR)
Correct figure 1M
Equation of line AB: y = 3/2(x+1)
Equation of line BC: y = -1/2 (x – 7)
Equation of line AC: y = ½ (x+1) 1½M
. . .
Required area of triangle = ∫𝐴𝐵 𝑦 𝑑𝑥 + ∫𝐵𝐶 𝑦 𝑑𝑥 − ∫𝐴𝐶 𝑦 𝑑𝑥 ½M
3 1 1 3 1 3
= 2 ∫−1(𝑥 + 1)𝑑𝑥 − 2 ∫1 (𝑥 − 7)𝑑𝑥 − ∫ (𝑥
2 −1
+ 1)𝑑𝑥 1M
1 3 3
3 𝑥2 1 𝑥2 1 𝑥2
= 2 [( 2 + 𝑥) ] − [( 2 − 7𝑥) ] − [( 2 + 𝑥) ] 1M
−1 2 1 2 −1
= 4 square units. 1M

28. The equation of family of planes through the intersection of the given planes is
2x + 3y – z + 1 + k( x + y – 2z + 3) = 0 1M
→(2 + k) x + (3 + k)y + (–1 –2k)z + 1 + 3k = 0 ---------------(1) ½M
Now since plane (1) is perpendicular to the plane 3x – y – 2z – 4 = 0
3(2 + k) –1(3 + k) – 2(–1 –2k) = 0 1M
→ k = –5/6 ½M
Therefore required plane equation is 7x + 13y + 4z = 9.------(2) 2M
Equation of xy plane z = 0
Dr’s of normal to xy plane: 0, 0 ,1
Dr’s of normal to plane (2): 7, 13, 4
Let ө be the angle between these planes then ө = cos-1(2√26/39) 1M
29. Let the number of units of grain transported from godown A to D = x
And the number of units of grain transported from godown A to E = y
Therefore the number of units of grain transported from godown A to F = 100 – (x+y)
Therefore the number of units of grain transported from godown B to D = 60 – x
Then the number of units of grain transported from godown B to E = 50 – y
Therefore the number of units of grain transported from godown B to F = x +y – 60
2M
Objective function C = 2.5x +1.5 y + 410, ½M
Subject to the constraints 60 – x ≥ 0, 50 – y ≥0, 100 – (x+y) ≥ 0 and x +y – 60 ≥ 0, x, y ≥ 0
L1 : x + y = 100, L2 : x + y= 60, L3 : x = 60, L4 : y = 50
Correct graph 1M
Corner points P =2.5x +1.5 y + 410
(60, 0) 560
(60,40) 620
(50,50) 610
(10,50) 510 Minimum 1M
Here transportation is minimum at point ( 10,50)
Hence the supplies should be as follows:
From godown A , 10 quintals, 50 quintals, and 40 quintals grains will be supplied to shops D, E,
and F respectively and from godown B, 50 quintals, 0 quintals and 0 quintals grains will be
supplied to shops D, E and F respectively. 1M
And the minimum cost = Rs.510. ½M

----xxxx----

GROUP-4

SUBMITTED BY:
NAGESH KUMAR MISHRA
PGT(MATHS)
KV IIT KANPUR

BLUE PRINT
CBSE Sample Paper
Class – XII
Subject: Mathematics (041)

S.No. UNIT VSA SA LA-I LA-II TOTAL


1M 2M 4M 6M Marks
1(a) Relations and Functions 1 1
10
(b) Inverse Trigonometric Functions 1 1
2 Matrices & Determinants 2 1 1 1 14
3(a) Continuity and differentiability 1 2
44
(b) Application of derivatives 1 1 1
(c) Integration 1 1 1
(d) Application of integrations 1
(e) Differential Equations 1
4(a) Vectors 2 16
(b) 3 – D Geometry 1 1 1
5 Linear Programming 1 6
6 Probability 1 2 10
TOTAL 4 8 11 6 100
SAMPLE QUESTION PAPER
CBSE - BOARD EXAMINATION
CLASS:XII TIME: 3 hrs
SUBJECT: MATHEMATICS MAX.MARKS:100

General Instructions:
(vii) All questions are compulsory.
(viii) This question paper contains 29 questions.
(ix) Question 1- 4 in Section A are very short-answer type questions carrying 1 mark each.
(x) Question 5-12 in Section B are short-answer type questions carrying 2 marks each.
(xi) Question 13-23 in Section C are long-answer-I type questions carrying 4 marks each.
(xii) Question 24-29 in Section D are long-answer-II type questions carrying 6 marks each.

SECTION – A

30. Let * be a binary operation defined by x*y = 5x + 7y. Find 2 * 4.

31. If Sin (sin—1(1/5) + cos—1x) = 1, then find the value of x.

32. If matrix A = (1 2 3), write AA’, where A’ is the transpose of the matrix A.

33. If f(x) = x2 – 5x + 7, write f(A), where A is a square matrix of order n.

SECTION – B

34. Evaluate: sin 100 – cos 100


sin 800 cos 800

35. Evaluate: ∫ 𝑡𝑎𝑛2 (2𝑥 + 1)𝑑𝑥

36. Find the derivative of log10 𝑥 w.r.t. x.

37. Find the angle ө between the vectors 𝑎⃗ and 𝑏⃗⃗ if | 𝑎⃗ . 𝑏⃗⃗ | = | 𝑎⃗ x 𝑏⃗⃗ |

38. Find the projection of the vector 𝑖̂ – 𝑗̂ on the vector 𝑖̂ + 𝑗̂ .

39. If a plane intersects the co-ordinate axes at (1,0,0), (0,1,0) and (0,0,1), find its equation.

40. A couple has 2 children. Find the probability that both are boys, if it is known that (i) one
of them is a boy (ii) the older child is a boy.

41. The sides of an equilateral triangle are increasing at the rate of 2 cm/sec. Find the rate at
which its area increases, when side is 10 cm long.
SECTION - C
4𝑥+3
42. If f(x) = , x ≠2/3, Show that f 0 f (x) = x, for all x ≠2/3. What is the inverse of f?
6𝑥−4
OR

Show that the function f:R→R defined by f(x) = 2x3 – 7, for x∊R is bijective.

43. Solve: 2 tan—1(cos x) = tan—1(2 cosec x).


OR
Prove that tan—1(63/16) = sin—1(5/13) + cos—1 (3/5).

44. Using properties of determinants, Prove that

a+b+2c a b
c b+c+2a b = 2(a+b+c)3
c a c+a+2b

45. Find the relation between a and b so that


f(x) = ax + 1, x ≤ 3
= bx + 3, x > 3 is continuous at x = 3.

𝑑𝑦
46. If x √1 + 𝑦 + y √1 + 𝑥 = 0, for |x| < 1, Prove that = –1 / (1 + x)2.
𝑑𝑥

OR
If x = a(cos t + t sin t) and y = a(sin t – t cos t), find d2y / dx2.

47. Find the intervals in which the function f(x) given by f(x) = sin x + cos x, 0 ≤ x ≤ 2π.
I) is strictly increasing ii) strictly decreasing

OR

Find the equation of tangent to the curve y = √3𝑥 − 2, which is parallel to the line
4x – 2y + 5 = 0.
1+sin 𝑥
48. Evaluate: ∫ 𝑒 𝑥 ( ) 𝑑𝑥
1+cos 𝑥

𝑥 𝑥
49. Solve: 2y 𝑒 𝑦 dx + ( y – 2x 𝑒 𝑦 ) dy = 0

50. Find the value of P so that the lines


1−𝑥 7𝑦−14 5𝑧−10 7−7𝑥 𝑦−5 6−𝑧
. = = and = = are perpendicular.
3 2𝑃 11 3𝑃 1 5

51. Given three identical boxes I, II and III, each containing two coins. In box I, both coins
are gold coins, in box II both are silver coins and in the box III there are one gold and one
silver coin. A person chooses a box at random and takes out a coin. If the coin is of gold,
what is the probability that the other coin in the box is also of gold?

52. Two numbers are selected at random (without replacement) from the first six positive
integers. Let X denotes the larger of the two numbers obtained. Find the expectation E(X)
of the random variable X.

SECTION – D

1 −1 2 −2 0 1
53. Use product (0 2 – 3) ( 9 2 −3) to solve the system of equations
3 −2 4 6 1 −2
x – y + 2z = 1, 2y – 3z = 1, 3x – 2y + 4z = 2.

54. Show that the height of the cylinder of greatest volume which can be inscribed in a right
circular cone of height h and semi vertical angle  is one-third that of the cone and the
4
greatest volume of cylinder is π h3 tan2.
27

(OR)

A tank with rectangular base and sides, open at the top is to be constructed so that its depth
is 2 m and volume is 8 m3. If building of tank costs Rs.70 per square meter for the base
and Rs.45 per square meter for sides. What is the cost of least expensive tank?
𝜋 𝑥 tan 𝑥
55. Evaluate : ∫0 𝑑𝑥.
sec 𝑥+tan 𝑥

56. Find the area of the region enclosed between the two circles x2 + y2 = 9 and (x –
3)2 + y2 = 9.
OR
Use the integration, find the area of triangle ABC, with the vertices A( --1, 0), B( 1, 3) and
C(3,2).
57. Find the equation of the plane passing through the intersection of the planes 2x + 3y – z +
1 = 0; x + y – 2z + 3 = 0 and perpendicular to the plane 3x – y – 2z – 4 = 0. Also find the
inclination of this plane with xy plane.

58. Two godowns A and B have grain capacity of 100 quintals and 50 quintals respectively.
They supply to 3 ration shops, D, E and F whose requirements are 60, 50, and 40 quintals
respectively. The cost of transportation per quintal from the godowns to the shops is given
in the following table.

Transportation cost per quintal ( in Rs)


From/To A B
D 6 4
E 3 2
F 2.50 3

How should the supplies be transported in order that the transportation cost is
minimum? What is the minimum cost? Write any one value reflected in the problem.

-----xxxx----
MARKING SCHEME
CBSE Sample Paper
Class – XII
Subject: Mathematics (041)

SECTION- A
30. 38 1M
31. 1/5 1M
32. AA1 = (14) 1M
33. f(A) = A2 – 5 A + 7 I, where I is an identity matrix of order n. 1M

SECTION - B

34. 1 2M
35. ½ tan(2x + 1) – x + c 2M
1
36. 𝑥 log10 𝑒 2M
37. π/4 2M
38. 0 2M
39. x + y + z = 1 2M
40. Sample space = { B1B2, B1G2, G1B2, G1G2} , B1 and G1 are the older boy and girl respectively.
Let E1 = both the children are boys;
E2 = one of the children is a boy ;
E3 = the older child is a boy
𝐸1∩𝐸2 1/4
Then, (i) P(E1/ E2) = P ( )= = 1/3 1M
𝐸2 3/4

𝐸1∩𝐸3 1/4
(ii) P(E1/ E3) = P ( 𝐸3
)= 2/4 =½ 1M
3
41. Here, Area(A) = √4 X2 where ‘x’ is the side of the equilateral triangle. 1M

𝑑𝐴
= 10√3 cm2/sec 1M
𝑑𝑡

SECTION – C

4𝑥+3
42. f0f(x) = f[f(x)] = f (6𝑥−4) ½M
f0f(x) = x 1½M
4𝑥+3
f—1(x) = 6𝑥−4 2M

OR
Proving one-one 2M
Proving onto 2M

2 cos 𝑥
43. tan−1 (1− 𝑐𝑜𝑠2 𝑥)= tan—1(2 cosec x) ½M
2 cos x/sin2x = 2 cosec x. ½M
2 cosecx (cot x – 1) = 0 1M
Cosec x = 0 or cot x = 1
Cosec x = 0 is not possible 1M
Cot x = 1
x = π/4 1M
OR
Sin 5/13 = tan—1 5/12
—1 cos—1 3/5 = tan—1 4/3 1M
5
+4/3
tan—1 5/12 + tan—1 4/3 = tan−1 ( 12
5 ) 2M
1− .4/3
12
63
= tan−1 16 1M
44. Operate: C1→ C1+ C2+ C3 1M
Taking 2(a+b+c) common from C1 ½M
R1 →R2 – R1, R3 →R3 – R1
Taking (a+b+c) common from R2 and R3 1M
For expanding the determinant and for result 1½M
45. LHL: Lt f(x) = 3a + 1 1M
x→3—
RHL: Lt f(x) = 3b + 3 1M
x→3+
since f is continuous at x = 3, LHL = RHL 1M
a = b + 2/3 1M
−𝑥
46. For bringing y = 1+𝑥 2M
dy/dx = –1 / (1 + x)2. 2M
OR

dx/dt = at cos t 1M

dy/dt = at sin t 1M
dy/dx = tan t 1M

d2y/dx2 = sec3 t/at 1M

47. f1(x) = cos x – sin x


f1(x) = 0 x = π/4, 5π/4 1M
f1(x) > 0 if x ∊(0, π/4) U (5π/4, 2π) 1M
f1(x) < 0 if x ∊ (π/4, 5π/4) 1M
f is increasing on (0, π/4) U (5π/4, 2π)
f is decreasing on (π/4, 5π/4) 1M

OR

Let (a, b) be the point of contact


b2 = 3a – 2 1M
𝑑𝑦
.(𝑑𝑥 )(a,b) = 3/2b 1M
3 −4
.2𝑏 = −2
a = 41/48, b = 3/4 1M
Equation of tangent: 48 x – 24 y – 23 = 0 1M

1 sin 𝑥
48. ∫ 𝑒 𝑥 (1+cos 𝑥 + 1+cos 𝑥) 𝑑𝑥 1M

1
= ∫ 𝑒 𝑥 (2 𝑠𝑒𝑐 2 𝑥/2 + tan 𝑥/2) 𝑑𝑥 1½M
𝑥
=𝑒 𝑥 tan 2 + c (∫ 𝑒 𝑥 (𝑓(𝑥) + 𝑓 1 (𝑥) ). 𝑑𝑥 = ex f(x) + c 1½M
𝑥
𝑑𝑥 2𝑥𝑒 𝑦 − 𝑦
49. 𝑑𝑦 = 𝑥
2𝑦𝑒 𝑦
𝑥 1
=𝑦– 𝑥 1M
2𝑒 𝑦
𝑥 𝑑𝑥 𝑑𝑣
Put 𝑦 = 𝑣 =𝑣+𝑦 ½M
𝑑𝑦 𝑑𝑦
𝑑𝑣 1
v + y 𝑑𝑦 = 𝑣 − ½M
2𝑒 𝑣
2 ∫evdv = -∫dy/y 1M
𝑥
2𝑒 𝑦 = -log 𝑦 + 𝑐 P 1M

50. Writing the given lines in standard form 2M


Perpendicular condition a1b1 + a2b2 + a3b3 = 0 ½M
Substitution of values ½M
Solving for P = 7 1M
51. Let E1, E2, E3 be the events that boxes I, II, III are chosen respectively.
A be the event that coin drawn is of gold. ½M
P(E1) = 1/3 P(A/E1) = 1
P(E2) = 1/3 P(A/E2) = 0
P(E3) = 1/3 P(A/E3) = ½ 1½M

By Bayes theorem

P(E1/A) = P(E1) P(A/E1)


P(E1) P(A/E1) + P(E2) P(A/E2) + P(E3) P(A/E3) ½M
= 1/3 x 1
1/3 x1 + 1/3 x 0 + 1/3 x ½ ½M
= 2/3 1M
52. First six positive integers are 1, 2, 3,4,5,6.
1 is the smallest positive integer
Therefore X = 2,3,4,5,6. 1M
P(X = 2) = 2/30 ½M
P(X = 3) = 4/30 ½M
P(X = 4) = 6/30 ½M
P(X = 5) = 8/30 ½M
P(X = 6) = 10/30 ½M
E(X) = ∑X P(X) 1M
= 2 x 2/30 + 3 x 4/30 + 4 x 6/30 + 5 x 8/30 +6 x 10/30 1M
= 14/3 ½M

53.
1 −1 2 −2 0 1
(0 2 – 3) ( 9 2 −3) = I 1½M
3 −2 4 6 1 −2

−2 0 1
A-1 = ( 9 2 −3) 1M
6 1 −2
Writing the given equations in matrix form AX = B
and X = A-1 B 1M
0
solving for X = (5) 1M
3

54. Let r be the radius and x be the height of the cylinder, then
𝑟
tan  = ℎ−𝑥 1M
Volume of cylinder V = πr2x
= π tan2 (h2x + x3 – 2h x2) 1M
𝑑𝑉
.𝑑𝑥 = π tan2  ( h2 + 3x2 – 4 hx) 1M
dV/dx = 0 x = h/3 1M
2 2
(d V/dx )x = h/3 < 0 1M
V is maximum when x = h/3 and
4
Maximum Volume = 27 π h3 tan2 1M
(OR)

Let length = x, breadth = y, h = 2 m

Given V = 8 m3 y = x/4 1M

Cost of tank = 70 xy + 45x 4(x + Y)

= 280 + 180(x + 4/x) = C (say) 1M


dC/dx = 180(1 – 4/x) 1M

dC/dx = 0 x=2 1M

(d2C / dx2)x=2 > 0 1M

Therefore cost is minimum when x = 2

Min. Cost = Rs.1000. 1M


𝜋 𝑥 tan 𝑥
55. I = ∫0 𝑑𝑥.
sec 𝑥+tan 𝑥

𝜋 (𝜋−𝑥) tan(𝜋−𝑥)
= ∫0 𝑑𝑥. 1M
sec(𝜋−𝑥)+tan(𝜋−𝑥)
𝜋 tan 𝑥
2I = 𝜋 ∫0 sec 𝑥+tan 𝑥 𝑑𝑥. 1½M
= π{sec x – tanx + x}0π 2M
= π(π – 2) 1M
𝜋
I = 2 (π – 2) ½M
56. Y2 = 9 – x2 -------- (1)
Y2 = 9 – (x – 3)2 ----------- (2)
Solving (1) and (2), x = 3/2
Correct figure 1½M
3/2 3
Area = 2{∫0 √9 − (𝑥 − 3)2 𝑑𝑥 + ∫3/2 √9 − 𝑥 2 𝑑𝑥 } 1M
𝑥−3 9 𝑥−3 3/2 𝑥 9 𝑥 3
2[ √9 − (𝑥 − 3)2 + sin−1 ( )] + 2[2 √9 − 𝑥 2 + 2 sin−1 (3)] 1½M
2 2 3 0 3/2
.
−3 9 9 −1 9
2[ 4 √9 − 4 + sin−1 ( 2 ) − 2 sin−1(−1)] +
2
.
.
9 −1 (1) 3 9 9 1
2[2 sin − 4 √9 − 4 − sin−1 (2)] 1M
2
.
= 6π – 9√3/2 square units. 1M
(OR)
Correct figure 1M
Equation of line AB: y = 3/2(x+1)
Equation of line BC: y = -1/2 (x – 7)
Equation of line AC: y = ½ (x+1) 1½M
. . .
Required area of triangle = ∫𝐴𝐵 𝑦 𝑑𝑥 + ∫𝐵𝐶 𝑦 𝑑𝑥 − ∫𝐴𝐶 𝑦 𝑑𝑥 ½M
3 1 1 3 1 3
= 2 ∫−1(𝑥 + 1)𝑑𝑥 − 2 ∫1 (𝑥 − 7)𝑑𝑥 − ∫ (𝑥
2 −1
+ 1)𝑑𝑥 1M
1 3 3
3 𝑥2 1 𝑥2 1 𝑥2
= 2 [( 2 + 𝑥) ] − [( 2 − 7𝑥) ] − [( 2 + 𝑥) ] 1M
−1 2 1 2 −1
= 4 square units. 1M

57. The equation of family of planes through the intersection of the given planes is
2x + 3y – z + 1 + k( x + y – 2z + 3) = 0 1M
→(2 + k) x + (3 + k)y + (–1 –2k)z + 1 + 3k = 0 ---------------(1) ½M
Now since plane (1) is perpendicular to the plane 3x – y – 2z – 4 = 0
3(2 + k) –1(3 + k) – 2(–1 –2k) = 0 1M
→ k = –5/6 ½M
Therefore required plane equation is 7x + 13y + 4z = 9.------(2) 2M
Equation of xy plane z = 0
Dr’s of normal to xy plane: 0, 0 ,1
Dr’s of normal to plane (2): 7, 13, 4
Let ө be the angle between these planes then ө = cos-1(2√26/39) 1M
58. Let the number of units of grain transported from godown A to D = x
And the number of units of grain transported from godown A to E = y
Therefore the number of units of grain transported from godown A to F = 100 – (x+y)
Therefore the number of units of grain transported from godown B to D = 60 – x
Then the number of units of grain transported from godown B to E = 50 – y
Therefore the number of units of grain transported from godown B to F = x +y – 60
2M
Objective function C = 2.5x +1.5 y + 410, ½M
Subject to the constraints 60 – x ≥ 0, 50 – y ≥0, 100 – (x+y) ≥ 0 and x +y – 60 ≥ 0, x, y ≥ 0
L1 : x + y = 100, L2 : x + y= 60, L3 : x = 60, L4 : y = 50
Correct graph 1M
Corner points P =2.5x +1.5 y + 410
(60, 0) 560
(60,40) 620
(50,50) 610
(10,50) 510 Minimum 1M
Here transportation is minimum at point ( 10,50)
Hence the supplies should be as follows:
From godown A , 10 quintals, 50 quintals, and 40 quintals grains will be supplied to shops D, E,
and F respectively and from godown B, 50 quintals, 0 quintals and 0 quintals grains will be
supplied to shops D, E and F respectively. 1M
And the minimum cost = Rs.510. ½M

----xxxx----

SAMPLE QUESTION PAPER ( Group IV)

MATHEMATICS

CLASS XII

TIME : 3 HOURS Maximum Marks: 100

General Instructions:

(xiii) All questions are compulsory.


(xiv) This question Paper contains 29 questions.
(xv) Question 1 - 4 in Section A are very short-answer type questions carrying 1 mark each.
(xvi) Question 5 - 12 in section B are short answer type question carrying 2 marks each.
(xvii) Question 13 - 23 in section C are long answer type I question carrying 4 marks each .
(xviii) Question 24 - 29 in section D are long answer type II question carrying 6 marks each .

SECTION - A

3. State the reason why the Relation R = { ( a , b ) : a ≤ 𝑏 3 } on the set R of real numbers is not

Reflexive.
0
2. Evaluate |cos 700 sin 200 |
𝑠𝑖𝑛70 𝑐𝑜𝑠200

𝑎 = 2𝑖̂ - 𝑗̂ + 2𝑘̂ and 𝑏⃗⃗ = -𝑖̂ + 𝑗̂ + 3𝑘̂


3. Find the unit vector in the direction of the sum of the vectors ⃗⃗⃗⃗

4. The binary operation * on R is denoted by a * b = 2a + b . Find ( 2 * 3 )


SECTION – B
1 1 2
5. Prove thattan−1 + tan−1 = tan−1
7 13 9

5 2 3 6
6. Find Matrix X and Y , if X + Y = | | and X - Y = | |
0 9 0 −1
2𝑥
7. Differentiate with respect to x , sin−1 (1+ 𝑥2 )

8. Find the rate of change of area of a circle with respect to its radius ‘ r ‘ when r = 6 cm.

9. Evaluate ∫ sin−1(cos 𝑥) 𝑑𝑥

10. Find the differential of the family of all straight lines.

11. Find the position vector of the mid – point of the vector joining the points P ( 2 , 3 , 4 ) and

Q ( 4 , 1 , -2 ).
3 7 9
12. If A and B are the two events such that P ( A ) = 5
, P(B)= 10
and P ( A ∪ 𝐵 ) = 10
, then find

P ( A∩ 𝐵 ) .

SECTION – C

13. Using properties of determinants , show that

1  a 2  b2 2ab 2b
 1  a 2  b 2 
3
2ab 1  a 2  b2 2a
2b 2a 1  a 2  b2

sin 𝑥
+ cos 𝑥, 𝑥 > 0
𝑥
14. Show that the function f (𝑥) given by 𝑓(𝑥) = 2 ,𝑥 = 0
4(1−√1−𝑥)
{ ,𝑥 < 0
𝑥

is continuous at 𝑥 = 0.

OR
1−𝑐𝑜𝑠4𝑥
, 𝑥>0
𝑥2
If the function f defined by f(x) = 𝑎 , 𝑥=0
√𝑥
, 𝑥>0
{ √16+√𝑥 −4 }
is continuous at x = 0, find the value of a.
−1 𝑥 𝑑2𝑦 𝑑𝑦
15. If y = 𝑒 𝑚 cos , prove that (1-𝑥 2 ) -x = m2y.
𝑑𝑥 2 𝑑𝑥

16. Find the intervals in which the function

1 − 12𝑥 − 9𝑥 2 − 2𝑥 3 is increasing or decreasing.

OR
Find the equation of the normal line to the curve 𝑦(𝑥 − 2)(𝑥 − 3) − 𝑥 + 7 = 0 at the point where

it meets the 𝑥 −axis.

17. Show that height of the cylinder of maximum volume that can be inscribed in a

2R
sphere of radius R is . Also find the maximum volume.
3

x 4 dx
18. Evaluate   x  1  x2  1

19. Find the general solution of the differential equation: (1 + 𝑦 2 )𝑑𝑥 = (tan−1 𝑦 − 𝑥)𝑑𝑦.

OR
𝑥 𝑥
Solve the differential equation : 2𝑦. 𝑒 𝑦 𝑑𝑥 + (𝑦 − 2𝑥𝑒 𝑦 ) 𝑑𝑦 = 0 .

20. Express the vector 𝑎⃗ = 5𝑖̂ − 2𝑗̂ + 5𝑘̂ as the sum of two vectors such that one is parallel to the

vector𝑏⃗⃗ = 3𝑖̂ + 𝑘̂ and other is perpendicular to 𝑏⃗⃗ .

21. Find the shortest distance between the lines whose vector equations are

𝑟⃗ = (1 − 2𝑡)𝑖̂+(1 − 𝑡)𝑗̂+(𝑡)𝑘̂ and


𝑟⃗ = (2 + 3𝑠)𝑖̂ +(1 − 5𝑠)𝑗̂+(2𝑠 − 1)𝑘̂

22. A speaks truth in 70% of the cases and B speaks truth in 80 % of the cases .In what

percentage of the cases :-

(i) They contradict each other in stating the same fact?

(ii) They agree each other in stating the same fact?

Truth is very closely related with our national movement. How?

23. By examining the chest X-ray , the probability that TB is detected when a person actually

suffering is 0.99 . The probability of incorrect diagnosis is 0.001. In a certain city one in

thousand persons suffer from TB . A person selected at random and is diagnosed to have TB.

What is the chance that he actually has TB.

SECTION – D

24. Let f : N → R be a function defined as f (x ) = 4x2 + 12x + 15 . Show that f : N → 𝑆 , where S is


the range of f , is invertible. Also find the inverse of f .

OR

A binary operation * is defined on the set X = R – { - 1 } by x * y = x + y + xy , ∀ x , y ∈ X.


Check whether * is commutative and Associative. Find its identity element and also find the
inverse of each element of X.

25. Two school A and B decided to award prizes to their students for three values

honesty(x),punctuality (y) and obedience(z).School A decided to award a total of Rs 11000

for the three values of 5,4 and 3 students respectively while school B decided to award

Rs 10700 for the three values of 4, 3 and 5 students respectively. If all the three prizes

together amount to Rs. 2700, then:

(v) Represent the above situation by a matrix equation and form linear equations using matrix
multiplication.
(vi) Is it possible to solve the system of equation so obtained using matrix? If yes, find the
award money for each value

OR

Using the properties of determinants , prove that:

𝑥 𝑥2 1 + 𝑝𝑥 3
|𝑦 𝑦2 1 + 𝑝𝑦 3 | = (1 + 𝑝𝑥𝑦𝑧)(𝑥 − 𝑦)(𝑦 − 𝑧)(𝑧 − 𝑥)
𝑧 𝑧2 1 + 𝑝𝑧 3

26. Using integration, find the area of the triangle whose vertices are 𝐴(1,0), 𝐵(2,2) 𝑎𝑛𝑑 𝐶(3,1)

 /2 x
27. .Evaluate 
0 sin x  cos x
dx

OR
2
Evaluate the following integral as a limit of a sum   3x  1 dx.
2

28. Find the equation of the plane passing through the point (-1, 3, 2) and perpendicular to each

of the planes 𝑥 + 2𝑦 + 3𝑧 = 5 and 3𝑥 + 3𝑦 + 𝑧 = 0. Also find the angle between this plane

and x-axis.

29. A dealer wishes to purchase number of fans and sewing machines . He has only Rs. 5760 to
invest and has a space for at most 20 items . A fan cost him Rs. 360 and sewing machine Rs. 240 .

His expectation is that .He can sell a fan at profit of Rs. 22and sewing machine at a profit of

Rs.18 . Assuming that he can sell all the items that he can buy , how should invest his money in

order to maximize the profit ? Formulate this as a linear programming problem and solve it

graphically .
MARKING SCHEME

26. For Reflexivity ( a , a ) ∈ R


1 1
Clearly ½ is a real no. , but 2 is not less than or equal to (2)3
1 1
( , ) not belongs to R
2 2
Hence , R is not Reflexive. 1 mark
27. Cos 700.cos200 - sin 700 sin 200
= cos (70 +20 ) = 0 1 mark
28. For getting unit vector 1mark
29. a * b = 2a + b
2*3=7 1 mark
1 1
30. tan−1 7 + tan−1 13
Applying formula 1 mark
2
= tan−1 91 mark
4 4
31. Getting value of X = [ ] 1 mark
0 4
1 −2
Y =[ ] 1 mark
0 5
32. Taking x = tan𝜃 ,
Getting y = 2𝜃 1 mark
𝑑𝑦
For getting 𝑑𝑥
=2 / ( 1+x2) 1 mark
33. For getting dA/dr = 2𝜋r 1 mark
(dA/dr ) at r = 6 = 12 𝜋 cm2/cm 1 mark
34. ∫ sin−1 cos x dx
𝜋
Putting cos x = sin ( 2 − x ) 1mark
𝜋
To get the result x – x2 / 2 + c 1 mark
2

35. For equation y = mx + c 1 mark


𝑑𝑦
𝑑𝑥
= m &for second derivative = 0 1 mark
36. For writing 𝑎⃗ =2i +3j+4k and 𝑏 ⃗⃗ = 4𝑖̂ + 𝑗 − 2𝑘̂ 1 mark
For finding P.V.ofmid point= 3i+2j+k 1 mark
37. For writing P(AUB)= P(A)+p(B)-P(A∩B) 1 mark
2
For getting correct answer P(A∩B) = 5
1 mark

1  a 2  b2 2ab 2b
2ab 1 a  b
2 2
2a
13.
2b 2a 1  a 2  b2
R1  R1  bR2 , R2  R2  aR3

………………………………(1+1/2)

1  a 2  b2 0 b 1  a 2  b 2 
0 1  a 2  b2 a 1  a 2  b 2 
2b 2a 1  a 2  b2

1 0 b
1  a 2
b 
2 2
0 1 a ………………………………(1+1/2)
2b 2a 1  a 2  b 2

1  a  b  1  a
2 2 2 2
 b2  2a 2  2b2 
………………………………..(1)
 1  a  b 
2 2 3

14. For finding: LHL=2 (1 mark)

For finding: RHL=2 (1 mark)

For finding:f (0) =2 (1 mark)

LHL=RHL= f(0) , therefor function is continuous at x=0 (1 mark)

OR

To find LHL= lim− 𝑓(𝑥) = 8


𝑥→0
To find RHL= lim+ 𝑓(𝑥)= 8
𝑥→0
Value = a =8
𝑑𝑦 𝑚𝑦
15. =-
𝑑𝑥 √1−𝑥 2
𝑑𝑦 2𝑥
√1−𝑥 2 ∗(−𝑚) −(−𝑚𝑦)
𝑑2𝑦 𝑑𝑥 2√1−𝑥2
= 2
𝑑𝑥 2 √1−𝑥 2

𝑑2𝑦 𝑑𝑦
(1-𝑥 2 ) -x = m2y.
𝑑𝑥 2 𝑑𝑥

16.

 𝑓 ′(𝑥) = −6(𝑥 + 1)(𝑥 + 2)  1


 Finding critical points 𝑥 = −1, −2  1
 Finding intervals for increasing and decreasing functions  1
 Writing answer: Increasing in (−2, −1), Decreasing in (−∞, −2) ∪ (−1, ∞)  1

OR

 Finding the point (7,0)


 1
𝑑𝑦
 Finding 𝑑𝑥  1
𝑑𝑦 1
 Finding 𝑑𝑥 at (7,0) = 20 
1
2

 Slope of normal −20 


1
2

 Finding equation of normal:20𝑥 + 𝑦 = 140


 1

17.

Let r and h be the radius and height of the cone respectively inscribed in a sphere of radius R.

1
mark

Let V be the volume of the cone.

Then,

Height of the cone is given by,

h = R + AB 1mark

1
mark
2
marks

∴ By second derivative test, the volume of the cone is the maximum when
1
mark
1
mark

𝑋4 𝑥 4 − 1+1
18. ∫ (𝑥−1)(𝑥2 +1 ) 𝑑𝑥 = ∫ (𝑥−1)(𝑥2 +1 ) 𝑑𝑥

𝑥4− 1 1
= ∫ (𝑥−1)(𝑥2 +1 ) 𝑑𝑥 + ∫ (𝑥−1)(𝑥2 +1 ) 𝑑𝑥

1
= ∫(𝑥 + 1 )𝑑𝑥 + ∫ (𝑥−1)(𝑥2 ) 𝑑𝑥
+1

𝑥2 1 𝑑𝑥 1 𝑥+1
= 2
+ 𝑥 + ∫
2 𝑥−1
− ∫
2 (𝑥 2 +1 )
𝑑𝑥

𝑥2 1 1 1
= + 𝑥 + 𝑙𝑜𝑔|𝑥 − 1| − log|(𝑥 2 + 1 )| - tan−1 𝑥 + 𝑐
2 2 4 2
19

𝑑𝑥
Writing the equation in the form: 𝑑𝑦 + 𝑃(𝑦). 𝑥 = 𝑄(𝑦) 𝑖. 𝑒.
𝑑𝑥 𝑥
+ 1+𝑦2 =  1
𝑑𝑦
tan−1 𝑦
1+𝑦 2
……… (1)

 Writing 𝑃(𝑦) = 1+𝑦2 , 𝑄(𝑦) =


1 tan−1 𝑦  1/2
1+𝑦 2

−1 𝑦
 Integrating factor = 𝑒 tan  1

 Multiplying integrating factor in (1) and changing the equation in form:  1/2

−1 𝑦 tan−1 𝑦 −1 𝑦
𝑒 tan . 𝑥 = ∫ 𝑒 tan
𝑑𝑦 .
1 + 𝑦2
−1
 Finding the general solution: 𝑥 = tan−1 𝑦 − 1 + 𝑐 𝑒 −tan 𝑦  1

OR
𝑥
𝑑𝑥 2𝑥𝑒 𝑦 −𝑦
 Writing D.E. in the form: 𝑑𝑦
= 𝑥 ………. (1)
2𝑦.𝑒 𝑦
 1
 Putting 𝑥 = 𝑣𝑦 𝑎𝑛𝑑
𝑑𝑥
=𝑣+
𝑑𝑣
𝑦 𝑑𝑦 ………….. (2)  1
𝑑𝑦
and converting D.E. in the form variable separable as
 1
𝑑𝑦
2𝑒 𝑣 𝑑𝑣 = −
𝑦

 Integrating both sides and getting general solution


𝑥
2. 𝑒 𝑦 = − log|𝑦| + 𝐶  1

⃗⃗⃗⃗1 +𝑏
20. .Let 𝑎⃗=𝑏 ⃗⃗⃗⃗⃗2

⃗⃗⃗⃗
𝑏1 =𝛽(3𝑖̂+𝑘̂)

⃗⃗⃗⃗⃗
𝑏2 =𝑥̂i+𝑦̂j+𝑧̂ k Marks 1

𝐹raming equation 3x+z=0 Marks 1

Ans𝛽=2 x=-1,y=-2,z=3 Marks 2

21. ⃗⃗⃗⃗⃗ 𝑎1 = 𝑖̂ − 𝑘̂
𝑎2 − ⃗⃗⃗⃗⃗ (1 mark)

⃗⃗⃗⃗⃗⃗⃗⃗⃗⃗⃗⃗⃗⃗⃗⃗
𝑏1 𝑋𝑏⃗⃗⃗⃗⃗2 = 3𝑖̂ − 𝑗̂ − 7𝑘̂ (1 mark)

⃗⃗⃗⃗1 𝑋𝑏
|𝑏 ⃗⃗⃗⃗⃗2 | = √59 (1 mark)
10
Shortest distance between the line:√59 (1 mark)

22. (i) They contradict each other in stating the same fact = 38% ( 1.5 mark)
(ii) They agree each other in stating the same fact= 0.62% (1.5 marks)

For value (1 mark)

23. Consider A1 : suffers from TB, P  A1   0.001 ------------------------------------(1)

A2 : person donot suffer fromTB, P  A2   0.999        (1)

C: Doctor diagnoses correctly

Then P(C / A1 )  0.99 and P(C / A2 )  0.001        (1)

P( A1 ).P(C / A1 ) 110
By Baye ’s theorem P( A1 / C )          (1)
P( A1 ).P(C / A1 )  P( A2 ).P(C / A2 ) 221

24. For proving one-one (2 Marks)

For proving onto (2 Marks)

For finding inverse (2 Marks)

OR

5 4 3 𝑥 11000
𝑦
25. [4 3 5] [ ] = [10700]
1 1 1 𝑧 2700
5x+4y+3z=11000

4x+3y+5z=10700

x +y + z=2700 (1marks)

for getting |A|= -3 (1 marks)

−2 −1 11
−1
getting A-1= 3 [ 1 2 −13] (2 marks)
1 −1 −1
getting x=1000,y=900 and z=800 (2 marks)

OR

For Commutative (2 marks)

For Associative (2 marks)

For finding Identity (1 mark)

For finding Inverse (1 mark)

25.
𝑥 𝑥2 1 𝑥 𝑥2 𝑝𝑥 3
|𝑦 𝑦2 1|+|𝑦 𝑦2 𝑝𝑦 3 |Mark 2
𝑧 𝑧2 1 𝑧 𝑧2 𝑝𝑧 3

Similarly 2 marks for using each properties

Getting proof (Marks 2)

26. Drawing correct rough sketch and obtaining point of intersection of given

lines ( 2 marks)
2 3 3 𝑥−1
𝑎𝑟𝑒𝑎 𝑜𝑓 ∆𝐴𝐵𝐶 = ∫1 2(𝑥 − 1)𝑑𝑥 + ∫2 (4 − 𝑥)𝑑𝑥 − ∫1 𝑑𝑥 (2 marks)
2

3
𝑎𝑟𝑒𝑎 𝑜𝑓 ∆𝐴𝐵𝐶 = 2 (2 marks)


2
x
27. I   dx
0
sin x  cos x


 2
1
2  sin x  cos x
2I  dx ……………………………….(1)
0


 2
1
2
2I  dx …………………………(1)
2 1 1
0 sin x  cos x
2 2

 2
  
2 
2I  cos ec  x   dx ………………………….(2)
2 0  4


I
2 2
log  
2 1 …………………………………………(2)

OR

By definition of limit sum we have

  3x  1dx  lim h  f 1  f 1  h   f 1  2h       f 1   n  1 h  


2
h 0
1

where nh  1              (1)

3 1  1  1     ntimes   1  1      ntimes  
 
= lim h 
h 0 

 3 h 2  22 h 2        n  1 h 2
2

      (1)


  3.2  h  2h        n  1 h  

 n  n  1 2n  1 n(n  1) 
= lim h  2n  3h   6h    6            (2)
2
h0
 6 2 
28. Equation of plane passing through (-1, 3,2) is A(x+1)+B(y-3) +C(z-2)=0 (1 mark)

Finding (A, B,C) = (-7, 8 -3) (2 marks)

Getting equation of plane: 7x-8y+3z+25=0 (2 marks)


7
Finding angle with x axis 𝑐𝑜𝑠 −1 √122 (1 mark)

29. Let x be the no. of fans and y be the no. of sewing machines

If p be the total profit ,

P= 22x+18y

360x+240y≤5760. For correct graph, ---------- (1)

i.e., 3x+2y≤48.

x+y≤20.

x≥0,y≥0 -------------------------------- (2)

E(0,24)

(0,20)D

C(8,12)

O (16,0)A (20,0)

po int s valueof p
(0, 0) 0
(16, 0) 352
(8,12) 392
(0, 20) 360

the dealer gets a maximum profit of Rs. 392 when he purchase and sells 8 fans and 12 sewing machines.

Investment in fans =360.×8=2880.


Investment in sewing machines =240×12= 2880.-------------------(3)

Sample Question Paper(Group –IV)

Sub: Maths MM 100 Duration: 3 hours

General Instructions:
1. All questions are compulsory.

2. This question paper consists of 29 questions divided into Four sections A, B, C andD. Section A comprises of
4 questions of one mark each, section B comprises of 8 questions of 02 marks each and section C comprises of
11 questions of 04 marks each and section D consists of 06 questions of 06 marks each.

________________________________________________________________________

SECTION - A (1 mark questions)

1. Let * be a binary operation on Q given by a*b=a+ab where a,b  Q .Is * commutative?


1  4 
2. Evaluate Sin  cos 1   
2  5 
3. Given that A, B are two symmetric matrices such that AB =BA .Is AB symmetric?
 x  y  z   9
   
4. Find the values of x, y and z if  x  z    5 
 y  z   7
   

SECTION - B (2 mark questions)

Sin30 0 Cos30 0
5. Evaluate
 sin 60 0 Cos60 0
6. Find a point on the curve y=x2-4x+5 where the tangent to the curve is parallel to the x axis
x  cos 6 x
7. Evaluate  3x 2
 Sin6 x
dx
      
8. Find the projection of the vector a  i  3 k on the vector b  3 i  j  4 k
     
9. If a is a unit vector and ( x  a )( x  a )  15 find x

2x  1 y2 z3
10. The Cartesian equation of a line AB is   .Find the direction cosines of a line
3 2 3
parallel to A

11. Let f: R  R be defined by f(x) = 3x+2.Show that f is invertible. Also find f  1 :R  R


 1 
12. Write in the simplest form : tan  1   where x  1
 
 x 1
2
SECTION - C (4 marks questions)

x 6 1
13. Show that x = 2 is a root of the equation 2  3 x x  3  0 and solve it completely.
 3 2x x2
 1  Cos2 x x0

 x2
14. Discuss the continuity of the function at x = 0: f ( x)  


5 x0

 
15. Verify Rolle’s theorem for the function f(x)= Sin2x in 0, 
 2
dy 1  y2
16. If 1  x 2  1  y 2  a( x  y ) prove that 
dx 1  x2
OR

If y=x2 +4 and x changes from 2 to 2.1 find the approximate change in y

2
17. Evaluate  e x dx as a limit of sum
0
OR

5
Evaluate   x  1x 2
4 dx
dy
18. Solve the differential equation Cos x
2
 y  tan x
dx
19. Find the differential equation for the family of circles which passes through the origin and have their
centre on the x axis
       

20. If a  b  c  0 and a  3 ; b  5 c  7 then show that the angle between a and b is 600

21. Find the vector equation of the plane passing through the intersection of the planes r .
      
( 2 i  7 j  4 k )  3 and r . ( 3 i  5 j  4 k )  11  0 and passing through the point (-2,1,3)
22. A coin is tossed three times and all the possible outcomes are assumed to be equally likely. Let E and F
be two events given by E: “both tail and head have occurred” F: “at most one tail has occurred”. Show
that E and F are independent.

23. The sum of three numbers is 6.Twice the third number when added to the first number gives 7.On
adding the sum of the second and third numbers to thrice of the first number we get 12.Find the
numbers using inverse of a matrix.
Or

 1 2 3 

Using elementary transformations find the inverse of the matrix 2 5 7 

  2  4  5

SECTION - D (6 mark questions)


24. Let R be a relation on N  N defined by (a, b)R (c, d)  ad=bc, for all (a,b) and (c,d)  N . Show that R is
an equivalence relation .
OR

Let * be the binary operation defined on QxQ by (a,b)*(c,d) = (ac,b+ad) where Q is the set of
rational numbers. Determine whether * is commutative and associative . Find the indentiy
element for * and the inverttible elements of QxQ

25. An open box with a square base is to be made out of a given quantity of metal sheet of area c 2.Show
c3
that the maximum volume of the box is
6 3

1 2
26: Evaluate  Sinx( 2  Cosx )
dx Or Evaluate  log Sinxdx
0

27: Find the area of region included between the parabolas y2=4ax and x2=4ay where a>0

28: Find the foot of the perpendicular drawn from the point A(1,0,3) to the line joining the points B(4,7,1)
C(3,5,3)

29: Mona wants to invest at most Rs 12000 in Savings Certificates (SC) and National Saving Bond (NSB). She
has to invest at least Rs 2000 in SC and at least Rs 4000 in NSB. If the rate of interest on Sc is 8% and the
rate of interest on NSB is 10% per annum, how much money should she invest to earn maximum yearly
income?

Sample Question paper(Group-IV)


SUBJECT: MATHEMATICS CLASS: XII
Time: 3hrs Max. Marks:100

GENERAL INSTRUCTIONS:
1 All questions are compulsory.
2 The question paperconsists of 29questions divided into four sections A, B ,C& D. Section A comprises
4 questions of one mark each, section B comprises of 8 questions of 2 marks each, section C comprises
of 11 questions of 4 marks each and section D consists of 6 questions of 6 marks each.
3.All questions in section A are to be answered in one word, one sentence or as per the exact
requirement of the question.

SECTION-A 4X1=4

Q1:* is a binary operation on R defined by a * b = a + b + ab. Find the identity element of the
binary operation.
5−𝑥 𝑥+1
Q2: For which value of x the matrix A= [ ] is singular.
2 4
Q3: For what value of p the projection of a = pi +j + 4k on b = 2i + 6j + 3k is 4 units.
1
Q4: Find the principal value of tan-1[2𝑐𝑜𝑠 (2𝑠𝑖𝑛−1 2)]

Section – B 8 X 2 = 16
2𝜋
Q5: Solve for x , 𝑡𝑎𝑛−1 𝑥 + 2𝑐𝑜𝑡 −1 𝑥 =
3
4 −2
Q6: Express [ ] as sum of symmetric and skew symmetric matrices.
7 3
𝑐𝑜𝑠 𝑥+𝑠𝑖𝑛 𝑥 𝑑𝑦
Q7: If y = tan-1( ), find 𝑑𝑥
𝑐𝑜𝑠 𝑥−𝑠𝑖𝑛 𝑥

Q8: Find the intervals in which the function f(x) = 4 – 2x - x2 is increasing and decreasing on R.
−1 1+𝑥+ 𝑥 2
Q9: Evaluate ∫ 𝑒 𝑡𝑎𝑛 𝑥 ( 2 )
dx .
1+ 𝑥
𝑑𝑦
Q10: Solve the differential equation log( ) = ax + by
𝑑𝑥
𝜃 1
Q11:If 𝜃 is the angle between two unit vectors a and b , the show that sin( 2) = 2 | 𝑎 − 𝑏|

Q12: The probability that A can solve a problem is 1/3 and b can solve the problem is ¼. If they try
independently, find the probability the question is solve by exactly one student.

Section – C 11 X 4 = 44

𝑎 −1 0
Q13: If f(x) = | 𝑎𝑥 𝑎 −1| using properties of determinanats find f(2x) – f(x)
𝑎 𝑥2 𝑎𝑥 𝑎
√1+ 𝑥 2 2𝑥
Q14: Differentiate 𝑡𝑎𝑛−1 ( ) with respect to 𝑠𝑖𝑛−1 (1+ 𝑥 2 )
𝑥

“or”

Find the value of a if following function is everywhere continuous


1  cos 4 x
 , if x  0
 x2
f ( x)   a , if x  0

 x
 16  x  4 , if x  0

Q15: If x = a cos t + b sin t and y = a sin t – b cost , prove that y2y2 – x y1 + y = 0

Q16: Find the value of p for which for which the curves x2 = 9p(9 – y) and x2 = p( y + 1) cut each other at
right angle. “or”

A water tank has the shape of an inverted right circular cone with its axis vertical and vertex lowermost.
Its semi-vertical angle is tan-1(0.5). Water is poured into it at a constant rate of 5 cubic metre per hour.
Find the rate at which the level of water is rising at the instant when the depth of water in the tank is 4
m.

Q17: A tank with rectangular base and rectangular walls open at the top is to be constructed so that its
depth is 2 m and volume is 8 m3. If the building of tank costs Rs 70 per sq. metre for the base and Rs 45
per sq. metre for the sides, what will be cost for least expensive tank?

𝑠𝑖𝑛 𝑥
Q18:Find ∫ dx
(𝑐𝑜𝑠2 𝑥+1)(𝑐𝑜𝑠2 𝑥+ 4)
Q19:Find the general solution of differential equation( 1 + tan y) ( dx – dy) + 2 x dy = 0

“or”
𝑦 𝑑𝑦 𝑦
Solve the differential equation x cos( ) = y cos( ) + x
𝑥 𝑑𝑥 𝑥

⃗⃗, and |𝑎| = 3, |𝑏| = 5, |𝑐| = 7 , show that angle between a and b is 600
Q20:If a +b + c =0

Q21: Find the equation of line passing through a point A( 1, 2, 3) and perpendicular to each of following lines
=( i + j + 2k) + 𝛾 ( 3i + j + 2k) and r = ( 2i – j + 2k) + 𝜇( 5i + 4j + 9k).

Q22: Two cards are drawn successively with replacement from a well shuffled pack of 52 cards.
Find the probability distribution of the number of heart cards. Also find mean of the distribution .

Q23: Suppose 5% of men and 0.25% of women have grey hair. A grey haired person is selected at
random. What is the probability of this person being a male? Assume that there are equal number of
males and females.

Section – D6 X 6 = 36
𝑥
Q24:Show that the function f : R → { x ∈ 𝑅 ∶ −1 < 𝑥 < 1 } defined by f(x) = is an invertible
1+|𝑥|
function. “or”

Let N denote the set of all natural numbers and R be the relation on N X N defined by

(a,b) R (c,d) iff ad( b + c ) = bc( a + d). Check whether R is an equivalence relation on N X N.

Q25: School A and B want to award their selected students on the values sincerity, truthfulness and
helpfulness. The school A wants to award Rs x each, Rsy each and Rs z each for the three respective
values to 3, 2 and 1 students respectively with a total award money of Rs 16,00. School B wants to
spend Rs 2,300 to award its 4, 1 and 3 students on respective values ( by giving the same award money
money the three values as before). If the total amount for one prize on each value is Rs 900, using
matrices, find the award money for each value.

Q26:Find the area of the region bounded by the two circles: x2 + y2 = 4 and (x – 2)2 + y2 = 4
“or”
Using integration, find the area of the following region:
{ ( x, y) : |𝑥 − 1| ≤ 𝑦 ≤ √5 − 𝑥 2 }
Q27: Find the distance of point(-1,-5,-10) from the plane x-y+z=5 measured parallel to the line
𝑥−2 𝑦+1 𝑧−2
= = .
3 4 12
𝜋⁄2
Q28: Evaluate ∫0 2 𝑠𝑖𝑛 𝑥 𝑐𝑜𝑠 𝑥 𝑡𝑎𝑛−1 (𝑠𝑖𝑛𝑥) dx “or”
3/2
Evaluate ∫−1 |𝑥𝑠𝑖𝑛(𝜋𝑥)| 𝑑𝑥

Q29: Anil wants to invest at most Rs.12,000 in Saving Certificates and National Saving Bonds and
According to the rules, he has to invest at least Rs. 2,000 in Saving Certificates and at least Rs. 4,000 in
National Saving Bonds. If the rate of interst on Saving Certificates is 8% per annum and on National
Saving Bonds is 10% per annum,how should he invest his money for maximum interest ? Solve the
problem graphically.
Sample Question Paper( Group-IV)

MATHEMATICS Max.Marks:100 Time Allowed: 3 hours

________________________________________________________________________

SECTION - A (1 mark questions)

59. Let * be a binary operation on Q given by a*b=a+ab where a,b  Q .Is * commutative?
1  4 
60. Evaluate Sin  cos 1   
2  5 
61. Given that A, B are two symmetric matrices such that AB =BA .Is AB symmetric?
 x  y  z   9
   
62. Find the values of x, y and z if  x  z    5 
 y  z   7
   
SECTION - B (2 mark questions)

Sin30 0 Cos30 0
63. Evaluate
 sin 60 0 Cos60 0
64. Find a point on the curve y=x2-4x+5 where the tangent to the curve is parallel to the x axis
x  cos 6 x
65. Evaluate  3x 2
 Sin6 x
dx
      
66. Find the projection of the vector a  i  3 k on the vector b  3 i  j  4 k
     
67. If a is a unit vector and ( x  a )( x  a )  15 find x

2x  1 y2 z3
68. The Cartesian equation of a line AB is   .Find the direction cosines of a line parallel
3 2 3
to AB
69. Let f: R  R be defined by f(x) = 3x+2.Show that f is invertible. Also find f  1 :R  R
 1 
70. Write in the simplest form : tan  1   where x  1
 
 x 1
2

SECTION - C (4 marks questions)

𝒙 𝒙 + 𝒚 𝒙 + 𝟐𝒚
71. |𝒙 + 𝟐𝒚 𝒙 𝒙 + 𝒚 | = 𝟗𝒚𝟐 (𝒙 + 𝒚)
𝒙 + 𝒚 𝒙 + 𝟐𝒚 𝒙
72. For which value of k, the function f defined below is continuous
𝒌 𝒄𝒐𝒔 𝒙 𝝅
𝒊𝒇 𝒙 <
𝝅−𝟐𝒙 𝟐
𝝅
73. 𝒇(𝒙) = 𝟑 𝒊𝒇 𝒙 = “or”
𝟐
𝟑 𝒕𝒂𝒏 𝟐𝒙 𝝅
𝒊𝒇 𝒙 >
{ 𝟐𝒙−𝝅 𝟐

𝒙𝟐 +𝟏
Find dy/dx if y = 𝒙𝒙 𝒄𝒐𝒔 𝒙 +
𝒙𝟐 −𝟏
dy 1  y2
74. If 1  x  1  y  a ( x  y ) prove that
2 2

dx 1  x2
2
75. Evaluate  e x dx as a limit of sum
0
OR

5
Evaluate   x  1x 2
4 dx
dy
76. Solve the differential equation Cos x
2
 y  tan x
dx
77. Find the differential equation for the family of circles which passes through the origin and have their centre
on the x axis
78. Let 𝐚⃗⃗ =𝐢̂ + 𝟒𝐣̂+ 2𝐤 ̂,𝐛
⃗ = 𝟑𝐢̂ − 𝟐𝐣̂+ 7𝐤
̂ and 𝐜⃗ =𝟐 ̂𝐢 − 𝐣̂+ 4𝐤
̂ , find a vector 𝐝⃗ which is perpendicular to both
⃗ 𝐚𝐧𝐝 𝐜.
𝐚⃗⃗&𝐛 ⃗⃗⃗ 𝐝⃗ = 𝟏8.
𝒙−𝟏 𝒚−𝟐 𝒛−𝟑 𝒙−𝟒 𝒚−𝟏
79. Show that lines = = and = = z intersect. Also find their point
𝟐 𝟑 𝟒 𝟓 𝟐

Of intersection.

80. A coin is tossed three times and all the possible outcomes are assumed to be equally likely. Let E and F be
two events given by E: “both tail and head have occurred” F: “at most one tail has occurred”. Show that E
and F are independent.

81. The sum of three numbers is 6.Twice the third number when added to the first number gives 7.On adding
the sum of the second and third numbers to thrice of the first number we get 12.Find the numbers using
inverse of a matrix.
Or

 1 2 3 

Using elementary transformations find the inverse of the matrix 2 5 7 

  2  4  5

SECTION - D (6 mark questions)

82. Let * be the binary operation defined on QxQ by (a,b)*(c,d) = (ac,b+ad) where Q is the set of rational
numbers. Determine whether * is commutative and associative . Find the indentiy element for * and the
inverttible elements of QxQ

83. Show that a closed right circular cylinder of given surface area S and maximum volume V is such
that its height is equal to the diameter of the base.

1
84. 26 Evaluate  Sinx(2  Cosx )dx
Or


2
Evaluate  log Sinxdx
0

27 Using integration, find the area of the circle x2 + y2 = 16 which is exterior to the parabola y2 = 6x
28 Find the foot of the perpendicular drawn from the point A(1,0,3) to the line joining the points B(4,7,1)
C(3,5,3) “or” Find the
vector equation of the plane through three points with position vectors i+j-2k,

29: Two tailors A and B earns Rs.150 and Rs.200 per day respectively. A can stitch 6 shirts and 4 pants per day
while B can stitch 10 shirts and 4 pants per day.How many days shall each work,if it is desired to produce at
least 60 shirts and 32 pants at a minimum labour cost ? Solve the problem graphically.

Sample Question Paper


MATHEMATICS CLASS – XII M.M. 100

GENERAL INSTRUCTIONS : 1. SECTION A : Consists four Questions carrying one mark each

SECTION B : Consists eight questions carrying two marks each.

SECTION C : consists eleven questions carrying four marks each.


SECTION D: Consists six questions carrying six marks each.

SECTION A:

Carrying one mark each question.

1. If the binary operation *, defined on Q, is defined as a*b = 2a + b – ab, for all a,b ∈ 𝑄, Find the value of
3*4.
2. If A is an invertible matrix of order 3 and |𝐴| = 5, then find |𝑎𝑑𝑗𝐴|.
𝑥 𝑥 𝑑𝑦 𝜋
3. If y = [sin 2 + cos 2]2 , find 𝑑𝑥 𝑎𝑡 𝑥 = 6 .
𝑑𝑥
4. Evaluate ∫ .
𝑥+𝑥𝑙𝑜𝑔𝑥

SECTION B
Each questions carrying two marks.
𝑐𝑜𝑠𝑥−𝑠𝑖𝑛𝑥 𝜋 𝜋
5. Simplify 𝑡𝑎𝑛−1 [𝑐𝑜𝑠𝑥+𝑠𝑖𝑛𝑥] , 𝑓𝑜𝑟 − 4 < 𝑥 < 4
.
3
6. Solve: cos(𝑡𝑎𝑛−1 𝑥) = sin(𝑐𝑜𝑡 −1 4).

7. A trust fund has Rs. 30,000 that is to be invested in two different types of bonds. The first bond pays 5%
interest per annum which will be given to orphanage and second bond pays 7% interest per annum which
will be given to financial benefits of the trust. Using matrix multiplication, determine how to divide Rs.
30,000 among two types of bonds, if the trust fund obtains an annual total interest of Rs. 1800.
1 3 2 1
8. Ind the value of x, if [1 𝑥 1] [ 2 5 1] [2] = 0.
15 3 2 𝑥
2𝑥+1 𝑑𝑦
9. If = 𝑐𝑜𝑠 −1 (1+ 4𝑥 ) , 𝑡ℎ𝑒𝑛 𝑓𝑖𝑛𝑑 𝑑𝑥
.
𝑥−1
10. If ∫ ( 𝑥 2 ) 𝑒 𝑥 𝑑𝑥 = 𝑓(𝑥)𝑒 𝑥 + 𝐶, 𝑡ℎ𝑒𝑛 𝑓𝑖𝑛𝑑 𝑡ℎ𝑒 𝑣𝑎𝑙𝑢𝑒 𝑜𝑓 𝑓(𝑥).

11. If the rate of change of volume ot the sphere is equals to the rate of its radius, find the radius of the
sphere.
12. Let A & B be two events . if P(A) =0.2, P(B) = 0.4 and P(AUB) =0.6, then find p (A/ B).
SECTION C
Each questions carry 4 marks.
2 3
13. Show that the matrix A = [ ] satisfies the equation A2 – 4A + I = O, where I is 2x2 identity matrix and
1 2
O is 2x2 zero matrix. Using this equation, find A-1.
14. Find the value of k, for which
√1+𝑘𝑥−√1−𝑘𝑥
, 𝑖𝑓 − 1 ≤ 𝑥 < 0
𝑓(𝑥) = { 𝑥
2𝑥+1
𝑥−1
, 𝑖𝑓 0 ≤ 𝑥 < 1

Is continuous at x = 0.
OR
Show that the function 𝑓(𝑥) = 2𝑥 − |𝑥| is continuous but not differentiable at x = 0.
𝑑𝑦 𝑐𝑜𝑠2 (𝑎+𝑦)
15. If cos y = x cos (a +y), with cosa ≠ ±1, 𝑡ℎ𝑒𝑛 𝑝𝑟𝑜𝑣𝑒 𝑡ℎ𝑎𝑡 𝑑𝑥
= sin 𝑎
. hence show that
𝑑2 𝑦 𝑑𝑦
sin a𝑑𝑥 2 + sin 2(𝑎 + 𝑦) 𝑑𝑥 = 0.

16. Find the intervals in which the function 𝑓(𝑥) = 3𝑥 4 − 4𝑥 3 − 12𝑥 2 + 5 is is


(a) Strictly increasing (b) strictly decreasing

OR

Find the equation of the normals to the curve y = x3 + 2x +6 which is parallel to the line

x + 14y +4 = 0.

𝑑𝑥
17. Evaluate ∫ 𝑐𝑜𝑠4 𝑥+𝑠𝑖𝑛4 𝑥.
𝑡 𝑑2 𝑦
18. 𝐼𝑓 𝑥 = 𝑎 sin 𝑡 𝑎𝑛𝑑 𝑦 = 𝑎 (cos 𝑡 + log tan ) , 𝑡ℎ𝑒𝑛 𝑓𝑖𝑛𝑑 .
2 𝑑𝑥 2

19. 𝑠𝑜𝑙𝑣𝑒 𝑡ℎ𝑒 𝑑𝑖𝑓𝑓𝑒𝑟𝑒𝑛𝑡𝑖𝑎𝑙 𝑒𝑞𝑢𝑎𝑡𝑖𝑜𝑛 (𝑡𝑎𝑛−1 𝑦 − 𝑥)𝑑𝑦 = (1 + 𝑦 2 )𝑑𝑥.


𝑂𝑅
Solve the differential equation (𝑥 2 − 𝑦 2 )𝑑𝑥 + 2𝑥𝑦 𝑑𝑦 = 0 given that y = 1 when x = 1.

20. Find the unit vector perpendicular to each of the vectors 𝑎⃗ + 𝑏⃗⃗ and 𝑎⃗ − 𝑏⃗⃗,
̂
𝑤ℎ𝑒𝑟𝑒 𝑎⃗ = 3𝑖̂ + 2𝑗̂ + 2𝑘 𝑎𝑛𝑑 𝑏⃗⃗ = 𝑖̂ + 2𝑗̂ − 2𝑘̂ .
𝑥+1 𝑦+3 𝑧+5 𝑥−2 𝑦−4 𝑧−6
21. Show that the lines 3
= 5
= 7
𝑎𝑛𝑑 1
= 3
= 5
intersect. Also find their point of

intersection.
22. A bag contains 1 white and 6 red balls, a second bag contains 4 white and 3 red balls. One of the bag is
picked up at random and a ball is randomly drawn from it, and is found to be white in colour. Find the
probability that the drawn ball was from the first bag.
23. Three cards are drawn at random (without replacement) from a well shuffled pack of 52 playing cards.
Find the probability distribution of the number of red cards. Hence fined the mean of the distribution.
SECTION D
Each question carry 6 marks.
24. Let f : N → 𝑅 be a function defined by 𝑓(𝑥) = 4𝑥 2 + 12𝑥 + 15.
Show that f : N → 𝑆 is invertible, where S is the range of f. hence find the inverse of f.

OR

Let N denote the set of all natural numbers and R be the relation on N x N defined by

(a,b) R (c,d) if ad (b + c) = bc (a + d). show that R is an equivalence relation.

25. Using the property of determinant , prove that


𝑥 𝑥2 1 + 𝑝𝑥 3
|𝑦 𝑦2 1 + 𝑝𝑦 3 | = (1 + 𝑝𝑥𝑦𝑧)(𝑥 − 𝑦)(𝑦 − 𝑧)(𝑧 − 𝑥).
𝑧 𝑧2 1 + 𝑝𝑧 3
OR
𝑎 + 𝑏 + 2𝑐 𝑎 𝑏
Prove that | 𝑐 𝑏 + 𝑐 + 2𝑎 𝑏 | = 2(𝑎 + 𝑏 + 𝑐)3 , by using the property of
𝑐 𝑎 𝑐 + 𝑎 + 2𝑏
determinant.
𝜋
𝑠𝑖𝑛2 𝑥
26. Evaluate : ∫02 𝑑𝑥 .
sin 𝑥+cos 𝑥

27. Sketch the graph of 𝑦 = ⌈𝑥 + 3⌉ and find the area under the curve 𝑦 = ⌈𝑥 + 3⌉ above x-axis and between
x = - 6 to x = 0.
28. Find the vector equation of the plane passing through the points (2, 1, - 1) and (- 1, 3, 4) and
perpendicular to the plane x – 2y +4z = 10. Also show that the plane thus obtained contains the line 𝑟⃗ =
− 𝑖̂ + 3𝑗̂ + 4𝑘̂ + λ(3𝑖̂ − 2𝑗̂ − 5𝑘̂ ).

29. A village has 500 hectares of land to grow two types of plants , X and Y. the contribution of total amount
of oxygen produced by plant X plant Y are 60% and 40% per hectare respectively. To control weeds, a
lquid herbicide has to be used for X and Y at rates 20 litres and 10 litres per hectare, respectively. Further
no more than 8000 litres of herbicides should be used in order to protect aquatic animals in a pond which
collects drainage from this land. How much land should be allocated to each crop so as to maximize the
total production the total production of oxygen?
(i) How do you think excess use of herbicides affects our environment ?
(ii) What are the general implications of this question towards planting trees around us?

Prepared by
Mr. L. G. SHAHARE PGT (Maths)
K. V. VSN NAGPUR
GROUP-5

MODEL QUESTION PAPER -1

TIME : 3 Hrs XII – MATHS M.M. 100

SECTION - A (FOURQUESTIONS OF ONE MARK EACH)

𝑥+𝑦 𝑦+𝑧 𝑧+𝑥


1. 𝑊𝑟𝑖𝑡𝑒 𝑡ℎ𝑒 𝑣𝑎𝑙𝑢𝑒 𝑜𝑓 ∆ = | 𝑧 𝑥 𝑦 |
−3 −3 −3
2. 𝐼𝑓 𝑎, ̂ 𝑐̂ 𝑎𝑟𝑒 𝑚𝑢𝑡𝑢𝑎𝑙𝑙𝑦 𝑝𝑒𝑟𝑝𝑒𝑛𝑑𝑖𝑐𝑢𝑙𝑎𝑟 𝑢𝑛𝑖𝑡 𝑣𝑒𝑐𝑡𝑜𝑟𝑠, 𝑡ℎ𝑒𝑛 𝑓𝑖𝑛𝑑 𝑡ℎ𝑒 𝑣𝑎𝑙𝑢𝑒 𝑜𝑓|2𝑎̂ + 𝑏̂ + 𝑐̂ |
̂ 𝑏,
3. If the binary operation * on the set of integers Z, is defined by a*b =
a+3𝑏 2 𝑡ℎ𝑒𝑛 𝑓𝑖𝑛𝑑 𝑡ℎ𝑒 𝑣𝑎𝑙𝑢𝑒 𝑜𝑓 2 ∗ 4.
4. Find the principal value of tan−1 √3 − sec −1(−2)

SECTION – B (EIGHT QUESTIONS OF TWO MARKS EACH)


5. 𝑊𝑟𝑖𝑡𝑒 𝑡ℎ𝑒 𝑖𝑛𝑡𝑒𝑔𝑟𝑎𝑡𝑖𝑛𝑔 𝑓𝑎𝑐𝑡𝑜𝑟 𝑜𝑓 𝑡ℎ𝑒 𝑓𝑜𝑙𝑙𝑜𝑤𝑖𝑛𝑔 𝑑𝑖𝑓𝑓𝑒𝑟𝑒𝑛𝑡𝑖𝑎𝑙 𝑒𝑞𝑢𝑎𝑡𝑖𝑜𝑛 ∶
𝑑𝑦
(1 + 𝑦 2 ) + (2𝑥𝑦 − 𝑐𝑜𝑡𝑦) =0
𝑑𝑥
6. 𝑊𝑟𝑖𝑒 𝑡ℎ𝑒 𝑢𝑛𝑖𝑡 𝑣𝑒𝑐𝑡𝑜𝑟 𝑝𝑒𝑟𝑝𝑒𝑛𝑑𝑖𝑐𝑢𝑙𝑎𝑟 𝑡𝑜 𝑏𝑜𝑡ℎ 𝑡ℎ𝑒 𝑣𝑒𝑐𝑡𝑜𝑟𝑠 𝑎, ̂ = 𝑖̂ + 𝑗̂ + 𝑘̂ 𝑎𝑛𝑑 𝑏̂ = 𝑖̂ + 𝑗̂
7. Express in the simplest form :
√1 + 𝑥 2 − 1
tan−1 ,𝑥 ≠ 0
𝑥
1 2
8. Find the inverse of the matrix A=[ ]
3 4
9. Find the derivative of y= 𝑙𝑜𝑔7 (𝑠𝑖𝑛𝑥)
10. Find the points on the curve y=𝑥 3 𝑎𝑡 𝑤ℎ𝑖𝑐ℎ 𝑡ℎ𝑒 𝑠𝑙𝑜𝑝𝑒 𝑜𝑓 𝑡ℎ𝑒 𝑡𝑎𝑛𝑔𝑒𝑛𝑡 is equal to y- coordinate of the
point.
1
11. Find ∫ 𝑥+𝑥𝑙𝑜𝑔𝑥 𝑑𝑥
2 1 1 𝐴̅
12. If P(A) = ,P(B)= , P(A∩B)= , then find P( ̅ )
5 3 5 𝐵

SECTION – C (ELEVEN QUESTIONS OF FOUR MARKS EACH)


13. 𝑈𝑠𝑖𝑛𝑔 𝑝𝑟𝑜𝑝𝑒𝑟𝑡𝑖𝑒𝑠 𝑜𝑓 𝑑𝑒𝑡𝑒𝑟𝑚𝑖𝑛𝑎𝑛𝑡𝑠 , 𝑝𝑟𝑜𝑣𝑒 𝑡ℎ𝑒 𝑓𝑜𝑙𝑙𝑜𝑤𝑖𝑛𝑔 ∶
𝑎2 𝑏𝑐 𝑎𝑐 + 𝑐 2
2
|𝑎 + 𝑎𝑏 𝑏 2
𝑎𝑐 |=4𝑎2 𝑏 2 𝑐 2
𝑎𝑏 𝑏 2 + 𝑏𝑐 𝑐2

14. 𝑆ℎ𝑜𝑤 𝑡ℎ𝑎𝑡 𝑡ℎ𝑒 𝑓𝑢𝑛𝑐𝑡𝑖𝑜𝑛 𝑓(𝑥) = |𝑥 − 1| + |𝑥 + 1|, ∀ 𝑥 ∈


𝑅𝑖𝑠 𝑛𝑜𝑡 𝑑𝑖𝑓𝑓𝑒𝑟𝑒𝑛𝑡𝑎𝑏𝑙𝑒 𝑎𝑡 𝑡ℎ𝑒 𝑝𝑜𝑖𝑛𝑡𝑠 𝑥 = −1𝑎𝑛𝑑 𝑥 = 1
−1 𝑥 𝑑2 𝑦 𝑑𝑦
15. 𝐼𝑓 𝑦 = 𝑒 𝑚 sin , 𝑡ℎ𝑒𝑛 𝑠ℎ𝑜𝑤 𝑡ℎ𝑎𝑡 (1 − 𝑥 2 ) 𝑑𝑥 2 − 𝑥 𝑑𝑥 + 𝑚2 𝑦 = 0
𝑥 3 −1
16. Find : ∫ 𝑥 3 +𝑥 𝑑𝑥
17. Find the differential equation for all the straight lines which are at a unit distance from the origin.
OR
𝑑𝑦
Show that the differential equation 2xy𝑑𝑥 = 𝑥 2 + 3𝑦 2 is homogeneous and solve it.
18. Show that the following two lines are coplanar:
𝑥−𝑎+𝑑 𝑦−𝑎 𝑧−𝑎−𝑑 𝑥−𝑏+𝑐 𝑦−𝑏 𝑧−𝑏−𝑐
= = 𝑎𝑛𝑑 = =
𝛼−𝛿 𝛼 𝛼+𝛿 𝛽−𝛾 𝛽 𝛽+𝛾
19. Show that four points A,B,C and D whose position vectors are 4𝑖̂+5𝑗̂+ 𝑘,-𝑗̂- 𝑘 , 3𝑖̂+9𝑗̂+4 𝑘̂and 4(-𝑖̂+𝑗̂+ 𝑘̂)
̂ ̂
respectively are coplanar.
20. A and B throw a die alternatively till one of them gets a number greater than four and wins the game.
If A starts the game, what is the probability of B winning?
21. A die is thrown three times. Events A and B are defined as below:
A: 5 on first and 6 in the second throw
B: 3 or 4 on the third throw.
Find the probability of B, given that A has already occurred.
22. Find the absolute maximum and minimum values of the function f given by f(x) = 𝑠𝑖𝑛2 𝑥 − 𝑐𝑜𝑠𝑥, 𝑥 ∈
[0, 𝜋]
23. Find the equation of tangent to the curve y=√3𝑥 − 2 , which is parallel to the line 4x-2y+5=0
OR
Prove that the curves x=𝑦 2 and xy = k cut at right angles if 8𝑘 2 = 1.

SECTION – D(SIX QUESTIONS OF SIX MARKS EACH)


24. . If the function f: R→ 𝑅 be defined by f(x)=2x-3 and g: R→ 𝑅 by g(x) = 𝑥 3 + 5 , then find the value of
(𝑓𝑜𝑔)−1 (𝑥).
OR
Let A=Q×Q,where Q is the set of all rational numbers, and * be a binary operations defined on A
by (a,b)*(c,d)=(ac,b+ad), for all (a,b),(c,d)∈ 𝐴. Then find the identity element of * in A and
invertible elements of A , and hence write the inverse of elements (5,3) and (½ , 4).

(𝑎+𝑏)2
𝑐
𝑐 𝑐
| (𝑏+𝑐)2 |
25. .Using properties of determinants, prove that
| 𝑎 𝑎
𝑎 |=2(𝑎 + 𝑏 + 𝑐)3
(𝑐+𝑎)2
𝑏 𝑏
𝑏
OR
𝑝 𝑞 𝑝𝛼 + 𝑞
If p≠ 0, 𝑞 ≠ 0 and | 𝑞 𝑟 𝑞𝛼 + 𝑟 |=0,then,using properties of determinants ,prove
𝑝𝛼 + 𝑞 𝑞𝛼 + 𝑟 0
that at least one of the statements is true (a) p,q,r are in G .P.(b) 𝛼 is a root of the equation px2
+2qx+r=0.
26. Using integration ,find the area of the region bounded by the lines y=2+x, y=2-xand x=2.
𝜋 𝑥 𝑡𝑎𝑛𝑥
27. Evaluate the following : ∫0 𝑠𝑒𝑐𝑥+tan 𝑥
𝑑𝑥
OR
4
Evaluate ∫0 (𝑥 + 𝑒 2𝑥 )𝑑𝑥 as the limit of a sum.

28. Find the direction ratios of the normal to the plane , which passes through the point (1,0,0) and (0,1,0) and
𝜋
makes angle 4 with the plane x+y=3 . Also find the equation of the plane.

29. The postmaster of the local post office wishes to hire extra helpers during the Deepawaliseason , because of a
large increase in the volume of mail handling and delivery. Because of the limited office space and the budgetary
conditions , number of temporary helpers must not exceed 10.According to past experience a man can handle
300 letters and 80 packages per day, on the average, and a woman can handle 400 letters and 50 packages per
day. The postmaster believes that the daily volume of extra mail and packages will be no less than 3400 and 680
respectively. A man receives Rs 225 a day and a woman receives Rs 200 a day. How many men and women
helpers should be hire to keep the pay roll at a minimum? Formulate an LPP and solve it graphically.

SOLUTIONS :
SECTION -A
1. 0
2. √6
3. 50
𝜋 2𝜋 𝜋
4. 3
− 3
= −3
SECTION-B
−2𝑦
𝑑𝑥 2𝑥𝑦 𝑐𝑜𝑡𝑦 ∫1+𝑦2𝑑𝑦
5. 𝑑𝑦
− 1+𝑦2 = 1+𝑦2 , I.F. = 𝑒 = 1+𝑦 2
⃗⃗
𝑎⃗⃗×𝑏 − 𝑖̂+𝑗̂
6. 𝑛̂ = ⃗⃗|
, 𝑎⃗ × 𝑏⃗⃗ = - 𝑖̂ + 𝑗̂ , |𝑎⃗ × 𝑏⃗⃗| = √2 , 𝑛̂ =
|𝑎⃗⃗×𝑏 √2
√1+𝑡𝑎𝑛2 𝛼−1
7. tan−1 , let x=tan𝛼
𝑥
𝛼
√𝑠𝑒𝑐 2 𝛼 − 1 1 − 𝑐𝑜𝑠𝛼 𝑠𝑖𝑛 2 𝛼 1
−1 −1 −1
tan = tan = tan 𝛼 = = tan−1 𝑥
𝑡𝑎𝑛𝛼 𝑠𝑖𝑛𝛼 𝑐𝑜𝑠 2 2 2
1 −4 2
8. Det A = -2, inverse of A =2 [ ]
3 −1
9. Base changing 1- mark
𝑑𝑦
𝑑𝑥
= 𝑐𝑜𝑡𝑥. 𝑙𝑜𝑔7 𝑒1- mark

10.
𝑠𝑙𝑜𝑝𝑒 𝑜𝑓 𝑡𝑎𝑛𝑔𝑒𝑛𝑡 𝑎𝑡 (𝑥1 , 𝑦1 ) = 3𝑥1 2
𝑟𝑒𝑞𝑢𝑖𝑟𝑒𝑑 𝑝𝑜𝑖𝑛𝑡𝑠 (0,0)𝑎𝑛𝑑(3,27)

1
11. ∫ 𝑥(1+𝑙𝑜𝑔𝑥) 𝑑𝑥 , let 1+logx = t , 1- mark
𝑐𝑜𝑟𝑟𝑒𝑐𝑡 𝑖𝑛𝑡𝑒𝑔𝑟𝑎𝑙 ∶ log(1 + 𝑙𝑜𝑔𝑥) + 𝑐 1- mark
𝐴̅ 𝑃(𝐴̅∩𝐵̅) 1−𝑃(𝐴∪𝐵)
12. P(𝐵̅) = 𝑃𝐵̅
= 1−𝑃(𝐵)
1- mark

𝐴̅
P(𝐵̅) = 7/10 1- mark
13. Getting two zero’s in any one row/column using appropriate properties of determinant 3- marks
To get correct answer by expansion 1 – mark
14. One mark each for calculating correct LHD at x=-1and x=1
One mark each for calculating correct LHD at x=-1and x=1
𝑑𝑦
15. Correct 𝑑𝑥
1- mark
2
𝑑 𝑦
Correct 𝑑𝑥 2
1 − 𝑚𝑎𝑟𝑘
To prove LHS=RHS 2- marks
𝑥 3 −1 𝐴 𝐵𝑥+𝑐
16. To get 𝑥 3 +𝑥
=1+ 𝑥
+ 𝑥 2 +1 with correct values of A,B and C 2- marks
Correct integrals 2 – marks
17. Getting y=mx±√𝑚2 +1 2 – marks
Required result : y=xy’±√𝑦′2 +1 2 – marks
OR
To prove the differential equation is homogeneous 1- mark
2 2 3
Getting a solution 𝑥 + 𝑦 = 𝑐𝑥 3 marks
18. Converting the equations into general form and applying condition of coplanarity 1- mark
Solution of condition of coplanarity 3- marks
⃗⃗⃗⃗⃗⃗ = -4𝑖̂-6𝑗̂-2 𝑘̂
19. 𝐴𝐵
⃗⃗⃗⃗⃗⃗ = −𝑖̂ + 4𝑗̂ + 3𝑘̂
𝐴𝐶
1
𝐴𝐷= -8𝑖̂-𝑗̂+3𝑘̂
⃗⃗⃗⃗⃗⃗ 12 marks
1
𝑏𝑜𝑥 𝑝𝑟𝑜𝑑𝑢𝑐𝑡 𝑜𝑓 ⃗⃗⃗⃗⃗⃗
𝐴𝐵 ,𝐴𝐶 ⃗⃗⃗⃗⃗⃗ ,𝐴𝐷
⃗⃗⃗⃗⃗⃗ = 0
2
𝑚𝑎𝑟𝑘
Solution of box product 2 marks
20. P(E) = 1/3 , P(E’) = 2/3 1 mark
Required probability = P(E’E)+P(E’E’E’E)+P(E’E’E’E’E’E)+… 1- mark
Calculation and getting result 2/5 2- marks
21. A={561,562,563,564,565,566}
B={all sample points havening 3 or 4 on the third throw} 2- mark
𝐵 𝑃(𝐵∩𝐴)
P( ) = = 1/32- marks
𝐴 𝑃(𝐴)
2
22. f(x) = 𝑠𝑖𝑛 𝑥 − 𝑐𝑜𝑠𝑥
f’(x) =2 𝑠𝑖𝑛𝑥. cos 𝑥 + 𝑠𝑖𝑛𝑥1 mark
2𝜋
getting critical points : 𝑥 = 0, 𝑥 = 3
2 mark
getting the required solution: absolute maximum value = 5/4
and absolute minimum value = -1 1 - mark
23. getting y’ = 3/2y ½ mark
let the coordinates of the point on parabola be (a,b)
3 −4
𝑚1 = 2𝑏 = −2 ½ mark
41 3
Getting the point on tangent are (48 , 4) and slope is 2 2 mark
Getting the required equation 48x-24y-23=0 1 - mark
OR
−1
1 1 𝑦
𝑚1 = = 1 , 𝑚2 = − = − 𝑘 3 2 mark
2𝑦 𝑥
2𝑘 3

𝑚1 × 𝑚2 = −1 𝑎𝑛𝑑 𝑔𝑒𝑡𝑡𝑖𝑛𝑔 𝑡ℎ𝑒 𝑑𝑒𝑠𝑖𝑟𝑒𝑑 𝑟𝑒𝑠𝑢𝑙𝑡 8𝑘 2 = 1 2 mark

24. getting fog(x) = 2𝑥 3 + 7 2- mark


to check the (𝑓𝑜𝑔)−1 𝑤𝑖𝑙𝑙 𝑒𝑥𝑖𝑠𝑡 𝑜𝑟 𝑛𝑜𝑡 2 − 𝑚𝑎𝑟𝑘
3 𝑋−7
(𝑓𝑜𝑔)−1 = √ 2 − 𝑚𝑎𝑟𝑘
2
OR
Getting the identity element in A is (1,0) 2 mark
Getting inverse of (a,b) is (1/a, b/a) 2 mark
Gettnng inverse of (5,3) = (1/5 , 3/5) 1 mark
Inverse of (1/2 , 4) = (2,-8) 1 mark

(𝑎 + 𝑏)2 𝑐2 𝑐2
1 2 2
25. LHS=𝑎𝑏𝑐 [ 𝑎 (𝑏 + 𝑐) 𝑎2 ] ½ mark
𝑏2 𝑏2 (𝑐 + 𝑎)2
Using 𝐶1 → 𝐶1 − 𝐶3 , 𝐶2 → 𝐶2 − 𝐶3

(𝑎 + 𝑏 + 𝑐)(𝑎 + 𝑏 − 𝑐) 0 𝑐2
1
=𝑎𝑏𝑐 [ 0 (𝑏 + 𝑐 + 𝑎)(𝑏 + 𝑐 − 𝑎) 𝑎2 ]
(𝑏 + 𝑐 + 𝑎)(𝑏 − 𝑐 − 𝑎) (𝑏 + 𝑐 + 𝑎)(𝑏 − 𝑐 − 𝑎) (𝑐 + 𝑎)2
(𝑎 + 𝑏 − 𝑐) 0 𝑐2
(𝑎+𝑏+𝑐)2
= 𝑎𝑏𝑐 | 0 (𝑏 + 𝑐 − 𝑎) 𝑎2 |
(𝑏 − 𝑐 − 𝑎) (𝑏 − 𝑐 − 𝑎) (𝑐 + 𝑎)2
(𝑎 + 𝑏 − 𝑐) 0 𝑐2
(𝑎+𝑏+𝑐)2
= 𝑎𝑏𝑐 | 0 (𝑏 + 𝑐 − 𝑎) 𝑎2 | R3→R3-(R1+R2)
(−2𝑎) (−2𝑐) 2𝑎𝑐
2
(𝑎𝑐 + 𝑏𝑐 − 𝑐 ) 0 𝑐2
(𝑎+𝑏+𝑐)2
=
𝑎𝑏𝑐𝑐𝑎
| 0 (𝑏𝑎 + 𝑐𝑎 − 𝑎2 ) 𝑎2 |
(−2𝑎𝑐) (−2𝑐𝑎) 2𝑎𝑐
(𝑎𝑐 + 𝑏𝑐) 𝑐2 𝑐2
(𝑎+𝑏+𝑐)2
= 𝑎𝑏𝑐𝑐𝑎 | 𝑎 2
(𝑏𝑎 + 𝑐𝑎) 𝑎2 | C1→(C1+C3),C2→(C2+C3)
0 0 2𝑎𝑐

(𝑎 + 𝑏) 𝑐 𝑐
(𝑎+𝑏+𝑐)2 2𝑐 2 𝑎2
= 𝑎𝑏𝑐𝑐𝑎
| 𝑎 (𝑏 + 𝑐) 𝑎| =2(𝑎 + 𝑏 + 𝑐)3
0 0 1

26. getting vertices of triangle A(0,2) , B (2,4), C (2,0) 1- mark


Correct figure 1- mark
2 2
Required area = ∫0 (2 + 𝑥). 𝑑𝑥 − ∫0 (2 − 𝑥). 𝑑𝑥 2- mark
Getting a result = 4 sq. unit 2- mark
𝑏 𝑏
27. using property : ∫𝑎 𝑓(𝑥). 𝑑𝑥 = ∫𝑎 𝑓(𝑎 + 𝑏 − 𝑥)𝑑𝑥
𝜋 𝜋 𝑡𝑎𝑛𝑥
And getting I = 2 ∫0 𝑠𝑒𝑐𝑥+𝑡𝑎𝑛𝑥 𝑑𝑥 2 – mark

𝜋 𝜋 𝑡𝑎𝑛𝑥(𝑠𝑒𝑐𝑥−𝑡𝑎𝑛𝑥)
I = ∫0 𝑑𝑥 1- mark
2 (𝑠𝑒𝑐𝑥+𝑡𝑎𝑛𝑥)(𝑠𝑒𝑐𝑥−𝑡𝑎𝑛𝑥)
𝜋 𝜋
I = 2 ∫0 (𝑠𝑒𝑐𝑥 𝑡𝑎𝑛𝑥 − 𝑠𝑒𝑐 2 𝑥 + 1)𝑑𝑥 1- mark
𝜋
Getting the result I = 2 (𝜋 − 2)2 – mark
OR
4
𝑓(𝑥) = (𝑥 + 𝑒 2𝑥 ) ,∫0 (𝑥 + 𝑒 2𝑥 )𝑑𝑥 = lim ℎ[𝑓(𝑎) + 𝑓(𝑎 + ℎ) + ⋯ 𝑛 𝑡𝑒𝑟𝑚𝑠] 1- mark
ℎ→0

𝑛+1 𝑒 2𝑛ℎ −1
lim ℎ[𝑛 ( )+ 𝑒 2𝑛ℎ ] 3 – mark
ℎ→0 2 𝑒 2ℎ −1
𝑒 8 −1
Simplifying limits ans getting the result 8+ 2
2 – mark
28. Let the equation of plane passing thorough the point (1,0,0) be
A(x-1)+b(y-0)+c(z-0) = 0 …(i)
This also passes through (0,1,0)
b=a 2 mark
𝜋
the angle between plane(i) and plane x+y=3 is . 4
solving the condition of angle between planes we get c=±√2𝑎 2 mark
required equation of plane is x+y±√2 𝑧 = 1 1- mark
required direction ratios are 1,1,±√2 1- mark
29. Mathematical formulation :
𝑥 + 𝑦 ≤ 10
3𝑥 + 4𝑦 ≥ 34
8𝑥 + 5𝑦 ≥ 68 2 - mark
𝑥 ≥ 0, 𝑦 ≥ 0
Correct figure 2 - mark
Getting the result: Minimum payroll 6 men and 4 women must be hired. 2 – mark
SAMPLE PAPER
CLASS XII : MATHEMATICS
BLUE PRINT
S.No. Topics 1 2 marks 4 marks 6 marks TOT
mark L
1 k) RELATIONS AND 1 1
FUNCTIONS
10
l) INVERSE TRIGONOMETRIC 1 2
FUNCTIONS
2 k) MATRICES 1 1
13
l) DETERMINANTS 1 1
3 z) CONTINUITY & 1 1
DIFFERENTIABILITY
aa) APPLICATION OF 1 3
DERIVATIVES
bb) INTEGRATION 1 1 1 44
cc) APPLICATION OF 1
INTEGRALS
dd) DIFFERENTIAL 1 1
EQUATIONS
4 k) VECTORS 1 1 1
l) 3-DIMENTIONAL 1 1 17
GEOMETRY
5 LINEAR PROGRMMING 1 6
6 PROBABILITY 1 2 10
TOTAL 4x1 = 8x2 = 16 11x4 = 44 6*6=36 100(
4 )
SAMPLE PAPER-2 : CLASS XII : MATHEMATICS

Time:03hours Max Marks: 100


General Instructions
 All questions are compulsory
 The question paper consists of 26 questions divided into three sections A, B and C
 Section A contains 6 questions of 1 mark each, Section B is of 13questions of four marks each and Section
C comprises of 7 questions of six marks each
 There is no overall choice. However, internal choice has been provided in 4 questions of four marks and 2
questions of six marks each. You have to attempt only one of the alternatives in all such questions.
 Use of calculators is not permitted
SECTION – A
1. Find gf(-3) if f(x) =|𝒙| and g(x) = |𝟓𝒙 − 𝟐|
3𝜋
2. Find the principal value of 𝑠𝑖𝑛−1 [𝑠𝑖𝑛 ]
5
3 1
3. If A = ( ), then find |𝑎𝑑𝑗 𝐴|
2 −3
4. If 𝑎⃗ = 𝑖̂ + 2𝑗̂ − 3𝑘 ̂ and 𝑏⃗⃗ = 2𝑖̂ + 4𝑗̂ + 9𝑘̂, find a unit vector parallel to⃗⃗⃗⃗
𝑎 + 𝑏⃗⃗
SECTION – B
sin 𝛼 cos 𝛼
5.If𝐴 = ( ), verify that A’ A = I.
−𝑐𝑜𝑠𝛼 sin 𝛼
13𝜋
6Find the principal value of the expression cos −1 (cos ).
6

5. Show that the relation R, defined on the set A of all triangles as: R = {(T1 , T2): T1 is similar to
T2} is an equivalence relation.
(OR)
𝑛 − 1 𝑖𝑓 𝑛 𝑖𝑠 𝑒𝑣𝑒𝑛
Let f: N {0}  N {0} defined by :𝑓(𝑥) = { .
𝑛 + 1 𝑖𝑓 𝑛 𝑖𝑠 𝑜𝑑𝑑
Show that f(x) is a bijective function.
1 1 1 1
6. Evaluate: 𝑡𝑎𝑛−1 + 𝑡𝑎𝑛−1 + 𝑡𝑎𝑛−1 + 𝑡𝑎𝑛−1
5 7 3 8
3𝑎𝑥 + 𝑏, 𝑖𝑓 𝑥 > 1
7. If the function f(x) given by: 𝑓(𝑥) = { 11 𝑥=1
5𝑎𝑥 − 2𝑏 𝑖𝑓 𝑥 < 1
is continuous at x = 1, find the value of a and b.
8 .Find the intervals in which the function f given by f(x) = 2𝑥 3 − 3𝑥 2 − 36𝑥 + 7 is:
(a) Strictly increasing (b) strictly decreasing.
2+sin 2𝑥 𝑥
9.Evaluate:∫ 𝑒 𝑑𝑥
1+cos 2𝑥
12 Form the differential equation of the family of hyperbolas having foci on x-axis and center at origin.
11.Find the area of the parallelogram whose adjacent sides are determined by the vectors
 
a  iˆ  ˆj  3kˆ and b  2iˆ  7 ˆj  kˆ .

12.A die is thrown twice and the sum of numbers appearing is observed to be 7. What is the conditional
probability that the number 2 has appeared at least once.
SECTION C

13,By using properties of determinants, show that


a b c
a 2
b c 2  (a  b)(b  c)(c  a)(a  b  c)
2

bc ca ab


14.If y = 𝑙𝑜𝑔{𝑥 + √𝑥 2 + 𝑎2 } prove that (x2 + a2) y2 + x y1 = 0

(𝑥 2 +1)(𝑥 2 +4)
15.Evaluate: ∫
(𝑥 2 +3)(𝑥 2 −5)
𝑑𝑥
16A window is in the form of a rectangle surrounded by a semi-circular opening. The total perimeter
of window is 10 meters. Find the dimensions of the window so as to admit maximum light through the
whole opening.
Your friend is constructing a house for him. What will you suggest him in order to have enough
light and air?
17. Find the intervals in which the function f given by f(x) = sinx +cos x, 0 ≤ x ≤ 2 is strictly
increasing or strictly decreasing.
OR
Prove that the curves x = y2 and xy = k cut at right angles if 8k2 = 1.
18.Show that semi-vertical angle of a right circular cone, of given surfacearea and maximum volume , is
1
sin−1 . On the surface is written “Respect for beliefs and cultures of various religion”. Do you believe in
3
mutual respect ?
OR
A window is in the shape of a rectangle surmounted by a semi-circle. If its perimeter is 10m, then find the
dimension of the window show that it may admit maximum light. Do you think that by getting maximum lightwe
can save electricity? Do you agree that we should save electricity
19.Solve the following differential equation: 𝑥 𝑑𝑦 − 𝑦 𝑑𝑥 = √(𝑥 2 + 𝑦 2 ) 𝑑𝑥
 
20. Let a = î + 4ĵ + 2 k̂ , b = 3 î - 2ĵ + 7 k and c = 2î - ĵ + 4 k . Find a vector d which is perpendicular
to

both a and b , and c . d = 15.


21.Find the shortest distance between the lines :

x 1 y 1 z 1 x 3 5 y z 7
  and  
7 6 1 1 2 1

22.In a bolt factory, machines A, B and C manufacture 25%, 35% and 40% of the total. Of their output
5%, 4%, 2% are defective. A bolt is drawn at random from the product. a) What is the probability that
the bolt drawn is defective? b) If the bolt drawn is found to be defective, find the probability that it is a
product of machine B?

23.An insurance company insured 2000 scooter drivers, 4000 car drivers and 6000 truck drivers. The
probabilityaccidents is 0.01, 0.03 and 0.15 respectively. One of the insured persons met with an
accident. What is the probability that he is a scooter driver? What moral value will you assign to all?.
SECTION D
24.Using Matrices, solve the following system of equations:
2x – 3y + 5z = 11; 3x + 2y – 4z = -5 ; x + y – 2z = -3
𝜋 𝑥
25Evaluate: ∫ dx
0 𝑎2 𝑐𝑜𝑠2 𝑥+ 𝑏 2 𝑠𝑖𝑛2 𝑥
26Find the area of the region: {(x , y) : x2 + y2 ≤ 1≤ x + y}
(OR)
Compute the area bounded by the lines x +2y = 2 ; y – x = 1and 2x + y = 7
𝑥
27.Find the distance of the point (1, 2, 3) from the plane x – y + z = 5 measured parallel to the line =
2
𝑦 𝑧
=
3 −6
28.A diet is to contain at least 90 units of vitamin A and 100 units of minerals. Two foods F1 and F2
are available. Food F1 cost Rs.4/- per unit and F2 costs Rs.6/- per unit. One unit of food F1 contains 3
units of vitamin A and 4 units of minerals. One unit of food F2 contains 6 units of vitamin A and 3
units of minerals. Formulate this as a linear programming problem. Find the minimum cost of the diet
that consists of mixture of these two foods and also meets the minimal nutritional requirements. What
is the importance of balanced diet in our daily life?

29.Consider f: R+(-5, ∞) given by f (x) = 9 x2 + 6x-5 show that f is invertible . Also find f-1 .

Answers:
1.gf(x) = |5|𝑥| − 2| gf(-3) = 13
3𝜋 2𝜋 2𝜋 2𝜋 𝜋 𝜋
2. 𝑠𝑖𝑛 = 𝑠𝑖𝑛 [𝜋 − ]= 𝑠𝑖𝑛 , ∈ [− , ]
5 5 5 5 2 2
2𝜋
PRINCIPAL VALUE =
5
−3 −1
3. 𝑎𝑑𝑗 𝐴 = ( ) |𝑎𝑑𝑗 𝐴| = -11
−1 3
𝑎 + 𝑏⃗⃗ =3 𝑖̂ + 6𝑗̂ + 6𝑘̂
4.⃗⃗⃗⃗
⃗⃗⃗⃗
𝑎+𝑏⃗⃗ ̂
3 𝑖̂+6𝑗̂ +6𝑘
Parallel unit vector = ⃗⃗|
=
|⃗⃗⃗⃗
𝑎+𝑏 9
sin 𝛼 − cos 𝛼 sin 𝛼 cos 𝛼 1 0
5.𝐴′ 𝐴 = ( )( )= ( )=𝐼
cos 𝛼 sin 𝛼 −𝑐𝑜𝑠𝛼 sin 𝛼 0 1
6.
𝜋
6
7.
8.I(-∞,-2) U(3,∞) and D(-2,3)
9.
𝑒 𝑥 𝑡𝑎𝑛𝑥
𝑥2 𝑦2
10.The equation of family of hyperbolas: − =1
𝑎2 𝑏2
Differentiate twice to eliminate a and b
The required D.E is xy y`` + x (y`)2 – y y` = 0
11.. Area of Parallelogram =a x b
𝒊 𝒋 𝒌
=|𝟏 −𝟏 𝟑|= 15√2 sq. units
𝟐 −𝟕 𝟏
12.A={(1,6), (6,1), (2,5), (5,2), (3,4), (4,3)}
B={(1,2),(2,1),(2,2),(2,3),(3,2),(4,2),(2,4),(5,2),(2,5),(6,2),(2,6)}
A B = (5,2) (2,5)
2
P(A/B)= 11
13.Apply C1→C1-C2, C2→C2-C3
1 1 𝑐
(a-b)(b-c). 𝑎+𝑏 𝑏+𝑐 𝑐2
−1 −1 𝑎+𝑏
Apply R1→R1+R3
0 0 𝑐+𝑎+𝑏
(a-b)(b-c). 𝑎+𝑏 𝑏+𝑐 𝑐2
−1 −1 𝑎+𝑏
Expanding along R1 and getting
(a-b)(b-c)(c-a)(a+b+c)
14.Find the first derivative and cross multiplying :y1√𝑥 2 + 𝑎2 = 1

Find the second derivate and getting the answer


15.PUT y = x2
(𝑥 2 +1)(𝑥 2 +4) 𝑦 2 + 5𝑦+4 7𝑦+19
= =1+
(𝑥 2 +3)(𝑥 2 −5) 𝑦 2 − 2𝑦−15 (𝑦+3)(𝑦−5)
7𝑦+19 1 27
1+ (𝑦+3)(𝑦−5)
= 1 + 4(𝑥 2 +3) + 4(𝑥 2 −5)
(𝑥 2 + 1)(𝑥 2 + 4) 1 −1
1 27 𝑥 − √5
∫ = 𝑥 + 𝑡𝑎𝑛 + 𝑙𝑜𝑔 | |
(𝑥 2 + 3)(𝑥 2 − 5) 4√3 √3 8√5 𝑥 + √5
16.Let x and y are length and breadth of the rectangle
20−(2+ 𝜋)𝑥 2 𝜋𝑥 2
Area of the figure = 4
+ 8
20
When A’(x) 0 gives x = 4+ 𝜋
Second derivative is negative at this value of x
Area is maximum when
20
Length =
4+ 𝜋
10
Breadth = 4+ 𝜋
10
Radius of semi-circle =4+ 𝜋

17.F(x)=sin 𝑥 +cos 𝑥
𝑓 −1 (x)=cos 𝑥 -sin 𝑥
𝑓 −1 (x)=0
Getting interval
𝜋 𝜋 5𝜋 5𝜋
(0, 4 ), ( 4 , 4 ),( 4 ,𝜋)
Getting increasing in
𝜋 5𝜋
(0, 4 ), ( 4 ,𝜋)
Getting decreasing in
𝜋 5𝜋
(4 , 4 )
OR
Finding point of intersection
(𝑘32 , 𝑘31)
𝑑𝑦
Finding 𝑑𝑥

1 𝑑𝑦 𝑑𝑦 −𝑦
= &
2𝑦 𝑑𝑥 𝑑𝑥
= 𝑥

1
Multiply - 2𝑥 =-1
Writing 8𝑘 2=1
18.Show that semi-vertical angle of a right circular cone, of given surface
1
area and maximum volume , is sin−1 3 .
Surface area of the cone S = πrl + πr 2
=πr(l + r)
s
l= −r ……………………. [1]
πr
1
V =3 πr 2 h
1
V 2 = s(sr 2 − 2πr 4 )
9
1
Z = s(sr 2 − 2πr 4 ) [1]
9
dz 2
dr
= 9 s(sr − 4πr 3 ) [1]
d2 z 2
dr2
= 9 s(s − 12πr 2 ) [1]

dz s
= 0, implies r=√
dr 4π

d2 z
<0 [1]
dr2

1
θ = sin−1 3 [1]
OR
2x+2r+πr = 10
2x+(2 + π)r = 10 [1]
1
Area = 2πx + 2 πr 2
1
= 10 – (2 + π)r 2 + 2 πr 2 `[1]
dA
= 10 − (4 + π)r [1]
dr
10
For maximum area r = [1]
4+π
d2 A
<0 …………… [1]
dr2

Dimension of the rectangle are


10 20
m, m For correct graph ……………………………. [1]
4+π 4+π
4x 2 + 4y 2 = 9
x2= y solving we get
1 3
y = 2 , x = √2, − √2 radius of the circle =2 [1]
1 3
3 2
Area = 2 ∫02 2√y dy + 2 ∫12 2√(2) − y 2 dy [1]
2
For correct integration ………………………………………….. [2]
1 9π 9 1
Area= [3 + 8 − 4 sin−1 3]sq.unit [1]
√2
OR
For correct graph ………………………………… [1]
y = 4x + 5
y = −x + 5
1 5
y = x + [2]
4 4
0 3 3 x 5
Area= ∫−1(4x + 5)dx + ∫0 (−x + 5)dx − ∫−1 (3 + 4) dx [1]
415
= 2
sq.units.

𝒅𝒚 𝒚+√𝒙𝟐 +𝒚𝟐 𝒚 𝒚 𝟐
19. . = = + √𝟏 + ( )
𝒅𝒙 𝒙 𝒙 𝒙
Homo generous differential equation
Put y=vx
𝑑𝑦 𝑑𝑣
=𝑣+𝑥
𝑑𝑥 𝑑𝑥
𝑑𝑣
v+x = 𝑣 + √1 + 𝑣 2
𝑑𝑥
𝑑𝑣 𝑑𝑥
=
√1 + 𝑣 2 𝑥
Log(v+√1 + 𝑣 2 ) = logcx 2 Marks
𝑦 𝑦2
+ √1 + = cx
𝑥 𝑥2
20.Writing 𝑑̂= 𝑥𝑖̂+y𝑗̂+𝑧𝑘̂
Writing x+4y+2z=0
3x-2y+7z=0
2x-y+4z=15
Writing x=32
y =⅄
z = -14⅄
15
writing⅄ = 7

180 15 220
𝑑̂= 7 𝑖̂ + 7 𝑗̂- 7 𝑘̂
⃗⃗⃗⃗⃗2 -⃗⃗⃗⃗⃗
21.𝑎 𝑎1 = 4i+6j+8k

Writing 𝑏⃗⃗⃗⃗1 * 𝑏⃗⃗⃗⃗⃗2 =-4i-6j-8k

Find | 𝑏⃗⃗⃗⃗1 * 𝑏⃗⃗⃗⃗⃗2 | = 116

Find SD=√116
22.
25 35 40
𝑃( 𝐴) = ; 𝑃(𝐵) = ; 𝑃(𝐵) =
100 100 100
e) 𝑃(𝐷) = 𝑃(𝐴)𝑃(𝐷/𝐴) + P(B) P (D / B) + P(C) P(D / C)
= 0.0345
28
f) P(B / D) = using Baye’s theorem
69
2
23.P(E1) =
12

4
P(E2)= 12

6
P(E3)=12
A = Ensured person meets accident

P(A/E1)= .01 P(A/E2)= .03


P(A/E3)= .15
2
∗ .01
12
P(E1/A)= 2 4 6
∗ .01+ ∗ .03+ ∗ .15
12 12 12

2
= 51
Moral – every driver should have insurance to protect families future and expenditures on treatment after
accident.
24.Expressing : AX = B
0 1 −2
Getting 𝐴−1 = (−2 9 −23)
−1 5 −13
x=1; y=2;z=3
25.Using the property of definite integrals:I
𝜋 1
=∫ dx
0 𝑎2 𝑐𝑜𝑠2 𝑥+ 𝑏 2 𝑠𝑖𝑛2 𝑥
2
Multiply and divide by Sec x, put t = tan x
Change the limits
𝜋2
I=
2𝑎𝑏
26.points of intersection (1,0)
1
Area = ∫0 (√1 − 𝑥 2 − (1 − 𝑥))𝑑𝑥

𝜋 1
Area = 4 − 2
27.The equaiton of the line through A and parallel to given line:
𝑥−1 𝑦+2 𝑧−3
= =
2 3 −6
28.The LPP is
Min Z = 4x + 6y subject to
3x + 6y  80 ; 4x + 3y  100, x,y0

Mimimum cost = Rs.104 when 24 units of


4
F1 and 3 units of F2 are mixed.
To get all neccesary nutrients for proper
growth and functioning of body, a
balanced diet is must for every body

29.f(𝑥1 )=f(𝑥2 )
9𝑥12 +6𝑥1 -5=9𝑥2𝟐+6𝑥2 -5
𝑥1 = x 2
f(x)=y
9𝑥 2 +6x-5=y
9𝑥 2 +6x-(5+y)=0

−6±√36+36(5+𝑦)
X= 18

Onto
Writing
−1+√6+𝑦
𝑓 −1 (x)=
3
KENDRIYA VIDYALYA SANGATHAN ,
Model Question Paper-3: CLASS XII
Class :XII Max Marks: 100

Subject: Maths Max Time:03 hours

General Instructions
 All questions are compulsory
 The question paper consists of 29 questions divided into three sections A, B ,C&D
 Section A contains 4questions of one mark each, Section B is of 8 questions of two
marks each and Section C comprises of 11 questions of four marks each,section D
comprises 6 questions of six marks each.
 There is no overall choice. However, internal choice has been provided in 30% questions
of four marks and six marks .You have to attempt only one of the alternatives in all such
questions.
 Use of calculators is not permitted

SECTION – A

1. Find f -1 when f={(1,a) (2,c) (3,b) }


3𝜋
2. Find the principal value of 𝑠𝑖𝑛−1 [𝑠𝑖𝑛 ]
5
3. If A and B are symmetric matrices of the same order, then show that AB – BA is a skew
symmetric matrix.
4. Find a unit vector parallel to the sum of vectors a  2iˆ  4 ˆj  5kˆ and b  iˆ  2 ˆj  3kˆ

SECTION – B
5. Define a Symmetric and Skew Symmetric Matrices with an example

1 1 1 1
6. Evaluate: 𝑡𝑎𝑛−1 + 𝑡𝑎𝑛−1 + 𝑡𝑎𝑛−1 + 𝑡𝑎𝑛−1
5 7 3 8

𝑑2 𝑦
7. If x=a cost and y=b sint then find 𝑑𝑥 2.
8. The volume of a cube is increasing at a rate of 9cubic cms per sec. How fast is the surface area
increasing when the lengh of an edge is 10 cm?
9. Evaluate:  ( cot x  tan x )dx

𝑑𝑦
10. . Find particular solution of the differential equation (x3+x2+x+1)𝑑𝑥=2x2+x;y=1when
x=0

11. A family has two children. What is the probability that both the children are boys given that at least
one of them is a boy?
12. Find the area of the triangle points A(1 , 1, 1), B(1,2,3) and C(2,3,1) as its vertices?

SECTION C
15. By using properties of determinants, show that
a b c
a 2
b c 2  (a  b)(b  c)(c  a)(a  b  c)
2

bc ca ab


𝑑𝑦 √1+𝑠𝑖𝑛𝑥+ √1−𝑠𝑖𝑛𝑥
16. Find for y = 𝑡𝑎𝑛 −1 [ ]
𝑑𝑥 √1+𝑠𝑖𝑛𝑥− √1−𝑠𝑖𝑛𝑥
2 2
15. If y = 𝑙𝑜𝑔{𝑥 + √𝑥 + 𝑎 } prove that (x2 + a2) y2 + x y1 = 0
𝜋 𝑥
16. Evaluate: ∫ dx
0 𝑎 𝑐𝑜𝑠 𝑥+ 𝑏 2 𝑠𝑖𝑛2 𝑥
2 2
17.Using vector, show that the points A(2, 3, 4), B(-1, -2, 1) and C(5, 8, 7) are collinear.
18. In a bolt factory, machines A, B and C manufacture 25%, 35% and 40% of the total.
Of their output 5%, 4%, 2% are defective. A bolt is drawn at random
from the product. If the bolt drawn is found to be defective, find the probability
that it is a product of machine B?
What value inherent above question?
19. An open top box is to be constructed by removing equal squares from each corners of a 3
m by 8 m rectangular shape of aluminium and folding up the sides.Find the volume of
the largest such box. What value shows this question?

(OR)
Manufacturer can sell x items at a price of Rs (5-x/100)each.The cost price of x item is Rs
(x/5)+100.Find the number of items he should sell to earn maximum profit?
What value indicate this problem?
20. Water is running in to a conical vessel, 15 cm deep and 5 cm in radius, at the rate of 0.1
cm3/sec. When the water is 6 cm deep, find at what rate is the level of water increasing.
OR
Find the intervals in which the function  2 x 3  9 x 2  12 x  1 is strictly increasing or decreasing.
21 Solve the differential equation: (tan-1y – x)dy = (1 + y2) dx
22 Find the equation of the plane passing through the line of intersection of the plans 2x + 6y +
12 = 0 and 3x – y + 4z = 0 which are at a unit distance from the origin.
23 .A bag A con tains 8 white and 7 black balls while the other bag B contains 5 white and 4
black balls. One ball is randomly picked up from bag A and mixed up with the balls in the bag B.
Then a ball is randomly drawn from it. Find the probability the ball drawn is white.
(OR)
Find the mean and variance of the number of heads in a two tosses of a coin

SECTION - D
24. Consider f: R +
[-5, ∞) given by f (x) = 9 x2 + 6x-5 show that f is invertible . Also find f-1 .

26 Using Matrices, solve the following system of equations: 2x – 3y


+ 5z = 11 ; 3x +2 y – 4z = -5 ; x + y – 2z = -3

26. Find the area lying above X axis and included between the circle x2+y2=8x and Parabola y2=4ax.

27. Find the distance of the pointA (-2, 3, -4) from the line (x+2)/3 = (2y+3)/3 =(3z+4)/5 measured to the
plane 4 x +12 y -3 z +1=0 ?
28. A diet is to contain at least 80 units of vitamin A and 100 units of minerals. Two foods F1 and F2
are available. Food F1 cost Rs.4/- per unit and F2 costs Rs.6/- per unit. One unit of food F1 contains 3
units of vitamin A and 4 units of minerals. One unit of food F2 contains 6 units of vitamin A and 3
units of minerals. Formulate this as a linear programming problem. Find the minimum cost of the diet
that consists of mixture of these two foods and also meets the minimal nutritional requirements.
What value you think this question contain?
 /4
29 Evaluate:  log 1  tan xdx
0
MARKING SCHEME

SECTION A

Q.1 For correct answer f -1 = {(a,1),(c,2),(b,3)}


2𝜋
Q.2 For correct answer 5

Q.3 Proving correctly AB-BA is a skew symmetric matrix


3 6 −2
Q.4 7 𝑖 + 7 𝑗 + 𝑘
7

SECTION B

. Q.5 For correct definition and example of symmetric and skew symmetric matrix
1 1 1 1
Q.6 (tan -1 5 + tan -1 7)+ (tan -1 3 + tan -1 8)

6 11
(ii). tan -1 17 + tan -1 23

325
(iii) tan -1 325

𝜋
(iv) For correct answer 4

𝑑𝑦 −𝑏
Q.7 (i) for correct = 𝑐𝑜𝑡 𝑡
𝑑𝑥 𝑎

(ii) for correct d2y/dx2 =(-b cosec3t)/a 2

𝑑𝑣
Q8.(i) For writing 𝑑𝑡 =9 cm3/sec

𝑑𝑥 3
(ii). V = x3 and S = 6x2 , for calculating = 𝑥2
𝑑𝑡

𝑑𝑆 36
(iii) For =
𝑑𝑡 𝑥

𝑑𝑆
(iv) For = 3.6 cm2/sec
𝑑𝑡

Q 9. (i) Put tanx = t2 , so that sec2x = 2t dt


𝑡2+1
(ii) I = 2∫ 𝑡 4 +1dt

𝑡𝑎𝑛𝑥−1
(iii) I=√2tan-1( )+c
√2𝑡𝑎𝑛𝑥

2𝑥 2 +𝑥
Q10 dy =(𝑥+1)(𝑥 2 +1)dx

1 3𝑥−1
∫ 𝑑𝑦=∫(2(𝑥+1)+2(𝑥 2 +1))dx

1 3 1
Y=2log(x+1)+4log(𝑥 2 +1)-2tan-1 x+c

1 3 1
Y= log(x+1)+ log(𝑥 2 +1)- tan-1 x+1
2 4 2
Q.11 S= { (b,b),(g,b),(b,g)(g,g)}

E = both the children are boys, F= atleast one of the child is a boy
3 1
P(f) =4 , P(E∩ 𝐹) = 4

1
P(E/F)= 3

Q.12 AB = j +2k , AC= i+2j

ABx AC = -4i+2j-k
1 1
Area of triangle = 2 |ABx AC|= 2 √21

Q.13 LHS= for applying C1 C1-C3 , C2 C2-C3

𝑎−𝑐 𝑏−𝑐 𝑐
|𝑎2 − 𝑐 2 𝑏2 − 𝑐 2 𝑐2 |
𝑐−𝑎 𝑐−𝑏 𝑎+𝑏

C1 C1+C2 ,

0 1 𝑐
(c-a)(b-c)|𝑏 − 𝑎 𝑏 + 𝑐 𝑐2 |
0 −1 𝑎 + 𝑏
Expanding along c1 = (a-b)(b-c)(c-a)(a+b+c)

𝑥 𝑥 𝑥 𝑥
Q14. (i). For putting 1 = sin22 + cos22 , sinx = 2sin2 cos2

𝜋 𝑥
(ii). For writing tan-1{tan(2 - 2)}

𝑑𝑦
(iii) for 𝑑𝑥 = - ½

𝜋 𝑑𝑥
Q16. 2I=𝜋 ∫0 𝑎2 𝑐𝑜𝑠2 𝑥+𝑏 2 𝑠𝑖𝑛2 𝑥
𝜋/2 𝑠𝑒𝑐 2 𝑥
I=𝜋 ∫0 𝑎2 +𝑏2𝑡𝑎𝑛2 𝑥 𝑑𝑥
𝜋2
I= 2𝑎𝑏

Q.18 B1 : bolt manufactured by machine A , B2 : bolt manufactured by machine B


B3 : bolt manufactured by machine C , E: bolt is defective

. (i)For correct probabilities of B1,B2and B3

(ii) For correct conditional probabilities P(E/B1)=.05, P(E/B2)=.04, P(E/B3)=.02


28
(iii) Use of Bayes theorem and correct solution P(B2/E) =69

For correct value answer

Q19 Let x m be the length of a side of the removed square. Then l = 8-2x, b= 3-2x

and h = x.V(x) = 4x3 -22x2+24x

V’(x) = 4(x-3)(3x-2)

V’’(x) = 24x-44

Critical point x = 2/3

V’’(2/3) < 0
200
Max. volume = m3
27

Writing correct values

OR

Let S(x) be the selling price of x items and let C(x) be the cost price of

x items.

𝑥2 𝑥
S(x) = 5x- , C(x) = 5 + 500
100

24 𝑥2
P(x) = 𝑥- -500
5 100

24 𝑥 1
P’(x) = - 50 P’’(x) = - 50 Critical
5
point x = 240

1
P’’(240) = - 50 < 0

Writing correct values

20. V = Volume of water in the cone


𝑟 5
tan  = ℎ = 15
h = 3r
𝜋
V = 27 ℎ3
𝑑ℎ 2.7 1
= =
𝑑𝑡 3𝜋ℎ2 40𝜋
OR
f ' x   6 x 2  18x  12   6x  2x  1
For intervals f ' x  0
 6x  2x  1  0
x  1 and x  2
Interval Value of f ' x  Sign of f ' x  Nature of
function
 ,2 f '  3  12 Negative Decreasing
 2,1 f '  1.5  1.5 Positive Increasing
 1,  f ' 0  6 Negative Decreasing
𝑑𝑥 𝑥 𝑡𝑎𝑛−1 𝑦
21. The given DE is + =
𝑑𝑦 1+𝑦 2 1+𝑦 2
𝑡𝑎𝑛−1 𝑦
I.F = 𝑒
−1
Solution: x = (tan-1y – 1 ) + 𝑐𝑒 𝑡𝑎𝑛 𝑦

22. Required plane : (2x + 6y + 12) +  (3x – y + 4z) = 0


Use perpendicular distance from (0,0,0) = 1
 = 2
The required planes are 2x + y + 2z + 3 = 0 and x – 2y +2 z – 3 = 0

23.

A = event of transferring a white ball


B = event of transferring a black ball
8 7
P(A) = 15 and P(B) = 15
E = event of selecting white ball from II bag
P(E) = P(AE or BE) = P(AE) + P(BE)
8 6 7 5 83
= 15 × 10 + 15 × 10 = 150
OR
X 0 1 2
P(X) 𝟏 𝟏 𝟏
𝟒 𝟐 𝟒
3
Mean = 2
3
Variance = 4

√𝑦+6 −1
Q24. ( i ). For correct x= g(y)= 3

(ii) gof = IR+

(iii) fog = I[-5, )

(iv) f -1 =g

Q.25 i) 2x-3y+5z= 11, 3x +2y -4z = -5 and x +y-2z = -3

ii) Write in matrix form AX = B


0 1 −2
iii) for correct A -1 = [−2 9 −23]
−1 5 −13
iv) For calculation and solution x=1,y=2 and z=3

Q.26 (i) For drawing the correct curve line and circle

(ii) for point of intersec tion of two curves ( 0,0) and (4,4), (4 √2,0)

4 4 √2
(iii)For writing ∫0 (𝑥 𝑑𝑥 + ∫4 (√32 − 𝑥 2 𝑑𝑥

(iv) For evaluating the answer 4𝜋 square unit.

4 1 4
Q16 ∫0 |𝑥 − 1|𝑑𝑥 = ∫0 −(𝑥 − 1)𝑑𝑥 + ∫1 (𝑥 − 1)𝑑𝑥

−𝑥 2 𝑥2
=[ + 𝑥] 10 + [ 2 − 𝑥] 41
2

=5

Q17 I=∫ √4 − (𝑥 + 1)2 dx

Put x+1=y;dx=dy

I=∫ √4 − 𝑦 2 𝑑𝑦
1 𝑥+1
I= (𝑥 + 1) √3 − 2𝑥 − 𝑥 2 + 2𝑠𝑖𝑛−1 ( )+𝑐 2
2 2

OR

S = { 1,2,3,4,5,6 } X is the random variable which can take values 1,2,3,4,5or6.

X 1 2 3 4 5 6

1/6 1/6 1/6 1/6 1/6 1/6


P(X)
21 91
For getting E(x)= 6 , E(x2)= 6 ,

35
For getting var(x)= 12

SECTION C

3 4
𝑥+2 𝑦+ 𝑧+
2 3
Q.23 Equation of the line = = 5
3 2
3

3 4 5
Coordinates of any point P on this line be (-2+3t, - 2 +2t, - 3 +3t )

9 5 8
Direction ratios of AP are 3t,2t- 2 ,3 t +,3 .

Direction ratios of normal to the plane 4x+12y-3z+1=0 are (4,12,-3)

Formulae a1a2+b1b2+c1c2=0

t=2

5
the refore point P(4, 2,2)

17
AP = units
2

27Equation of plane containing the given point is

A(x-1) +B(y-1)+C(z-2)=

2A+3B-2C=0

A + 2B – 3C = 0

5x-4y-z= 7

Q28. Let x units of food F1 and y units of F2 be mixed to get the desire diet.

Min Z = 4x +6y

Subject to the constraints

3x +6y ≥ 80
4x +3y ≥ 100

x, y 0

For finding correct feasible region

Corner points : Value of Z = 4x +6y

80 2
A( 3 , 0) 1063

4 104 (Min)
B(24, 3)

100 200
C(0, )
3

The feasible region has no point common with 4x +6y < 104.

Min cost of diet = Rs.104.

For correct value answer

a a
29. Applying  f x dx   f a  x dx correctly
0 0

Getting up to log 2x   I 4


0


Getting I= log 2
8

SAMPLE QUESTION PAPER - 4

MATHEMATICS

CLASS XII

TIME : 3 HOURS Maximum Marks: 100

General Instructions:

(xix) All questions are compulsory.


(xx) This question Paper contains 29 questions.
(xxi) Question 1 - 4 in Section A are very short-answer type questions carrying 1 mark each.
(xxii) Question 5 - 12 in section B are short answer type question carrying 2 marks each.
(xxiii) Question 13 - 23 in section C are long answer type I question carrying 4 marks each .
(xxiv) Question 24 - 29 in section D are long answer type II question carrying 6 marks each .

SECTION - A

4. State the reason why the Relation R = { ( a , b ) : a ≤ 𝑏 3 } on the set R of real numbers is not

Reflexive.
0
2. Evaluate |cos 700 sin 200 |
𝑠𝑖𝑛70 𝑐𝑜𝑠200

𝑎 = 2𝑖̂ - 𝑗̂ + 2𝑘̂ and 𝑏⃗⃗ = - 𝑖̂ + 𝑗̂ + 3𝑘̂


3. Find the unit vector in the direction of the sum of the vectors ⃗⃗⃗⃗

4. The binary operation * on R is denoted by a * b = 2a + b . Find ( 2 * 3 )

SECTION – B
1 1 2
5. Prove that tan−1 7 + tan−1 13 = tan−1 9

5 2 3 6
6. Find Matrix X and Y , if X + Y = | | and X - Y = | |
0 9 0 −1
2𝑥
7. Differentiate with respect to x , sin−1 ( )
1+ 𝑥 2

8. Find the rate of change of area of a circle with respect to its radius ‘ r ‘ when r = 6 cm.

9. Evaluate ∫ sin−1(cos 𝑥) 𝑑𝑥

10. Find the differential of the family of all straight lines.

11. Find the position vector of the mid – point of the vector joining the points P ( 2 , 3 , 4 ) and

Q ( 4 , 1 , -2 ).
3 7 9
12. If A and B are the two events such that P ( A ) = 5
, P(B)= 10
and P ( A ∪ 𝐵 ) = 10
, then find

P(A∩𝐵).

SECTION – C

13. Using properties of determinants , show that

1  a 2  b2 2ab 2b
 1  a 2  b 2 
3
2ab 1  a 2  b2 2a
2b 2a 1  a 2  b2

sin 𝑥
+ cos 𝑥, 𝑥 > 0
𝑥
14. Show that the function f (𝑥) given by 𝑓(𝑥) = 2 ,𝑥 = 0
4(1−√1−𝑥)
{ 𝑥
,𝑥 < 0

is continuous at 𝑥 = 0.

OR
1−𝑐𝑜𝑠4𝑥
, 𝑥>0
𝑥2
If the function f defined by f(x) = 𝑎 , 𝑥=0
√𝑥
, 𝑥>0
{ √16+√𝑥 −4 }
is continuous at x = 0, find the value of a.
−1 𝑥 𝑑2𝑦 𝑑𝑦
15. If y = 𝑒 𝑚 cos , prove that (1-𝑥 2 ) -x = m2y.
𝑑𝑥 2 𝑑𝑥

16. Find the intervals in which the function

1 − 12𝑥 − 9𝑥 2 − 2𝑥 3 is increasing or decreasing.

OR
Find the equation of the normal line to the curve 𝑦(𝑥 − 2)(𝑥 − 3) − 𝑥 + 7 = 0 at the point where

it meets the 𝑥 −axis.

17. Show that height of the cylinder of maximum volume that can be inscribed in a

2R
sphere of radius R is . Also find the maximum volume.
3

x 4 dx
18. Evaluate 
 x  1  x 2  1
19. Find the general solution of the differential equation: (1 + 𝑦 2 )𝑑𝑥 = (tan−1 𝑦 − 𝑥)𝑑𝑦.

OR
𝑥 𝑥
Solve the differential equation : 2𝑦. 𝑒 𝑦 𝑑𝑥 + (𝑦 − 2𝑥𝑒 𝑦 ) 𝑑𝑦 = 0 .

20. Express the vector 𝑎⃗ = 5𝑖̂ − 2𝑗̂ + 5𝑘̂ as the sum of two vectors such that one is parallel to the

vector 𝑏⃗⃗ = 3𝑖̂ + 𝑘̂ and other is perpendicular to 𝑏⃗⃗ .

21. Find the shortest distance between the lines whose vector equations are

𝑟⃗ = (1 − 2𝑡)𝑖̂+(1 − 𝑡)𝑗̂+(𝑡)𝑘̂ and


𝑟⃗ = (2 + 3𝑠)𝑖̂ +(1 − 5𝑠)𝑗̂+(2𝑠 − 1)𝑘̂

22. A speaks truth in 70% of the cases and B speaks truth in 80 % of the cases .In what

percentage of the cases :-

(i) They contradict each other in stating the same fact?

(ii) They agree each other in stating the same fact?

Truth is very closely related with our national movement. How?

23. By examining the chest X-ray , the probability that TB is detected when a person actually
suffering is 0.99 . The probability of incorrect diagnosis is 0.001. In a certain city one in

thousand persons suffer from TB . A person selected at random and is diagnosed to have TB.

What is the chance that he actually has TB.

SECTION – D

24. Let f : N → R be a function defined as f (x ) = 4x2 + 12x + 15 . Show that f : N → 𝑆 , where S is


the range of f , is invertible. Also find the inverse of f .

OR

A binary operation * is defined on the set X = R – { - 1 } by x * y = x + y + xy , ∀ x , y ∈ X.

Check whether * is commutative and Associative. Find its identity element and also find the
inverse of each element of X.

25. Two school A and B decided to award prizes to their students for three values

honesty(x),punctuality (y) and obedience(z).School A decided to award a total of Rs 11000

for the three values of 5,4 and 3 students respectively while school B decided to award

Rs 10700 for the three values of 4, 3 and 5 students respectively. If all the three prizes

together amount to Rs. 2700, then:

(vii) Represent the above situation by a matrix equation and form linear equations using matrix
multiplication.
(viii) Is it possible to solve the system of equation so obtained using matrix? If yes, find the
award money for each value

OR

Using the properties of determinants , prove that:

𝑥 𝑥2 1 + 𝑝𝑥 3
|𝑦 𝑦2 1 + 𝑝𝑦 3 | = (1 + 𝑝𝑥𝑦𝑧)(𝑥 − 𝑦)(𝑦 − 𝑧)(𝑧 − 𝑥)
𝑧 𝑧2 1 + 𝑝𝑧 3

26. Using integration, find the area of the triangle whose vertices are 𝐴(1,0), 𝐵(2,2) 𝑎𝑛𝑑 𝐶(3,1)

 /2 x
27. . Evaluate 
0 sin x  cos x
dx

OR
2
Evaluate the following integral as a limit of a sum   3x  1 dx.
2

28. Find the equation of the plane passing through the point (-1, 3, 2) and perpendicular to each
of the planes 𝑥 + 2𝑦 + 3𝑧 = 5 and 3𝑥 + 3𝑦 + 𝑧 = 0. Also find the angle between this plane

and x-axis.

29. A dealer wishes to purchase number of fans and sewing machines . He has only Rs. 5760 to
invest and has a space for at most 20 items . A fan cost him Rs. 360 and sewing machine Rs. 240 .

His expectation is that .He can sell a fan at profit of Rs. 22and sewing machine at a profit of

Rs.18 . Assuming that he can sell all the items that he can buy , how should invest his money in

order to maximize the profit ? Formulate this as a linear programming problem and solve it

graphically .

MARKING SCHEME

38. For Reflexivity ( a , a ) ∈ R


1 1
Clearly ½ is a real no. , but 2 is not less than or equal to (2)3
1 1
( 2 , 2 ) not belongs to R
Hence , R is not Reflexive. 1 mark
39. Cos 700.cos200 - sin 700 sin 200
= cos (70 + 20 ) = 0 1 mark
40. For getting unit vector 1mark
41. a * b = 2a + b
2*3=7 1 mark
1 1
42. tan−1 + tan−1
7 13
Applying formula 1 mark
2
= tan−1 9 1 mark
4 4
43. Getting value of X = [ ] 1 mark
0 4
1 −2
Y =[ ] 1 mark
0 5
44. Taking x = tan𝜃 ,
Getting y = 2𝜃 1 mark
𝑑𝑦
For getting 𝑑𝑥
=2 / ( 1+x2 ) 1 mark
45. For getting dA/dr = 2𝜋r 1 mark
(dA/dr ) at r = 6 = 12 𝜋 cm2/cm 1 mark
46. ∫ sin−1 cos x dx
𝜋
Putting cos x = sin ( 2 − x ) 1mark

𝜋
To get the result 2
x – x2 / 2 + c 1 mark

47. For equation y = mx + c 1 mark


𝑑𝑦
𝑑𝑥
=m & for second derivative = 0 1 mark
48. For writing 𝑎⃗ =2i +3j+4k and 𝑏 ⃗⃗ = 4𝑖̂ + 𝑗 − 2𝑘̂ 1 mark
For finding P.V.of mid point= 3i+2j+k 1 mark
49. For writing P(AUB)= P(A)+p(B)-P(A∩B) 1 mark
2
For getting correct answer P(A∩B) = 5
1 mark
1  a 2  b2 2ab 2b
2ab 1 a  b
2 2
2a
13.
2b 2a 1  a 2  b2
R1  R1  bR2 , R2  R2  aR3

………………………………(1+1/2)

1  a 2  b2 0 b 1  a 2  b 2 
0 1  a 2  b2 a 1  a 2  b 2 
2b 2a 1  a 2  b2

1 0 b
1  a 2
b 
2 2
0 1 a ………………………………(1+1/2)
2b 2a 1  a 2  b 2

1  a  b  1  a
2 2 2 2
 b2  2a 2  2b2 
………………………………..(1)
 1  a  b 
2 2 3

14. For finding: LHL=2 (1 mark)

For finding: RHL=2 (1 mark)

For finding:f (0) =2 (1 mark)

LHL=RHL= f(0) , therefor function is continuous at x=0 (1 mark)

OR

To find LHL= lim− 𝑓(𝑥) = 8


𝑥→0
To find RHL= lim+ 𝑓(𝑥) = 8
𝑥→0
Value = a =8
𝑑𝑦 𝑚𝑦
15. =-
𝑑𝑥 √1−𝑥 2

𝑑𝑦 2𝑥
√1−𝑥 2 ∗(−𝑚) −(−𝑚𝑦)
𝑑2𝑦 𝑑𝑥 2√1−𝑥2
= 2
𝑑𝑥 2 √1−𝑥 2

𝑑2𝑦 𝑑𝑦
(1-𝑥 2 ) -x = m2y.
𝑑𝑥 2 𝑑𝑥

16.
 𝑓 ′(𝑥) = −6(𝑥 + 1)(𝑥 + 2)  1
 Finding critical points 𝑥 = −1, −2  1
 Finding intervals for increasing and decreasing functions  1
 Writing answer: Increasing in (−2, −1), Decreasing in (−∞, −2) ∪ (−1, ∞)  1

OR

 Finding the point (7,0)


 1
𝑑𝑦
 Finding 𝑑𝑥
 1
𝑑𝑦 1
 Finding 𝑑𝑥 at (7,0) = 20 
1
2

 Slope of normal −20 


1
2

 Finding equation of normal:20𝑥 + 𝑦 = 140


 1

17.

Let r and h be the radius and height of the cone respectively inscribed in a sphere of radius R.

1
mark

Let V be the volume of the cone.

Then,

Height of the cone is given by,

h = R + AB 1mark

1
mark
2
marks

1
mark

∴ By second derivative test, the volume of the cone is the maximum when

1
mark
𝑋4 𝑥 4 − 1+1
18. ∫ (𝑥−1)(𝑥2 +1 ) 𝑑𝑥 = ∫ (𝑥−1)(𝑥2 +1 ) 𝑑𝑥

𝑥4− 1 1
= ∫ (𝑥−1)(𝑥2 +1 ) 𝑑𝑥 + ∫ (𝑥−1)(𝑥2 +1 ) 𝑑𝑥

1
= ∫(𝑥 + 1 )𝑑𝑥 + ∫ (𝑥−1)(𝑥2 +1 ) 𝑑𝑥

𝑥2 1 𝑑𝑥 1 𝑥+1
= 2
+ 𝑥 + ∫
2 𝑥−1
− ∫
2 (𝑥 2 +1 )
𝑑𝑥

𝑥2 1 1 1
= 2
+ 𝑥 + 2
𝑙𝑜𝑔|𝑥 − 1| − 4
log|(𝑥 2 + 1 )| - 2 tan−1 𝑥 + 𝑐

19

𝑑𝑥
Writing the equation in the form: 𝑑𝑦 + 𝑃(𝑦). 𝑥 = 𝑄(𝑦) 𝑖. 𝑒.
𝑑𝑥 𝑥
+ 1+𝑦2 =  1
𝑑𝑦
tan−1 𝑦
……… (1)
1+𝑦 2


1
Writing 𝑃(𝑦) = 1+𝑦2 , 𝑄(𝑦) =
tan−1 𝑦  1/2
1+𝑦 2

−1 𝑦
 Integrating factor = 𝑒 tan  1

 Multiplying integrating factor in (1) and changing the equation in form:  1/2

tan−1 𝑦
−1 𝑦 −1 𝑦
𝑒 tan . 𝑥 = ∫ 𝑒 tan
𝑑𝑦 .
1 + 𝑦2
−1
 Finding the general solution: 𝑥 = tan−1 𝑦 − 1 + 𝑐 𝑒 −tan 𝑦  1

OR
𝑥
𝑑𝑥 2𝑥𝑒 𝑦 −𝑦
 Writing D.E. in the form: = 𝑥 ………. (1)
𝑑𝑦
2𝑦.𝑒 𝑦
 1
 Putting 𝑥 = 𝑣𝑦 𝑎𝑛𝑑
𝑑𝑥
=𝑣+
𝑑𝑣
𝑦 𝑑𝑦 ………….. (2)  1
𝑑𝑦
and converting D.E. in the form variable separable as
 1
𝑑𝑦
2𝑒 𝑣 𝑑𝑣 = −
𝑦

 Integrating both sides and getting general solution


𝑥
2. 𝑒 𝑦 = − log|𝑦| + 𝐶  1

⃗⃗⃗⃗1 +𝑏
20. . Let 𝑎⃗=𝑏 ⃗⃗⃗⃗⃗2

⃗⃗⃗⃗
𝑏1 =𝛽(3𝑖̂+𝑘̂)

⃗⃗⃗⃗⃗
𝑏2 =𝑥̂i+𝑦̂j+𝑧̂ k Marks 1

𝐹raming equation 3x+z=0 Marks 1


Ans𝛽=2 x=-1,y=-2,z=3 Marks 2

21. ⃗⃗⃗⃗⃗ 𝑎1 = 𝑖̂ − 𝑘̂
𝑎2 − ⃗⃗⃗⃗⃗ (1 mark)

⃗⃗⃗⃗⃗⃗⃗⃗⃗⃗⃗⃗⃗⃗⃗⃗
𝑏1 𝑋𝑏⃗⃗⃗⃗⃗2 = 3𝑖̂ − 𝑗̂ − 7𝑘̂ (1 mark)

⃗⃗⃗⃗1 𝑋𝑏
|𝑏 ⃗⃗⃗⃗⃗2 | = √59 (1 mark)
10
Shortest distance between the line:√59 (1 mark)

22. (i) They contradict each other in stating the same fact = 38% ( 1.5 mark)

(ii) They agree each other in stating the same fact= 0.62% (1.5 marks)

For value (1 mark)

23. Consider A1 : suffers from TB, P  A1   0.001 ------------------------------------(1)

A2 : person donot suffer fromTB, P  A2   0.999        (1)

C: Doctor diagnoses correctly

Then P(C / A1 )  0.99 and P(C / A2 )  0.001        (1)

P( A1 ).P(C / A1 ) 110
By Baye ’s theorem P( A1 / C )          (1)
P( A1 ).P(C / A1 )  P( A2 ).P(C / A2 ) 221

24. For proving one-one (2 Marks)

For proving onto (2 Marks)

For finding inverse (2 Marks)

OR

5 4 3 𝑥 11000
25. [4 3 5 ] [ 𝑦 ] = [ 10700]
1 1 1 𝑧 2700
5x+4y+3z=11000

4x+3y+5z=10700

x +y + z=2700 (1marks)

for getting |A|= -3 (1 marks)

−2 −1 11
-1 −1
getting A = 3 [1 2 −13] (2 marks)
1 −1 −1
getting x=1000,y=900 and z=800 (2 marks)
OR

For Commutative (2 marks)

For Associative (2 marks)

For finding Identity (1 mark)

For finding Inverse (1 mark)

25.

𝑥 𝑥2 1 𝑥 𝑥2 𝑝𝑥 3
|𝑦 𝑦2 1|+|𝑦 𝑦2 𝑝𝑦 3 | Mark 2
𝑧 𝑧2 1 𝑧 𝑧2 𝑝𝑧 3

Similarly 2 marks for using each properties

Getting proof (Marks 2)

26. Drawing correct rough sketch and obtaining point of intersection of given

lines ( 2 marks)
2 3 3 𝑥−1
𝑎𝑟𝑒𝑎 𝑜𝑓 ∆𝐴𝐵𝐶 = ∫1 2(𝑥 − 1)𝑑𝑥 + ∫2 (4 − 𝑥)𝑑𝑥 − ∫1 𝑑𝑥 (2 marks)
2

3
𝑎𝑟𝑒𝑎 𝑜𝑓 ∆𝐴𝐵𝐶 = 2 (2 marks)


2
x
27. I   dx
0
sin x  cos x


 2
1
2  sin x  cos x
2I  dx ……………………………….(1)
0


 2
1
2
2I  dx …………………………(1)
2 1 1
0 sin x  cos x
2 2

 2
  
2 
2I  cos ec  x   dx ………………………….(2)
2 0  4


I
2 2
log  2 1  …………………………………………(2)

OR

By definition of limit sum we have


2

  3x  1dx  lim h  f 1  f 1  h   f 1  2h       f 1   n  1 h  


2
h 0
1

where nh  1              (1)

3 1  1  1     ntimes   1  1      ntimes  
 
= lim h 
h 0 

 3 h 2  22 h 2        n  1 h 2
2

      (1)


  3.2  h  2h        n  1 h  

 n  n  1 2n  1 n(n  1) 
= lim h  2n  3h   6h    6            (2)
2
h0
 6 2 

28. Equation of plane passing through (-1, 3,2) is A(x+1)+B(y-3) +C(z-2)=0 (1 mark)

Finding (A, B,C) = (-7, 8 -3) (2 marks)

Getting equation of plane: 7x-8y+3z+25=0 (2 marks)


7
Finding angle with x axis 𝑐𝑜𝑠 −1 √122 (1 mark)

29. Let x be the no. of fans and y be the no. of sewing machines

If p be the total profit ,

P= 22x+18y

360x+240y≤5760. For correct graph, ---------- (1)

i.e., 3x+2y≤48.

x+y≤20.

x≥0,y≥0 -------------------------------- (2)

E(0,24)

(0,20)D

C(8,12)

O (16,0)A (20,0)
po int s valueof p
(0, 0) 0
(16, 0) 352
(8,12) 392
(0, 20) 360

the dealer gets a maximum profit of Rs. 392 when he purchase and sells 8 fans and 12 sewing machines.

Investment in fans =360.×8=2880.

Investment in sewing machines =240×12= 2880.-------------------(3)

SAMPLE QUESTION PAPER - 5

MATHEMATICS

CLASS XII

TIME : 3 HOURS Maximum Marks: 100

General Instructions:
1. All questions are compulsory.

2. This question paper consists of 29 questions divided into Four sections A, B, C andD. Section A comprises of
4 questions of one mark each, section B comprises of 8 questions of 02 marks each and section C comprises of
11 questions of 04 marks each and section D consists of 06 questions of 06 marks each.

3. All questions in Section A are to be answered in one word, one sentence or as per

the exact requirement of the question.

4. There is no overall choice. However, internal choice has been provided in 03

Questions of four marks each and 02 questions of six marks each. You have to

attempt only one of the alternatives in all such questions.

5. Use of calculators is not permitted. You may ask for logarithmic tables, if

required.
________________________________________________________________________

SECTION - A (1 mark questions)

1. Let * be a binary operation on Q given by a*b=a+ab where a,b  Q .Is * commutative?


1  4 
2. Evaluate Sin  cos 1   
2  5 
3. Given that A, B are two symmetric matrices such that AB =BA .Is AB symmetric?
 x  y  z   9
   
4. Find the values of x, y and z if  x  z    5 
 y  z   7
   
SECTION - B (2 mark questions)

Sin30 0 Cos30 0
5. Evaluate
 sin 60 0 Cos60 0
6. Find a point on the curve y=x2-4x+5 where the tangent to the curve is parallel to the x axis
x  cos 6 x
7. Evaluate  3x 2
 Sin6 x
dx
      
8. Find the projection of the vector a  i  3 k on the vector b  3 i  j  4 k
     
9. If a is a unit vector and ( x  a )( x  a )  15 find x

2x  1 y2 z3
10. The Cartesian equation of a line AB is   .Find the direction cosines of a line
3 2 3
parallel to A

11. Let f: R  R be defined by f(x) = 3x+2.Show that f is invertible. Also find f  1 :R  R


 1 
12. Write in the simplest form : tan  1   where x  1
 
 x2  1 
SECTION - C (4 marks questions)

x 6 1
13. Show that x = 2 is a root of the equation 2  3 x x  3  0 and solve it completely.
 3 2x x2
 1  Cos2 x x0

 x2
14. Discuss the continuity of the function at x = 0: f ( x)  


5 x0

 
15. Verify Rolle’s theorem for the function f(x)= Sin2x in 0, 
 2
dy 1  y2
16. If 1  x  1  y  a ( x  y ) prove that
2 2

dx 1  x2

OR

If y=x2 +4 and x changes from 2 to 2.1 find the approximate change in y

2
17. Evaluate  e x dx as a limit of sum
0
OR

5
Evaluate   x  1x 2
4 dx
dy
18. Solve the differential equation Cos x
2
 y  tan x
dx
19. Find the differential equation for the family of circles which passes through the origin and have their
centre on the x axis
       

20. If a  b  c  0 and a  3 ; b  5 c  7 then show that the angle between a and b is 600

21. Find the vector equation of the plane passing through the intersection of the planes r .
      
( 2 i  7 j  4 k )  3 and r . ( 3 i  5 j  4 k )  11  0 and passing through the point (-2,1,3)
22. A coin is tossed three times and all the possible outcomes are assumed to be equally likely. Let E and F
be two events given by E: “both tail and head have occurred” F: “at most one tail has occurred”. Show
that E and F are independent.

23. The sum of three numbers is 6.Twice the third number when added to the first number gives 7.On
adding the sum of the second and third numbers to thrice of the first number we get 12.Find the
numbers using inverse of a matrix.
Or

 1 2 3 

Using elementary transformations find the inverse of the matrix 2 5 7 

  2  4  5

SECTION - D (6 mark questions)

24. Let R be a relation on N  N defined by (a, b)R (c, d)  ad=bc, for all (a,b) and (c,d)  N . Show that R is
an equivalence relation .
OR

Let * be the binary operation defined on QxQ by (a,b)*(c,d) = (ac,b+ad) where Q is the set of
rational numbers. Determine whether * is commutative and associative . Find the indentiy
element for * and the inverttible elements of QxQ
25. An open box with a square base is to be made out of a given quantity of metal sheet of area c 2.Show
c3
that the maximum volume of the box is
6 3
1
26. Evaluate  Sinx(2  Cosx )dx
Or


2
Evaluate  log Sinxdx
0

27. Find the area of region included between the parabolas y2=4ax and x2=4ay where a>0
28. Find the foot of the perpendicular drawn from the point A(1,0,3) to the line joining the points B(4,7,1)
C(3,5,3)
29. Mona wants to invest at most Rs 12000 in Savings Certificates (SC) and National Saving Bond (NSB). She
has to invest at least Rs 2000 in SC and at least Rs 4000 in NSB. If the rate of interest on Sc is 8% and the
rate of interest on NSB is 10% per annum, how much money should she invest to earn maximum yearly
income?

******

ANSWER KEY WITH MARKING SCHEME

Q.No Value points Marks Total


Marks

1 Not Commutative (as a*b=a+ab  b+ba=b*a) 1M 1M

2 1 4 4  1M 1M
(put cos  1 =   cos  = Given expression =Sin =
10 5 5 2
4
1
1  Cos 5= 1
=
2 2 10

3 AB is Symmetric. (AB) t =B t A t =BA=AB 1M 1M

4 x=2 ; y= 4 ; z= 3 1M 1M

( by equating corresponding entries we get x+y+z = 9 ; x+z =5 ; y+z


= 7)

5 0 0
Sin30 Cos 60 +Cos30 0 Sin60 0 1M 2M
=Sin 900=1 IM

6 dy 1M 2M
(2,1) is the point .  2 x  4 =0  x  2 putting x =2 in y=x2-
dx
4x+5 we get y = 1

7 1 1 1M 2M
6 t
Put 3 x  Sin6 x = u  Given integral =
2
dt =

1
log 3 x 2  Sin6 x  c
6

8     1M 2M
projection of the vector a  i  3 k on the vector b 
 
   a .b
3 i  j 4 k =

b

3  0  12 15
= =
9  1  16 26
1M

9    1M 2M
x 2
-a 2
=15  x 2


=15+1  x = 4 1M

10 1 1M 2M
x
2  y  2  z  3 dr’s of AB are 3 ,4,6
3 4 6

3 4 6
Hence dc’s of AB are , ,
55 55 55
1M

11 f ( x1 )  f ( x2 )  3 x1  2  3 x 2  2  x 1 =x 2 Thus f is one 1M
to one

y2 y2 1M
y=3x+2  x= Given any y  R there exists =x  R
3 3
s.t. f(x)=y showing that the function is onto.
2
1 x2
f (x)= is the inverse of f
3
1
12 x=Cosec    =Cosec x

 
1  1  1M
tan  1   = tan 1  
   
 x 1  Co sec   1 
2 2

2
  1M
 = tan 1 
1 
 = tan Tan = 
1
 1 1
= tan
   Cot 
 Cot  
2

1
=Cosec x

13 x  2 3x  6  x  2 1M
5  5x  5  0 by performing R1  R1  R2 and
3 2x x2
R2  R2  R3

1 3 1 1M
5(x-2) 1  x 1  0
 3 2x x2

0 3 1 1M
5(x-2) 0 x  1  0 C1  C1  C 3 4
x  1 2x x2

0 3 1 ½M
5(x-2)(x-1) 0  x 1  0
1 2x x2

= -5(x-2)(x-1)(x+3) = 0  x  2,1,3 ½M

14 2 Sin 2 x
Here f(0) = 5 and Lt f(x) = Lt =2
x 0 x 0 x2

Sinx Sinx
2 Lt Lt =2  1  1 =2
x 0 x x 0 x

As Lt f(x)  = f(0) ,f is not continuous


x 0

15   ½M
Consider the function f(x) = Sin2x in 0,  . Here f is
 2
 
continuous in 0,  as f(x)= Sin x is continuous
 2

  1M
f ' (x) = 2 Cos2x exists in ( 0, ) thus f is differentiable on ( 0, )
2 2
  1M
f(0)=Sin (0) = 0 and f   =Sin  = 0
 2

Conditions for Rolle’s thm are satisfied. Hence there should be at ½M


 
least one c   0,  such that f (c ) = 0
'
4
 2

   1M
Let 2Cos2c = 0  c =   0, 
4  2

16 Let x=Sin A and y=Sin B ½M

Cos A + Cos B =a(Sin A –Sin B) ½M

A B A B A B A B 1M
2Cos Cos =a  2Cos Sin
2 2 2 2 4

A B 1M
Cot =a  Sin-1x-Sin-1y=2Cot-1a
2

1 1 ½M
Differentiating , - y ' =0
1  x2 1  y2

½M
dy 1  y2

dx 1  x2

OR

x= 2 x  0.1 ½M

y  f ' ( x )x =2x x =2  2 0.1 =0.4 1½M

When x=2 y = 22+4=8 ½M 4

y+  y =8+0.4=8.4 1M

y changes from 8 to 8.4 ½M

17. ba 2 ½M
a=0 b=2 h=  nh=2 f(x)=ex
n n
2 1M
e
x
dx =
0

Lt h  f (0)  f (0  h)  f (0  2h)  ......  f (0  (n  1)h)


h 0

h 0

= Lt h 1  e  e  e  .....  e
h 2h 3h  n 1 h
 ½M
Lt h.

1. e h  1  n
 1M
h 0 eh  1 4

= Lt
h

e 2  1 =e 2 -1  1M
h 0 e 1
h

Or

5 A Bx  C 1M
 2
 =
 x  1 x  4 x  1 x  4
2

To get A =1 B = -1 and C = 1 1M

5 1  x 1 ½M
  x  1x dx =  dx  
2
4  x1 x2  4
dx

x 1 1M
=log x  1 -  x 4
2
dx   2
x 4
dx 4

1 1 x ½M
=log x  1 - log x 2  4  tan 1  c
2 2 2

18 dy 1M
Dividing Cos x
2
 y  tan x by Cos2x to get
dx
dy
 ySec 2 x  tan xSec 2 x
dx

I.F. =etanx ½M
4
Solution is y(IF)=  tan xSec 2 x etanxdx+ k 1M

 Put tan x =u  y etanx =  ue u du  k ½M

 y etanx =e u ( u  1)  k ½M

 y =tanx-1+ke  tan x ½M

19 1M

Let a be the radius Then centre is


(a,0)

= n to the Circle is (x-a) 2 +y 2 =a 2  x  y  2ax  0 ….(*) 1M


2 2
Differentiating ,2x+2yy’-2a=0  a=x+yy ' =a 1M

4
Putting this value of a in (*) we get 2xyy ' -y  x =0 1M
2 2

20   ½M
Any vector parallel to b is of the form k b for some scalar k

     1M
Let a = k b + c where c is perpendicular to b

    ½M
Then c =( a - k b )  b

1M
      
 ( a - k b ). b  ( a . b )-k( b . b )=0  k=2 4
    ½M
k b =2 b = 6 i  2 k

      ½M
c =( a - 2 b )=(  i  2 j  k )

OR

      ½M
a b c  0  a b   c

   ½M
 ( a  b ) 2  ( c ) 2

      ½M
 ( a  b ).( a  b )  (  c ).(  c )

     1M
2 2 2
a +b +2 a . b = c

  15 ½M
a .b =
2
  1M
a .b 15 1
Cos  = =    60
0
=
 
2 3 5 2 4
a b

21     1½M
r . { ( 2  3 ) i  ( 7  5 ) j  (4  4 ) k }  3  11  0 …(*)

4
   1½M
(*) Passes through the point with position vector ( 2 i  j  3 k )
1
 
6
 1M
Putting the value of  in (*) we get r .
  
(15 i  47 j  28 k )  7  0

22. E: “both tail and head have occurred” ½M+½M


={HTT,THT,TTH,HHT,HTH,THH}

F: “at most one tail has occurred”= {HHH,HHT,THH,HTH}

E  F ={HHT,HTH,THH} ½M

6 4 3 1½M
P(E) = P(F)= P( E  F )= 4
8 8 8

3 6 4 1M
As P( E  F )=P(E) P(F)    is true ,we conclude that E
8 8 8
and F are independent.

23 Let the first, second and third numbers be x, y and z respectively. 1M


Then x + y + z=6 : x + 2z =7 ; 3x + y + z=12

 1 1 1  x 6 ½M
     
 AX=B where A=  1 0 2  X=  y  B=  7 
 3 1 1 z  12 
     

A  4  0  A is invertible ½M

 2 0 2  1M
  4
adjA=  5  2  1 
 1 2  1 

 2 0 2   6   3 1M
1 1     
X=A B=  5  2  1  7  =  1 
A  1
 2  1   12   2 

The required numbers are 3 ,1 and 2 ½M

OR

 1 2 3  ½M

Let A= 2 5 7 

  2  4  5

 1 2 3  1 0 0 1M

A=IA  2 5 7  = 0 1 0 A

  2  4  5 0 0 1 6
1 0 0
By performing elementary transformations ,to get 0 1 0 =
 
0 0 1
 3  2  1
  4 1  1 A 3½M
 
 2 0 1 

 3  2  1 1M
 A =   4 1  1
1

 2 0 1 

24

(a,b) R (a,b)  ab=ba Reflexive 1.5M

(a,b) R (c,d)  ad=bc  cb=da  (c,d) R (a,b) symmetric 1.5M 6

(a,b) R (c,d) and (c,d) R (e,f)  ad=bc and cf=de  ad.cf=bc.de 2½ M


 af=be  (a,b) R (e,f) Transitive

Thus R is an equivalence relation ½M

25 Let x be the side of the square base and y be the height. 1½M

c2  x2
V=x2y ,Surface area S= x2+4xy=c2  y  4 x

1
V= (c x  x )
4
2 3

dx 4

dV 1 2
 c  3x2  d 2V  3
dx 2

2
x
1½M

dV c d 2V   3c 1M
0 x  2 
 0
dx 3 dx  x  c 2 3
3

c c 1M
V is Maximum when x  . Then y 
3 2 3 6

c3 1M
V Max =
6 3

26 Sinx ½M
 Sin x( 2  Cosx )
2
dx by multiplying the Nr and Dr by Sinx

Sinx 1 1M
=  (1  Cos 2 x)(2  Cosx)dx =  (1  t 2 )( 2  t )dx by letting
Cosx=t
1 ½M
=  (1  t )(1  t )( 2  t )dx
1 A B C 1M
=  
(1  t )(1  t )( 2  t ) 1  t 1  t 2  t

1 1 1 1½M
To get A= B= C=
6 2 3
6
1 1M
 (1  t )(1  t )( 2  t )
dx =

1 1 1
log 1  t  log 1  t  log 2  t  k
6 2 3

1
 sin x(2  Cosx )dx =
1 1 1
 log 1  Cosx  log 1  Cosx  log 2  Cosx  c
6 2 3
½M

OR

   1M
2 2   2
I=  log Sinxdx =  log Sin  x dx =  log Cosxdx
0 0 2  0

  1M
2 2 2 SinxCosx
2I=  log SinxCosxdx =  log dx
0 0 2

 2M
2 2 SinxCosx
=  log dx =
0 2 6

2  
 log Sin2 xdx  log 2  I 1  log 2 ………(*)
0 2 2

 1M
1  1 2
I1 =  log S int dt =  2  log Sinxdx =I
2 0 2 0

  1M
(*)  2I=I- log 2  I= log 2
2 2
27 2M

Solving the two equations given we get x=0 and x=4a 1M

4a 4a x 2 2M
Required area =  4ax dx   dx
0 0 4a
6

32a 2 16a 2 16a 2 1M


=   sq.units
3 3 3

28 Let P be the foot of the perpendicular from A on BC. If P divides BC ½M


 3k  4 5k  7 3k  1 
in the ratio k:1 then P is  , , 
 k 1 k 1 k 1 
1M

Dr’s of BC are 1,2,-2 ½M

3k  4 5k  7 3k  1 1M
Dr’s of AP are  1,  0, 3
k 1 k 1 k 1

7 2M
Since AP  BC ,dot product =0  k 
4

 5 7 17  1M
So the foot of the perpendicular is  , ,  6
3 3 3 

29 Suppose that she invests Rs x in SC and Rs y in NSB ½M

8 x 10 y 1½M
Then LPP is to maximize Z=  subjecting to constraints x
100 100
 2000 y  4000 x+y  12000
2½M

(To
draw the graph, to identify the feasible region and to get
coordinates)

8 x 10 y 1M
To put the coordinates of the vertices in Z=  to get
100 100 6
values Rs 560, Rs 1040 Rs 1160

Rs 2000 should be invested in in savings certificates and Rs 10000 ½M


in National Savings Bonds to get a maximum yearly income of Rs
1160

MATHEMATICS

Time allowed:- 3 hours Maximum marks :- 100

General Instructions:-

1. All questions are compulsory.


2. The question paper consists of 29 question , divided into four sections A , B, C & D. Section A comprises of
4 questions of one mark each,Section B comprises of 8 questions of 2 marks each,Section C comprises of
11 questions of 4 marks each & Section D comprises of 6 questions of 6 marks each.
3. All questions in section A are to be answered in word , one sentence or as per the exact requirement of
the question.
4. There is no overall choice however internal choice has been provided in 3 questions of 4 marks each and
3 questions of 6 marks each.You have to attempt only one of the alternative in all such questions.
5. Use of calculator is not permitted.You may ask for logarithmic table if required.

Section A

1. Determine the value of k for which the following function is continuous at x=5
𝑘𝑥 + 1 𝑖𝑓 𝑥 ≤ 5
F(x) ={
3𝑥 − 5 𝑖𝑓 𝑥 > 5
2. Let A be a non singular square matrix of order 3× 3 then find the value of l A l
𝑠𝑖𝑛2 𝑥− 𝑐𝑜𝑠2 𝑥
3. Find: ∫ 𝑠𝑖𝑛𝑥.𝑐𝑜𝑠𝑥
𝑑𝑥
4. Find the length of the perpendicular drawn from the origin to the plane 2x-3y+6z+21=0 ?

Section B

5.Find the intervals in which the function f(x) = x2+2x-5 is strictly increasing or decreasing.
6.The volume of cube is increasing at a rate of 9 cubic centimetres per second.How fast is the surface area
increasing when the length of an edge is 10cm.

7.If A and B are symmetric matrices, prove that AB-BA is a skew symmetric matrix.

8.Verify mean value theorem if f(x) = x2-4x-3 in the interval [1,4].

9.A family has two children . What is the probability that both the children are boys given that at least one of
them is a boy?

1−cos 2𝑥
10.Find ∫ √ 1+𝑐𝑜𝑠2𝑥 dx ?

11.Solve the following linear programming problem graphically

Minimise z=200x +500y


Subject to the constraints: x+2y≥ 10
3x+4y ≤24
x≥ 0 , y≥ 0

12.What are the direction cosine of the line which makes equal angles with the coordinate axes ?

Section C

13.Prove that :-

√1+𝑥−√1−𝑥 𝜋 1 1
Tan-1{ } = − cos−1 𝑥 , - ≤𝑥≤1
√1+𝑥+√1−𝑥 4 2 √2

14.Two cards are drawn simultaneously (without replacement) from a well shuffled pack of 52 cards.Find
the probility distribution of number of aces.Also find the mean of distribution?

15.Two groups are competeting for the post of board of directors of corporation.The probabilities that the
first and second group wins are 0.6 and 0.4 respectively.Further if the first groups wins the probability of
introducing a new product is 0.7 and the corresponding probability is 0.3 if the second group wins.Find the
probability that the new product was introduced by second group?what is the importance of healthy
competetation in our life.

16.Let 𝑎⃗ = 𝑖̂ +4𝑗̂ + 2𝑘̂ , 𝑏⃗⃗ = 3𝑖


̂ −2𝑗̂ + 7𝑘̂ and 𝑐⃗ = 2𝑖̂ −𝑗̂ + 4𝑘̂. Find the 𝑝⃗ which is perpendicular to both
⃗⃗⃗⃗and 𝑝⃗.𝑐⃗ = 18 ?
𝑎⃗&𝑏

17.Prove that for any three vectors 𝑎⃗, 𝑏⃗⃗ 𝑎𝑛𝑑 𝑐⃗.

[𝑎⃗ + 𝑏⃗⃗𝑏⃗⃗ + 𝑐⃗𝑐⃗ + 𝑎⃗]= 2[𝑎⃗𝑏⃗⃗𝑐⃗]

18.Using properties of determinants prove that

1 + 𝑎2 − 𝑏 2 2𝑎𝑏 −2𝑏
| 2𝑎𝑏 1 − 𝑎2 + 𝑏 2 2𝑎 | = (1+a2+b2)3
2𝑏 −2𝑎 1 − 𝑎2 − 𝑏 2
OR
1 2 2
If A = ⟦2 1 2⟧ then prove that A2 -4A -5I = 0
2 2 1

2
19.If yx =ey-xthen prove that dy/dx=(1+logy) /logy
OR
2 2 2 2 2
If y=(tan-1x) then show that (1+x2) d y/dx +2x(1+x )dy/dx=2

𝜋
20.Evaluate ∫02 𝑙𝑜𝑔𝑠𝑖𝑛𝑥dx

Or

2
Evaluate∫−1|𝑥 3 − 𝑥|dx

21. A marchant plans to sell two types of personal computers,a desktop model and a portable Model that will
cost Rs 25000 and Rs 40000 ,respectively. He estimates that the total Monthly demand of computers will not
exceed 250 units . determine the number of Unit of each type of computers which the merchant should stock to
get maximum profit, If he does not want to invest more then Rs70 lakh and his profit on the desktop model is Rs
4500 and on the portable model is Rs 5000 . Make an LPP and solve it graphically
sin 𝑥+cos 𝑥
22. Evaluate, ∫ 𝑑𝑥
√sin 2𝑥

23.Solve the differential equation,

(tan−1 𝑦 − 𝑥)𝑑𝑦 = (1 + 𝑦 2 )𝑑𝑥

SECTION-D

24. Show that the semi-vertical angle of a right circular cone of maximum volume and given slant height is
tan-I√2 .

25.Find the equation of the plane passing through the line of intersection of the plane

⃗⃗⃗(𝑖̂ + 𝑗̂ + 𝑘̂)=1
𝑟.

⃗⃗⃗(2𝑖̂ + 3 − 𝑘̂)-4=0 and parallel to x-axis .


𝑟.

OR

Find the distance of the point (2 ,3 , 4 )from the line


𝑥+3 𝑦−2 𝑧
3
= 6
= 2
measured parallel to the plane 3x +2y+2z -5 =0

26. Show that the relation S in set A ={x ∈ 𝑧 ∶ 0 ≤ 𝑥 ≤ 12 } given by

S ={ (a , b ) : a , b ∈ 𝑧 , |𝑎 − 𝑏|𝑖𝑠 𝑑𝑖𝑣𝑖𝑠𝑖𝑏𝑙𝑒 𝑏𝑦 4} 𝑖𝑠 𝑎𝑛 𝑒𝑞𝑢𝑖𝑣𝑎𝑙𝑒𝑛𝑐𝑒 𝑟𝑒𝑙𝑎𝑡𝑖𝑜𝑛 . 𝑓𝑖𝑛𝑑 𝑡ℎ𝑒 𝑠𝑒𝑡 𝑜𝑓 all elements


related to A .

OR

Consider the binary operations ∗ :R× 𝑅 → 𝑅 𝑎𝑛𝑑 𝑜: 𝑅 × 𝑅 → 𝑅 defind as a∗ 𝑏 = |𝑎 − 𝑏| and

a ob = a for all a , b ∈ R ,show that ∗ is commutative but not associative and o is associative but not
commutative .

27.Using matrices , the following system of linear equations

x +y –z =3
2x + 3y + z = 10

3x – y -7z = 1

28.Using the integration,find the area of the region in the first quadrant enclosed by the x-axis, the line y = x
and circle x2+y2=18.

OR

Using integration, find the area of triangle PQR, coordinate of whose vertices are P(2,0),Q(4,5)&R(6,5).
𝑑𝑦 𝑥𝑦
29.Find the particular solution of the differential equation,𝑑𝑥 = 𝑥 2 +𝑦2

given that y=1 when x=0.

MARKING SCHEME

CLASS: XII

SUBJECT : MATH

SECTION: A

1. lim− 𝑘𝑥 + 1= lim+ 3𝑥 − 5 ½
𝑋→5 𝑥→5

5k +1 = 3×5 – 5

K = 9/5 ½

2.Iadj A I =|𝐴|2 1

𝑠𝑖𝑛𝑥 𝑐𝑜𝑠𝑥
3.∫ dx -∫ dx 1/2
𝑐𝑜𝑠𝑥 𝑠𝑖𝑛𝑥

= log secx – log sinx 1/2

4. Length of perpendicular from origin is

2.0−3.0+6.0+21
=| | 1/2
√22 +(−3)2 +62

=3 units 1/2

SECTION-B


5.f (x)=2x+2 1/2

2x+2=0

x= -1 1/2

f(x) is st.decreasing if 𝑓 ′ (x)<0 in (-∞,-1) 1/2

f(x) is st. increasing if 𝑓 ′ (x)>0 in (-1,∞) 1/2


6. V=𝑥 3 , S=6𝑥 2
𝑑𝑉 𝑑𝑥
𝑑𝑡
=3𝑥 2 𝑑𝑡

𝑑𝑥
9=3𝑥 2 𝑑𝑡

𝑑𝑥 3
𝑑𝑡
=𝑥 2

𝑑𝑆 𝑑𝑥
𝑑𝑡
=12𝑥 𝑑𝑡

𝑑𝑆 3
= 12𝑥 ( 2 ) (𝑓𝑟𝑜𝑚 𝑎𝑏𝑜𝑣𝑒 𝑟𝑒𝑠𝑢𝑙𝑡)
𝑑𝑡 𝑥
𝑑𝑆 36
𝑑𝑡
=𝑥 2

𝑑𝑆
[ 𝑑𝑡 ] = 3.6 𝑐𝑚2 1
𝑥=10

7.𝐴′ = 𝐴

𝐵′ = 𝐵 1/2

(𝐴𝐵 − 𝐵𝐴)′ = (𝐴𝐵)′ − (𝐵𝐴)′ 1/2

=𝐵′ 𝐴′ − 𝐴′ 𝐵′

=(𝐵𝐴 − 𝐴𝐵)1/2

=−(𝐴𝐵 − 𝐵𝐴)1/2

therefore(𝐴𝐵 − 𝐵𝐴)is a skew symmetric matrix

8. since𝒇(𝒙)is apolynomialfunction. Therefore it is continuous and differentiable in


[1,4]. 1/2

𝑓(𝑥) = 𝑥 2 − 4𝑥 − 3

Thus, 𝑓(1) = 12 − 4 × 1 − 3 = −6

𝑓(4) = 42 − 4 × 4 − 3 = −3 1/2

then by mean value theorem ∃ c such that,


𝑓(4)−𝑓(1)
𝑓 ′ (𝑐) = 4−1
1/2

−3 − (−6)
2𝑐 − 4 =
3

2𝑐 = 1 + 4
5
𝑐 = 2 𝜖 [1,4]1/2
9. Let b stand for boys and g stand for girls.

S={(b,b),(b,g),(g,b),(g,g)} 1/2

E=both the children are boys = {(b,b)}

F=Atleast one of the child is a boy={(b,b),(b,g),(g,b)} 1/2

E∩F={(b,b)}
𝑃(E∩F)
P(E/F)= 𝑃(𝐹)
1/2

1/4 1
= = 1/2
3/4 3

𝟐sin𝟐 𝒙
10.∫ √𝟐𝒄𝒐𝒔𝟐𝒙 𝑑𝑥 1/2

=∫ tan 𝑥 𝑑𝑥 1/2

= log sec 𝑥 + 𝑐 1

11.correct figure 1

Corner point Value of Z


(0,5) 2500
(4,3) 2300 (minimum) 1
(0,6) 3000

12.let α,β and γ be the angle made by the line with coordinate axes.

Thus, cosα=cosβ=cosγ 1/2

l=m=n 1/2
2 2 2
l +m +n =1

3l2 = 1
1
l=±
√3

1 1 1
thus ,d.c. of a line are ± ,± ,± 1
√3 √3 √3

SECTION-C

13.put x=cos2θ, θ=1/2cos−1 𝑥


√1 + cos2θ − √1 − cos2θ
tan−1
√1 + cos2θ + √1 − cos2θ
√2cos2 θ−√2sin2 θ
= tan−1 1
√2cos2 θ+√2sin2 θ

cosθ−sinθ
=tan−1 (cosθ+sinθ)

1−tan θ
=tan−1 ( )1
1+tan θ

𝜋
=tan−1 tan ( 4 − θ) 1

𝜋
=4 − θ

𝜋 1
=4 − 2 cos−1 𝑥 (on putting value of θ) 1

14.p(getting an ace card ) = 4C1/52C1=1/13

P(X=0)=4C0× 48C2 / 52C2 = 188/221 1

P(X=1)=4C1× 48C1/ 52C2 = 32/221

P(X=2)=4C2× 48C0/ 52C2 = 1/221 1

Therefore, the probability distribution is as follows,

X 0 1 2 1
P(X) 188/221 32/221 1/221

188 32 1
𝜇 = ∑ 𝑥𝑖 𝑝𝑖 = 0 × +1× +2×
221 221 221
34
𝜇 = 221 1

15. E1 : First group wins

E2 : Second group wins

A: The new product is introduced 1

P(E1)=0.6

P(E2)=0.4

P(A/E1)=0.7

P(A/E2)=0.3
P(E2 )P(A/E2 )
P(E2/A)=P(E )P(A/E
1
1 1 )+P(E2 )P(A/E2 )
0.4×0.3
= 1
0.6×0.7+0.4×0.3

=2/9

Any two values… 1

16.Let𝑝⃗ = 𝑥𝑖̂ + 𝑦𝑗̂ + 𝑧𝑘̂

It is given that 𝑎⃗ is perpendicular to 𝑝⃗, then 𝑝.


⃗⃗⃗⃗ ⃗⃗⃗⃗=0
𝑎

(𝑥𝑖̂ + 𝑦𝑗̂ + 𝑧𝑘̂).( 1𝑖̂ + 4𝑗̂ + 2𝑘̂)=0

⇒𝑥 + 4𝑦 + 2𝑧 = 0 … … … … … … … (𝑖)

Also given that 𝑏⃗⃗ is perpendicular to 𝑝⃗, then 𝑝.


⃗⃗⃗⃗ 𝑏⃗⃗=0

(𝑥𝑖̂ + 𝑦𝑗̂ + 𝑧𝑘̂).( 3𝑖̂ − 2𝑗̂ + 7𝑘̂)=0

⇒3𝑥 − 2𝑦 + 7𝑧 = 0 … … … … … … … (𝑖𝑖)

⃗⃗⃗⃗ 𝑐⃗⃗⃗=18
𝑝. (given that)

(𝑥𝑖̂ + 𝑦𝑗̂ + 𝑧𝑘̂).( 2𝑖̂ − 𝑗̂ + 4𝑘̂)=18


1
⇒2𝑥 − 𝑦 + 4𝑧 = 18 … … … … … … … (𝑖𝑖𝑖) 12

On solving equation (i)(ii) and (iii),we get

1
𝑥 = 64 , 𝑦 = −2 , 𝑧 = −281
2

𝑝⃗ = 64𝑖̂ − 2𝑗̂ − 28𝑘̂ 1

17.L.H.S.

=[𝑎 ⃗⃗⃗⃗𝑏
⃗⃗⃗⃗ + 𝑏 ⃗⃗⃗⃗ + 𝑐⃗⃗⃗𝑐⃗⃗⃗ + ⃗⃗⃗⃗]
𝑎

⃗⃗⃗⃗ + ⃗⃗⃗⃗
=(𝑎 𝑏 ).[ ( ⃗⃗⃗⃗
𝑏 + 𝑐⃗⃗⃗) × ( 𝑐⃗⃗⃗ + ⃗⃗⃗⃗
𝑎 )] 1

= ⃗⃗⃗⃗
𝑎 .(𝑏⃗⃗⃗⃗ × 𝑐⃗⃗⃗) + ⃗⃗⃗⃗
𝑎 .(𝑏⃗⃗⃗⃗ × ⃗⃗⃗⃗)
𝑎 + ⃗⃗⃗⃗ 𝑎 + 𝑏⃗⃗ .(𝑏
𝑎 .(𝑐⃗⃗⃗ × ⃗⃗⃗⃗) ⃗⃗⃗⃗ × 𝑐⃗⃗⃗) + ⃗⃗⃗⃗
𝑏 .(𝑏⃗⃗⃗⃗ × ⃗⃗⃗⃗)
𝑎 +)𝑏 ⃗⃗⃗⃗ .(𝑐⃗⃗⃗ × ⃗⃗⃗⃗)1
𝑎

=[𝑎 ⃗⃗⃗⃗𝑐⃗⃗⃗]+[𝑎
⃗⃗⃗⃗𝑏 ⃗⃗⃗⃗⃗⃗⃗⃗]+[𝑎
⃗⃗⃗⃗𝑏 𝑎 ⃗⃗⃗⃗𝑐⃗⃗⃗𝑎 ⃗⃗⃗⃗⃗⃗⃗⃗
⃗⃗⃗⃗]+[𝑏 ⃗⃗⃗⃗⃗⃗⃗⃗
𝑏 𝑐⃗⃗⃗]+[𝑏 𝑏𝑎 ⃗⃗⃗⃗𝑐⃗⃗⃗𝑎
⃗⃗⃗⃗]+[𝑏 ⃗⃗⃗⃗] 1

=2[𝑎 ⃗⃗⃗⃗𝑐⃗⃗⃗]
⃗⃗⃗⃗𝑏 1

18.on applying C1→C1-bC3 and C2→C2+aC3

1 + 𝑎2 + 𝑏 2 0 −2𝑏
2 2 |2
L.H.S.=| 0 1+𝑎 +𝑏 2𝑎
𝑏(1 + 𝑎2 + 𝑏 2 ) −𝑎(1 + 𝑎2 + 𝑏 2 ) 1 − 𝑎2 − 𝑏 2

1 0 −2𝑏
=(1 + 𝑎2 + 𝑏 2 )2 |0 1 2𝑎 |
2 2
𝑏 −𝑎 1−𝑎 −𝑏

=(1 + 𝑎2 + 𝑏 2 )2 [1(1 − 𝑎2 − 𝑏 2 + 2𝑎2 ) − 2𝑏(−𝑏)] 1


=(1 + 𝑎2 + 𝑏 2 )2 (1 + 𝑎2 + 𝑏 2 )

= (1 + 𝑎2 + 𝑏 2 )31

OR

1 2 2 1 2 2
A2= [2 1 2] [2 1 2]
2 2 1 2 2 1
9 8 8 1
= [8 9 8] 1
2
8 8 9

L.H.S= A2-4A-5I

9 8 8 1 2 2 1 0 0
1
=[8 9 8] − 4 [ 2 1 2] − 5 [ 0 1 0]12
8 8 9 2 2 1 0 0 1
0 0 0
=[0 0 0]1
0 0 0

19.Taking log on both side, we get

𝑥𝑙𝑜𝑔𝑦 = (𝑦 − 𝑥) log 𝑒

𝑥𝑙𝑜𝑔𝑦 = (𝑦 − 𝑥)1

Diff. Both sides with respect to x, we get


𝑥 𝑑𝑦 𝑑𝑦
𝑦 𝑑𝑥
+ log 𝑦 = 𝑑𝑥 − 11

𝑑𝑦 𝑦(1 + log 𝑦)
=
𝑑𝑥 (𝑦 − 𝑥)
𝑑𝑦 𝑦(1+log 𝑦)
𝑑𝑥
= 𝑦 1
(𝑦− )
1+log 𝑦

𝑑𝑦 (1+log 𝑦)2
𝑑𝑥
= log 𝑦
1

OR

On diff. Both side ,w.r.t. x


𝑑𝑦 1
𝑑𝑥
= 2 tan−1 𝑥 .1+𝑥2 1

𝑑𝑦
(1 + 𝑥 2 ) 𝑑𝑥 = 2 tan−1 𝑥 1

Again diff. w.r.t. x,


𝑑2 𝑦 𝑑𝑦 2
(1 + 𝑥 2 ) + (2𝑥) = (1+𝑥2 ) 1
𝑑𝑥 2 𝑑𝑥

𝑑2 𝑦 𝑑𝑦
(1 + 𝑥 2 )2 + 2𝑥(1 + 𝑥 2 ) 𝑑𝑥 = 2 1
𝑑𝑥 2

𝜋
20. I=∫02 log sin 𝑥𝑑𝑥
𝜋
π
I=∫02 log sin( 2 − 𝑥) 𝑑𝑥 1/2

𝜋
I=∫02 log cos 𝑥 𝑑𝑥
𝜋 𝜋
2I=∫02 log sin 𝑥 𝑑𝑥 + ∫02 log cos 𝑥𝑑𝑥
𝜋 𝜋
1
2I=∫02 log sin 2𝑥 𝑑𝑥 − ∫02 log 2𝑑𝑥 1
2

𝑑𝑡 𝜋
Put 2𝑥 = 𝑡 , 𝑑𝑥 = 2
, 𝑤ℎ𝑒𝑛 𝑥 = 0 , 𝑡ℎ𝑒𝑛 𝑡 = 0, 𝑎𝑛𝑑 𝑤ℎ𝑒𝑛 𝑥 = 2
𝑡ℎ𝑒𝑛 𝑡 = 𝜋 1/2

1 𝜋 𝜋/2
2I= ∫0 log sin 𝑡𝑑𝑡 − log 2[𝑥]0
2

2 𝜋/2 π
2I=2 ∫0 log sin 𝑡𝑑𝑡 − 2 log 2 1/2

π π
2I=I − log 2, ⇒ I = − log 2 1
2 2

OR

We observe that,

𝑥3 − 𝑥 𝑤ℎ𝑒𝑛 − 1 < 𝑥 < 0


|𝑥 3 − 𝑥| = {−(𝑥 3 − 𝑥) 𝑤ℎ𝑒𝑛 0 < 𝑥 < 1 1
3
𝑥 −𝑥 𝑤ℎ𝑒𝑛 1 < 𝑥 < 2
0 1 2
Therefore, I =∫−1(𝑥 3 − 𝑥)𝑑𝑥 + ∫0 −(𝑥 3 − 𝑥)𝑑𝑥 + ∫1 (𝑥 3 − 𝑥)𝑑𝑥 1

0 1 2
𝑥4 𝑥2 𝑥4 𝑥2 𝑥4 𝑥2
= [4 − ]
2 −1
−[4 − 2 0
] +[4 − 2 1
] 1

1 1 1 1 1 1
=− + − + +4−2− +
4 2 4 2 4 2
11
= 4 sq. Unit 1

21. Let the marchant stock, x desktop computers and y portable computers.

Now,

Types Numbers Cost per Profit


computer
Desktop x 25000 4500 x
Portable y 40000 5000 y
TOTAL At most 250 At most Z=4500x +5000y
7000000

The required LPP is, max Z = 4500 x +5000 y,

subject to constraints , x + y ≤ 250

25000 x + 40000 y ≤ 7000000

i.e. 5 x + 8 y ≤1400 1

& x≥0,y≥0

Figure, 1

From diagram, corner points of the feasible region are (0 , 0),(250 , 0),(200 , 50) & (0 , 175)

Now we calculate value of Z for these corner points,

Corner points Value of Z 1


(0 , 0) 0
(250 , 0) 1125000
(200 , 50) 1150000 (max)
( 0 , 175) 875000

Hence from above table ,the maximum profit is 1150000 /- when 200 desktop &

50 portable computers are stocked. 1

sin 𝑥+cos 𝑥 sin 𝑥+cos 𝑥


22.∫ 𝑑𝑥 = ∫ 𝑑𝑥
√sin 2𝑥 √1+sin 2𝑥 −1

sin 𝑥+cos 𝑥
=∫ 𝑑𝑥 1
√1−(1−sin 2𝑥)

sin 𝑥+cos 𝑥
=∫ 𝑑𝑥 1
√1−(sin 𝑥−cos 𝑥)2

Put sin 𝑥 − cos 𝑥 = 𝑡

⇒(cos 𝑥 + sin 𝑥)𝑑𝑥 = 𝑑𝑡


𝑑𝑡
=∫ 1
√1−𝑡 2

= sin−1 𝑡 + 𝐶

=sin−1(sin 𝑥 − cos 𝑥) + 𝐶 1

23.Given differential equation is,

(tan−1 𝑦 − 𝑥)𝑑𝑦 = (1 + 𝑦 2 )𝑑𝑥


𝑑𝑦 (tan−1 𝑦 − 𝑥)
⇒ =
𝑑𝑥 (1+𝑦 2 )

𝑑𝑥 1 tan−1 𝑦
Or, + 𝑥 = 1
𝑑𝑦 (1+𝑦 2 ) (1+𝑦2 )

Integrating factor (I.F.) is,


1
∫(1+𝑦2 )𝑑𝑦 −1 𝑦
I.F. = 𝑒 ∫ 𝑃𝑑𝑦 = 𝑒 = 𝑒 tan 1/2

We know that, 𝑦 × 𝐼. 𝐹. = ∫(𝑄 × 𝐼. 𝐹. )𝑑𝑦 + 𝐶

−1 𝑦 tan−1 𝑦 −1 𝑦
Thus, 𝑦 × 𝑒 tan = ∫ ( (1+𝑦2 ) × 𝑒 tan ) 𝑑𝑦 + 𝐶 1/2

1
Put ,tan−1 𝑦 = 𝑡 ⇒ (1+𝑦 2 )
𝑑𝑦 = 𝑑𝑡

−1 𝑦
thus, 𝑦 × 𝑒 tan = ∫(𝑡. 𝑒 𝑡 )𝑑𝑡 + 𝐶1 1

= 𝑡𝑒 𝑡 − ∫ 𝑒 𝑡 𝑑𝑡 + 𝐶1
−1 𝑦
=𝑒 tan (tan−1 𝑦 − 1) + 𝐶1 1

24.Let height, slant height ,radius of the base and semi vertical angle of the right circular cone be ℎ , 𝑙, 𝑟 and
α resp.

Then , r2 = l2 – h2 ..........(i) 1

Figure.......
1

1
V=3πr2h

𝜋
V=3 (𝑙 2 . ℎ − ℎ3 )

𝑑𝑉 𝜋 2
= (𝑙
𝑑ℎ 3
− 3ℎ2 ) 1

𝑑𝑉
=0
𝑑ℎ

⇒𝑙 2 = 3ℎ2

⇒r 2 + ℎ2 = 3ℎ2

r =√2ℎ 1

In right triangle CVA,

𝑟 √2 ℎ
tan 𝛼 = 𝑜𝑟, tan 𝛼 =
ℎ ℎ

𝛼 = tan−1 √2 1

𝑑2 𝑉 𝜋
Also = (−6ℎ)<0 as h>0
𝑑ℎ 2 3
Therefore, V is maximum.

Hence volume is maximum, when 𝛼 = tan−1 √2 1

25.Equation of plane in Cartesian form is given by,

𝑥+𝑦+𝑧−1=0 … (𝑖)

2𝑥 + 3𝑦 − 𝑧 + 4 = 0 … (𝑖𝑖) 1

equation of plane passing through the line of intersection of planes (𝑖)&(𝑖𝑖) be,

(𝑥 + 𝑦 + 𝑧 − 1) + 𝜆( 2𝑥 + 3𝑦 − 𝑧 + 4 ) = 0

(1 + 2𝜆)𝑥 + (1 + 3𝜆)𝑦 + (1 − 𝜆)𝑧 + (−1 + 4𝜆) = 0 … (𝑖𝑖𝑖) 1

It is given that plane (𝑖𝑖𝑖) is parallel to x-axis,

Therefore, 1(1 + 2𝜆) + 0(1 + 3𝜆) + 0(1 − 𝜆) = 0 2

⇒1+2𝜆=0
1
⇒𝜆= − 2 1

On putting the value of 𝜆 in equation (𝑖𝑖𝑖), we get required equation of plane,

𝑦 − 3𝑧 + 6 = 0

In vector form,equation of plane is

𝑟⃗⃗⃗. (𝑗̂ − 3𝑘̂ ) + 6 = 0 1

OR

Given equation of line,


𝑥+3 𝑦−2 𝑧
3
= 6 = 2= (say)

Therefore, co-ordinate of any point on this line are,

1
𝑥 = (3λ − 3) , 𝑦 = (6λ + 2) , 𝑧 = 2λ1
2

Now, direction ratio of line PT

= (3λ − 3 − 2 , 6λ + 2 − 3 , 2λ − 4)

or , = (3λ − 5 , 6λ − 1 , 2λ − 4) 1

given that PT is parallel to the plane, therefore

1
3(3λ − 5) + 2(6λ − 1) + 2(2λ − 4) = 01
2

⇒25𝜆−25 = 0
⇒ 𝜆=1 1

Therefore ,co-ordinate of point T ={ (3λ − 3) , (6λ + 2) , 2λ }

=(0 ,8 ,2)

Required distance between points P & T is given by,

=√(0 − 2)2 + (8 − 3)2 + (2 − 4)2

=√3 units 1

26.set A can be written as,

A={0 , 1, 2 ,3 ,4.........12} 1

1.Reflexive:- for any 𝑥 ∈ 𝐴, we get |𝑥 − 𝑥| = 0,which is divisible by 4.

⇒ (𝑥, 𝑥) ∈ 𝑆 ∀𝑥 ∈𝐴
1
Therefore, S is reflexive. 12

2.Symmetric :-for any (𝑥, 𝑦) ∈ 𝑆, we get

|𝑥 − 𝑦|is divisible by 4

⇒|𝑦 − 𝑥| = 4𝜆 ,for some 𝜆∈ 𝑍

⇒ (𝑥, 𝑦) ∈ 𝑆 ∀ 𝑥, 𝑦 ∈ 𝐴

Thus,(𝑥, 𝑦) ∈ 𝑆 ⇒ (𝑦, 𝑥) ∈ 𝑆
1
Therefore, 𝑆 is symmetric. 1
2

3.Transitive:- for any (𝑥, 𝑦) , (𝑦, 𝑧) ∈ 𝑆

⇒|𝑥 − 𝑦|&|𝑦 − 𝑧|are divisible by 4.

⇒|𝑥 − 𝑦| = 4𝜆 and |𝑦 − 𝑧| = 4𝜇 , for some 𝜆 , 𝜇 ∈ 𝑍

⇒𝑥 − 𝑧 = 𝑥 − 𝑦 + 𝑦 − 𝑧

= ± 4𝜆 ± 4𝜇

= ± 4(𝜆 + 𝜇 )

⇒|𝑥 − 𝑧|is divisible by 4.

⇒(𝑥, 𝑧) ∈ 𝑆∀ 𝑥 , 𝑦 , 𝑧 ∈ 𝑍
1
⇒S is transitive. 12

Since S is reflecive,symmetric& transitive. Therefore S is an equivalence relation.

Set of all elements related to A,


1
A={0 , 1, 2 ,3 ,4.........12}
2
OR

(1)For binary operation *:-

Commutative:-

a ∗ b = |a − b| ∀ a, b ∈ R

𝑏 ∗ 𝑎 = |𝑏 − 𝑎| ∀ 𝑎, 𝑏 ∈ 𝑅

= |−(𝑎 − 𝑏)|

= |𝑎 − 𝑏|

Thus , 𝑎 ∗ 𝑏 = 𝑏 ∗ 𝑎 ∀ 𝑎, 𝑏 ∈ 𝑅
1
Therefore ,* is commutative. 1
2

Associative :-

𝑎 ∗ (𝑏 ∗ 𝑐) = 𝑎 ∗ |𝑏 − 𝑐|

= |𝑎 − |𝑏 − 𝑐||.............(i)

&(𝑎 ∗ 𝑏) ∗ 𝑐 = |𝑎 − 𝑏| ∗ 𝑐

= ||𝑎 − 𝑏| − 𝑐|.............(ii)

From (i)&(ii), we have,

𝑎 ∗ (𝑏 ∗ 𝑐) ≠ (𝑎 ∗ 𝑏) ∗ 𝑐
1
Hence , * is not associative. 12

(2)for binary operation o:-

Commutative :-

𝑎𝑜𝑏 =𝑎 ∀ 𝑎, 𝑏 ∈ 𝑅

𝑏𝑜𝑎 =𝑏 ∀ 𝑎, 𝑏 ∈ 𝑅

⇒𝑎𝑜𝑏 = 𝑏𝑜𝑎
1
Thus, 𝑜 is not commutative. 1
2

Associative :-

𝑎 𝑜 (𝑏 𝑜 𝑐) = 𝑎 𝑜 𝑏 ∀ 𝑎, 𝑏 , 𝑐 ∈ 𝑅

⇒ 𝑎 𝑜 (𝑏 𝑜 𝑐) = 𝑎......(iii)

(𝑎 𝑜 𝑏) 𝑜 𝑐 = 𝑎 𝑜 𝑐 ∀ 𝑎, 𝑏 , 𝑐 ∈ 𝑅

⇒ (𝑎 𝑜 𝑏) 𝑜 𝑐 = 𝑎 ......(iv)

From equation (iii) & (iv), we have

𝑎 𝑜 (𝑏 𝑜 𝑐) = (𝑎 𝑜 𝑏) 𝑜 𝑐 , ∀ 𝑎, 𝑏 , 𝑐 ∈ 𝑅
1
Hence ,ois associative. 12

27. 𝐴𝑋 = 𝐵

𝑋 = 𝐴−1 𝐵.......(i) 1

1 1 −1 𝑥 3
Where 𝐴 = [2 3 1 ] , 𝑋 = [𝑦] , 𝐵 = [10]
3 −1 −7 𝑧 1

Now, |𝐴| = 1(−21 + 1) − 1(−14 − 3) − 1(−2 − 9)

= 8 ≠ 0........(ii) 1

Cofactors of [𝐴] →

𝐶11 = −20𝐶12 = 17𝐶13 = −11

𝐶21 = 8𝐶22 = −4𝐶23 = 4

1
𝐶31 = 4𝐶32 = −3𝐶33 = 11
2

Therefore ,

−20 8 4
𝑎𝑑𝑗 𝐴 = [ 17 −4 −3].........(iii) 1/2
−11 4 1
𝑎𝑑𝑗 𝐴
We know that 𝐴−1 = |𝐴|
........(iv)

Thus , from equation (i),(ii),(iii) & (iv), we get


𝑎𝑑𝑗 𝐴
𝑋= |𝐴|
∙𝐵 1/2

−20 8 4 3
1
Or, = [ 17 −4 −3] [10]
8
−11 4 1 1
𝑥 3
Thus , 𝑋 = [𝑦] = [1] 1
𝑧 1

i.e. 𝑥 = 3 , 𝑦 = 1 , 𝑧 = 1 1/2

28. FIG. 1

Equation of given circle,

𝑥 2 + 𝑦 2 = 18…..(i)

& equation of line ,𝑦 = 𝑥 ……(ii)

On solving equation (i) & (ii) , we get

𝑥 2 + 𝑥 2 = 18
𝑥 = ±3 1

We select 𝑥 = 3 because 𝑥 = −3 does not lies in first quadrant.

On putting 𝑥 = 3 in equation (ii), we get


𝑦=3

Thus,point of intersection of circle and given line is P (3 , 3).

And circle cut x-axis at A(3√2, 0).

Therefore, required area in first quadrant,

3 3√2
=∫0 𝑦(𝑙𝑖𝑛𝑒)𝑑𝑥 + ∫3 𝑦(𝑐𝑖𝑟𝑐𝑙𝑒)𝑑𝑥

3 3√2 2
= ∫0 𝑥 𝑑𝑥 + ∫3 √(3√2) − 𝑥 2 𝑑𝑥 1

3√2
3
𝑥2 𝑥 2 (3√2)2 −1 𝑥
= [2] + [ √(3√2) − 𝑥2 + sin ] 1
0 2 2 3√2
3

9 9 √3
= + [9 sin−1(1) − − 9 sin−1 ] 1
2 2 2

9 𝜋 9 𝜋
= + [9 × − − 9 × ]
2 2 2 3

9 9𝜋 9 9𝜋
=2 + 2
−2− 3

3𝜋
= sq. Units 1
2

OR

Figure. 1

Equation of line PQ,

5−0
𝑦−0= (𝑥 − 2)
4−2
5
or, 𝑦 = 2 (𝑥 − 2) ....(i) 1/2

Equation of lineQR,

3−5
𝑦−5= (𝑥 − 4)
6−4

or, 𝑦 =9−𝑥 ....(ii) 1/2

Equation of lineRP,

0−3
𝑦−3= (𝑥 − 6)
2−6
3
or, 𝑦 = (𝑥 − 2) ....(iii) 1/2
4

therefore, area of triangular region PQR


4 6 6
=∫0 𝑦(𝑙𝑖𝑛𝑒 𝑃𝑄)𝑑𝑥 + ∫4 𝑦(𝑙𝑖𝑛𝑒 𝑄𝑅)𝑑𝑥 − ∫2 𝑦(𝑙𝑖𝑛𝑒 𝑅𝑃)𝑑𝑥 1

45 6 63
=∫0 (𝑥 − 2)𝑑𝑥 + ∫4 (9 − 𝑥 )𝑑𝑥 − ∫2 (𝑥 − 2)𝑑𝑥
2 4

4 6 6
5 2
1 2
3 2
1
= [ (𝑥 − 2) ] − [ (9 − 𝑥) ] − [ (𝑥 − 2) ] 1
4 2 2 4 8 2 2

= 5+ 8 −6

=7 sq. Units 1

𝑑𝑦 𝑥𝑦
29.given differential equation is, = .....(i)
𝑑𝑥 𝑥 2 +𝑦2

𝑥𝑦
Let 𝑓(𝑥, 𝑦) =
𝑥 2 +𝑦 2

On replacing 𝑥 by 𝜆𝑥 and 𝑦 by 𝜆𝑦,

𝜆2 𝑥𝑦
𝑓(𝜆𝑥, 𝜆𝑦) = = 𝜆0 𝑓(𝑥, 𝑦) 1
𝜆2 (𝑥 2 +𝑦 2 )

Thus 𝑓(𝑥, 𝑦)is a homogeneous function and given differential equation is a homogeneous differential
equation.

Now put, 𝑦 = 𝑣𝑥
𝑑𝑦 𝑑𝑣
⇒𝑑𝑥 = 𝑣 + 𝑥 𝑑𝑥 1

Thus from equation (i),

𝑑𝑣 𝑣
𝑣+𝑥 =
𝑑𝑥 1 + 𝑣 2
𝑑𝑣 𝑣
𝑥 = −𝑣
𝑑𝑥 1 + 𝑣 2

𝑑𝑣 𝑣 − 𝑣 − 𝑣 3
𝑥 =
𝑑𝑥 1 + 𝑣2
1+𝑣 2 −𝑑𝑥
𝑣3
𝑑𝑣 = 𝑥
1

On, integrating both side, we get

1 1 𝑑𝑥
∫ 3
𝑑𝑣 + ∫ 𝑑𝑣 = − ∫
𝑣 𝑣 𝑥
1
⇒− 2𝑣 2 + log|𝑣| = − log|𝑥| + 𝐶

𝑥2 𝑦
⇒− + log | | = − log|𝑥| + 𝐶 1
2𝑦 2 𝑥

𝑥2
⇒− 2𝑦2 + log|𝑦| − log|𝑥| = − log|𝑥| + 𝐶

𝑥2
or, − 2𝑦2 + log|𝑦| = 𝐶 1

now put 𝑦 = 1 when 𝑥 = 0 , we get 𝐶 = 0


𝑥2
⇒ log|𝑦| = ,which is the required solution. 1
2𝑦 2

1
GROUP-6
CLASS XII SUBJECT MATHEMATICS

1. Question Paper Design

S.N Typology of Very Short Long Long Marks %


questions Short Answer Answer I Answe Weightag
Answer (2 (4 marks) r II e
(1 marks) (6mark
marks) s)

1 Remembering 2 2 2 1 20 20%
2 Understanding 1 3 4 2 35 35%
3 Application 1 - 3 2 25 25%
4 Hots - 3 1 - 10 10%

5 Evaluation - - 1(VBQ) 1 10 10%

Total 1x4=4 2x8=16 4x11=44 6x6=36 100 100%

2. Question –wise break-up


Type of Question Marks per questions Total No. Question Total Marks
VSA 1 4 4

SA 2 8 16
LA-I 4 11 44
LA-II 6 6 36
Total - 29 100

3. Difficulty level
Difficulty level Weightage

Easy 20%
Average 60%
Difficult 20%
4. Choice (s):
There will be no overall choice in the question paper. However, 30% internal choices will be
given in 4 marks and 6 marks questions.

BLUE PRINT
CLASS XII SUBJECT MATHEMATICS

CHAPTERWISE

S.No. Topics VSA SA(2) LAI(4) LA(6) TOTAL


(1)
1 m) RELATIONS AND 1
FUNCTIONS
n) INVERSE 1 1 10
TRIGONOMETRIC
FUNCTIONS
2 m) MATRICES 1 1
13
n) DETERMINANTS 1 1
3 ee) CONTINUITY & 1 2 1
DIFFERENTIABILITY
ff) APPLICATION OF 1 1 1
DERIVATIVES
gg) INTEGRATION 1 2 1 44
hh) APPLICATION OF 1
INTEGRALS
ii) DIFFERENTIAL 2 1
EQUATIONS
4 m) VECTORS 1 1 1
n) 3-DIMENTIONAL 1 1 17
GEOMETRY
5 LINEAR PROGRMMING 1 06
6 PROBABILITY 1 1 10
TOTAL 4*1=4 8*2=16 11*4=44 6*6=36 29(100)
Sample paper

Time: 3 hours Class XII (MATHEMATICS) MM: 100

General Instructions:

4. All the questions are compulsory.


5. This paper contains 29 questions.
6. Question 1-4 in Section A are very short-answer type questions carrying 1 mark each.
7. Question 5-12 in Section B are short-answer type questions carrying 2 marks each.
8. Question 13-23 in Section C are long-answer-I type questions carrying 4 marks each.
9. Question 24-29 in Section D are long-answer-II type questions carrying 6 marks each.
SECTION –A
Questions 1 to 4 carry 1 mark each.
( 2i−3j)
1. Obtain a 2x2 matrix A= [aij] where aij = 2
2. Examine the continuity of the function 𝑓(𝑥) = 2𝑥 2 -1 at x=3
𝑥 𝑥
3. Integrate : ∫ (𝑠𝑖𝑛2 2 − 𝑐𝑜𝑠 2 2) 𝑑𝑥

4. Let 𝑏⃗⃗ = 𝑖̂ + 2𝑗̂ − 2𝑘̂ and 𝑎⃗ = 8𝑖̂ + 𝑗̂ , find the projection of 𝑎⃗ on 𝑏⃗⃗
SECTION –B
Question 5 to 12 carry 2 marks each.
𝑥
5. Simplify : 𝑡𝑎𝑛−1 (√𝑎2 )
−𝑥 2
6. If A and B are the symmetric matrices of the same order, then show that AB – BA is
skew symmetric matrix.
7. If x=a cos t and y=b sin t then find d2y/dx2.
dy √1+sinx+√1−sinx
8. Find dx for y = tan-1
√1+sinx −√1−sinx

9. Find the area of parallelogram, whose adjacent sides are determined by the vectors 𝑎⃗ =
𝑖̂ − 2𝑗̂ + 3𝑘̂ , 𝑏⃗⃗ = 2𝑖̂ + 𝑗̂ − 4𝑘̂
4
10. Evaluate: ∫0 |x − 1| dx
11. Integrate: √3 − 2x − x 2

12. The volume of a cube is increasing at the rate of 9 cubic cm per sec .H=ow fast is the
surface area increasing when the length of an edge is 10 cm.

SECTION-C
Question 13 to 23 carry 4 marks each.

13. By using properties of determinants, show that

a b c

a2 b2 c2 = (a-b)(b-c)(c-a)(a+b+c)

b+c c+a a+b


14. Evaluate: tan-1 1/5 +tan-1 1/7 +tan-1 1/3 +tan-1 1/8
15. Consider f: R+ [-5, ∞) given by f(x) = 9x2 +6x -5, show that f is invertible. Also find f -
1
.
−1 𝑥 𝑑2 𝑦 𝑑𝑦
16. If 𝑦 = 𝑒 𝑎𝑐𝑜𝑠 , −1 ≤ 𝑥 ≤ 1 then show that: (1 − 𝑥 2 ) −𝑥 − 𝑎2 𝑦 = 0
𝑑𝑥 2 𝑑𝑥
(OR)

𝑑𝑦 1−𝑦 2
If √1 − 𝑥 2 + √1 − 𝑦 2 = 𝑎(𝑥 − 𝑦), show that 𝑑𝑥 = √1−𝑥 2

17. The length 𝑥 of a rectangle is decreasing at the rate of 5 cm minute and the width 𝑦 is
increasing at the rate of 4 cm minute , when 𝑥 = 8 cm and 𝑦 = 6 cm, find the rate of
change of
(a) The perimeter and (b) the area of rectangle.

18. Find the shortest distance between the lines whose vector equations are
𝑟⃗ = (1 − 2𝑡)𝑖̂ +(1 − 𝑡)𝑗̂+(𝑡)𝑘̂ and
𝑟⃗ = (2 + 3𝑠)𝑖̂ +(1 − 5𝑠)𝑗̂+(2𝑠 − 1)𝑘̂
2+𝑠𝑖𝑛2𝑥
19. Evaluate ∫ 𝑒 𝑥 (1+𝑐𝑜𝑠2𝑥) 𝑑𝑥
OR
𝑒𝑥
Evaluate: ∫ √5−4𝑒 𝑥 𝑑𝑥
−𝑒 2𝑥

dy
20. Find the particular solution of the differential equation ( x3+x2+x+1 )dx = 2x2+x : given y
=1 and x=0.
OR
dy y
Solve the differential equation x dx –y + xsin =0.
x
𝑑𝑦 𝑦
21. Solve the differential equation : 𝑥 𝑑𝑥 = 𝑦 − 𝑥 𝑡𝑎𝑛 (𝑥 )
22. A family has two children. What is the probability that both the children are boys given
that at least one of them is boy?
OR
Find the variance of the number obtained on a throw of an unbiased die?
23. If 𝑎⃗ = 𝑖̂ + 𝑗̂ + 𝑘̂ and 𝑏⃗⃗ = 𝑗̂ − 𝑘̂ , find a vector 𝑐⃗ such that 𝑎⃗ × 𝑐⃗ = 𝑏⃗⃗ and 𝑎⃗. 𝑐⃗ = 3
OR
Express the vector 𝑎⃗ = 5𝑖̂ − 2𝑗̂ + 5𝑘̂ as the sum of two vectors such that one is

SECTION –D

Question 24 to 29 carry 6 marks each.

24. Using matrices, solve the following system of equations :


2x-3y+5z=11 ; 3x+2y-4z=-5 ; x+y-2z=-3
25. An open top box is to be constructed by removing equal squares from each corners of a
3m by 8m rectangular shape of aluminum and folded up the sides. Find the volume of the
largest such box. What values shows in this question?
OR
x x
Manufacture can sell x items at a price of Rs (5 - 100) each. The cost price of x item is Rs ( 5 +
500). Find the number of items he should sell to earn maximum profit? What values is being
indicated by this problem.
26. Find the area above X-axis and included between the circle x2+y2=8x and parabola
y2 =4ax.
27. Find the equation of the plane that contain the points (1,-1, 2) and is perpendicular to each
of the planes 2x+3y-2z=5 and x+2y-3z=8?
OR
Find the foot of perpendicular and image of the point (1, 2,-3) in the line
x+1 y−3 z
= =
2 −2 −1
28. A diet is to contain at least 80 units of vitamin A and 100 units of minerals. Two foods F1
and F2 are available. Food F1 cost Rs 4/- per unit and F2 costs Rs 6/- per unit .One unit of
food F1 contains 3 units of vitamin A and 4 units of minerals. One unit of food F2 contains
6units of vitamin A and 3 units of minerals. Formulate this as a linear programming
problem. Find the minimum cost of the diet that consists of mixture of these two foods and
also meets the minimal nutritional requirements. What value you think this question
contain?
29. In a bolt factory, machine A, B and C manufacture 25%, 35% and 40% of the total. Of
their output 5%, 4%, 2% are defective .A bolt is drawn at random from the product. If the
bolt drawn is found to be defective, find the probability that it is a product of machine B?
OR
If a machine is correctly set up, it produces 90% acceptable items. If it is incorrectly set
up, it produces only 40% acceptable items. Past experience show that 80% of the set ups
are correctly done. If after a certain setup, the machine produces 2 acceptable items, find
the probability that the machine correctly set up.

Marking Scheme

Ques.no. Steps Mar


ks
SECTION - A
Q 1. 1
For correct matrix A = -1/2 -2

1/2 -1
Q 2. Continuous at x = 3 1
Q 3. 2x 1
Q 4. 10 1
√13

SECTION-B
Q 5. x 2
sin−1
a
Q 6. Proving correctly AB – BA is a skew symmetric matrix 2

Q 7. dy −b 1
i) for correct dx = cot t
a

ii) for correct d2y/dx2=(-b cosec3t)/a2 1

x x x x
Q 8. i) for putting 1=sin2 2 + cos2 2 ,sinx=2sin2cos2 1
1
ii) for writing tan-1(tan(π/2-x/2))
dy
iii) fordx = −1/2

Q 9. Writing correct formula 1


5√6 square unit 1
Q.10 4 1 4 1
∫0 |x − 1| = ∫0 −(x − 1)dx + ∫1 (x − 1)dx
−x2 x2 1
=[ + x]10 +[ − x]14
2 2

=5
Q.11 I=∫ √4 − (x + 1)2 dx 1
Put x+1=y; dy=dx
1
I=∫ √4 − y 2 dy
1 x+1
I=2(x+1)√3 − 2x − x 2 +2sin−1 ( )+c
2

Q.12 dy
i)for writing =9cm3/sec
dt 1
dx 3
ii)V=x3 and S=6x2 for calculating =
dt x2
ds 36 1
iii) for dt = x
ds
iv) for dt =3.6 cm2/sec
SECTION-C
Q.13 LHS =for applying C1→ C1 –C3,C2→C2 –C3 1
a−c b−c c
|a2 − c 2 b2 − c 2 c2 | 1
c−a c−b a+b

C1→ C1 +C2
0 1 c 2
(c-a) (b-c)|b − a b + c c2 |
0 −1 a + b
Expanding along c1=(a-b)(b-c)(c-a)(a+b+c)
Q.14 i. (tan-1 1/5 + tan-1 1/7)+(tan-11/3 +tan-11/8 ) 1
ii. tan-1 6/17+tan-151/23 1
iii. tan-1 325/325 1
iv. for correct answer π/4 1

Q.15 √y+6 −1 1
i. For correct x = g(y) = 3 1
ii. gof = IR+
iii. fog = I(-5,∞) 2
iv. f-1 = g
Q.16 dy
for getting, √1 − x 2 dx = −ay For finding the second derivative and 2
d2 y dy
2
getting,(1 − x 2 ) 2 − x − a2 y = 0
dx dx
Or
dy
For getting correct value of dx 2
2
dy 1−y2
For getting = √ 2
dx 1−x

Q.17 The rate of change of 2


(a) The rate of change of the perimeter =-2 cm /m
(b) The rate of change of the area of rectangle= 2 square cm /min 2

Q.18 . ⃗⃗⃗⃗⃗
a2 − ⃗⃗⃗⃗ a1 = î − k̂ 1
⃗⃗⃗⃗⃗
b1 Xb ⃗⃗⃗⃗⃗2 = 3î − ĵ − 7k̂ 1
⃗⃗⃗⃗⃗1 Xb
⃗⃗⃗⃗⃗2 | = √59 1
|b
10 1
Shortest distance between the line: √59

Q. 19 For getting form ex { f(x)+f‫(׳‬x))} 2


Result =ex tanx+c 2
or
2
x dt
for substituting e = t and getting ∫ √a2 2 2
−t
ex +2
for getting sin−1 ( )+c
3

Q. 20 2x2 +x
dy=(x+1)(x2 +1)dx
1
1 3x−1
∫ dy=∫( 2(x+1)+2(x2 +1) dx
1 3 1 1
Y=2log(x+1)+4log(x2+1)2tan-1x+c 2
1 3 1
Y=2log(x+1)+4log(x2+1)2tan-1x+1

Or 2
put y=vx 2
dy dv
=v+xdx
dx
Solving and ingtegrating
y y
Ans: cosecx - cotx.x=c
Q. 21.
For putting, y=vx 2
2
dy dx
For getting tanv = − x

y
For getting, xsin x = c
Q. 22. S={(b,b),(g,b),(b,g),(g,g)} 1
E= both the children are boys ,F= atleast one of the child is boy 1
3 1
P(F)=4 P(E ∩ F)=4
1 2
P(E/F)=2
OR
s={1,2,3,4,5,6} X is the random variable which can take value 1,2,3,4,5or 6 2
X 1 2 3 4 5 6
P(X) 1/6 1/6 1/6 1/6 1/6 1/6
21 91
For getting E(x)= 6 E(x2)= 6
35
For getting var(x)=12 2

Q. 23 (1) Let c⃗ =x̂i+ŷj+ẑk 2


x+y+z=3

⃗⃗⃗⃗⃗X
a c⃗ =(z-y)î+(z − x)ĵ+(y-x)k̂ 2

For values of x=5/3,y=2/3,z=2/3

OR
⃗⃗⃗⃗⃗1 +b
⃗⃗⃗⃗⃗2 2
Let a⃗⃗=b
⃗⃗⃗⃗⃗
b1 = (3î+k̂)
⃗⃗⃗⃗⃗
b2 =x̂i+ŷj+ẑk 2
raming equation 3x+z=0
=2 x=-1,y=-2,z=3

SECTION-D
Q. 24 i)2x-3y+5z=11, 3x+2y-4x=-5, x+y-2z=-3 1
ii)write in matrix form AX=B
0 1 −2 3
iii)for correct A-1 = [−2 9 −23]
−1 5 −13 2
iv)for calculation and solution x=1,y=2 and z=3
Q. 25 Let x m be the length of a side of the removed square. then l=8-2x,b=3-2x 1
and h-x,v(x)=4x3-22x2+24x
V ′ (x)=4(x-3)(3x-2) 2
V ′′ =24x-44
Critical point x=2/3 2
V ′′ (2/3)<0 1
500
Max.volume= 27 m3
Or
Let S(x) be the selling price of x items and let C(x) be the cost price of x items 1
x2 x
S(x)=5x-100, c(x)=5+500 2
24 x2 2
P(x)= 5 x − 100 − 500
24 x
P ′ (x)= x − 50 1
5
−1
P ′′ (x) =
50
Critical point x= -240
−1
P ′′ (240)= 50 < 0
Writing correct values
Q. 26 i)for drawing the correct curve line and circle 2
ii)for point of intersection of two curves (0,0)and (4,4),(4√2,0) 2
4 4√2 2
iii) for writing ∫0 xdx + ∫0 √32 − x 2 dx
iv)for evaluating the answer 4π square unit
Q. 27 Equation of the plane containing the given point is 1
A(x-1)+B(y-1)+C(z-2)=0 1
12
2A +3B-2C=0 1
A+2B-3C=0 12
5x-4y-z=7 2
OR
Writing general point on the line P(2K-1,-2K+3,-K) 1
Finding direction ratio of the given and general point 2K-2,-2K+1,-K+3 1
Finding the value of K as 1 1
Foot of perpendicular (1,1,-1) 1
And image (1,0,1) 2

Q.28 Let x be units of food and y units of F2 be mixed to get the desire diet
Min Z=4x+6y
Subject to the constraints
3x+6y≥80
4x+3y≥100 1
12
X,y≥0
For finding correct feasible region 1
12
Corner points: Value of Z=4x+6y
A(80/3,0) 2
1063
B(24,4/3) 104(min)
C(0,100/3) 200

The feasible region has no point common with 4x+6y<104 1


Min cost of diet =Rs104 1
For correct value answer 1

Q. 29 B1:bolt manufactured by machine A B2:Bolt manufactured by machine B


B3: bolt manufactured by machine C, E:bolt is defective (Assumptions) 1
i)for correct probabilities B1,B2 and B3 1
ii)for Correct conditional probabilities P(E/B1)=0.5, P(E/B2)=0.04 ,
P(E/ B3)=0.02 2
iii)use of Bayes theorem and getting correct solution(B2/E)=18/60 2
OR
Let A be the event that the machine produces 2 acceptable items also B1 2
represent the event of correct set up and B2 of incorrect set up, then P(B1)=0.8, 2
P(B2)=0.2 2
P (A|B1)=0.9X0.9, P(A|B2)=0.4X0.4
Applying Bay’s theorem
Getting P (B1|A) =0.95

GROUP-7
CLASS: XII

MATHEMATICS Max.Marks:100

Time Allowed: 3 hours

General Instructions:
1. All questions are compulsory.
2. This question paper consists of 29 questions divided into Four sections A, B, C andD. Section A comprises of
4 questions of one mark each, section B comprises of 8 questions of 02 marks each and section C comprises of
11 questions of 04 marks each and section D consists of 06 questions of 06 marks each.

3. All questions in Section A are to be answered in one word, one sentence or as per

the exact requirement of the question.

4. There is no overall choice. However, internal choice has been provided in 03

Questions of four marks each and 02 questions of six marks each. You have to

attempt only one of the alternatives in all such questions.

5. Use of calculators is not permitted. You may ask for logarithmic tables, if

required.

________________________________________________________________________

SECTION - A (1 mark questions)

1. Let * be a binary operation on Q given by a*b=a+ab where a,b  Q .Is * commutative?


1  4 
2. Evaluate Sin  cos 1   
2  5 
3. Given that A, B are two symmetric matrices such that AB =BA .Is AB symmetric?
 x  y  z   9
   
4. Find the values of x, y and z if  x  z    5 
 y  z   7
   
SECTION - B (2 mark questions)

Sin30 0 Cos30 0
5. Evaluate
 sin 60 0 Cos60 0
6. Find a point on the curve y=x2-4x+5 where the tangent to the curve is parallel to the x axis
x  cos 6 x
7. Evaluate  3x 2
 Sin6 x
dx
      
8. Find the projection of the vector a  i  3 k on the vector b  3 i  j  4 k
     
9. If a is a unit vector and ( x  a )( x  a )  15 find x

2x  1 y2 z3
10. The Cartesian equation of a line AB is   .Find the direction cosines of a line
3 2 3
parallel to A

11. Let f: R  R be defined by f(x) = 3x+2.Show that f is invertible. Also find f  1 :R  R


 1 
12. Write in the simplest form : tan  1   where x  1
 
 x2  1 
SECTION - C (4 marks questions)

x 6 1
13. Show that x = 2 is a root of the equation 2  3 x x  3  0 and solve it completely.
 3 2x x2
 1  Cos2 x x0

 x2
14. Discuss the continuity of the function at x = 0: f ( x)  


5 x0

 
15. Verify Rolle’s theorem for the function f(x)= Sin2x in 0, 
 2
dy 1  y2
16. If 1  x 2  1  y 2  a( x  y ) prove that 
dx 1  x2

OR

If y=x2 +4 and x changes from 2 to 2.1 find the approximate change in y

2
17. Evaluate  e x dx as a limit of sum
0
OR

5
Evaluate   x  1x 2
4 dx
dy
18. Solve the differential equation Cos x
2
 y  tan x
dx
19. Find the differential equation for the family of circles which passes through the origin and have their
centre on the x axis
       

20. If a  b  c  0 and a  3 ; b  5 c  7 then show that the angle between a and b is 600

21. Find the vector equation of the plane passing through the intersection of the planes r .
      
( 2 i  7 j  4 k )  3 and r . ( 3 i  5 j  4 k )  11  0 and passing through the point (-2,1,3)
22. A coin is tossed three times and all the possible outcomes are assumed to be equally likely. Let E and F
be two events given by E: “both tail and head have occurred” F: “at most one tail has occurred”. Show
that E and F are independent.

23. The sum of three numbers is 6.Twice the third number when added to the first number gives 7.On
adding the sum of the second and third numbers to thrice of the first number we get 12.Find the
numbers using inverse of a matrix.
Or
 1 2 3 

Using elementary transformations find the inverse of the matrix 2 5 7 

  2  4  5

SECTION - D (6 mark questions)

24. Let R be a relation on N  N defined by (a, b)R (c, d)  ad=bc, for all (a,b) and (c,d)  N . Show that R
is an equivalence relation .
OR

Let * be the binary operation defined on QxQ by (a,b)*(c,d) = (ac,b+ad) where Q is the set of
rational numbers. Determine whether * is commutative and associative . Find the indentiy
element for * and the inverttible elements of QxQ

25. An open box with a square base is to be made out of a given quantity of metal sheet of area c 2.Show
c3
that the maximum volume of the box is
6 3
1
26. Evaluate  Sinx(2  Cosx )dx
Or


2
Evaluate  log Sinxdx
0

27. Find the area of region included between the parabolas y2=4ax and x2=4ay where a>0
28. Find the foot of the perpendicular drawn from the point A(1,0,3) to the line joining the points B(4,7,1)
C(3,5,3)
29. Mona wants to invest at most Rs 12000 in Savings Certificates (SC) and National Saving Bond (NSB). She
has to invest at least Rs 2000 in SC and at least Rs 4000 in NSB. If the rate of interest on Sc is 8% and the
rate of interest on NSB is 10% per annum, how much money should she invest to earn maximum yearly
income?

******

ANSWER KEY WITH MARKING SCHEME

Q.No Value points Marks Total


Marks

1 Not Commutative (as a*b=a+ab  b+ba=b*a) 1M 1M

2 1 4 4  1M 1M
(put cos  1 =   cos  = Given expression =Sin =
10 5 5 2
4
1
1  Cos 5= 1
=
2 2 10

3 AB is Symmetric. (AB) t =B t A t =BA=AB 1M 1M

4 x=2 ; y= 4 ; z= 3 1M 1M

( by equating corresponding entries we get x+y+z = 9 ; x+z =5 ; y+z


= 7)

5 0
Sin30 Cos 60 +Cos30 0 Sin60 0
0 1M 2M

=Sin 900=1 IM

6 dy 1M 2M
(2,1) is the point .  2 x  4 =0  x  2 putting x =2 in
dx
y=x2-4x+5 we get y = 1

7 1 1 1M 2M
6 t
Put 3 x  Sin6 x = u  Given integral =
2
dt =

1
log 3 x 2  Sin6 x  c
6

8     1M 2M
projection of the vector a  i  3 k on the vector b 
 
   a .b
3 i  j 4 k =

b

3  0  12 15
= =
9  1  16 26
1M

9    1M 2M
x 2
-a 2
=15  x 2


=15+1  x = 4 1M
10 1 1M 2M
x
2  y  2  z  3 dr’s of AB are 3 ,4,6
3 4 6

3 4 6
Hence dc’s of AB are , ,
55 55 55
1M

11 f ( x1 )  f ( x2 )  3 x1  2  3 x 2  2  x 1 =x 2 Thus f is one 1M
to one

y2 y2 1M
y=3x+2  x= Given any y  R there exists =x  R
3 3
s.t. f(x)=y showing that the function is onto.
2
x2
f  1 (x)= is the inverse of f
3

1
12 x=Cosec    =Cosec x

 1   1  1M
tan  1   = tan 1  
   
 x2  1   Co sec   1 
2

2
  1M
 = tan 1 
1 
 = tan Tan = 
1
 1 1
= tan
    
 Cot   Cot
2

1
=Cosec x

13 x  2 3x  6  x  2 1M
5  5x  5  0 by performing R1  R1  R2 and
3 2x x2
R2  R2  R3

1 3 1 1M
5(x-2) 1  x 1  0
 3 2x x2

0 3 1 1M
5(x-2) 0 x  1  0 C1  C1  C 3 4
x  1 2x x2

0 3 1 ½M
5(x-2)(x-1) 0  x 1  0
1 2x x2
= -5(x-2)(x-1)(x+3) = 0  x  2,1,3 ½M

14 2 Sin 2 x
Here f(0) = 5 and Lt f(x) = Lt =2
x 0 x 0 x2

Sinx Sinx
2 Lt Lt =2  1 1 =2
x 0 x x  0 x

As Lt f(x)  = f(0) ,f is not continuous


x 0

15   ½M
Consider the function f(x) = Sin2x in 0,  . Here f is
 2
 
continuous in 0,  as f(x)= Sin x is continuous
 2

  1M
f ' (x) = 2 Cos2x exists in ( 0, ) thus f is differentiable on ( 0, )
2 2

  1M
f(0)=Sin (0) = 0 and f   =Sin  = 0
 2

Conditions for Rolle’s thm are satisfied. Hence there should be at ½M


 
least one c   0,  such that f (c ) = 0
'
4
 2

   1M
Let 2Cos2c = 0  c =   0, 
4  2

16 Let x=Sin A and y=Sin B ½M

Cos A + Cos B =a(Sin A –Sin B) ½M

A B A B A B A B 1M
2Cos Cos =a  2Cos Sin
2 2 2 2 4

A B 1M
Cot =a  Sin-1x-Sin-1y=2Cot-1a
2

1 1 ½M
Differentiating , - y ' =0
1  x2 1  y2

½M
dy 1  y2

dx 1  x2

OR

x= 2 x  0.1 ½M

y  f ' ( x )x =2x x =2  2 0.1 =0.4 1½M


When x=2 y = 22+4=8 ½M 4

y+  y =8+0.4=8.4 1M

y changes from 8 to 8.4 ½M

17. ba 2 ½M
a=0 b=2 h=  nh=2 f(x)=ex
n n
2 1M
 e dx =
x

Lt h  f (0)  f (0  h)  f (0  2h)  ......  f (0  (n  1)h)


h 0


= Lt h 1  e h  e 2 h  e 3 h  .....  e
h 0
 n 1 h
 ½M

Lt h.

1. e h  1 
n
 1M
4
h 0 eh  1

= Lt
h

e 2  1 =e 2 -1  1M
h 0 e 1
h

Or

5 A Bx  C 1M
 2
 =

 x  1 x  4 x  1 x  4
2

To get A =1 B = -1 and C = 1 1M

5 1  x 1 ½M
  x  1 x 2  4 dx =  dx  
  x1 x2  4
dx

x 1 1M
=log x  1 - x 2
4
dx   2
x 4
dx 4

1 1 x ½M
=log x  1 - log x 2  4  tan 1  c
2 2 2

18 dy 1M
Dividing Cos x
2
 y  tan x by Cos2x to get
dx
dy
 ySec 2 x  tan xSec 2 x
dx

I.F. =etanx ½M
4
Solution is y(IF)=  tan xSec 2 x etanxdx+ k 1M

 Put tan x =u  y etanx =  ue u du  k ½M


 y etanx =e u ( u  1)  k ½M

 y =tanx-1+ke  tan x ½M

19 1M

Let a be the radius Then centre is


(a,0)

= n to the Circle is (x-a) 2 +y 2 =a 2  x 2  y 2  2ax  0 ….(*) 1M

Differentiating ,2x+2yy’-2a=0  a=x+yy ' =a 1M 4

Putting this value of a in (*) we get 2xyy ' -y  x =0 1M


2 2

20   ½M
Any vector parallel to b is of the form k b for some scalar k

     1M
Let a = k b + c where c is perpendicular to b

    ½M
Then c =( a - k b )  b

1M
      
 ( a - k b ). b  ( a . b )-k( b . b )=0  k=2 4
    ½M
k b =2 b = 6 i  2 k

      ½M
c =( a - 2 b )=(  i  2 j  k )

OR

      ½M
a b c  0  a b   c
   ½M
 ( a  b ) 2  ( c ) 2

      ½M
 ( a  b ).( a  b )  (  c ).(  c )

     1M
2 2 2
a +b +2 a . b = c

  15 ½M
a .b =
2
  1M
a .b 15 1
Cos  = =    60
0
=
 
2 3 5 2
a b
4

21     1½M
r . { ( 2  3 ) i  ( 7  5 ) j  (4  4 ) k }  3  11  0 …(*)

4
   1½M
(*) Passes through the point with position vector ( 2 i  j  3 k )
1
 
6
 1M
Putting the value of  in (*) we get r .
  
(15 i  47 j  28 k )  7  0

22. E: “both tail and head have occurred” ½M+½M


={HTT,THT,TTH,HHT,HTH,THH}

F: “at most one tail has occurred”= {HHH,HHT,THH,HTH}

E  F ={HHT,HTH,THH} ½M

6 4 3 1½M
P(E) = P(F)= P( E  F )= 4
8 8 8

3 6 4 1M
As P( E  F )=P(E) P(F)    is true ,we conclude that E
8 8 8
and F are independent.

23 Let the first, second and third numbers be x, y and z respectively. 1M


Then x + y + z=6 : x + 2z =7 ; 3x + y + z=12

 1 1 1  x 6 ½M
     
 AX=B where A=  1 0 2  X=  y  B=  7 
 3 1 1 z  12 
     

A  4  0  A is invertible ½M

 2 0 2  1M
  4
adjA=  5  2  1 
 1 2  1 

 2 0 2   6   3 1M
1     
X=A  1 B=  5  2  1  7  =  1 
A  1
 2  1   12   2 

The required numbers are 3 ,1 and 2 ½M

OR

 1 2 3  ½M

Let A= 2 5 7 

  2  4  5

 1 2 3  1 0 0 1M

A=IA  2 5 7  = 0 1 0 A

  2  4  5 0 0 1 6

1 0 0
 
By performing elementary transformations ,to get 0 1 0 =
 
0 0 1
 3  2  1
  4 1  1 A 3½M
 
 2 0 1 

 3  2  1 1M
 A =   4 1  1
1

 2 0 1 

24

(a,b) R (a,b)  ab=ba Reflexive 1.5M

(a,b) R (c,d)  ad=bc  cb=da  (c,d) R (a,b) symmetric 1.5M 6

(a,b) R (c,d) and (c,d) R (e,f)  ad=bc and cf=de  ad.cf=bc.de 2½ M


 af=be  (a,b) R (e,f) Transitive

Thus R is an equivalence relation ½M

25 Let x be the side of the square base and y be the height. 1½M

c2  x2
V=x2y ,Surface area S= x2+4xy=c2  y  4 x

1
V= (c x  x )
4
2 3 dV 1 2
dx 4

 c  3x2  d 2V  3
dx 2

2
x
1½M
dV c d 2V   3c 1M
0 x  2 
 0
dx 3 dx  x  c 2 3
3

c c 1M
V is Maximum when x  . Then y 
3 2 3
6
c3 1M
V Max =
6 3

26 Sinx ½M
 Sin x( 2  Cosx )
2
dx by multiplying the Nr and Dr by Sinx

Sinx 1 1M
=  (1  Cos 2 x)(2  Cosx)dx =  (1  t 2 )( 2  t )dx by letting
Cosx=t

1 ½M
=  (1  t )(1  t )( 2  t )
dx

1 A B C 1M
=  
(1  t )(1  t )( 2  t ) 1  t 1  t 2  t

1 1 1 1½M
To get A= B= C=
6 2 3 6

1 1M
 (1  t )(1  t )( 2  t )
dx =

1 1 1
log 1  t  log 1  t  log 2  t  k
6 2 3

1
 sin x(2  Cosx )dx =

1 1 1
 log 1  Cosx  log 1  Cosx  log 2  Cosx  c
6 2 3
½M

OR

   1M
2 2   2
I=  log Sinxdx =  log Sin  x dx =  log Cosxdx
0 0 2  0
  1M
2 2 2 SinxCosx
2I=  log SinxCosxdx =  log dx
0 0 2

 2M
2 2 SinxCosx
=  log dx =
0 2
 6
2  
 log Sin2 xdx  log 2  I 1  log 2 ………(*)
0 2 2

 1M
1 1 2
I1 =  log S int dt =  2  log Sinxdx =I
2 0 2 0

  1M
(*)  2I=I- log 2  I= log 2
2 2

27 2M

Solving the two equations given we get x=0 and x=4a 1M

4a 4a x 2 2M
Required area =  4ax dx   dx
0 0 4a 6

32a 2 16a 2 16a 2 1M


=   sq.units
3 3 3

28 Let P be the foot of the perpendicular from A on BC. If P divides BC ½M


 3k  4 5k  7 3k  1 
in the ratio k:1 then P is  , , 
 k 1 k 1 k 1 
1M

Dr’s of BC are 1,2,-2 ½M

3k  4 5k  7 3k  1 1M
Dr’s of AP are  1,  0, 3
k 1 k 1 k 1
7 2M
Since AP  BC ,dot product =0  k 
4

 5 7 17  1M
So the foot of the perpendicular is  , , 
3 3 3 
6
29 Suppose that she invests Rs x in SC and Rs y in NSB ½M

8 x 10 y 1½M
Then LPP is to maximize Z=  subjecting to constraints x
100 100
 2000 y  4000 x+y  12000

2½M

(To
draw the graph, to identify the feasible region and to get
coordinates)

8 x 10 y 1M
To put the coordinates of the vertices in Z=  to get 6
100 100
values Rs 560, Rs 1040 Rs 1160

Rs 2000 should be invested in in savings certificates and Rs 10000 ½M


in National Savings Bonds to get a maximum yearly income of Rs
1160

BLUE PRINT
SAMPLE PAPER
Sub:- Mathematics
CLASS-XII
SL. NAME OF THE KNOWLEDGE UNDERSTANDING APPLICATION TOTA
NO CHAPTERS L
VSA SA LA-I LA-II VSA SA LA-I LA-II VSA SA LA-I LA-II

1 RELATIONS 1(4) 1(6) 2(10)


AND
FUNCTIONS
INVERSE
TRIGONOMETR
C FUNCTIONS
2 MATRICES & 1(1) 1(2) 1(4) 1(6) 4(13)
DETERMINANTS
3 CONTINUITY 1(1) 1(1) 1(2) 3(4)
AND
DIFFERENTIABI
LITY
4 APPLICATION 2(4) 1(4) 1(6) 4(14)
OF
DERIVATIVES
5 INTEGRALS 1(1) 2(8) 1(2) 1(4) 5(15)

6 APPLICATION 1(6) 1(6) 2(12)


OFINTEGRALS
7 VECTOR 1(4) 1(4)
ALGEBRA

8 THREE 1(2) 1(4) 1(6) 3(12)


DIMENSIONAL
GEOMETRY
9 LINEAR 1(2) 1(4) 2(6)
PROGRAMMING
PROBLEMS
10 PROBABILITIES 1(2) 2(8) 3(10)

11 Total 2(2) 2(2) 7(14) 9(36) 5(30) 1(2) 2(8) 1(6) 29(100)

BLUE PRINT Sub:- Mathematics

2016-2017

CLASS-XII

1. Question Paper Design:

S.NO. Typology of VSA SA LA-I LA-II Marks Weightage


Question (1Mark) (2 Marks) (4Marks) (6Mark)
1. Remembering 2 2 2 1 20 20%
2. Understanding 1 3 4 2 35 35%
3. Application 1 - 3 2 25 25%
4. HOTS - 3 1 - 10 10%
5. Evaluation - - 1(VBQ) ∗ 1 10 10%
Total 1x 4= 4 2 x 8 = 16 4 x 11=44 6 x6=36 100 100%
∗ Value based question of 04 marks.
2. Question –wise break-up:
Type of Marks per Total no. of Total Marks
Question question questions
VSA 1 4 4
SA 2 8 16
LA-I 4 11 44
LA-II 6 6 36
Total - 29 100

3. Difficulty level :

Difficulty level Weightage


Easy 20%
Average 60%
Difficult 20%

4. Choice(s) :
There will be no overall choice in the question paper. However, 30% internal choice will be given in 4
marks and 6 marks questions
KENDRIYA VIDYALAYA SANGATHAN
SAMPLE QUESTION PAPER
AISSCE- 2017
SUBJECT: MATHEMATICS Class- XII
TIME : 3 HRS. Max. Marks :100
General Instructions:

1. All questions are compulsory


2. This question paper consists of 29 questions divided into Four sections A, B C
and D. Section A comprises of 4 questions of one mark each, section B
comprises of 8 questions of two marks each, section C comprises of 11 questions
of four marks each and section D comprises of 6 questions of six marks each
3. All questions in Section A are to be answered in one word, one sentence or as
per the exact requirement of the question
4. There is no overall choice. However, internal choice has been provided in
04 questions of four marks each and 02 questions of six marks each. You
have to attempt only one of the alternatives in all such questions.
5. Use of calculators is not permitted. You may ask for logarithmic tables, if
required
SECTION ---A MARKS
1 If | 𝐴 | = 2, where A is a 2 × 2 matrix, find | 𝑎𝑑𝑗 𝐴 | 1

2 Show that the function f defined as follows is continuous at 𝑥 = 2. 1


3𝑥 − 2 , 0<𝑥≤1
f (𝑥) = {2𝑥 2 − 𝑥 , 1<𝑥≤2
5𝑥 − 4 , 𝑥>2
0 1
3 Evaluate ∫−6|𝑥 + 3|𝑑𝑥 .
4−𝑥 3−𝑦 𝑧+2 1
4 Equation of a line is given by = = . Write the direction cosines of a parallel to
2 −3 6
given line.
SECTION ---B
5 1 2 3 −7 −8 −9 2
If X[ ]=[ ] , then find the matrix X .
4 5 6 2 4 6
6 Verify Rolle’s theorem for the function: 𝑓(𝑥) = 𝑥 2 + 2, 𝑥 ∈ [−2, 2] 2

3
7 Find the approximate value of √26 using derivative . 2
8 Show that the function given by f(x)=𝑒 2𝑥 is strictly increasing on R 2
9 Find the shortest distance between the lines +1 = 2𝑦 = −12 z , 𝑥 = (y + 2)/2 = 6z − 6 2
.
10 A problem in Mathematics is given to 4 students A, B, C, D. Their chances of solving the 2
1 1 1 2
problem, respectively, are 3 . 4 , 5, and3. What is the probability that the problem will
be solved?
11 Solve the following linear programming problem graphically: 2
Maximize Z = 3x+2y
Subject to x + y ≤ 5 , 𝑥, 𝑦 ≥ 0
𝑑𝑦 2
12 Find the integrating factor of the following differential equation : 𝑥 log 𝑥 𝑑𝑥 + 𝑦 =
2 log 𝑥 .
SECTION – C
1−𝑥 1 2
13 Solve for x: 𝑡𝑎𝑛−1 (1+𝑥) = 2 𝑡𝑎𝑛−1 𝑥.
14 If 𝑥, 𝑦, 𝑧 are in GP, then using properties of determinants, show that 4
𝑝𝑥 + 𝑦 𝑥 𝑦
|𝑝𝑦 + 𝑧 𝑦 𝑧 | = 0, where𝑥 ≠ 𝑦 ≠ 𝑧 and 𝑝 is any real numbers.
0 𝑝𝑥 + 𝑦 𝑝𝑦 + 𝑧
OR
Using properties of determinant prove that:
−bc b2 + bc c 2 + bc
2
| 𝑎 + 𝑎c −𝑎c c 2 + 𝑎c| = (𝑎b + bc + c𝑎)3 .
𝑎2 + 𝑎b b2 + 𝑎b −𝑎b
15 Form the differential equation representing the family of curves given by (𝑥 − 𝑎)2 + 4
2𝑦 2 = 𝑎2 , where a is an arbitrary constant.

OR
𝑑𝑦
Show that the differential equation 2𝑥𝑦 = 𝑥 2 + 3𝑦 2 is homogeneous and solve it.
𝑑𝑥

3 4
16 Evaluate: ∫1 (2𝑥 2 + 3𝑥) 𝑑𝑥 using limit of sum.
17 Evaluate:∫(√cot 𝑥 + √tan 𝑥) dx 4

OR
3
2
∫ |𝑥. cos(𝜋𝑥)| 𝑑𝑥 4
0

18 𝑑2 𝑦 𝑑𝑦 1 4
If 𝑦 = √𝑥 + 1 − √𝑥 − 1, prove that (𝑥 2 − 1) 𝑑𝑥 2 + 𝑥 −4 𝑦 =0.
𝑑𝑥

19 Find a unit vector perpendicular to the plane of the triangle ABC, where the coordinates 4
of its vertices are A (3, −1,2), B (1, −1, −3) and C (4, −3,1).
20 Find the distance of the point (1, −2, 3) from the plane 𝑥 − 𝑦 + 𝑧 = 5 measured parallel 4
to the line whose direction cosines are proportional to 2, 3, −6.

21 Two numbers are selected at random (without replacement) from the first six positive 4
integers. Let X denotes the larger of the two numbers obtained. Find the probability
distribution of the random variable X, and hence find the mean of the distribution .
22 Suppose a boy throws a die. If he gets a 1 or 2, he tosses a coin three times and notes 4
down the number of heads. If he gets 3, 4, 5 or 6 he tosses the coin once and notes down
whether a head or a tail is obtained. If he obtains exactly one head, what is the probability
that he obtained 3, 4, 5 or 6 with the die?

23 A dealer in rural area wishes to purchase some sewing machines. He has only Rs.57,600 4
to invest and has space for at most 20 items. An electronic machine costs him Rs.3,600
and a manually operated machine costs Rs.2,400.He can sell an electronic machine at a
profit of Rs.220 and a manually operated machine at a profit of Rs.180. Assuming that he
can sell all the machines that he buys, how should he invest the money in order to
maximize his profit? Make it as a LPP and solve it graphically .
SECTION –D
24 1 −1 0 6
If A = [2 5 3], find A-1, using elementary row operations.
0 2 1

OR
2 −2 −4 6
Express the matrix A = [−1 3 4 ] as the sum of a symmetric and a skew
1 −2 −3
symmetric matrix.

25 Let 𝑓: [0, ∞) → 𝑅 𝑏𝑒 𝑎 𝑓𝑢𝑛𝑐𝑡𝑖𝑜𝑛 𝑑𝑒𝑓𝑖𝑛𝑒𝑑 𝑏𝑦 𝑓(𝑥) = 9𝑥 2 + 6𝑥 − 5. prove that f is 6


invertible. Modify, only the codomain of f invertible and then find its inverse.
OR
Let 𝑅 be a relation defined on 𝑄𝑥𝑄 𝑏𝑦 (𝑎, 𝑏)𝑅(𝑐, 𝑑) = (𝑎 + 𝑑, 𝑏 + 𝑐), where Q is set of
rational numbers. Determine R is equivalence relation or not.

26 A wire of length 28 cm is to be cut into two pieces. One of the two pieces is to be made 6
into a square and other into a circle .What should be the lengths of two pieces so that
the combined area of the circle and the square is minimum .

27 Using integration, find the area of the region bounded between the parabola 4𝑦 = 3𝑥 2 6
and the line: 2𝑦 = 3𝑥 + 12.
28 𝑑2 𝑦 𝑑𝑦 6
If 𝑦 = 𝑠𝑖𝑛−1 𝑥, 𝑠ℎ𝑜𝑤 𝑡ℎ𝑎𝑡 (1 − 𝑥 2 ) 𝑑𝑥 2 − 𝑥 𝑑𝑥 = 0.

29 Find the equation of the plane through the intersection of the planes 6
𝑥 + 3𝑦 + 6 = 0 𝑎𝑛𝑑 3𝑥 − 𝑦 − 4𝑧 = 0 and whose perpendicular distance from origin is
unity.
Or

Find the distance of the point (1, – 2, 3) from the plane x – y + z = 5 measured 6
parallel to the line whose direction cosines are proportional to 2, 3, – 6.
.
KENDRIYA VIDYALAYA SANGATHAN
SAMPLE QUESTION PAPER

MARKING SCHEME

SUBJECT: MATHEMATICS Class- XII


TIME : 3 HRS. Max. Marks :100

Q1 |𝐴𝑑𝑗 𝐴| = |𝐴|1 ½

|𝐴𝑑𝑗 𝐴| = 2 ½

Q2 L.H.L : lim− 2𝑥 2 − 𝑥 = lim− 2(2 − ℎ)2 − (2 − ℎ) = 6 1


𝑋→2 𝑋→2
R.H.D : lim+ 5𝑥 − 4 = lim+5(2 + ℎ) − 4 = 6
𝑋→2 𝑋→2 1
F(2) = 6
F(x) is continuous AT X= 2 as L.H.L = R.H.L =F(2)

Q3 −3 0
∫−6 (−𝑥 − 3) 𝑑𝑥 + ∫−3(𝑥 + 3)𝑑𝑥
= 9 sq unit 1
Q4 𝑥−4 𝑦−3 𝑧+2
= =
−2 3 6
Dr’s of line parallel to given line : -2 , 3 , 6 ½
−2 3 6
Dc’s of line parallel to given line : 17 , 17 , 17 1
Q5 𝑎 𝑏 ½
Let X = [ ]
𝑐 𝑑
𝑎 + 4𝑏 2𝑎 + 5𝑏 3𝑎 + 6𝑏 −7 −8 −9
 [ ]=[ ] ½
𝑐 + 4𝑑 2𝑐 + 5𝑑 3𝑐 + 6𝑑 2 4 6
 Solving for a , b , c ,d
1
 a=1 , b = -2 , c = 2 , d = 0
Q6 (1) Continuity ½
(2) Differentiality ½
(3) f(a)=f(b) ½
find c=0 ½

3
Q7 Let f(x) = √𝑥 , x = 27 , x + ∆𝑥 = 26 ½
∆𝑥 = −1 ½
𝑑𝑦
∆𝑦~ × ∆𝑥 ½
𝑑𝑥
−1
∆𝑦~
27
3
√26 = y + ∆𝑦 = 2.62(𝑎𝑝𝑝) ½

Q8 F’(x)=2𝑒 2𝑥 ½

=2(+ve) when x> 0 ½

1 ½
=2(+ve) when x< 0
½
=2 when x=0
Q9 𝑥+1 𝑦 𝑧 𝑥 𝑦+2 𝑧−1 ½
𝐿1 : = = , 𝐿2 = = =
1 1/2 −1/12 1 2 1/6
1 1 1
𝑎1 = −𝑖̂, ⃗⃗⃗⃗⃗
⃗⃗⃗⃗⃗ 𝑎2 = −2𝑗̂ + 𝑘 ̂ , ⃗⃗⃗⃗
𝑏1 = 𝑖̂ + 2 𝑗̂ − 12 𝑘̂ , ⃗⃗⃗⃗⃗
𝑏2 = 𝑖̂ + 2𝑗̂ + 6 𝑘̂
1 1 3
½
𝑎1 = 𝑖̂ − 2𝑗̂ + 𝑘̂ , ⃗⃗⃗⃗⃗
𝑎2 − ⃗⃗⃗⃗⃗
⃗⃗⃗⃗⃗ 𝑏2 × ⃗⃗⃗⃗𝑏1 = 𝑖̂ − 𝑗̂ + 𝑘̂ 4 4 2

|𝑏 ̂2 | = √38
⃗⃗⃗⃗1 × 𝑏 ½
4
(𝑎
⃗⃗⃗⃗⃗⃗−𝑎 ⃗⃗⃗⃗⃗⃗⃗×𝑏
⃗⃗⃗⃗⃗⃗).(𝑏 ⃗⃗⃗⃗⃗)
Distance (d) = 2 |𝑏⃗⃗⃗⃗⃗1 ×𝑏⃗⃗⃗⃗⃗2| 1
1 2
9 ½
=
√38
Q10 P(A) = 1/3 , P(B) = ¼ , P(C) = 1/5 , P(D) = 2/3 ½

P(problem is solved ) = 1 – P(problem is not solved)


= 1 – P(𝐴̅)P(𝐵̅)P(𝐶̅ )P(𝐷
̅) 1½
2
= 1 - 15
13
=15
Q11 Graph ½
A(0 , 5) B(5,0) are the x intercept and y-intercept of the graph
Value of Z at A = 10 1
Value of Z at B = 15
Max z = 15 ½
Q12 𝑑𝑦 𝑦 2 ½
+ =
𝑑𝑥 𝑥 𝑙𝑜𝑔𝑥 𝑥
1 ½
P(x) = 𝑥 𝑙𝑜𝑔𝑥
1
∫𝑥 𝑙𝑜𝑔𝑥 𝑑𝑥 1
 Integrating factor = 𝑒 = 𝑒 log(log 𝑥) = log x

Q13 2(1 − 𝑥 2) 1
=𝑥
4𝑥
1 1 1
𝑥= ,−
√3 √3
Q14 𝑥, 𝑦, 𝑧 𝑎𝑟𝑒 𝑖𝑛 𝐺. 𝑃 => 𝑦 2 = 𝑥𝑧 -------------(1) ½
𝑝𝑥 + 𝑦 𝑥 𝑦 𝑐1 →𝑐1 −(𝑝𝑐2 +𝐶3 )
|𝑝𝑦 + 𝑧 𝑦 𝑧 |→
0 𝑝𝑥 + 𝑦 𝑝𝑦 + 𝑧 1
0 𝑥 𝑦
| 0 𝑦 𝑧 | ½
2
−(𝑝 𝑥 + 2𝑝𝑦 + 𝑧) 𝑝𝑥 + 𝑦 𝑝𝑦 + 𝑧
𝑥 𝑦
= −(𝑝2 𝑥 + 2𝑝𝑦 + 𝑧) |𝑦 𝑧 |
= −(𝑝2 𝑥 + 2𝑝𝑦 + 𝑧)(𝑥𝑧 − 𝑦 2 ) = 0 (from eq (1))

(OR)

−𝑏𝑐 𝑏 2 + 𝑏𝑐 𝑐 2 + 𝑏𝑐
| 𝑎2 + 𝑎𝑐 −𝑎𝑐 𝑐 2 + 𝑎𝑐| 1
𝑎2 + 𝑏𝑎 𝑏 2 + 𝑏𝑎 −𝑏𝑎
𝑎𝑅1 +𝑏𝑅2 +𝑐𝑅3 1 −𝑎𝑏𝑐 𝑎𝑏 2 + 𝑎𝑏𝑐 𝑎𝑐 2 + 𝑎𝑏𝑐 1
→ |𝑏𝑎2 + 𝑎𝑏𝑐 −𝑎𝑏𝑐 𝑏𝑐 2 + 𝑎𝑏𝑐 |
𝑎𝑏𝑐
𝑐𝑎2 + 𝑐𝑏𝑎 𝑐𝑏 2 + 𝑏𝑐𝑎 −𝑐𝑏𝑎

1/2
−𝑏𝑐 𝑎𝑏 + 𝑎𝑐 𝑎𝑐 + 𝑎𝑏
|𝑎𝑏 + 𝑏𝑐 −𝑎𝑐 𝑏𝑐 + 𝑎𝑏 |
𝑐𝑎 + 𝑏𝑐 𝑏𝑐 + 𝑐𝑎 −𝑏𝑎
𝑅1 →𝑅1 +𝑅2 +𝑅3 𝑎𝑏 + 𝑏𝑐 + 𝑐𝑎 𝑎𝑏 + 𝑏𝑐 + 𝑐𝑎 𝑎𝑏 + 𝑏𝑐 + 𝑐𝑎 ½
→ | 𝑎𝑏 + 𝑏𝑐 −𝑎𝑐 𝑏𝑐 + 𝑎𝑏 |
𝑐𝑎 + 𝑏𝑐 𝑏𝑐 + 𝑐𝑎 −𝑏𝑎

1 1 1 𝐶1 →𝐶1 −𝐶2 & 𝐶2 →𝐶2 −𝐶3


=( 𝑎𝑏 + 𝑏𝑐 + 𝑐𝑎) |𝑎𝑏 + 𝑏𝑐 −𝑎𝑐 𝑏𝑐 + 𝑎𝑏| →
1
𝑐𝑎 + 𝑏𝑐 𝑏𝑐 + 𝑐𝑎 −𝑏𝑎
0 0 1
=( 𝑎𝑏 + 𝑏𝑐 + 𝑐𝑎) |𝑎𝑏 + 𝑏𝑐 + 𝑎𝑐 −(𝑎𝑐 + 𝑎𝑏 + 𝑏𝑐) 𝑏𝑐 + 𝑎𝑏|
0 𝑏𝑐 + 𝑐𝑎 + 𝑎𝑏 −𝑏𝑎

𝑎𝑏 + 𝑏𝑐 + 𝑐𝑎 −(𝑎𝑏 + 𝑏𝑐 + 𝑐𝑎)
=( 𝑎𝑏 + 𝑏𝑐 + 𝑐𝑎) | |
0 𝑎𝑏 + 𝑏𝑐 + 𝑐𝑎
3 1 −1
=(𝑎𝑏 + 𝑏𝑐 + 𝑐𝑎) | |
0 1
3
=(𝑎𝑏 + 𝑏𝑐 + 𝑐𝑎)
Q15 (𝑥 − 𝑎)2 + 2𝑦 2 = 𝑎2
𝑦2 11/2
Or, x + 2 𝑥 = 2a
𝑑 𝑦2 𝑑
Or, 𝑑𝑥 (x + 2 𝑥 ) = 𝑑𝑥 (2𝑎)
Or, 𝑥 2 + 4𝑦𝑥𝑦 , − 2𝑦 2 = 0 2½
OR
𝑑𝑦 𝑥 2 +3𝑦 2
= = f(x,y)
𝑑𝑥 2𝑥𝑦
1
𝑘 2 𝑥 2 +3𝑘 2 𝑦2 𝑥 2 +3𝑦 2
F(kx,ky) = = = 𝑓(𝑥, 𝑦)
2𝑘 2 𝑥𝑦 2𝑥𝑦
Differential equ. Is homogeneous.
Let y= vx ½
𝑑𝑣 1+3𝑣 2
 V+x𝑑𝑥 = 2𝑣
2𝑣 𝑑𝑥
 ∫ 1+𝑣2 𝑑𝑣 = ∫ 𝑥 2½
 Log|1 + 𝑣 2 | = 𝑙𝑜𝑔𝑥 + 𝑙𝑜𝑔𝑐
 1 + 𝑣 2 = 𝑥𝑐
 𝑥2 + 𝑦2 = 𝑥3𝑐
Q16 a=1 , b= 3 , f(x)= 2𝑥 2 + 3𝑥
𝑏−𝑎 2 1
n= 2 =ℎ
10 7 2
f(a+h)+f(a+2h)+f(a+3h)+…+f(a+nh) = + ℎ (2 + ℎ) + 3 (2 + ℎ)(4 + ℎ) 1

3 10 7 2
∫1 (2𝑥 2 + 3𝑥) 𝑑𝑥 = lim ℎ[ ℎ + ℎ (2 + ℎ) + 3 (2 + ℎ)(4 + ℎ)]
ℎ→0 2
88
= 3
Q17 I = ∫ √cot 𝑥 + √tan 𝑥 dx
1+tan 𝑥 1
=∫ 𝑑𝑥
√tan 𝑥

1+𝑡 2
=2 ∫ 1+𝑡 4 𝑑𝑡 ( let tan x = 𝑡 2 & 𝑠𝑒𝑐 2 𝑥 𝑑𝑥) 1
1
1+ 2
𝑡
= 2∫ 1 2
𝑑𝑡 1
(𝑡− ) +2
𝑡
𝑑𝑢
= 2∫ 𝑢2 +2
tan 𝑥−1 1
= √2 tan−1 tan 𝑥 + 𝑐
√2
OR
3/2 1/2 3/2 1½
𝐼= ∫0 |𝑥 cos 𝜋𝑥| = ∫0 𝑥 cos 𝜋𝑥 𝑑𝑥 + ∫1/2 −𝑥 cos 𝜋𝑥 𝑑𝑥
1 1
Now ∫ 𝑥 𝑐𝑜𝑠𝜋𝑥 𝑑𝑥 = 𝜋 sin 𝜋𝑥 + 𝜋2 cos 𝜋𝑥 + 𝑐 1½
5 1 1
∴ I = 2𝜋 − 𝜋2
Q18 𝑑𝑦 1 1 1
= −
𝑑𝑥 2√𝑥 + 1 2√𝑥 − 1
2𝑦 ′ √𝑥 2 − 1 = √𝑥 + 1 − √𝑥 − 1
𝑥 1 1 1
2𝑦 ′′ √𝑥 2 − 1 + 2𝑦 ′ √𝑥 2 = 2√𝑥−1 − 2√𝑥+1
−1
′′ ′ 𝑥 √𝑥+1−√𝑥−1 1
2𝑦 √𝑥 2 − 1 + 2𝑦 √𝑥 2 −1
= 2
𝑑2𝑦 𝑑𝑦 1 1
(𝑥 2 − 1) 2 +𝑥 − 𝑦=0
𝑑𝑥 𝑑𝑥 4

Q19 ⃗⃗⃗⃗⃗⃗
𝐴𝐵 = −2𝐼̂ − 5𝐽̂ , ⃗⃗⃗⃗⃗⃗ 𝐴𝐶 = 𝑖̂ − 2𝑗̂ − 𝑘̂ 1
⃗⃗⃗⃗⃗⃗
𝐴𝐵 × 𝐴𝐶 ⃗⃗⃗⃗⃗⃗ = −10𝑖̂ − 7𝑗̂ + 4𝑘̂ 1
⃗⃗⃗⃗⃗⃗ × 𝐴𝐶
⃗⃗⃗⃗⃗⃗ | = √165 1
|𝐴𝐵
∴ 𝑅𝑒𝑞𝑢𝑖𝑟𝑒𝑑 𝑢𝑛𝑖𝑡 𝑣𝑒𝑐𝑡𝑜𝑟 𝑝𝑒𝑟𝑝𝑒𝑛𝑑𝑖𝑐𝑢𝑙𝑎𝑟 𝑡𝑜 𝑡ℎ𝑒 𝑝𝑙𝑎𝑛𝑒 𝑜𝑓 𝑡𝑟𝑖𝑎𝑛𝑔𝑙𝑒 𝐴𝐵𝐶 1
−10 −7 4
= 𝑖̂ + 𝑗̂ + 𝑘̂
√165 √165 √165
Q20 P: 𝑥 − 𝑦 + 𝑧 = 5
A (1 ,-2 ,3)
AM is distance to the Line L parallel to the line having dc’s 2 , 3 , -6
𝑥−1 𝑦+2 𝑧−3 1 /2
Equation of AM is 1 = 3 = −6 .
𝑥−1 𝑦+2 𝑧−3
Taking is = = =𝑘 1/2
2 3 −6
∴ 𝑀 = (2𝑘 + 1,3𝑘 − 2, −6𝑘 + 3)
M lies on the plane P .
2K+1-3k+2-6k+3=5
1 1 1/2
K= 7
9 −11 15 1/2
So, M = (7 , 7 , 7 ) 1
Req . distance AM = 1unit

Q21 Random variable X : larger no. of the two nos.


Range of X ={2,3,4,5,6} 1/2
P(X=2)=1/15 , P(X=3) = 2/15 ,P(X=4) = 3/15 ,P(X=5) = 4/15 ,P(X=6)=5/16 2
Probability distribution is:
X=w 2 3 4 5 6 ½
P(X=w) 1/15 2/15 3/15 4/15 5/15

E(X) = 14/3 1
Q22 E1 = He obtained 3 , 4 , 5 or 6
E2 = He obtained 1 or 2 1
A = He obtains exactly one head .
P(E1) = 4/6 , P(E2) = 2/6 , P(A/E1) = ½ , P(A/E2) = 3/8 1 1/2
P(A) = P(E1). P(A/E1) + P(E2) . P(A/E2) = 11/24 1
𝐴
𝑃(𝐸1).𝑃( )
𝐸1
P(E1/A) = = 8/11. 1/2
𝑃(𝐴)
Q23 Let x number of electronic machine & y number of manually operated machine ½
were purchased .
L.P.P is Max Z = 220x + 180y
Subject to the constraints 1
X + y ≤ 20 , 3𝑥 + 2𝑦 ≤ 48
Such that x,y≥ 0
Changing inequalities to equalities we get X + y = 20 , 3𝑥 + 2𝑦 = 48

X 0 2
0
y 2 0 1
0

X 0 1
6
y 2 0 ½
4
For correct graph with correct feasible region
Corner point table : at A(16 , 0) Z = 3520
At B(0 , 20 ) Z = 3600
At C(8 , 12) Z = 3920

Max Z = Rs. 3920 for x = 8 & y = 12


Q24 A = IA ½
1 −1 0 1 0 0
=> (2 5 3) = (0 1 0) 𝐴
0 2 1 0 0 1
1 −1 0 1 0 0 1½
 (0 7 3) = (−2 1 0) 𝐴 ( 𝑅2 → 𝑅2 − 2𝑅1)
0 2 1 0 0 1
1 −1 0 1 0 0
 (0 1 0) = (−2 1 −3) 𝐴 (𝑅2 → 𝑅2 − 3𝑅3) 1½
0 2 1 0 0 1
1 0 0 −1 1 −3
 (0 1 0) = (−2 1 −3) 𝐴 1½
0 0 1 4 −2 7
−1 1 −3 1
 𝐴−1 = (−2 1 −3)
4 −2 7

OR 1
2 −2 −4 2 −1 1
𝑇
A = (−1 3 4 ) , 𝐴 = (−2 3 −2)
1 −2 −3 −4 4 −3
2
4 −3 −3 0 −1 −5
𝑇 𝑇
A + 𝐴 = (−3 6 2 ) , A −𝐴 = (1 0 6)
−3 2 −6 5 −6 0
2 −3/2 −3/2 ½
1 𝑇
LET P=2 (𝐴 + 𝐴 ) = ( −3/2 3 1 )
−3/2 −1 −3/2
0 −1/2 −5/2 ½
1 1
Q = 2 (𝐴 − 𝐴𝑇 ) = (1/2 0 3 )
5/2 −3 0
1
P is symmetric , Q is skew symmetric
P + Q = A (SHOW)
Q25 Conversion f into completing square 1
Proof that f is invertible 1

One-one, onto 2
−1 1
𝑓 −1 = [−1 + √𝑥 + 6 ]
3
OR
2
Reflexive
2
Symmetric
2
Transitive
Q26 Let xcm is used for a square and 28-x is used for a circle . ½
𝑥 28−𝑥
Side of the square = 4 , radius of the circle = 2𝜋
𝑥2 (28−𝑥)2
½
Area A = 16 + 4𝜋2 ½
𝑥 28 − 𝑥 1
𝐴′ = −
8 2𝜋
′′
1 1 1
𝐴 = +
8 2𝜋
112 ½
Critical points = 𝑥 = 𝜋+4
112 ½
At 𝑥 = 𝜋+4 𝐴′′ > 0 ½
28𝜋
28-x = 𝜋+4
112 28𝜋 1
Length of two pieces are 𝜋+4 & .
𝜋+4
Q27
y-axis

X=-2 x=4
2
A
D x-asix
C o

Area (AOBA) = 𝑎𝑟 (𝐴𝐶𝐷𝐵𝐴) − ar(ACO) − ar(ODBO) 1


1 4 1 3𝑥 2 4 1
Now 𝑎𝑟 (𝐴𝐶𝐷𝐵𝐴) = ∫ (3 𝑥 + 12)d𝑥 = 2 [ + 12𝑥] −2 = 45
2 −2 2
[𝑥 3 ] 2
3 4 1
and ar(ACO) + ar(ODBO) = 4 ∫−2 𝑥 2 𝑑𝑥 = = 18 1
4 −1
Therefore Area (AOBA) = 45 − 18 = 27 sq units Ans: 27 sq units
1
Q28 𝑑𝑦 1 2
=
𝑑𝑥 √1 − 𝑥 2
𝑑𝑦 2 2
𝑜𝑟 (1 − 𝑥 2 ) ( ) = 1
𝑑𝑥
𝑑2𝑦 𝑑𝑦
𝑜𝑟 (1 − 𝑥 2 ) 2
−𝑥 =0 2
𝑑𝑥 𝑑𝑥
Q29 𝑒𝑞𝑢𝑎𝑡𝑖𝑜𝑛 𝑜𝑓 𝑝𝑙𝑎𝑛𝑒 𝑖𝑠 𝑥 + 3𝑦 + 6 + 𝜇(3𝑥 − 𝑦 − 4𝑧) = 0 1½
𝑝𝑒𝑟𝑝𝑒𝑛𝑑𝑖𝑐𝑢𝑙𝑎𝑟 𝑑𝑖𝑠𝑡𝑎𝑛𝑐𝑒 𝑡𝑜 𝑡ℎ𝑒 𝑝𝑙𝑎𝑛𝑒 𝑓𝑟𝑜𝑚 𝑜𝑟𝑖𝑔𝑖𝑛 𝑖𝑠 = 1
𝑓𝑖𝑛𝑑 𝑑𝑖𝑠𝑡𝑎𝑛𝑐𝑒 𝑓𝑟𝑜𝑚 𝑜𝑟𝑔𝑖𝑛 𝑡𝑜 𝑝𝑙𝑎𝑛𝑒 𝑎𝑛𝑑 𝑝𝑢𝑡 𝑖𝑡 𝑒𝑞𝑢𝑎𝑙 𝑡𝑜 1 1½
𝑤𝑒 𝑓𝑖𝑛𝑑 𝜇 = ±1 1
𝑟𝑒𝑞𝑢𝑟𝑖𝑒𝑑 𝑒𝑞𝑢𝑎𝑡𝑖𝑜𝑛𝑠 𝑜𝑓 𝑝𝑙𝑎𝑛𝑒𝑠 𝑎𝑟𝑒 2𝑥 + 𝑦 − 2𝑧 + 3 = 0 1
𝑎𝑛𝑑 𝑥 − 2𝑦 − 2𝑧 − 3 = 0 1

OR
x−1 y+2 z−3 1m
Any line through (1,-2,3) with D.r’s as 2,3,-6 is 2 = 3 = −6
x−1 y+2 z−3
= = = α(say) ⇒ x=2 α + 1, y=3 α − 2, z = −6α + 3
2 3 −6
1m
This point lie on the plane x-y+z=5
1m
∴2 α + 1 − 3α + 2 − 6α + 3 = 5
1
⇒ α=
7 1m
9 11 15
Required point is (7 , − 7
, 7 ).

9 2 11 2 15 2 1m
Required distance =√(7 − 1) + (− + 2) + ( 7 − 3)
7
1
=7 √4 + 9 + 36

=1
1m

MATHEMATICS
CLASS - XII
Blue Print of Question Paper

Topic/Unit Section A Section B Section C Section D Total


Relation & - - 4(1) 6(1) 10(2)
Functions
Algebra 1(1) 2(1) 4(1) 6(1) 13(4)
Calculus 2(2) 8(4) 16(4) 18(3) 44(13)
Vectors & Three 1(1) 2(1) 8(2) 6(1) 17(5)
Dimensional
Geometry
Linear - 2(1) 4(1) - 6(2)
Programming
Probability - 2(1) 8(2) - 10(3)
Total 4(4) 16(8) 44(11) 36(6) 100(29)

Note: Number of questions are given within brackets and marks outside the brackets
MATHEMATICS
CLASS XII
Time Allowed ; 3 Hours Max. Marks : 100

General Instructions:
(1) All the questions are compulsory.
(2) This question paper contains 29 questions.
(3) Questions 1-4 in Section A are very short answer type questions carrying 1 mark each.
(4) Questions 5-12 in Section B are short answer type questions carrying 2 marks each.
(5) Questions 13-23 in Section C are long answer –I type questions carrying 4 marks each.
(6) Questions 24-29 in Section D are long answer –II type questions carrying 6 marks each.

SECTION A
Questions from 1 to 4 are of 1 mark each.
(𝑥+3)2 − 36
1. Determine the value of k for which the following function is continuous at x = 3 : f(x) ={ , x≠ 3
𝑥−3
= k , x=3
2. A and B are square matrices of order 3 each, |𝐴|=2 and |𝐵|=3. Find|3𝐴𝐵|.
5−𝑥 𝑦+4 6−𝑧
3. Write the vector equation of the line −3 = 7 = 2 .
𝑠𝑖𝑛2 𝑥−𝑐𝑜𝑠2 𝑥 𝑑𝑥
4. Find : ∫ 𝑠𝑖𝑛𝑥−cos 𝑥

SECTION B
Questions from 5 to 12 are of 2 marks each.
5. Find the intervals in which the function f given by f(x) = 2x3 – 3x2 -36x + 7 is strictly increasing.
7𝑦 5 −21 5
6. If [ ]= [ ], then find values of x and y.
2𝑥 − 3𝑦 −3 11 −3
𝑑𝑦
7. If 𝑒 𝑦 (x+1) = 1, Show that 𝑑𝑥 =−𝑒 𝑦
8. Find the sum of the order and the degree of the following differential equation.
𝑑2 𝑦 3 𝑑𝑦
+ √𝑑𝑥 + (1+x) =0
𝑑𝑥 2
9. Find the coordinates of the image of the point (1,3,4) in the plane 2x- y – 3 =0
10. Two tailors, A and B, earn Rs. 300 and Rs. 400 per day respectively. A can stitch 6 shirts and 4 pairs of
trousers while B can stitch 10 shirts and 4 pairs of trousers per day. To find how many days should each of
them work and if it desired to produce at least 60 shirts and 32 pairs of trousers at a minimum labour cost,
formulate tis as an LPP.
11. A couple has 2 children. Find the probability that both are boys, if it is known that (1) one of them is a boy
(2) the older child is a boy.
12. An edge of a variable cube is increasing at the rate of 5 cm/s. How fast is the volume increasing ,when the
side is 15 cm?
SECTION C
Questions from 13 to 23 are of 4 marks each.
𝑎2 𝑏𝑐 𝑎𝑐 + 𝑐 2
13. Using properties of determinants prove the following: |𝑎 + 𝑎𝑏
2
𝑏 2
𝑎𝑐 2 | = 4a2b2c2
𝑎𝑏 𝑏 2 + 𝑏𝑐 𝑐2
OR
𝑥 𝑥2 1 + 𝑥2
If x, y and z are different and ∆ =|𝑦 𝑦 2 1 + 𝑦 2 |= 0, then show that 1 +xyz =0
𝑧 𝑧2 1 + 𝑧2
14. Verify Rolle’s theorem for the function f(x) = x2 + 2x – 8, x ∈ [-4, 2}
OR
Find the point on the curve y = x3-11x + 5 at which the tangent is y = x-11.
8
15. Solve for x : tan-1(x +1) + tan-1(x -1) = tan-1(31)
2
16. Evaluate ∫0 (𝑥 2 + 3) dx as a limit of a sum.
17. An experiment succeeds twice as often as it fails. Find the probability that in the next six trials, there will be
at least 4 successes
𝑦 𝑦
18. Solve the differential equation : (x dy – y dx) y sin(𝑥 ) = (y dx + x dy) x cos (𝑥 ).
OR
Find the particular solution of the differential equation: (1 + 𝑒 2𝑥 ) dy + (1 + y2) ex dx =0 given that y = 1 when
x =0.
19. If 𝑎⃗, 𝑏⃗⃗ and 𝑐⃗ are the position vectors of the vertices A, B and C of triangle ABC, show that the area of ∆ABC
1
is 2 | 𝑎⃗ 𝑥 𝑏⃗⃗ + 𝑏⃗⃗ 𝑥 𝑐⃗ + 𝑐⃗ 𝑥 𝑎⃗|.
𝑥−8
20. Find the vector equation of line passing through the point (1, 2, -4) and perpendicular to the two lines: =
3
𝑦+19 𝑧−10 𝑥−15 𝑦−29 𝑧−5
= 𝑎𝑛𝑑 = = .
−16 7 3 8 −5
21. Differentiate the following equation with respect to x : y =𝑥 𝑠𝑖𝑛𝑥 + (𝑠𝑖𝑛 𝑥)𝑐𝑜𝑠𝑥
22. One kind of cake required 200 gm flour and 25 gm of fat and another kind of cake requires 100 gm of flour
and 50 gm of fat. Find the maximum number of cakes which can be made from 5 kg of flour and 1 kg of fat
assuming that there is no shortage of other ingredients using in making the cakes? Formulate this problem
as an LPP and solve it graphically.
23. A card from a pack of 52 cards is lost. From the remaining cards of the pack, two cards are drawn and are
found to be both diamonds. Find the probability of the lost card being a diamond.
SECTION D
Questions from 24 to 29 are of 6 marks each.
𝜋
3 𝑠𝑖𝑛𝑥+cos 𝑥
24. Evaluate : ∫ 𝜋 dx
√𝑠𝑖𝑛2𝑥
6
OR
√𝑥 2 +1
Find ∫ [ log(𝑥 2 +1) -2logx ] dx
𝑥4
√𝑦+6 −1
25. Consider f:R+ →[-5, ∞) given by f(x) = 9x2 + 6x – 5. Prove that f is invertible with f-1(y) = ( ).
3
OR
A binary operation * is defined on the set X = R – {-1} by x * y = x + y + x y, ∀ 𝑥, 𝑦 ∈ 𝑋. Check whether * is
commutative and associative. Find its identity element and also find the inverse of each element of X.
2𝑅
26. Show that the height of the cylinder of maximum volume that can be inscribed in a sphere of radius R is .
√3
2 −3 5
27. If A = [3 2 −4], find A-1. Using A-1 solve the system of equations : 2x – 3y + 5z = 11
1 1 −2
3x + 2y -4z = -5, x + y -2z = -3
28. Find the area of the region bounded by the curves y = x2 + 2 , y = x, x=0 and x =3.
29. Find the equation of the plane which contains the line of intersection of the planes 𝑟⃗.(𝑖̂ + 𝑗̂ + 𝑘̂) = 1 and
̂ - 𝑘̂) + 4 =0 and parallel to X-axis.
𝑟⃗.(2𝑖̂ + 3𝑗

QUESTION PAPER : MATHEMATICS


CLASS – XII
MARKING SCHEME (Suggestive)
QUESTION EXPECTED ANSWERS VALUE POINTS
NUMBER SECTION A
1 LHL = RHL =f(3) 1/2
lim 𝑓(𝑥) = lim 𝑓(𝑥) =k 1/2
𝑥→3− 𝑥→3+
K = 12
2 Given order of matrices A and B =3 and |𝐴| =2 and |𝐵| =3
|3𝐴𝐵| = 33 |𝐴||𝐵| 1/2
= 27 x 2 x 3 = 162 1/2
3 𝑥−5 𝑦−(−4) 𝑧−6
Equation of line (Cartesian form) 3 = 7 = 6 1/2
1/2
Vector form 𝑟⃗ = (5𝑖̂ - 4𝑗̂ + 6𝑘̂) + 𝛿(3𝑖̂ + 7𝑗̂ - 6𝑘̂)
4 𝑠𝑖𝑛2 𝑥 − 𝑐𝑜𝑠 2 𝑥 𝑑𝑥
∫ ½
𝑠𝑖𝑛𝑥 − cos 𝑥
=∫(𝑠𝑖𝑛𝑥 𝑑𝑥 –∫ cos 𝑥 𝑑𝑥 ½
= - cos x+ sin x + C

SECTION B
5 F(x) is a polynomial function. Hence it is continuous in R
F’(x) = 6 (x-3) (x+2)
F’(x) > 0 (1)
(x-3)(x+2) > 0
X < -2 or X > 3 (1)
F is strictly increasing 0n (-∞, 2) ∪ (3, ∞)
6 By comparing elements y = -3 (1)
2x- 3y= 11
Or x = 2 (1)
7 Given ey (x + 1) =1
𝑑𝑦
On differentiating both side w. r .t. x, we get ey + (x + 1 )ey 𝑑𝑥
(1)
=0 1
𝑑𝑦 (2)
Or ey +1 𝑑𝑥 =0 1
𝑑𝑦 (2)
Or 𝑑𝑥 =-1
8 On cubing both side, we get (1)
𝑑2 𝑦 𝑑𝑦
{𝑑𝑥 2 + (1 + x)}3 = - 𝑑𝑥
(1)
Hence , order is 2 & degree is 3. So sum is 5
9 let 𝑥1 , 𝑦1 𝑎𝑛𝑑 𝑧1 be the image of (1,3,5)
1+𝑥1 3+𝑦1 5+ 𝑧1 (1)
so coordinate of mid point is ( , , )
2 2 2
But this point lies on the plane 2x-y-3 =0 which is (1)
satisfied by the mid point.
So coordinate of image = (-3, 5,2)
10 For objection function (1)
For inequations (1)
11 Let B1,B2 denote boys andG1,G2 denote girls, then sample 1
(2)
space = {B1B2, B1G2, G1B2,G1G2}
(1)
Again , let E1= both the children are boys , E2= one of the 1
children is a boy ( )
2
E3 =the older child is a boy (1)
E1={ B1B2} E2 ={ B1G2, G1B2,G1G2 } E3 ={ B1B2, G1B2}
E1 1 E1 1
P( E2) =3 P(E3) = 2
12 Let x be the edge of cube and v be the volume of cube at any
time t.
𝑑𝑥
Given 𝑑𝑡 =5 cm/s and x =15 cm (1)
V =x3
𝑑𝑣 𝑑𝑥
=3x3 𝑑𝑡
𝑑𝑡
𝑑𝑣
=3 x (15)2 x 5 = 3375 cm3/s (1)
𝑑𝑡
SECTION C
13 1 1 1
Apply c1→ 𝑎 c1 , c2→ 𝑏 c2 and c3 → 𝑐 c3 (1)
C1→ c1 + c2 - c3 (1)
R2 → R2 – R1 (1)
Expend along C1 and get result (1)
OR
Using Property 5 (change determinants in two determinants)
C3↔C2 and then C1↔C2 (1)
R2 → R2 – R1 and R3 → R3 – R1 (1)
Take common factor (y-x) and (z-x) from R3 and along C1 (1)
For final proof
(1)
14 Step 1 f(x) is continuous in [-4, 2]. (1/2)
Step 2 f(x) is differential in (-4, 2). (1/2)
Step 3 f(-4) = f(2) so all conditions satisfied. (1)
Stepn4 f’(c) = 0 (1)
2c + 2 =0
C= -1 ∈ [-4, 2] hence Rolle’s theorem is verified with (1)
c = -1
OR

y = x3-11x + 5 ..........(1)
y = x-11…………….(2) (2)
𝑑𝑦
slope of (2) = 1 and from (1) 𝑑𝑥 = 3x2 – 11
(1)
3x2 – 11= 1
Or x = ± 2
Y = { -9, 19}
(1)
So points are (2,-9) and (-2, 19) where slope of tangent is 1
Equation of tangent at (2, -9) is y = x -11

15 8
tan-1(x +1) + tan-1(x -1) = tan-1(31)
2𝑥 8
after applying formula we get tan-1(2−𝑥 2 ) = tan-1(31) (2)
1
x =4 and x = -8 but x = -8 is not possible
1
hence x = 4 (2)

16 F(x) = x2 + 3, a = 0 , b = 2 and nh = 2 (1)


2
∫0 (𝑥 2 + 3) dx = lim ℎ[𝑓(𝑎) + 𝑓(𝑎 + ℎ) + 𝑓(𝑎 + 2ℎ) + (1)
ℎ→0 (1)
⋯ + 𝑓{𝑎 + (𝑛 − 1)ℎ}
= lim ℎ[𝑓(0) + 𝑓(ℎ) + 𝑓(2ℎ) + ⋯ + 𝑓{(𝑛 − 1)ℎ} and (1)
ℎ→0
solving
(𝑛−1)𝑛(2𝑛−1)
= lim ℎ[3𝑛ℎ + ℎ3 { }
ℎ→0 6
16 26
=6+ =
6 3

17 Let probability of success = 2x


X + 2x = 1
X=3
1 (1)
1 2
If p = 3 then q = 3 (1)
Let X be be the random variables that represents the number of
success in six trials.
By binomial distribution, we obtain P(X= x) = 𝑛𝑥𝐶 𝑝𝑛−𝑥 𝑞 𝑥 (1)
Probability of at least 4 successes = P(X≥ 4) = 𝑃(𝑋 = 4) + (1)
𝑃(𝑋 = 5 ) + 𝑃(𝑋 = 6) By putting values and after
31 2 4
calculation we get result= ( )
9 3
𝑦 𝑦
18 (x dy – y dx) y sin(𝑥 ) = (y dx + x dy) x cos (𝑥 )
𝑦 𝑦 𝑦2 𝑦
𝑑𝑦 cos ( ) + ( ) 𝑠𝑖𝑛 (𝑥 )
𝑥 𝑥 𝑥2
= 𝑦 𝑦 𝑦
𝑑𝑥
𝑥 𝑠𝑖𝑛 (𝑥 ) − 𝑐𝑜𝑠 (𝑥 ) (1)
Put y = vx
𝑑𝑦 𝑑𝑣
= 𝑣 + 𝑥 𝑑𝑥 after solving (1)
𝑑𝑥
𝑣𝑠𝑖𝑛𝑣−𝑐𝑜𝑠𝑣 1
Therefore ∫ dv = 2 ∫ 𝑥dx (1)
𝑣𝑐𝑜𝑠𝑣
1 1
∫ 𝑡𝑎𝑛𝑣𝑑𝑣 - ∫ 𝑣dv =2 ∫ 𝑥dx
𝑠𝑒𝑐𝑣 𝑦 (1)
Applying formula we get = ± C and replacing v by 𝑥
𝑣𝑥 2
𝑦
Sec(𝑥 ) = C xy which is the general solution .
OR

(1)
Integrating both sides, we get:
(1)

Substituting these values in equation (1), we get:

(1)
Now, y = 1 at x = 0.
Therefore, equation (2) becomes

Substituting in equation (2), we get:

(1)
19 Let the position vector of A, B and C be 𝑎⃗ ,𝑏⃗⃗, 𝑐⃗ respectively
1
Area of ∆𝐴𝐵𝐶 = 2 |𝐴𝐵 ⃗⃗⃗⃗⃗⃗ 𝑥 ⃗⃗⃗⃗⃗⃗
𝐴𝐶 |
1
(1)
⃗⃗⃗⃗⃗⃗ − ⃗⃗⃗⃗⃗⃗
= |(𝑂𝐵 𝑂𝐴)𝑥( 𝑂𝐶 ⃗⃗⃗⃗⃗⃗ − ⃗⃗⃗⃗⃗⃗
𝑂𝐴)|
2
1
= |(𝑏⃗⃗ − 𝑎⃗)𝑥( 𝑐⃗ − 𝑎⃗)| (1)
2
1
= 2 |𝑏⃗⃗𝑥𝑐⃗ − 𝑏⃗⃗𝑥𝑎⃗ − 𝑎⃗𝑥𝑐⃗ + 𝑎⃗𝑥𝑎⃗| (1)
1
= |𝑏⃗⃗𝑥𝑐⃗ + 𝑎⃗𝑥𝑏⃗⃗ + 𝑐⃗𝑥𝑎⃗|
2
1
=2 | 𝑎⃗ 𝑥 𝑏⃗⃗ + 𝑏⃗⃗ 𝑥 𝑐⃗ + 𝑐⃗ 𝑥 𝑎⃗| (1)
20 Any line through (1,2,-4) can be written as
𝑥−1
= (1)
𝑎
𝑦−2 𝑧+4
= 𝑐 ……….(1) (1)
𝑏
(1) is at right angles to the given lines with d.n. <3, -
16 ,7 > and <3, 8 ,-5>
if 3a -16b +7c =0 ……….(2)
3a +8b -5c =0 …………(3) (1)
𝑎 𝑏 𝑐
By cross multiplication, we get 24 = 36 = 72
𝑎 𝑏 𝑐
Or 2 = 3 = ……….(4) (1)
6
𝑥−1 𝑦−2 𝑧+4
From (1) and (4) we find line = =
2 3 6
Vector form 𝑟⃗ = (𝑖̂ + 2𝑗̂ − 4𝑘̂) + 𝛿 (2𝑖̂ + 3𝑗̂ + 6𝑘̂)
21

(1)

Differentiating both sides with respect to x, we obtain

(1)

Differentiating both sides with respect to x, we obtain

(1)

From (1), (2), and (3), we obtain


(1)
22 Let there be x cakes of first kind and y cakes of second
kind. Therefore,
x ≥ 0 and y ≥ 0
The given information can be complied in a table as
follows (1)
Flour (g) Fat (g)
Cakes of first kind, x 200 25
Cakes of second kind, y 100 50
Availability 5000 1000
(1)

Total numbers of cakes, Z, that can be made are, Z = x


+y
The mathematical formulation of the given problem is
Maximize Z = x + y … (1)
subject to the constraints,

The feasible region determined by the system of (1)


constraints is as follows.

The corner points are A (25, 0), B (20, 10), O (0, 0), and
C (0, 20).
The values of Z at these corner points are as follows.
Corner point Z=x+y
A(25, 0) 25 (1)
B(20, 10) 30 → Maximum
C(0, 20) 20
O(0, 0) 0
Thus, the maximum numbers of cakes that can be made
are 30 (20 of one kind and 10 of the other kind).

23 Let E1 lost card is a diamond , E2 : lost card is not a diamond


E1 and E2 are mutually exclusive and exhaustive.
13 1 39 3
P(E1) = 52 = 4 and P(E2) = 52 = 4 (1)
Let E : two cards drawn from the remaining pack are diamonds
13 13
𝐸 𝐶 𝐸 𝐶
Then P( 𝐸 ) = 512𝐶 and P( 𝐸 ) = 512𝐶 (1)
1 2 2 2
𝐸
𝐸1 P( P(E1)
𝐸1
Required probability P( 𝐸 ) = 𝐸 𝐸 (1)
P( )P(E1)+P( )P(E2)
𝐸1 𝐸2
11
Put above values get probability = 50 (1)

24

(1)

(1)

(1)

(1)

(1)

(1)

OR
(1)

(2)

(1)

(2)

25 f:R+ →[-5, ∞) let y = f(x) = 9x2 + 6x – 5


√𝑦+6−1 (2)
after solving x = { }
3
√𝑦+6−1
g(y) = { 3 } (1)
gof(x) =x and fog(y) = y
√𝑦+6−1
(2)
hence f is invertible and is defined by f-1(y) ={ } (1)
3
OR
for proof of commutative (1.5)
for proof of associative (1.5)
for identity element e= 0 (1.5)
𝑥
for inverse = - 1+𝑥 (1.5)

26 A sphere of fixed radius (R) is given.


Let r and h be the radius and the height of the cylinder
respectively

(1)

From the given figure, we have (1)


The volume (V) of the cylinder is given by,

(1)

(1)

(1)

(1)
Now, it can be observed that at so
volume is maximum.
2𝑟 2
The volume is the maximum when r2 = then height of
3
2𝑅
cylinder is
√3

27

(1)
(2)
Now, the given system of equations can be written in the
form of AX = B, where

(1)

(2)

28
(2)

The area bounded by


the curves, y = x2 + 2, y = x, x = 0, and x = 3, is
represented by
the shaded area OCBAO as
Then, Area OCBAO = Area ODBAO – Area ODCO (2)

(1)

(1)

29 Given planes are 𝑟⃗.(𝑖̂ + 𝑗̂ + 𝑘̂) = 1 and 𝑟⃗.(2𝑖̂ + 3𝑗 ̂ - 𝑘̂) + 4


=0 (1)
i.e. x + y +z -1 =0 and 2x + 3y – z +4 =0 (1)
any plane through the line of intersection of planes (. x + y +z -
1) + k(2x + 3y – z +4) =0
or x(1 +2k) +y(1 +3k) +z(1-k) = 0 ……………..(1) (2)
d. c. of plane (1) < 1+2k, 1+3k, 1-k > and d.c. of X –axis are <1,
0, 0>
this plane is parallel to X –axis if (1 +2k) .1 +(1 +3k).0 +(1-k) .0
=0
k = -2
1 (2)
put the value of k in (1)
now plane (1) is y- 3z +6 =6
S.N. TOPIC DELIVERED BY

1 AIMS & OBJECTIVES OF INSERVICE SHRI SUNEEL KUMAR(COURSE DIRECTOR)


COURSE
2 RELATIONS & FUNCTIONS SH. YOGENDRA BAHADUR (RESOURCE PERSON)

3 EFFECTIVE COMMUNICATION SKILLS SH. D.K VERMA (EXTERNAL GUEST SPEAKER)

4 AEP SMT. USHA PAL,TGT(ENGLISH) KV SITAPUR

5 ACTION RESEARCH MS. NEHASHREE SRIVASTAV (EXTERNAL GUEST SPEAKER)

6 LIMITIS AND CONTINUITY SH. YOGENDRA BAHADUR (RESOURCE PERSON)

7 UNIFORM SYSTEM OF ASSESSMENT SH. SATISH KUMAR (ASSOCIATE COURSE DIRECTOR)

8 INVERSE TRIGONOMETRIC FUNCTIONS SH. S.N. TRIPATHI, (RESOURCE PERSON)

9 PERMUTATIONS & COMBINATIONS SH. SATISH KUMAR (ASSOCIATE COURSE DIRECTOR)

10 PLOTTING OF ALGEBRAIC CURVES AND SH. S.N. TRIPATHI, (RESOURCE PERSON)


GRAPHICAL TRANSFORMS
11 DERIVATIVES SH. YOGENDRA BAHADUR (RESOURCE PERSON)

12 YOGA/ WELLNESS PROGRAMME/STRESS SH. S.N.TRIPATHI,STC&YOGA TEACHER, KV


MANAGEMENT CANTT,LUCKNOW
13 CONSTRUCTIVISM/MULTIPLE MS. NEHASHREE SRIVASTAV (EXTERNAL GUEST SPEAKER)
INTELLIGENCE
14 APPLICATION OF INTEGRAL SH. YOGENDRA BAHADUR (RESOURCE PERSON)

15 LEADERSHIP AND PERSONALITY SH. SATISH KUMAR (ASSOCIATE COURSE DIRECTOR)


DEVELOPMENT
16 FIRST AID & EMERGENCY DR. RAJEEV SRIVASTAVA, VMC MEMBER, KV RDSO,
LUCKNOW
17 CONDUCT OF SUBJECT COMMITTEE SH. SATISH KUMAR (ASSOCIATE COURSE DIRECTOR)
MEETING
18 VECTORS & 3-D GEOMETRY SH. SATISH KUMAR (ASSOCIATE COURSE DIRECTOR)

19 DIFFRENTIAL EQUATION SH. YOGENDRA BAHADUR (RESOURCE PERSON)

20 SOP FOR TERRORIST ATTACK/DISASTER SH. RAVI SHANKAR MISHRA,HOD(DEPT OF CIVIL


MANAGEMENT ENG.),AIMT,LUCKNOW
21 HUMAN RIGHTS AND CHILD RIGH SH. RAVI SHANKAR MISHRA,HOD(DEPT OF CIVIL
ENG.),AIMT,LUCKNOW
22 3 D GEOMATERY SH. SATISH KUMAR (ASSOCIATE COURSE DIRECTOR)

23 CODE OF CONDUCT INCLUDING ARTICLE SHRI SUNEEL KUMAR(COURSE DIRECTOR)


81(b)
24 PROBABILITY SH. S.N. TRIPATHI, (RESOURCE PERSON)

25 APPLICATION OF DERIVATIVES SH. S.N. TRIPATHI, (RESOURCE PERSON)


KV RDSO LUCKNOW
Pre Test
In- Service Course for PGT(Maths)- 2017
Time – 3 hours MM- 100
General Instructions-
(i) All questions are compulsory.
(ii) The question paper consists of 29 questions divided into three sections A,B, C
and D. Section A contains 4 questions of 1 mark each, Section B contains 8
questions of 2 marks each ,section C contains 11 questions of 4 marks each and
section D contains 6 questions of 6 marks each.
Section – A
1. Write the name of Chairman of KVS.
2. Write full form of SGFI.
3. When KVS foundation day is celebrated.
4. Find the domain of the function y = f(x) given by the equation 10x + 10y = 10.
Section – B
1.5
5. Evaluate ∫0 [𝑥]𝑑𝑥, where [x] is greatest integer function.
6. Evaluate sin-1 (sin10).
7. What is the total number of events if a coin is tossed two times ?
8. Write two matrices such that their product is a null matrix but neither of them is
the null matrix.
9. Name the number system used in computer programming.
10. Describe RTE 2009 briefly.
11. Prove that function ‘f’ is invertible iff ‘f’ is one –one and onto.
𝑑𝑦 𝑥+2𝑦+3
12. Solve: = .
𝑑𝑥 2𝑥+3𝑦+4
Section – C

13. If (xr ,yr), where r= 1,2,3 be the coordinates of the points A,B,C respectively , then
prove that the equation of the median through A is
𝑥 𝑦 1 𝑥 𝑦 1
|𝑥1 𝑦1 1| + |𝑥1 𝑦1 1| = 0.
𝑥2 𝑦2 1 𝑥3 𝑦3 1
𝑥 2 −3𝑥−3
14. Find the maximum and minimum value of the expression , x∈ 𝑅.
2𝑥 2 +2𝑥+1
200 2
15. What will be the remainder , when x + 1 is divided by x +1.
16. Find the radius of the smallest circle which touches the straight line 3x – y =6 at (1,
-3) and also touches the line y = x.
17. Find the condition that the equation ax2 + 2hxy + by2 + 2gx + 2fy + c = 0 will
represent two straight lines.
18. What is constructivism? Describe briefly.
19. Prove that every differentiable function is continuous, but its converse need not be
true.
20. A man takes a step forward with probability 0.4 and backward with probability
0.6. Find the probability that at the end of eleven steps he is one step away from
the starting point.
0 0
21. If A=[ ] and f(x) = 1+ x + x2 + …..+ x16 , find the value of f(A).
0 5
𝑥2 𝑦2
22. Find the area of the curve + = 1using integration.
225 121

1
23. Evaluate ∫ dx .
(𝑥 2 −1)√𝑥 2 +1

Section – D

tan−1 𝑥−sin−1 𝑥
24. Evaluate: lim .
𝑥→0 𝑠𝑖𝑛3 𝑥
25. Using elementary column operation , find inverse of the matrix
2 −3 3
A = [2 2 3]
3 −2 2
26. Find the shortest distance and equation of line of shortest distance between the
lines given by 𝑟⃗ = (3𝑖̂ + 8𝑗̂ + 3𝑘̂) + 𝜆(3𝑖̂ − 𝑗̂ + 𝑘̂) and 𝑟⃗ = (−3𝑖̂ − 7𝑗̂ + 6𝑘̂) +
𝜇(−3𝑖̂ + 2𝑗̂ + 4𝑘̂).
27. Find the area common to the curve 5x2 – y = 0 and 2x2 – y + 9 = 0.
𝑑𝑦
28. Solve = cos(𝑥 + 𝑦) + sin(𝑥 + 𝑦).
𝑑𝑥
29. A point on the hypotenuse of a right triangle is at a distance ‘a’ and ‘b’ from the
sides of the triangle. Show that the minimum length of the hypotenuse is
3
2 2 2
(𝑎 + 𝑏 ) .
3 3
IN-SERVICE COURSE FOR PGT ( Maths ) :2017-18
Time : 1h Mid-Test MM : 40
General Instructions:

I. All questions are compulsory


II. Question paper consists four sections A, B, C and D.
III. Section A comprises 4 questions of 1 mark each, Section B comprises 4 questions of 2 marks
each , Section C comprises 4 questions of 4 marks each and section D comprises 2 question
of 6 marks each
Section A

1: Let R be the equivalence relation in a set A = {0, 1, 2, 3, 4, 5} given by R ={ (a, b): 2


divides (a – b) }. Write the equivalence class [0].
2: Find the slope of normal to the curve y = 7 x3 + 11 at x = 2
3: What is action research?
4: What are different types of communication?
Section B
𝟏
( )
−𝟐 𝟏−𝒙
5: If y = 𝟐 , then find 𝐥𝐢𝐦+ 𝒚
𝒙→𝟏

6: Using differentials to find the approximate of √𝟎. 𝟎𝟑𝟕


7: Draw the graph of cosec x. Also, write its domain and range.
8: How action research can be effective for betterment of students ?

Section C
9: Draw the graph of y = 3x2 – 2x3. Also, write the point(s) of inflexion.
𝟐
10: Find the value of a and b so that the function f(x) = { 𝒙 + 𝟑𝒙 + 𝒂 ; 𝒙≤𝟏
𝒃𝒙 + 𝟐 ∶ 𝒙>1
is differentiable at each x ∈R
𝒅𝟐 𝒚 𝝅
11. If x = a (1 – cos 3t ) and y= a sin3 t , then find at t =
𝒅𝒙𝟐 𝟔

12. Write short notes on Multiple Intelligence.

Section D
13: Find the greatest and least value of (𝐬𝐢𝐧−𝟏 𝒙)𝟐 + (𝐜𝐨𝐬−𝟏 𝒙)𝟐
14. Let A = R – {3} and B = R –{1}. Consider a function f : A → B defined by f(x)
𝒙−𝟐
= . Show that f is an invertible function and hence find f - 1
𝒙−𝟑
IN-SERVICE COURSE FOR PGT ( Maths):2017-18
Time : 1h Post-TestMM : 40
General Instructions:

I. All questions are compulsory


II. This question paper consistsof four sections A, B,C and D.
III. Section A comprises 4 questions of 1 mark each, Section B comprises 4 questions of 2 marks each ,
Section C comprises 4 questions of 4 marks each and section D comprises 2 question of 6 marks each
Section A
𝟐
1:Find the primitive of𝒆𝒙 , if exists.
𝒅𝒚
2:Find the integrating Factor of differential equation x.log x𝒅𝒙 + y = log x

3:What is the full form of CCS and RDSO.

4: Solve : 3 𝐬𝐢𝐧−𝟏 𝒙 + 2 𝐜𝐨𝐬 −𝟏 𝒙 = 𝝅

Section B
𝝅
⃗⃗ of magnitude 5√𝟐, making an angle of 450 with x – axis , with y-axis and an acute angle 𝜽
5:Find a vector 𝒂 𝟐
with z axis
𝟏
6:Evaluate: ∫ 𝒅𝒙
𝟏−𝒕𝒂𝒏 𝒙

𝒅𝒚 𝟏−𝒚𝟐
7:If√𝟏 − 𝒙𝟐 + √𝟏 − 𝒚𝟐 = a (x - y) , prove that =√
𝒅𝒙 𝟏− 𝒙𝟐
8:Form the differential equation of the family of circles having radii 3, whose centre lies on y-axis.

Section C
𝟏+𝒙𝟐
9:Evaluate : ∫ dx
(𝟏−𝒙𝟐 )√𝟏+𝒙𝟐 +𝒙𝟒

10: A manufacturing company makes two types of television sets; one is black and white and the other is
colour. The company has resources to make at most 300 sets a week. It takes Rs 1800 to make a black and
white set and Rs 2700 to make a coloured set. The company can spend no more than Rs 648000 a week to
make television sets. If it makes a profit of Rs 510 per black and white set and Rs 675 per coloured set.
Formulate this problem as a LPP so that company has maximum profit.

11.Find the foot of perpendicular drawn from the point 𝟐𝒊̂ - 𝒋̂ + 5𝒌̂ to the line ⃗⃗⃗𝒊̂
𝒓 𝒋̂𝒌̂  𝝀(10𝒊̂4
̂)
𝒋̂𝒌

12. (i) Why is the first aid very necessary in Emergency? Describe briefly.
(ii) Describe briefly about Article 81(B)?

Section D
13: Find area of that part of the circle x2 + y2 = 16 which is exterior to the parabola y2 = 6x
14.A card from a pack of 52 cards is lost. From the remaining cards of the pack three cards are drawn at random
(without replacement) and are found to be all spades. Find the probability of the lost card being a spade.
Recipe for Being a Leader
 Listen effectively
 Encourage teamwork and participation
 Empower team members
 Communicate effectively
 Emphasize long-term productivity
 Make sound and timely decisions
 Treat each person as an individual
 Know yourself and your team
 Protect your team
 Have vision, courage and commitment

También podría gustarte